Você está na página 1de 504

A-PDF Merger DEMO : Purchase from www.A-PDF.

com to remove the watermark

ANGLO-CHINESE JUNIOR COLLEGE


MATHEMATICS DEPARTMENT

MATHEMATICS
Higher 2 9740 / 01
Paper 1 19 August 2010
JC 2 PRELIMINARY EXAMINATION
Time allowed: 3 hours

Additional Materials: List of Formulae (MF15)

READ THESE INSTRUCTIONS FIRST

Write your Index number, Form Class, graphic and/or scientific calculator model/s on the cover page.
Write your Index number and full name on all the work you hand in.
Write in dark blue or black pen on your answer scripts.
You may use a soft pencil for any diagrams or graphs.
Do not use paper clips, highlighters, glue or correction fluid.

Answer all the questions.


Give non-exact numerical answers correct to 3 significant figures, or 1 decimal place in the case of
angles in degrees, unless a different level of accuracy is specified in the question.
You are expected to use a graphic calculator.
Unsupported answers from a graphic calculator are allowed unless a question specifically states
otherwise.
Where unsupported answers from a graphic calculator are not allowed in the question, you are
required to present the mathematical steps using mathematical notations and not calculator commands.
You are reminded of the need for clear presentation in your answers.

The number of marks is given in brackets [ ] at the end of each question or part question.
At the end of the examination, fasten all your work securely together.

This document consists of 6 printed pages.

[Turn Over
ANGLO-CHINESE JUNIOR COLLEGE
MATHEMATICS DEPARTMENT
JC2 Preliminary Examination 2010

MATHEMATICS 9740
Higher 2
Paper 1
/ 100
Index No: Form Class: ___________
Name: _________________________
Calculator model: _____________________

Arrange your answers in the same numerical order.


Place this cover sheet on top of them and tie them together with the string provided.

Question no. Marks


1

5
6

10

11

12

13

14

Anglo-Chinese Junior College


H2 Mathematics 9740: 2010 JC 2 Preliminary Examination Paper 1
Page 2 of 6
n +1
1 The nth term of a sequence is given by un = ( −1) n 2 , for n ≥ 1 . The sum of the first n
terms is denoted by Sn. Use the method of mathematical induction to show that
n +1 n ( n + 1)
Sn = ( −1) for all positive integers n. [4]
2

2 The 3 flavours of puddings produced by a dessert shop are mango, durian and strawberry. A
mango pudding requires 5g of sugar and 36ml of water. A durian pudding requires 6g of
sugar and 38ml of water. A strawberry pudding requires 4g of sugar and 40ml of water. The
puddings are sold in pairs of the same type at $1.60, $2.20 and $1.80 for mango, durian and
strawberry respectively.
On a particular day, 754g of sugar and 5972ml of water were used to make the puddings and
all the puddings made were sold except for a pair of strawberry puddings. The collection
from the sale of puddings was $142.40. Formulate the equations required to determine the
number of each type of pudding made on that day. [4]

3 The diagram below shows the graph of y = f(x) . The curve passes through the origin and
has a maximum point at A ( 4 , 4 ) and asymptotes x = −2 and y = 2 .
y

y = f(x) ( 4 , 4)
2 A

x
−2

Sketch on separate diagrams, the graphs of


(i) y = 1 [3]
f(x)
(ii) y = f '(x) [3]
showing clearly asymptotes, intercepts and coordinates of turning points where possible.


4 The complex number w has modulus 3 and argument . Find the modulus and argument of
3
−i −i
, where w* is the complex conjugate of w. Hence express in the form a + ib , where
w* w*
a and b are real, giving the exact values of a and b in non-trigonometrical form. [4]
n
 −i 
Find the possible values of n such that   is purely imaginary. [2]
 w* 
[Turn Over

Anglo-Chinese Junior College


H2 Mathematics 9740: 2010 JC 2 Preliminary Examination Paper 1
Page 3 of 6
5 Solve the equation z 4 − i =0 , giving the roots in the form reiα , where r > 0 and
−π < α ≤ π . [3]
The roots represented by z1 and z2 are such that arg ( z1 ) > arg ( z2 ) > 0 . Show z1 , z2 and
z1 + z2 on an Argand diagram. Deduce the exact value of arg ( z1 + z2 ) . [3]

6 An economist is studying how the annual economic growth of 2 countries varies with time.
The annual economic growth of a country is measured in percentage and is denoted by G
and the time in years after 1980 is denoted by t. Both G and t are taken to be continuous
variables.
(i) Country A is a developing country and the economist found that G and t are can be modeled
dG G + 1
by the differential equation = . Given that, when t = 0 , G = 0 , find G in terms of t.
dt 2
[4]
(ii) Comment on the suitability of the above differential equation model to forecast the future
economic growth of Country A. [1]
(iii) Country B is a developed country and the economist found that G and t can be modeled by
dG  G +1
the differential equation = − .
dt  2 
Given that Country B has been experiencing decreasing economic growth during the period
of study, sketch a member of the family of solution curves of the differential equation model
for Country B. Hence, comment on the economic growth of Country B in the long term. [2]

7 (i) Given that f ( x ) = ecos , where −1 ≤ x ≤ 1 , find f ( 0 ) , f ′( 0 ) and f ′′( 0 ) . Hence write down
−1
x

the first three non-zero terms in the Maclaurin series for f ( x ) . Give the coefficients in terms
of e kπ , where k ∈  . [4]
(ii) Given that g= ( x ) tan x + sec x , where x is sufficiently small for x and higher powers of x
3

to be neglected. Deduce the first three non-zero terms in the series expansion of g ( x ) .
π π π
Hence, show that f ( x ) + e 2 g ( x ) ≈ 2e 2 + x 2 e 2 . [3]
π π π
f ( x ) + e 2 g ( x ) dx ≈ ∫ 2e 2 + x 2 e 2 dx ,
a a
(iii) Explain clearly why it is inappropriate to state that ∫ −a −a
where a ∈  . [1]

8 (i) Show that 1 − 2 + 1 Ar 2 + Br + C , where A, B and C are constants to be


=
r ! ( r + 1) ! ( r + 2 ) ! ( r + 2 )!
found. [2]
n
3r 2 + 3r − 3 .
(ii) Hence find ∑ ( r + 2 )! [3]
r =1

3r 2 + 3r − 3 converges, and write down its value.
(iii) Give a reason why the series ∑ r + 2 )! [2]
r =0 (

Anglo-Chinese Junior College


H2 Mathematics 9740: 2010 JC 2 Preliminary Examination Paper 1
Page 4 of 6
9 Relative to the origin O, two points A and B have position vectors given by a = 3i + j + 3k
and b = 5i − 4 j + 3k respectively.
 
(i) Find the length of the projection of OA on OB . [2]
(ii) Hence, or otherwise, find the position vector of the point C on OB such that AC is
perpendicular to OB. [2]
(iii) Find a vector equation of the reflection of the line AB in the line AC. [3]

10(a) The first 2 terms of a geometric progression are a and b ( b < a ). If the sum of the first n
terms is equal to twice the sum to infinity of the remaining terms, prove that a n = 3b n .
[3]
(b) The terms u1 , u2 , u3 ,... form an arithmetic sequence with first term a and having non-zero
common difference d.
(i) Given that the sum of the first 10 terms of the sequence is 105 more than 10u5 , find
the common difference. [3]
(ii) If u26 is the first term in the sequence which is greater than 542, find the range of
values of a. [3]

11 2 y 2 a ( 2a − x ) , where a > 0 ,
The region R in the first quadrant is bounded by the curve =
and the line joining ( 2a, 0 ) and ( 0, a ) . The region S, lying in the first quadrant, is bounded
2 y 2 a ( 2a − x ) and the lines x = 2a and y = a .
by the curve =
(i) Draw a sketch showing the regions R and S. [1]
(ii) Find, in terms of a, the volume of the solid formed when S is rotated completely about the
x-axis. [4]
(iii) By using a suitable translation, find, in terms of a, the volume of the solid formed when R is
rotated completely about the line x = 2a . [4]

The curve C has the equation y = 3x + ax + 2 where a is a constant.


2
12
x+a
dy
(i) Find and the set of values of a if the curve has 2 stationary points. [4]
dx
(ii) Sketch the curve C for a = 1, stating clearly the exact coordinates of any points of
intersection with the axes and the equations of any asymptotes. [3]
Hence, find the range of values of k such that the equation 3x + x + 2= k(x + 1) has exactly
2

2 real roots. [2]

13 The curve has the parametric equations


x = 5 2 , y = tan −1 t
1+ t
(i) Sketch the curve for −2 ≤ t ≤ 2 . [1]
(ii) Find the cartesian equations of the tangent and the normal to the curve at the point
where t = 1 . [5]
(iii) Find the area enclosed by the x-axis, the tangent and the normal at the point where t = 1. [3]
[Turn Over

Anglo-Chinese Junior College


H2 Mathematics 9740: 2010 JC 2 Preliminary Examination Paper 1
Page 5 of 6
14 The functions f and g are defined as follows:
 π π
f : x  sin x , x ∈ − ,  ,
 2 2
π
( x + 1)( 3 − x ) ,
g:x  x∈ .
8
(i) Sketch the graph of the function g, labeling clearly the exact values of the coordinates of
turning point(s) and intersections with the axes, if any. [1]
State the range of the function g in exact values. [1]

(ii) Given that gf exists as a function. By considering the graphs of f and g, explain why
π π
gf (α ) ≠ gf ( β ) if − ≤α < β ≤
. [2]
2 2
Hence what can be said about the function gf ? [1]
Without sketching the graph of gf , find the range of gf in the form [ a, b ] , giving the exact
values of a and b. [1]

(iii) (a) Give a reason why fg does not exist as a function. [1]
(b) Find the greatest exact value of k for which fg is a function if the domain of g is
restricted to the interval [1, k ] . [2]

- End of Paper -

Anglo-Chinese Junior College


H2 Mathematics 9740: 2010 JC 2 Preliminary Examination Paper 1
Page 6 of 6
ANGLO-CHINESE JUNIOR COLLEGE
MATHEMATICS DEPARTMENT

MATHEMATICS
Higher 2 9740 / 02
Paper 2 23 August 2010
JC 2 PRELIMINARY EXAMINATION
Time allowed: 3 hours

Additional Materials: List of Formulae (MF15)

READ THESE INSTRUCTIONS FIRST

Write your Index number, Form Class, graphic and/or scientific calculator model/s on the cover page.
Write your Index number and full name on all the work you hand in.
Write in dark blue or black pen on your answer scripts.
You may use a soft pencil for any diagrams or graphs.
Do not use paper clips, highlighters, glue or correction fluid.

Answer all the questions.


Give non-exact numerical answers correct to 3 significant figures, or 1 decimal place in the case of
angles in degrees, unless a different level of accuracy is specified in the question.
You are expected to use a graphic calculator.
Unsupported answers from a graphic calculator are allowed unless a question specifically states
otherwise.
Where unsupported answers from a graphic calculator are not allowed in the question, you are
required to present the mathematical steps using mathematical notations and not calculator commands.
You are reminded of the need for clear presentation in your answers.

The number of marks is given in brackets [ ] at the end of each question or part question.
At the end of the examination, fasten all your work securely together.

This document consists of 5 printed pages.

[Turn Over
ANGLO-CHINESE JUNIOR COLLEGE
MATHEMATICS DEPARTMENT
JC2 Preliminary Examination 2010

MATHEMATICS 9740
Higher 2
Paper 2
/ 100
Index No: Form Class: ___________
Name: _________________________
Calculator model: _____________________

Arrange your answers in the same numerical order.


Place this cover sheet on top of them and tie them together with the string provided.

Question no. Marks


1

5
6

10

11

12

13

14

Anglo-Chinese Junior College


H2 Mathematics 9740: 2010 JC 2 Preliminary Examination Paper 2
Page 2 of 5
Section A: Pure Mathematics [40 marks]
1
1
1 Find the exact value of ∫
−1
e2 x −
e 2 ( x −1)
dx. [4]

2 The variable complex numbers z and w are such that z − 2 − i =3 and arg ( w − 5 + 3i ) =π.
(i) Illustrate both of these relations on a single Argand diagram. [2]
(ii) State the least value of z − w . [1]
(iii) Find the greatest and least possible values of arg ( z + 3) , giving your answers in radians
correct to 3 decimal places. [4]

 a
3 Find in terms of a, the range of values of x that satisfy the inequality ln  2 x −  ≥ 0 ,
 x
where a > 1 . [4]

4 (a) State the derivative of cos x3 . Hence, find ∫x


5
sin x3 dx. [4]
2 5 3

∫ (x − 5)

(b) Find the exact value of 2 2 dx , in the form a 2 + b 3 , using the
10

substitution x = 5 sec θ . [6]

x +1 z −1
5 The plane p1 has equation x + 2 y − z =3 . The line l1 has equation = y= .
2 4
(i) Show that the line l1 is parallel to, but not contained in the plane p1. [2]
(ii) Find the cartesian equation of the plane p2 which contains the line l1 and is perpendicular to
the plane p1. [3]
(iii) Find, in scalar product form, the vector equation of the plane p3 which contains the point
( 4,1, −1) and is perpendicular to both p1 and p2. [2]
2
 
Another line l2 which is parallel to the vector  0  intersects the line l1 at the
 −3 
 
point A ( −1, 0,1) .
(iv) Given that the line l2 meets the plane p1 at the point B, find the coordinates of B. [4]
(v) Find the sine of the acute angle between the line l2 and the plane p1, and hence, find the
length of the projection of the line segment AB on the plane p1, giving your answer in surd
form. [4]

Section B: Statistics [60 marks]


6 Mr Raju, who owns a supermarket wishes to find out what customers think about the goods
that he sells. He has been advised that he should take a random sample of his customers for
this purpose. State, with reasons, which of the following sampling procedures is preferable.
A. Select every 10th customer on each day in a typical week.
B. Select the first 20 customers on each day in a typical week [2]

[Turn Over

Anglo-Chinese Junior College


H2 Mathematics 9740: 2010 JC 2 Preliminary Examination Paper 2
Page 3 of 5
7 One day, Pinocchio went shopping and bought a pair of size 30 Wood Shoes. The right
shoes have lengths which are normally distributed with mean 20 cm and standard deviation
0.14 cm. The left shoes have lengths which are normally distributed with mean 20.1 cm and
standard deviation 0.11 cm. The length of the right shoe is independent of the length of the
left shoe.
When wearing the pair of shoes, Pinocchio takes six steps, heel to toe as shown in the
diagram. Calculate the probability that the distance AB, from the back of the first step to the
front of the sixth step, exceeds 120 cm.

A B [3]

8 On Ulu Island the weights of adult men and women may both be taken to be independent
normal random variables with means 75kg and 65 kg and standard deviations 4 kg and 3 kg
respectively.
Find the probability that the weight of a randomly chosen man and the weight of a randomly
chosen woman differ by more than 1 kg. [3]
Explain if this is equal to the probability that the difference in weight between a randomly
chosen married woman and her husband is more than 1kg. [1]

9 Research has shown that before using an Internet service, the mean monthly family
telephone costs is $72. A random sample of families which had started to use an Internet
service was taken and their monthly telephone costs were :
$70, $84, $89, $96, $74
Stating a necessary assumption about the population, carry out a test at the 5% significance
level, whether there is an increase in the mean monthly telephone costs. [5]
If the assumption stated above still holds, and if the standard deviation of the monthly
telephone costs is $9.89, find the range of values of the mean monthly family telephone
costs µ 0 that would lead to a reverse in the decision to the above test. [3]

10 Six overweight men registered at a slimming centre for a slimming programme. The
following table records x, the height (to the nearest cm) and y, the weight (to the nearest 0.1
kg) of these six men.
Man A B C D E F
x (height in cm) 150 157 160 162 167 170
y (weight in kg) 65.1 73.2 85 k 80.9 89.9
(i) Given that the least square regression line of x on y line =
is x 103.6 + 0.726 y , show that
the value of k to the nearest 0.1 kg is 80.3. Hence or otherwise, find the least square
regression line of y on x in the form =
y ax + b , giving the values of a and b to the nearest
3 decimal places. [5]
(ii) Based on the data given, use an appropriate regression line to predict the weight of an
overweight man who is 165 cm tall. [2]
(iii) Find the value of the product moment correlation coefficient between x and y and sketch
the scatter diagram of y against x. A particular man among the 6 men who registered for
the slimming programme is unusually overweight. Indicate who this man is. [3]

Anglo-Chinese Junior College


H2 Mathematics 9740: 2010 JC 2 Preliminary Examination Paper 2
Page 4 of 5
11 In a hotel, large number of cups and saucers are washed each day. The number of cups that
are broken each day while washing averages 2.1. State in context, a condition under which
a Poisson distribution would be a suitable probability model. [1]
Assume that the number of broken cups and saucers follow a Poisson distribution.
(i) Show that on any randomly chosen day, the probability that at least 3 cups are broken is
0.350 correct to 3 significant figures. [1]
The probability that there will be at least two days in n days with at least 3 broken cups is
more than 0.999. Find the least value of n. [3]
(ii) The number of saucers broken each day averages 1.6, independently of the number of cups
broken. The total number of cups broken and saucers broken during a week of 7 days is
denoted by T. State a possible model for the distribution of T. [2]
A random sample of 100 weeks is chosen. Using a suitable approximation, find the
probability that the average weekly total number of broken cups and saucers does not exceed
26. [3]

12 Fish are bred in large batches and allowed to grow until they are caught at random for sale.
When caught, only 20% of the fish measure less than 8 cm long.
(i) What is the probability that the 10th fish caught is the sixth fish that is less than 8 cm long?
[2]
(ii) A large number, n, of fish are caught and the probability of there being 10 or fewer fish in
the catch which measures less than 8 cm long is at most 0.0227 .
Using a suitable approximation, derive the approximate inequality
10.5 − 0.2n ≤ −0.8 n . [4]
Hence find the least possible number of fish to be caught. [2]

13 An automated blood pressure machine is being tested. Members of the public, p % of whom
have high blood pressure (hypertension), try it out and are then seen by a doctor. She finds
that 80% of those with hypertension and 10% of those with normal blood pressure have
been diagnosed as hypertensive by the machine. The probability that a randomly chosen
patient who was diagnosed as hypertensive by the machine actually has hypertension is 2 .
3
(i) Find the value of p [3]
(ii) Hence, find the probability that a randomly chosen patient does not have hypertension,
given that the machine diagnosed him as having normal blood pressure. [2]
Comment on the usefulness of the machine. [1]

14 Ten balls are identical in size and shape of which 2 are red, 3 are blue and 5 are green. The
two red balls are labeled ‘1’ and ‘2’, the three blue balls are labeled ‘1’, ‘2’ and ‘3’, and the
five green balls are labeled ‘1’, ‘2’, ‘3’, ‘4’ and ‘5’.
(i) Find the number of ways of choosing 2 balls of identical colour. [2]
(ii) Find the number of ways of choosing 6 balls if it includes at least one ball of each colour. [4]
(iii) A person arranged 3 balls in a row with the numbered sides facing him forming a 3-digit
number. Among these 3 balls, none of them are green. Find the number of possible 3-digit
numbers facing that person.
[The number formed is independent of the colours of the balls used. i.e. the number 112 is
counted as one number whether the colour of the ball labeled ‘2’ is red or blue.] [3]
- End of Paper -
Anglo-Chinese Junior College
H2 Mathematics 9740: 2010 JC 2 Preliminary Examination Paper 2
Page 5 of 5
Anglo-Chinese Junior College
H2 Mathematics 9740 3
y
2010 JC 2 PRELIM Marking Scheme (ii) y = f '(x)
Paper 1:
1 n +1 n ( n + 1)
Let Pn denote the statement Sn = ( −1)
2
1( 2 )
LHS = S1 = u1 = ( −1) 12 = 1 RHS = ( −1)
2 2
For n = 1, =1
2
LHS = RHS
∴ P1 is true.
k ( k + 1)
k +1 (4,0)
Assume Pk true for some k ∈ ℤ + , i.e. Sk = ( −1) x
2 −2 A’
k + 2 ( k + 1)( k + 2 )
Prove that Pk+1 is true, i.e. S k +1 = ( −1)
2
LHS = S k +1 = S k + uk +1
k ( k + 1)
= ( −1) + ( −1) ( k + 1)
k +1 k +2 2

2 4 −i 1
=
k +1 ( k + 1) w* 3
= ( −1)  k − 2 ( k + 1) 
2   −i  π  2π  π
arg   = arg(−i ) − arg( w*) = − −  − =
k +1 ( k + 1)  w*  2  3  6
= ( −1) [ −k − 2]
2 −i 1  π π  1 3 1  3 1
=  cos + i sin  =  + i = + i
k + 2 ( k + 1)( k + 2 ) w* 3  6 6  3  2 2  6 6
= ( −1) = RHS
2 n n
Since P1 is true, and Pk is true ⇒ Pk+1 is true,  −i   1   nπ nπ 
  =    cos + i sin 
by the principle of mathematical induction, Pn is true ∀ n ∈ ℤ + .  w*   3   6 6 
n
2 Let x be the no of mango puddings produce.  −i  nπ
Let y be the no of durian puddings produce.   is purely imaginary, cos =0
 w*  6
Let z be the no of strawberry puddings produce.
π π
5 x + 6 y + 4 z = 754 ……………………………(1) n = ( 2k + 1) , k ∈ ℤ ,
6 2
36 x + 38 y + 40 z = 5972 ………………………(2)
∴ n = 3 ( 2k + 1) , k ∈ ℤ .
0.8 x + 1.1y + 0.9( z − 2) = 142.4 ……………….(3)
Solving (1), (2) & (3) using GC,
5 π
x = 46, y = 42 and z = 68. i
z 4 − i = 0 ⇒ z 4 = i ⇒ z 4 = 1e 2
y
y= 1 π 
i + 2 kπ 
3 f(x) z4 = e2 , k = −2, −1, 0,1
(i) 1 π 
i  + 2 kπ 
4 2 
z=e , k = −2, −1, 0,1
 7π   3π  π   5π 
i −  i −  i  i  
z= e  8 ,e  8 ,e  8 ,e  8 
y = 0.5 y
z2 + z2
A’ (4, 0.25)
z1
x
−2

8 z2 x
π
O 8
x=0
1 2
 −1   −1  cos−1 x  −1 
−3

 5π 
i 
f ′′( x ) =  
 1− x  1− x
2 2
e

( −1

 2 
)
+ −ecos x   (1 − x 2 ) 2 ( −2 x )
z1 = e 8 
−3 π

z2 =
π 
i 
e8
=
1 cos−1 x
1 − x2
e (
− xecos x (1 − x
−1

) )
2 2
⇒ f ′′ ( 0 ) = e 2

π 1  4π  3π
arg ( z1 + z2 ) = +  =
8 2 8  8 π
π π
(ii) e2 2
∴f ( x) = e 2 − e 2 x + x + ...
6 dG G + 1 2
=
dt 2
1 1 g ( x ) = tan x + sec x
∫ G + 1 = ∫ 2 dt
dG
−1
1  x2    x2  
ln G + 1 = 0.5t + C = tan x + ≈ x + 1 −  = x +  1 + ( −1)  −  + ... 
cos x  2    2  
G + 1 = ±e 0.5t +C
x2
G = −1 + Ae0.5t , where A = ±eC ≈ 1+ x +
2
When t = 0 , G = 0 ,  π π

π π
A =1 e 2 2  π2  x2 
(iii) f ( x) + e 2 g ( x) ≈  e 2 − e 2 x + x + e 1 + x + 
∴ G = −1 + e0.5t  2   2 
 
 
Examples of possible comments: π π

The model is not suitable because … = 2e 2 + e 2 x 2 (Shown)


 The economist is assuming that that there are no fluctuations in the economic
π π π
growth in the future. a a

 The economist is assuming that the country will enjoy perpetual economic The statement ∫ f ( x ) + e 2 g ( x ) dx ≈ ∫ 2e 2 + x 2 e 2 dx is inappropriate as
−a −a
growth in the long term. π π π

 The economist is assuming Country A is always experiencing positive and f ( x ) + e 2 g ( x ) ≈ 2e 2 + x 2 e 2 only when a is sufficiently small.
increasing economic growth in the future.
 Factors affecting economic growth remains unchanged. 8 1− 2 + 1
(i) r ! ( r + 1) ! ( r + 2 ) !
G

t =
( r + 1)( r + 2 ) − 2 ( r + 2 ) + 1
0
( r + 2 )!
G = −1 + Be −0.5t , B = 1 2
= r + r −1
−1
( r + 2 )!
A = 1, B = 1, C = −1.
(ii) n 2
∑ 3r( r++32r)−! 3
r =1
n 2
= 3∑ r + r − 1
In the long term, Country B is expected to be still in recession with an economic r =1 ( r + 2 )!
growth decreasing towards -1%. n
 
π = 3∑  1 − 2 + 1 
7(i)
f ( x ) = ecos
−1
x
⇒ f (0) = e 2 r =1 
r ! ( r + 1) ! ( r + 2 ) ! 
π
−1
f ′( x ) = ⇒ f ′ ( 0 ) = −e 2
−1
ecos x

1− x 2

3 4
 1 2 1 
  1! − 2! + 3! 


 5λ − 3   5 
     
 1 2 1   − 4λ − 1  .  − 4  = 0
 + − +    3λ − 3   −3 
  2! 3! 4!     
 1 2 1 
 + − +   2
  3! 4! 5!   λ=
 1 2 1  5
+ − +  
 4! 5! 6! 
= 3 
⋮  5
   2  
  1 2 1  OC =  −4 
 +  − + 
 5 
  ( n − 2 ) ! ( n − 1) ! n !    3
 
 +  1 − 2 + 1   5
(iii)   ( n − 1) ! n ! ( n + 1) !   
    (iii)  2  1  1 
 1
Since OC = OB , OC : CB = 2 : 3 OB ' = − OB = −  −4 
2 1    5 5 5 
 +  − +   = 3 1 − 1 + 1 
  n ! ( n + 1) ! ( n + 2 ) !    3
 2 ( n + 1) ! ( n + 2 ) ! 
Or use midpoint theorem,
 1  
n 2
∑ 3r( r++32r)−! 3 2
(
OC = OB ' + OB )
r =0
  5   5   5
  
( ) 2 
OB ' = 2OC − OB = 2   −4   −  −4  = −  −4 
2 n 1
= − 3 + ∑ 3r + 3r − 3
2! r =1 ( r + 2 ) !  5  3   3  5 
      3
= ( − 3 ) + 3 1 − 1 + 1 
 
2  2 ( n + 1) ! ( n + 2 ) !   5   3  20 
1 → 0 and 1 → 0.    1    1 
As n → ∞, AB ' = OB ' − OA = −  −4  −  1  = −  1 
( n + 1)! ( n+2 )! 5    5 
∴ the series converges to 0.  3   3  18 

 
9(i) Length of the projection of OA on OB
 3  20 
 3  5    
Vector equation of line AB ' is r =  1  + α  1  , α ∈ ℝ
=  1  ⋅
1   20 4
 −4  = 50 = 2 2 or 2 ɶ    18 
 3  50  3   3  
   
10 GP : a = a, r = b
(ii) Method 1: (a) a

A From (i), OC = 2 2 n
The sum to infinity of the remaining terms, S∞ = ar
1− r
  5   5 Sn = 2S∞
  1   2 
OC = 2 2  −4 = −4
O C B 
50   
 3  
5  
 3
(
a 1− rn )= 2ar
n

1− r 1− r
Method 2:
1 − r n = 2r n
 0  5
Line OB: r =  0  + λ  −4  3r n = 1
 0  3 rn = 1
    3
 5   3   5λ − 3 
( ba ) = 13
n
  
AC = OC − OA = λ  −4  −  1  =  −4λ − 1
 3   3   3λ − 3 
      3b n = a n
 
Since AC ⊥ OB ,
(b) (i)

5 6
S10 = 105 + 10u5 (i) 2
y = 3x + ax + 2
x+a
2[ ] [ ]
10 2a + 9d = 105 + 10 a + 4d

10a + 45d = 105 + 10a + 40d =


2
(
dy ( x + a )( 6x + a ) − 3x + ax + 2 )
d = 21
dx ( x + a) 2

(ii)
=
(
3x 2 + 6ax + a 2 − 2 )
u25 ≤ 542 u26 > 542 ( x + a )2
a + 24(21) ≤ 542 and a + 25(21) > 542
For stationary points,
a ≤ 38 a > 17 dy
=0
dx

∴17 < a ≤ 38 (
3x 2 + 6ax + a 2 − 2 = 0 )
11 y
(i) For 2 stationary points,
a
2 y 2 = a ( 2a − x )
R
S ( 6a ) 2 − 4 ( 3 ) ( a 2 − 2 ) > 0
x 24a 2 > −24
O 2a a 2 > −1
a 2 is always positive.
∴ a ∈ ℝ.

(ii) When S is rotated completely about the x-axis, 2


y = 3x + x + 2 = 3x − 2 + 4
Required volume = π a 2 ( 2a ) − π ∫
2a a x +1 x +1
0 2
( 2a − x ) dx (ii)
2a
π a  ( 2a − x ) 
2
= 2π a 3 −  
2  −2
  0
πa
= 2π a 3 −
2
( )
2a 2

= π a 3 cu. units

Points of intersection with the axes : (0,2)


(iii) After a translation of 2a units in the negative x-direction, Asymptotes : y = 3x − 2 and x = −1 .
2 y2
New equation is 2 y 2 = a ( 2a − ( x + 2a ) ) ⇒ x = −
a
When R is rotated completely about the line x = 2a , The range of values of k such that the equation 3x 2 + x + 2 = k(x + 1) has exactly 2
2
1 a 2y 
2 real roots :
Required volume = π ( 2a ) ( a ) − π ∫  −
2
 dy k > 1.93 or k < −11.9
3 0
 a 
a 13
4  4 y5 
= π a3 − π
 2 (i)
3  5a  0
4 3 4 3 8
= π a − π a = π a 3 cu. units
3 5 15

12
7 8
π
and g : x ֏ ( x + 1)(3 − x), x ∈ ℝ .
8

(i)
5 y

π (1, π )
2
2

(ii)
dy
= 1 and dx = -5 ( 2t ) 8
y = g ( x)
dt 1 + t 2
( )
dt 2
1+ t2
dy dy dt
= .
dx dt dx -1 1 3 x
0
( )
2
2
= 1 . 1+t
1 + t 2 −10t

=−
(1 + t )2

10t

When t = 1, π
Rg = (−∞, ]
dy 2
= −1 (ii)
dx 5

Equation of tangent :
y
y−π
4 = −1 y = f ( x)
x− 5 5 1
(iii) 2
f (α )
y = −1 x+ 1 + π
5 2 4 ( )
π π x From the graph of y = f ( x) ,
Gradient of normal is 5 − α 0 β
π π
2 2 f (α ) ≠ f ( β ) if − ≤α < β ≤ .
Equation of normal: 2 2
f (β ) In fact, −1 ≤ f (α ) < f ( β ) ≤ 1 .
y−π -1
4 =5
y
x− 5
2 y = g ( x)
(
y = 5x − 25 − π
2 4 ) π 2
3π 8

2 (( ) (
Area = 1 × 5 1 + π − 1 25 − π × π
2 4 5 2 4 4 ))
2
= 13π = 1.60 (3s.f )
80
x
-1 0 1

14 Functions From the graph of y = g ( x) ,


gf (α ) < gf ( β )
 π π
Given f : x ֏ sin x, x ∈  − , 
 2 2 or gf (α ) ≠ gf ( β )

9 10
if −1 ≤ f (α ) < f ( β ) ≤ 1 .

gf is one-one (or increasing).


π
Rgf = [0, ] .
2

π π π
(iii) Rg = (−∞, ] ⊄ [− . ] = D f .
(a) 2 2 2

Alternative:
5π 5π π π
fg (4) = f [ g (4)] = f [−
] is undefined because − ∉ [− , ] = D f .
8 8 2 2
Hence, fg does not exist as a function.

(iii) y
(b)
(1, π )
2

y = g ( x)

-1 1 3 x
0
π
y=−
2

π
g ( x) =
2
π π
( x + 1)( 3 − x ) = −
8 2
( x + 1)( 3 − x ) = −4
− x 2 + 2 x + 3 = −4
x2 − 2 x − 7 = 0
2 ± 32
x=
2
x = 1± 8
Since x ≥ 1, x = 1 + 8 .
1 ≤ k ≤ 1+ 8
Greatest value of k is 1 + 8 .

11 12
Anglo-Chinese Junior College
H2 Mathematics 9740
∫ x sin x dx
5 3

2010 JC 2 PRELIM Marking Scheme


Paper 2: = ∫ x ( x sin x ) dx
3 2 3

1
1  1   1  2
1 ∫ e2 x −
e2 ( x −1)
dx = x 3  − cos x3  − ∫  − cos x3  3 x dx
−1  3   3 
1
2 1 x3
1 1 = − cos x3 + ∫ x 2 cos x3 dx
=− ∫−1
e2 x −
e 2 ( x −1)
dx + ∫
1
e2 x −
e 2 ( x −1)
dx 3
2
x3 1
1 1 = − cos x3 + sin x3 + c
1 1  2 1 1  3 3
= −  e 2 x + e−2 ( x −1)  +  e 2 x + e −2 ( x −1) 
2 2  −1  2 2  12
4 (b) π
x = 5 sec θ when x = 10, θ =
=
2
( e + e − 4e + e−2 + 1)
1 4 2
4
dx π
= 5 sec θ tan θ when x = 2 5, θ =
2 (i) dθ 3
y
2 5 3

∫ (x − 5)
2 −
2 dx
3 z 10
π
26 3 3

∫π ( 5sec θ − 5)

i (2,1) = 2 2 5 sec θ tan θ dθ
3
−3 O 2 x 4
3 π
1 3
sec θ
w
(5,−3) =
5 ∫π tan 2 θ

o 4
π π
1 3
cos θ 1 3
=
5 ∫π sin 2 θ
dθ or
5 ∫π cot θ cos ec θ dθ
2 (ii) Least value of z − w = 1 4 4
π
1 1  3 1 π
= − = [ − cos ec θ ]π 34
−1  1  −1  3  5  sin θ  π 5
2 (iii) Greatest arg ( z + 3) = tan   + sin   = 0.826 (3 dp) 4
5  26  1 2 1 2
 3  = 2− 3 i.e. a = b=−
−1  1 
Least arg ( z + 3) = tan   − sin −1   = −0.432 (3 dp)
5 15 5 15
5  26  1  −1 2
 
3  a p1 : r ⋅  2  = 3 l1 : r =  0  + λ  1  , λ ∈ ℝ
ln  2 x −  ≥ 0 where a > 1 . 5 ɶ  
 −1 
ɶ  
1
 
4
 x
 2  1   −1  1 
a  1  . 2  = 2 + 2 − 4 = 0   
2x − ≥ 1     0  .  2  = −1 + 0 − 1 ≠ 3
(i)      
x  4   −1  1   −1
2 x2 − x − a ⇒ l1 is parallel to p1. ⇒ l1 is not contained in p1.
≥0
x
Alternative method:
 1 + 1 + 8a )   1 − 1 + 8a )   −1 + 2λ   1 
2 x −   x − 
 4 4  λ i 2  = −2 ≠ 3
   ≥0   
 1 + 4λ   −1
  
x
Since no solution for λ , ∴ l1 is parallel and not contained to p1
1 − 1 + 8a 1 + 1 + 8a
≤ x < 0 or x ≥  1   2  −3 
4 4 (ii)      
4 (a)  2  ×  1  = −3  2 
d
dx
( cos x3 ) = − 3 x 2 ( sin x 3 )  −1   4 
   
1
 

1 2
 −3   −1   −3 
p2 : r ⋅  2  =  0 i 2  = 3 + 0 + 1 = 4
No , (i) The weight of a husband and wife may not be independent
p2 : −3x + 2 y + z = 4
ɶ      Or (ii) Randomness is not there ( a randomly chosen women but spouse is not
1 11 randomly chosen)
Or (iii) Distribution of weight of married woman is different from distribution of
adult woman.
 2  4   2  2 Etc
(iii)         9 Telephone costs are assumed to be normally distributed.
p3 : r ⋅  1  =  1  ⋅  1  = 8 + 1 − 4 = 5 p3 : r ⋅  1  = 5
ɶ       ɶ  
 4   −1   4   4

 −1  2 1
(iv)      
l2 : r =  0  + µ  0  , µ ∈ ℝ p1 : r ⋅  2  = 3
ɶ   ɶ   To test H0 : µ = 72
 −3 
1
     −1 against H1: µ > 72 at 5% level of significance
 −1 + 2 µ   1 
  
 0  .  2  = 3 ⇒ − 2 + 5µ = 3 ⇒ µ = 1 ∴ point B is (1, 0, −2 )
_
x− µ 0
 1 − 3µ   −1 Under H0 , T= t(5-1)
   s n

2 1 _
x− µ 0 82.6 − 72
1   1   5 Test statistics : T= = = 2.2219
(v) sin θ = 0 ⋅ 2 =
13   6   78 s n 10.6677 / 5
 −3   −1
p value = P(T > 2.2219) = 0.0452 < 0.05

Reject H0 at the 5% level of significance. We conclude that there is sufficient


 evidence at the 5% level of significance that there is evidence of an increase in
Length of the projection of AB on p1 = AB cos θ
mean monthly costs.
2
 53 
=  0  1 −
25 53 1
= 13   = = 318
 −3  78  78  6 6
  To test H0 : µ = µ 0
against H1: µ > µ 0 at 5% level of significance
6 Procedure A is preferable as it is unbiased.
Early customers may not be typical customers in general. _
x− µ0
Under H0 , Z = N(0,1)
7 Let R be the r.v for the length of a right shoe and L for the left shoe 9.89 n
R N(20, 0.142) and L N(20.1 , 0.112)
Method 1 _

X = R + L N(40.1, 0.0317) x − µ 0 82.6 − µ0 5


Test statistics : Z = = = (82.6 - µ 0 )
3X N(40.1 x 3, 0.0317 x 32) σ n 9.89 / 5 9.89
P(X > 120) = 0.713
Method 2 5
Do not reject H0 if P(Z > (82.6 - µ 0 ) ) > 0.05
R + L N(40.1, 0.0317) 9.89
P(R + L > 120
3 )
= 0.713 5
(82.6 - µ 0 ) < 1.64485...........(1)
8 Let M and W be the rv for the weight of an adult man and woman respectively. 9.89
M N(75,42) and W N(65,32)
µ0 > 75.3
W M ~ N(-10, 52)
10
P( W − M > 1 ) = P(W – M > 1) + P(W – M< -1) = 0.978 (i) x = 161 (from calculator or computation)
Or P( W − M > 1 ) = 1 - P( W − M < 1 ) = 1 – P(-1<W – M<1) = 0.978

3 4
when x = 161 , x = 103.6 + 0.726 y P(X ≥ 2) > 0.999
1 – P(X ≤ 1) > 0.999............(1)
y = (161 − 103.6) / 0.726 P(X ≤ 1) < 0.001
= 79.06336088 Using GC,
using y = ∑ y n n P(X ≤ 1)
1 21 0.00145
79.06336088 = (65.1 + 73.2 + 85 + k + 80.9 + 89.9) 22 0.000984
6
k = 80.3 least n is 22
Use G.C. to find regression line of y on x:
(ii) T Po(2.1x7 + 1.6 x 7)
y = −97.593 + 1.097 x T Po(25.9)
Method 1:
Since n is large,
_
(ii) Use y on x line to predict weight. 25.9
T (25.9, ) approx by central limit theorem
100
When x = 165 , y = −97.593 + 1.097(165) _
P( T ≤ 26) = 0.578 (to 3 sig fig)
y = 83.4 (1 d.p.) – using 3 d.p. of a and b to compute.
Method 2:
or 100 weeks, Y Po(2590)
y = 83.5 - using full accuracy of a and b to compute.
λ = 2590 > 10 . Normal approx to Poisson
(iii) Y N(2590,2590) approx
Using G.C., r = 0.893
P(Y ≤ 2600) = P( Y < 2600.5) (With cc)
= 0.578 (to 3 sig fig)
y
12 Let X be the r.v for the number of fish which measures less than 8 cm long.
9 1 4 1
89.9 (i) C5 ( )5 ( )4 ( ) = 0.00330 ( to 3 sf)
5 5 5
85.0
80.9 X B(n,0.2)
80.3 (ii) Since n large and p= 0.2 , X N(0.2n,(0.2)(0.8)n) approx
P(X ≤ 10) ≤ 0.0227
73.2 P(X < 10.5) ≤ 0.0227
10.5 − 0.2n
65.1 x P(Z < ) ≤ 0.0227
150 157 160 162 167 170 0.4 n
10.5 − 0.2n
≤ -2.000929..........(1)
0.4 n
C is unusually overweight.
10.5 – 0.2n ≤ -0.800372 n
Hence 10.5 – 0.2n ≤ -0.8 n approx
11 Breakages occur randomly or Method 1
Breakages occur independently or
Using GC Y1 = 10.5 -0.2x + 0.8 x
Mean number of breakages is a constant → Table Ans : 91

Method 2 : Use GC and graph


(i) Let A be the r.v for the number of broken cups per day
A Po(2.1)
P(A ≥ 3) = 1 – P(A ≤ 2) = 0.350369 = 0.350 (3 sig figs)

Let X be the r.v for the number of days with a least 3 broken cups out of n cups.
X
5 6
Method 3: 7
balls:   [Note: We can’t exclude green balls because total
Hence (10.5 – 0.2n)2 ≥ (-0.8 n )2 6
Hence 4n2 - 484n + 11025 ≥ 0 approx……(2) number of red and blue balls is only 5.]
From GC : n ≥ 90.6 or n ≤ 30.4
 10   8   7  
n ≥ 91 or n ≤ 30 (NA because does not satisfy (1) ) No. of ways =   −   +    = 210 − (28 + 7) = 175
Least n = 91  6   6   6  
13 Let H be the event that the member of public has hypertension
Let D be the event that the machine diagnosed hypertension
(i) Alternative Method:
D
0.8 Case Green Blue Red No. of ways
k H  5   3  2 
0.2
1 4 1 1   ×   ×   = 30
D’  4 1  1 
0.1 D 5  3  2
1-k H’ 2 3 2 1  3  ×  2  ×  1  = 60
     
(i) 0.9 D  5   3  2 
2 P ( H ∩ D)
P(H/D) = =
’ 3 3 1 2   ×   ×   = 30
 3 1  2
3 P( D)
5  3  2
4 2 3 1   ×   ×   = 60
2 0.8k  3  2  1 
=
3 0.8k + (1 − k )(0.1) (1)  5  3  2
5 2 2 2  2  ×  2  ×  2  = 20
     
k = 0.2  5   3  2 
p% = 20 % 6 1 3 2  1  ×  3 ×  2  = 5
p = 20      
P ( H ′ ∩ D′) (1 − k )(0.9) Total 175
(ii) P(H’/D’) = = = 0.947
P ( D′) 0.2k + (1 − k )(0.9) (iii) Excluding the green balls, we only have
1, 1, 2, 2, 3. Since we are ignoring the colours of
Examples of possible comments: the balls, we are forming 3-digit numbers from the
(1) If it does not find you hypertensive then you can be reasonably confident 5 digits 1, 1, 2, 2, 3.
that your blood pressure is normal. Case 1: All 3 digits are distinct.
(2) If it diagnoses hypertension, then you should consult your doctor for further The 3 digits are 1, 2, 3 and the number of ways of
tests. arranging them are 3!
(3) Any other logical comments with reference to the context of the question Case 2: 2 digits are identical.
Step 1: Choose 2 digits that are identical
14 2 (1, 1 or 2, 2): 2
(i) Case 1: 2 red balls -   = 1
2 Step 2: Choose a digit from the remaining digits
(1, 3 or 2, 3): 2
 3
Case 2: 2 blue balls -   = 3 Step 3: Arrange the 3 chosen digits in a row:
 2 3!
=3
5 2!
Case 3: 2 green balls -   = 10
2  3! 
No. of ways = 3!+   ( 2 )( 2 ) = 6 + 12 = 18
No. of ways = 1 + 3 + 10 = 14  2! 

(ii) Case 1: No red ball.


Choose 6 balls from a total of 8 (blue and green)
8
balls:   .
 6
Case 2: No blue ball.
Choose 6 balls rom a total of 7 (red and green)

7 8
Anderson Junior College
Preliminary Examination 2010
H2 Mathematics Paper 1 (9740/01)

1 Without using a graphic calculator, find the exact solution of the inequality
12 x + 29
≤ 4, x ≠ ± 5 .
5 − x2
x(12 + 29 x)
Hence solve the inequality ≤4 . [7]
5x2 −1

 x π
 tan 2 for 0 < x ≤ ,
2
2 It is given that f(x) = 
 2x π
for < x ≤ π.
 π 2

(i) Sketch the graph of y = f(x) for 0 < x ≤ π . [2]


(ii) Find the exact volume of revolution when the region bounded by the curve in
(i), the line y = 2 and the y - axis is rotated completely about the x-axis. [5]

dy
3 Given that ln y = sin −1 2 x , show that 1 − 4x 2 = 2 y . [1]
dx
By repeated differentiation of this result,
(i) Find the series expansion of y in ascending powers of x, up to and including
the term in x 3 . [4]
π
(ii) Hence deduce the approximate value of e 3 , giving your answer in the form of
1
8
( )
a + b 3 , a, b ∈ Ζ where a and b are to be determined. [2]

1
4 The curve C is given by the equation y = x x where x > 0. The variable point P
moves along the x−axis from x = 10 to x = 1 at a constant rate of 1 unit per second.
Another variable point Q moves along the curve C so that the x−coordinates of both
P and Q are the same at any point in time.
dy
(i) Find in terms of x. [3]
dx
(ii) The area of triangle OPQ is represented by A. Show that the rate of change of
1 3
A is ln 2 − unit2/s when t = 6. [4]
2 2

Page 1 of 4
AJC / 2010 Preliminary Examination / 9740 / P1
d   x  1
5 Show that  tan    = . [2]
dx   2   1 + cos x
x + sin x
Hence, or otherwise, show that ∫ = dx x f ( x) + C , where f (x) is a single
1 + cos x
trigonometric function to be determined and C is an arbitrary constant. [5]

π
6 The function f is defined by < x ≤π .
f : x → 4 + sec x ,
2
(i) Show that the inverse function f -1 exists and find f -1 in similar form. [3]
(ii) On a single, clearly labelled diagram, sketch the graphs of f and f −1 . [2]
 1 
Another function g is defined by g : x → ln   , x>2 .
 x−2
Find the maximal domain of the function f for the composite function gf to be
defined. Find the corresponding range of gf . [3]

7 A sequence of real numbers u1 , u2 , u3 , ….. satisfies the recurrence relation


4u=n +1 a un − 2 , a ∈  for all positive integers n and u1 = 1 . Express un in terms
of a and n . [4]
Find the range of values of a for which the sequence converges. [1]
If the sequence converges, find the limit in terms of a . [1]

c
8 The curve G has equation a y = x + b + where x ≠ 1 and a, b and c are
x −1
constants with a > 0.
Given that the curve G has two stationary points P and Q,
(i) Find the range of all possible values of c. [2]
(ii) Find the coordinates of P and Q in terms of a, b and c. [2]

The point (0,1) lies on one of the asymptotes of G and the lines y = −1 and y = 5
are tangents to G. Find the values of a, b and c. [4]

9 The non-zero complex numbers z1 and z2 satisfy the equation


z2 − z1 z2 + z1 =
2 2
0
z
Find the complex number 2 , given that its imaginary part is positive. [3]
z1
(i) The points P and Q represent z1 and z2 respectively in an Argand diagram.
Describe, with reasons, the geometrical properties of triangle OPQ where O is
the origin. [3]
Find the values of n for which z1 + z2 =
n n
(ii) 0. [2]

Page 2 of 4
AJC / 2010 Preliminary Examination / 9740 / P1
10 (a) The diagram below shows the graph of y = f(x). The curve cuts the x-axis at
x = 0 , x = 3 and x = 5 . It has asymptotes x = 4 and y = −1 . There is a
5 
minimum at the point A(-2, -3) and a maximum at the point C  ,1 .
2 
y
y = f(x)

C ( 52 ,1)
×
x
B (0, 0) 3 4 5

-1

×
A (-2, -3)

Sketch the graph of =y 2 − f ( x) , indicating the asymptotes and corresponding


points for A, B and C clearly. [4]

(b) Describe a series of linear transformations to show how the graph of


 1 
y = ln  2 , 0 <x <3
 x − 6x + 9 
can be obtained from the graph of y = ln x , x > 0. [4]

11 Part of an isosceles triangle, formed by numbers following a particular pattern is


shown below:

1
1 3 5
5 7 9 11 13
21 23 25 27 29 31 33
73 75 77 79 81 83 85 87 89
…….

(i) State the middle term in the nth row. [1]


(ii) Find the first and last term of the nth row. [2]
(iii) Show that the sum of the terms in the nth row is (2n − 1)3n −1 . [2]
(iv) Find the minimum number of rows in the triangle for the number of terms in
the triangle to exceed 500. [3]

Page 3 of 4
AJC / 2010 Preliminary Examination / 9740 / P1
1
12(a) By using the substitution u = , or otherwise, show that
t
5
∞ t 1 u2
∫1 (1 + t 3 )3 ∫0 (1 + u 3 )3 du .
dt =

Hence evaluate the exact value of this integral. [4]

(b) The diagram below shows the part of the curve C with parametric equations
t t2
= x = , y , for t ≥ 1 . y
1+ t 3 1+ t 3

x
0 1
2

(i) C approaches to a point when t → ∞. Find the coordinates of this point. [1]
1
(ii) The region bounded by the curve, the x-axis and the line x = is denoted by R.
2
Using the result found in (a), find the exact area of R. [4]

13 The equations of a plane π1 and a line l are shown below:


 2
 
π1 : r ⋅  1  = 6
 −2 
 
x −1 y + 4
l: = = z −5
3 2
The point A has position vector 3i − j + 4k .
(i) Find the distance between point A and the plane π1 . [2]


(ii) B is another point such that AB =−5 j − 2k . Find the length of projection of


AB onto the plane π1 . [2]
(iii) Using your answers in (i) and (ii), find the area of triangle ABC, where C is
the reflection of A in the plane π1 . [2]
(iv) Find the equation of the plane π2 which contains the line l and the origin.
Hence, find the line of intersection between the planes, π1 and π2 . [4]

END OF PAPER

Page 4 of 4
AJC / 2010 Preliminary Examination / 9740 / P1
Anderson Junior College
Preliminary Examination 2010
H2 Mathematics Paper 2 (9740/02)

Section A: Pure Mathematics (40 marks)

9 + ax
1 Find the values of a and b if the expansion of in ascending powers of x up
1 + bx 2
35 2
to and including the term in x 2 is 3 + x + x . With these values of a and b, state
6
the range of values of x for which expansion is valid. [6]

x3 + 3x 2 + 2 x + 1
2 Express f ( x) = in partial fractions. [2]
x 2 + 3x + 2

Hence, or otherwise, show that

( N + N + 1) −
N
1 2 1
∑x =1
f (=
x)
2 N +2
. [3]

=
Using the sketch of y f ( x) where x > 0 shown below, or otherwise, show that
N N +1
∑ f ( x) < ∫
x =1
1
f ( x) dx .
y
[3]
y= f(x)

x
0

3 A complex number z satisfies z − a = a, a ∈  + .


1
(i) The point P represents the complex number w, where w = , in an Argand
z
diagram. Show that the locus of P is a straight line. [2]
(ii) Sketch both loci on the same diagram and show that the two loci do not
1
intersect if 0 < a < . [4]
2
1  1
(iii) For a = , find the range of values of arg  z −  , give your answer correct
2  a
°
to 0.1 . [3]
 1
State the limit of arg  z −  when a approaches zero. [1]
 a

Page 1 of 5
AJC / 2010 Preliminary Examination / 9740 / P2
 cos t 
 
4 Relative to the origin O , the points A and B have position vectors a =  sin t  and
 −1 
 
 cos 2t 
 
b =  − sin 2t  respectively, where t is a real parameter such that 0 ≤ t < π .
 12 
 
(i) Show that a ⋅ b = m + cos(nt ) where m and n are constants to be determined. [2]
(ii) Hence, find the exact value of t for which ∠AOB is a maximum. [3]
r
 
(iii) Another point C has position vector c =  s  where r and s are real constants.
0
 
π
Given that A, B and C are collinear when t = , find the values of r and s. [3]
2

5 An underground storm canal has a fixed capacity of 6000 m3 and is able to


discharge rainwater at a rate proportional to V , the volume of rainwater in the
storm canal.

On a particular stormy day, rainwater is flowing into the canal at a constant rate of
300 m3 per minute. The storm canal is initially empty. Let t be the time in
minutes for which the rainwater had been flowing into the storm canal,
300(1 − e − kt )
(i) show that V = , where k is a positive constant. [4]
k

A first alarm will be sounded at the control room when the volume of rainwater in
the storm canal reaches 4500 m3 and a second alarm will be sounded when the
storm canal is completely filled. Given that the first alarm was sounded 20 minutes
after the rainwater started flowing into the storm canal.

(ii) Find the time interval between the first and second alarm. (Assuming the
weather condition remains unchanged). [3]

(iii) Briefly discuss the validity of the model for large values of t . [1]

Page 2 of 5
AJC / 2010 Preliminary Examination / 9740 / P2
Section B: Statistics (60 marks)

6 Mary Lim has 7 cousins. In how many ways can she invite some or all of them to
her birthday party? [2]

At her birthday party, Mary sets up a round table of 8 seats with a different
welcome gift at each seat. If all her cousins turn up for the party and given that 4 of
them are from the Lee family, 2 are from the Yeo family and 1 is from the Tan
family, find the number of ways they can be seated with Mary for a meal at the
table if family members of the same surname are seated together but members of
the Lee and Yeo families are not adjacent to each other. [3]

After the meal, Mary and her cousins start to play a game. The game requires a
formation of 2 facilitators and 2 teams of 3 members each. Find the number of
possible formations if not all the members in each team have the same surname. [3]

7 Chemical X will react with chemical Y to form compound Z. A scientist at a


chemical plant wants to study the relation between chemicals X and Y by varying
the amount of chemical Y (in milligrams) placed in a reaction flask containing
50mg of chemical X. When chemical Y is completely used up, the amount of
chemical X (in milligram) left is recorded as shown in the table below.
Chemical Y used (y mg) 20 40 60 80 100
Chemical X left (x mg) 29.1 16.2 8.9 5.1 3.8
(a) (i) Find the equations of the least square regression lines of y on x and x on y. [2]
(ii) Using the appropriate regression line found in (i), estimate the least amount of
chemical Y used given that chemical X has completely used up. Comment on
the validity of your estimation. [2]

(b) (i) A group of scientists propose alternative models of the form w = a + by ,


where w is a function of x, to describe the relation between chemicals X and
Y.
Model A: w = x 2
1
Model B: w =
x
Model C: w = ln x

State, with a reason, which model is the most appropriate and find the
corresponding least square regression line by performing a suitable
transformation. [2]

(ii) Hence, estimate the change in w when y increases by 5. [1]


(iii) In the same experiment conducted earlier, the amount of compound Z formed,
z (in miligrams), is measured. From the data collected, the linear product
moment correlation coefficient between z and x is found to be − 0.9 and
the least square regression line of z on x is given by z = −1.5 x + 12 . Find
the least square regression line of x on z. [4]

Page 3 of 5
AJC / 2010 Preliminary Examination / 9740 / P2
8 In year 2009, the average length of time for cars parked at the Integrated Resort
Haven (IRH) was 12 hours. The facilities are upgraded in 2010 and the
management of IRH wants to find out if there is difference in the mean length of
time for cars parked at IRH.

Assuming that the resort and the car park are open at all times. The length of time,
x hours, for cars parked at IRH were recorded for 200 randomly selected cars on a
particular day and the following results were obtained:

∑ ( x=
− 12) 80 , ∑ (x =
− 12) 2
1425 .

(i) Denoting the population mean and variance of the parking times by µ and
σ 2 respectively, find unbiased estimates of µ and σ 2 . [2]
(ii) Given that an appropriate hypothesis test carried out could not provide
sufficient evidence to indicate a difference in the mean length of parking time,
find the range of values of the significance level of this test. [3]
(iii) State, with a reason, the range of the significance level of another test (without
carrying out the test) such that the sample could not provide sufficient
evidence that there is an increase in the mean length of parking time. [1]
(iv) The sample of 200 cars could also be obtained using systematic sampling.
Describe how this can be done. [2]

9 A multiple-choice question (MCQ) consists of 5 suggested answers, only one of


which is correct. For each of the questions set for a particular topic, there is a
probability of p that a student, Alice, knows the correct answer, and whenever she
knows the correct answer she selects it. If she does not know the correct answer, she
randomly selects one of the 5 suggested answers. The events K and C are defined
as follows:
K: Alice knows the correct answer.
C : Alice selects the correct answer.

(i) Find the probability, in terms of p, that Alice selects the correct answer. [2]
(ii) Describe what the event K’∩ C represents in the context of this question. [1]
1
(iii) Given that P(K’|C) = , find the value of p. [2]
16

Taking p to be 0.3 and Alice answers 1 MCQ daily from Monday to Friday.

(iv) Given that she answers 3 MCQ correctly, find the probability that this
happens in 3 consecutive days. [2]

(v) Alice scores 3 marks for each correct answer, but loses 1 mark for each
incorrect answer. Find the probability that Alice obtains a negative score for
the 5 MCQ she attempted. [2]

Page 4 of 5
AJC / 2010 Preliminary Examination / 9740 / P2
10 At a newly opened shop, the number of orders for herbal chicken soup received in a
randomly chosen 30-minute interval follows a Poisson distribution with mean 2.3.
The shop is opened for 8 hours daily, from 11 am to 7 pm. (Assume that the orders
received are independent.)
(i) Find the probability that there are at least 6, but less than 10 orders received in
a randomly chosen one-hour period. [2]
(ii) Find the probability that in 100 randomly chosen one-hour periods, the shop
receives an average of more than 5 orders in a one-hour period. [2]
(iii) The shop owner incurs a fixed operating cost of $250 per day. The cost price
and selling price of a bowl of herbal chicken soup are $8 and $20 respectively.
By using a suitable approximation, find the probability that he makes a profit
of at least 40% of his total cost incurred per day. [4]
(iv) After operating the shop for half a year, the shop owner wishes to assess if he
should continue with the business. He decides to observe the lunch time
crowd from 12 pm to 2 pm for 25 days selected at random. If there are less
than 14 days with more than 10 orders during the lunch period, he will close
down the shop. Comment on whether he should close down the shop. [3]
Explain whether the Poisson distribution is a good model for the number of orders
for herbal chicken soup in a day. [1]

11 Peter bought an ice-cream machine. The amount of time taken by the machine to
produce a large tub of ice-cream follows a normal distribution with mean µ
minutes and standard deviation σ minutes. It is found that there is a 88% chance
that the machine will take less than 60 minutes and a 70% chance that it will be
more than 50 minutes. The amount of time taken by the machine to produce a small
tub of ice-cream also follows a normal distribution with mean 20 minutes and
standard deviation 2 minutes. The amount of time taken by the machine to produce
a large tub of ice-cream and a small tub of ice-cream are independent of each other.

(i) Find µ and σ . [3]

(ii) Find the probability that the difference between the amount of time taken by
the machine to produce 5 large tubs of ice-cream and thrice the amount of
time taken to produce 3 small tubs of ice cream is more than 1 hour. [4]

(iii) After using the machine for a year, Peter decides to test the functionality of
the machine by using it to produce n large tubs of ice-cream (where n is large)
and observing the time taken for each production. He will consider buying a
new machine if there are more than 20 times that it takes at least 60 minutes to
produce a large tub of ice-cream. Using a suitable approximation, find the
greatest value of n such that the probability that he needs to consider buying a
new machine is less than 0.2. [5]

END OF PAPER
Page 5 of 5
AJC / 2010 Preliminary Examination / 9740 / P2
AJC H2 Maths _Prelim 2010_P1 (Solutions) 4 x 2 16 x 3 8x 3
∴ y = 1 + 2x + + + ... ∴ y ≈ 1 + 2 x + 2 x 2 +
1 12 x + 29 (12 x + 29) − 4(5 − x 2 ) 2! 3! 3
≤4 ⇒ ≤0
5 − x2 5 − x2 π
−1

⇒ ( 5 − x)( 5 + x)(2 x + 3) ≤ 0
2 Let e 3 = e sin 2x

π 2 3
 3   3  8 3 
⇒ x= -3/2, x ≤ − 5 or x ≥ 5 x= -3/2, x < − 5 or x > 5 ( x ≠ ± 5 ) π
3
= sin −1 2 x ⇒ x =
4
3
e 3 ≈ 1 + 2
4
 + 2   
1
(
  4  + 3  4  = 8 11 + 5 3 )
     
x(12 + 29 x) ∴ a = 11, b = 5
≤4
5x 2 − 1 4 1
1 1 (i) y = x1/ x
⇒ ln y = ln x
Replace x by 1/x, ⇒ x= -2/3, < − 5 or > 5 x
x x 1 dy 1 1
1 1 ⇒ = − 3/ 2 ln x + 3/ 2
⇒ x= -2/3, − < x < 0 or 0 < x < y dx 2x x
5 5
dy y  ln x  x1/ x  ln x 
1 1 ⇒ = 1 −  = 3/ 2 1 − 
⇒ x= -2/3, − <x< as x = 0 is a solution to the inequality dx x3/ 2  2  x  2 
5 5
1 dA 1 1 dy
2 (i) (ii) A = xy ⇒
= y+ x
2 dx 2 2 dx
dy 1
2 When t = 6, x = 4 so y = 2 and = (1 − ln 2 )
dx 4
dA 1 1 1  3 1
so = ( 2 ) + ( 4 )  (1 − ln 2 )  = − ln 2
dx 2 2 4  2 2
dx
Given that = −1 ,
dt
dA dA dx 3 1  1 3
0 we have = × =  − ln 2  ( −1) = ln 2 − units/sec
π x dt dx dt 2 2  2 2
π
2 5 d   x  1 x 1 1
tan    = sec2   = = .
π dx   2  2 2  x  1 + cos x
x π 2 2 cos 2  
(ii) Volume required = π ( 2)2 (π ) − π ∫ 2 tan 2 dx − π ∫π x dx 2
2 2 π
0
Using integration by parts,
π
 x 2 π 7 
= 2π − π  2 tan − x  −  x 2  π
2
=  π 2 − 2π  units3  x + sin x   x x
−1
 2 0 2 4  ∫  1 + cos x  dx = ( x + sin x) tan  2  -∫ tan  2  (1 + cos x) dx
3 ln y = sin 2 x
x  x x
 1  dy 2 dy = ( x + sin x) tan   - ∫ tan   (2 cos 2  ) dx
 y  dx = ⇒ 1 − 4 x 2 = 2 y (shown) ------- (1) 2 2 2
  1 − 4x 2 dx
x  x
= x tan   + sin x tan   − ∫ sin x dx
dy  − 4 x  d2y  dy 
d2y
dx 2
1 − 4 x 2 +  =2

dy

dx 2
( )
1 − 4 x 2 − 4 x  = 4 y ---- (2) 2 2
dx  1 − 4 x 2  dx  dx  x x x x
= = x tan   + 2sin   cos   tan   + cos x + A
d y 2 2 2 2
( )  dy 
3 2 2
d y d y dy
1 − 4 x 2 + 2 (− 8 x ) − 4  − 4 x 2  = 4
dx 3 dx  dx   dx  dx x 2 x 2 x
= x tan   + 2sin   + 1 − 2sin   + A
d3y  dy  2 2 2
dx 3
( d2y
)
1 − 4 x 2 + 2 (− 12 x ) − 8  = 0 ---------------- (3) x
dx  dx  = x tan   + C , where C = 1 + A
dy d2y d3y 2
When x = 0, y = 1 , = 2, 2
= 4, = 16
dx dx dx 3

Page 1 of 6 Page 2 of 6
6 Function f is a one-one function from (ii) From (i), the stationary points of C are at x = 1 ± c .
the sketch of y = f ( x) , hence f-1 exist.
1+ b − 2 c
Let y = 4 + sec x When x = 1 − c , ay = 1 − c + b − c ⇒ y=
a
1 y = f −1(x)
⇒ sec x = = y−4 1+ b + 2 c
cos x When x = 1 + c , ay = 1 + c + b + c ⇒ y =
1 a
−1
Hence f : x → cos ( ) , x ≤ 3.
-1
x+b
x−4 y = f (x)
(iii) The equations of the asymptotes of C are x = 1 and y = .
a
x+b
| | Given (0, 1) is a point on y = , we get a – b = 0 - - - (1)
1 0 π a
g : x → ln = − ln( x − 2) , x > 2 3 π
2 Given that the line y = k does not pass through any point on C only for values of k in the
x−2
y=x interval (0, 2), the min turning pt corresponds to y = 0 and the max turning pt corresponds to
For Rf to be a subset of Dg , range of f need to be
y = 2:
2π 1+ b − 2 c
restricted to (2,3] , therefore the maximal domain of f is ( ,π ] . y= = −1 ⇒ a + b – 2 c = –1
3 a
Corresponding range of gf is [0, ∞) 1+ b + 2 c
y= = 5 ⇒ 5a – b – 2 c = 1
7 a 1 a
4un +1 = aun − 2 ⇒ un +1 = un −
4 2 3 9
Hence a = 1, b = 1 and c = , i.e. c = .
aa 1 1 2 4
⇒ un +1 =  un −1 −  − 9 2
44 2 2 z  z   z  1 ± 1 − 4 1 ± 3i
z2 − z1 z2 + z1 = 0 ⇒  2  −  2  + 1 = 0 ⇒  2  = =
2 2
2
a 1 a  z
 1  1 z  z1  2 2
⇒ un +1 =   un −1 − 1 + 
4 2 4   z2  1 + 3i
2 Since imaginary part is positive, ⇒   =
a a 1  1 a   z1  2
⇒ un +1 =    un − 2 −  − 1 + 
4 4 2  2 4  z2 z 
(i) = (1/ 2) 2 + ( 3 / 2) 2 = 1 and arg  2  = tan −1 3 = π / 3
1 a a 
3 2
a z1  z1 
⇒ un +1 =   un − 2 − 1 + +  
4 2  4  4   |z2| = | z1| and arg (z2) = arg(z1) + π/3
( n −1) +1 n −1 Triangle OPQ form an equilateral triangle as OP = OQ and ∠POQ = 60o
1 a a a 
2
a
⇒ un +1 =   un − ( n −1) − 1 + +  + ... +    z 
n

4 2  4  4   4   (ii) z1 + z2 = 0 ⇒  2  = −1
n n

 n
  z1 
a
n 1(1 −    n
 i π3  nπ
a 1 4 
⇒ un +1 =   u1 − 
i (π + 2 k π )
e  = e ⇒ = π (1 + 2k ) ⇒ n = 3(1 + 2k ) for any integer k
4 2  1− a    3
 4  10

n n −1 a
6−a  a  2 6−a  a  2
⇒ un +1 =   − ∴ un =   −
4−a  4 4−a 4−a 4 4−a
a 2
If the sequence converges then < 1 ⇒ a < 4 . The sequence converges to − .
4 4−a
8 c
(i) ay = x + b +
x −1
dy 1  c 
⇒ = 1 − 2
 = 0 when (x – 1) = c.
dx a  ( x − 1) 2 
This equation has two real and distinct solutions only when c > 0. 10  1 
 = ln ( x − 3) = −2 ln(3 − x)
−2
b y = ln  2
 x − 6x + 9 
Page 3 of 6 Page 4 of 6
The graph can be obtained from y = ln x by 3  2
1. translate of 3 unit in the negative x direction y = ln( x + 3)    
 −1  ⋅  1 
2. reflect in the y-axis y = ln(− x + 3)  4   −2  6 −1− 8
=     −
6
3. scale by factor 2, parallel to the y-axis = − 2 = 3 unit
4. reflect in the x-axis 22 + 12 + 22 22 + 12 + 2 2 3
11 ) i) For the nth row, middle term = 3n−1
ii) For the nth row, first term = 3n −1 − 2(n − 1) , last term = 3n −1 + 2(n − 1)
iii) Sum of terms in nth row  2  2
 0
=
2n − 1 n −1
( 3 − 2(n − 1) ) + ( 3n −1 + 2(n − 1) ) 
   ⌢
ii) AB =  −5  , n =
1  1  = 1 1 
2     3 
 −2  ɶ
 
2 + 1 + (−2) 2
2 2
 −2   −6 
=
2n − 1 n −1
(2)(3 ) = (2n − 1)3 n −1    
2 length of projection
n
 0  2
iv) No. of terms in each nth row row = (2n-1) Therefore, total no. of terms in n rows = ∑ (2r − 1) 
= AB × nˆ =  −5  ×  1 
1
r =1
n ɶ 3    Alternatively,
∑ (2r − 1) > 500  −2   −2  
r =1
use AB inˆ ,
 12   6 ɶ
  2  2 2 followed by
n
1 2
2∑ r − n > 500 =   3  −2  = 3 6 + 2 + 5 = 3 65 units
− 4 = 2 2
Pythagoras thm
r =1 3  10   5
   
 n(n + 1) 
2 − n > 500 ⇒ n 2 − 500 > 0
 2  1 2
Min no. of rows = 23 iii) Area of triangle ABC = ( AC )( 65)
2 3
1 2
12 ∞ t5 1
u2 = (3 × 2)( 65) = 2 65 units 2
∫1
1
(i) dt = 0 u5  1  = ∫0 (u 3 + 1)3 du (shown) 2 3
(1 + t 3 )3 ∫1 1  2
− du
1  3 1  3
(1 + 3 )3  u         
u iv) l : r =  −4  + λ  2  , λ ∈ℜ , Two vectors // to π2 are  −4  and  2,
1 3 −2 1 5 1  5 1
3u (1 + u 3 ) −3 du =  (1 + u ) 
1 1 2 1 1 1
= ∫
3 0 3 −2
= − + =
24 6 8
   
 1   3   −14   −1
   
 0
       
normal to π2 is // to  −4  ×  2  =  14  = 14  1 
(ii) As t → ∞ , x, y → 0. The point is (0, 0)  5   1   14   
1
      1
1 t2 (1 − 2t 3 ) ∞ 2t 5 − t 2  −1
(iii) Area required =
∫0
2 y dx = ∫
∞ 1+ t3

(1 + t 3 ) 2
dt =
∫1 (1 + t 3 )3
dt
 
Equation of π2 : r ⋅  1  = 0 , ie. − x + y + z = 0
∞ 2t 5 ∞ t2 1
=
∫1 3 3
(1 + t )
dt − ∫
1 (1 + t 3 )3
dt  
To find line of intersection:
1 ∞ −x + y + z = 0
= 2( ) − 1 3t 2 (1 + t 3 ) −3 dt
8 3 ∫1 2 x + y − 2z = 6

3 −2  −1 1 1 0   1 0 −1 2 
= 1 − 1  (1 + t )  = 1 + 1  0 − 1  = 5 units
2
The augmented matrix, M is   , RREF (M) =  
     2 1 −2 6  0 1 0 2
4 3  −2 1 4 6 4  24
 x   2 + z   2 1  2 1
13 i) distance between point A and the plane π1 x−z =2            
 y  =  2  =  2  +  0  z ie. Equation of line: r =  2  + µ  0  , µ ∈ℜ
y=2  z   z  0 1 0 1
           
[Alternatively, Cartesian equation of line: x − 2 = z , y = 2 ]

Page 5 of 6 Page 6 of 6
AJC H2 Maths _Prelim 2010_P2 (Solutions) AP a 0.5 1
1 sin θ = = = =
 1 1  AB 1 − a 2 − 0.5 3 P
9 + ax  −  2 
1
2 2 a  a
= ( 9 + ax ) (1 + bx ) = 3 1 + x +  x  + ...  (1 − bx + ...)
2 −1 a
2  2

1 + bx 2  18 2!  9   ∴θ = 19.5 o
A θ B
   1  1 2
  Hence 160.5o ≤ arg  z −  ≤ 180o or −180o < arg  z −  ≤ −160.5o
1/2

 a2 2   2
  a  a
x + ...  (1 − bx 2 + ...)
a a a
= 3 1 + x − ≈ 3+ x +− − 3b  x 2
 18 648  6  216   1
As a approaches 0, arg  z −  → 180o
By comparing coefficients, a = 6 and b = -2  a
9 + ax 1 4i  cos t   cos 2t 
The valid range for expansion of is x < .     1 1 1
1 + bx 2 2 a ⋅ b =  sin t  ⋅  − sin 2t  = cos t cos 2t − sin t sin 2t − = cos(t + 2t ) − = cos 3t −
ɶ ɶ     2 2 2
2 x3 + 3x 2 + 2 x + 1 1 1  −1   1 2 
f ( x) = = x+ −
x 2 + 3x + 2 x +1 x + 2 1
cos 3t −
a ⋅b 2 2 1
4ii cos ∠AOB = = =  cos 3t − 
x + 3x + 2 x + 1 N 
( )
3 2
N N
1 1  a b 2 5 2

x =1
f ( x) = ∑
x =1 x 2 + 3x + 2
= ∑ x +
x =1 
− 
x +1 x + 2 
cos 2 t + sin 2 t + 1 cos 2 2t + sin 2 2t + 1
2
N
 1 1  For maximum ∠AOB , since 0 ≤ ∠AOB ≤ π and cos θ is a decreasing function over [ 0, π] , we aim
= ∑ x + − 
x =1  x +1 x + 2  2 1
to minimize cos ∠AOB , ie.  cos 3t −  .
N N
 1 1  5 2
= ∑ x + ∑ − 
x =1  x + 1 x+2 π
x =1
Thus, cos 3t = −1 , ie. t = (since 0 ≤ t < π ).
N 1 1 3
= ( N + 1) + −
2 2 N +2 0  −1   −1   0   −1  r  0   r 
4iii When t = π , a =  1  , b =  0  AB =  0  −  1  =  −1  and AC =  s  −  1  =  s − 1
N
 
y= f(x)
∑ f ( x) = sum of N rectangles under the curve. y 2  
 −1 

 
      
1 2   −1  3 / 2 
    
 0   −1   1 

x =1   1 2             
N +1
 −1   r 
∫ f ( x) dx = sum of the area under the curve  
1 Since A, B and C are collinear, AB = k AC ⇒  −1  = k  s − 1
As shown in the diagram, as the curve is concave 3/ 2  1 
upwards,    
Area of N rectangles < area bounded by the curve Looking at the z component, k= 3/2, so r = -2/3 and s = 1/3
N+1
x
N N +1 0 1 2 3 N
∴ ∑ f ( x) < ∫ f ( x) dx 5i dV 1
x =1 = 300 − kV ⇒ ∫ dV = ∫ 1 dt
1
dt 300 − kV
1
3 1 ⇒ − ln 300 − kV = t + C
Let w = , k
z
⇒ 300 − kV = e− k ( t +C )
1 1 − aw 1
|z – a | = a ⇒ −a = a⇒ = a ⇒ w− = w ⇒ 300 − kV = Ae − kt
w w a
300(1 − e − kt )
The locus of w is a straight line (the perpendicular bisector of w1 =
1
& w2=0 When t = 0 , V = 0 ⇒ 300 − 0 = Ae0 ⇒ A = 300 ⇒V = (Shown).
a k
1 1 300(1 − e −20 k )
If 0 < a < ⇒ 0 < 2a < 1 and >1 When t = 20 , V = 4500 , ⇒ 4500 = ⇒ 15k = (1 − e −20 k )
2 2a k
1 1 1 From the GC, k = 0.030293
Hence 2a < 2a a 300(1 − e −0.030293t )
2a 2a 2nd alarm : when V = 6000 ∴ 6000 = ⇒ t = 30.7
Therefore the 2 loci do not intersect. a 0.030293
The residents will have 10.7 minutes between the 1st and 2nd alarm.
1
a= 300
2 t → ∞ ⇒V → = 9903 m3 which is impossible as the canal has only a fixed volume of
0.030293
6000 m3 . The model is not valid for large values of t.
Page 1 of 5 Page 2 of 5
6 No of ways to invite her guests = (2)7 – 1 = 127 (iv) The management must have a sampling frame (the list of all cars parked). If there are N cars,
choose a random number, k, from 1 to N/200 (take the nearest integer value), then select
No of ways to arrange a round table = 2! X 4! X 2! X 8 = 768 every (N/200)th car until a sample of 200 is obtained.

No of ways to select 2 facilitators and 2 teams (Without any restrictions) 9 1


= 8C2 x (6C3 x 3C3) ÷ 2! = 280 (i) P(C) = p ×1 + (1 − p ) ×
5
No of ways to select the 2 facilitators and 2 teams with Lee family forming 1 team (ii) K’∩C represents the event where Alice does not know the correct answer but she
= 4C3 x 5C2 x 3C3 = 40 answers correctly.
1
No. of possible formations = 280 – 40 = 240 P( K '∩ C ) 1 (1 − p ) × 1
(iii) P(K’|C) = = 5 = solving, get p =0.75.
P(C ) 16 1 16
7 (a)(i) For x on y, x = −0.3085 y + 31.13 ⇒ x = −0.309 y + 31.1 (3 s.f.) p × 1 + (1 − p ) ×
5
For y on x, y = −2.8526 x + +95.999 ⇒ y = −2.85 x + 96.0 (3s.f.)
When p = 0.3, P(C) = 0.3+0.7×0.2=0.44
(ii) Since chemical Y is the controlled variable, use regression line of x on y.
(0.44)3 (0.56) 2 × 3 3
0 = −0.3085 y + 31.13 ⇒ y = 100.91 P(3 consecutive correct | 3 correct answers) = =
The estimation is not valid as this is an extrapolation, linear relation may not hold outside the (0.44)3 (0.56) 2 × C35 10
range of data.
P(negative score) = P(0 correct or 1 correct) = (0.56)5 + C15 (0.56)4 (0.44) = 0.271
1(b)(i) By comparing the linear product moment correlation for the 3 models, Model C is the
most appropriate with the highest value of r = 0.993 as it best describes the data given. 10 Let X be the r.v. denoting the number of orders for herbal chicken soup in 30 min.
X ~ Poi (2.3)
Using linear transformation w = ln x , Let Y be the r.v. denoting the number of orders for herbal chicken soup in 1 hour.
Regression line of w on y is w = −0.026136 y + 3.8294 ⇒ w = −0.0261 y + 3.83 (3 s.f.) Y ~ Poi(2.3 × 2) i.e. Y ~ Poi(4.6)

(ii) Change in w = −0.026136(5) = −0.13068 ≈ −0.131 (3 s.f.) (i) P ( 6 ≤ Y < 10 ) = P (Y ≤ 9 ) − P (Y ≤ 5) = 0.29471 ≈ 0.295(3.s. f )
w decreases by 0.131. (ii) Y ~ Poi (4.6) , Mean = variance = 4.6
4.6
(b)(iii) r = (− 0.9 ) = bd where b = −1.5
2 2
Hence d =
(− 0.9)2 = −0.54
Since n = 100 is large, by CLT, Y ∼ N (4.6,
100
) approximately
− 1.5 P(Y > 5) = 0.031090 ≈ 0.0311(3.s. f )
Since ( x , z )lies on the reg line z = −1.5 x + 12 ,
(iii) Let T be the r.v. denoting the number of orders for herbal chicken soup in a day (8 hours).
z = −1.5 x + 12
T ~ P (2.3 × 16) i.e. T ~ P (36.8)
29.1 + 16.2 + 8.9 + 5.1 + 3.8 Since λ > 10 , T ~ N (36.8,36.8) approximately.
x= = 3.3220655
P ( 20T ≥ 1.4 ( 8T + 250 ) )
5
z = −1.5(3.3220655) + 12 = 7.0169
 350 
Hence, reg line of x on z is x − (3.3220655) = −0.54( z − 7.0169) = P ( 8.8T − 350 ≥ 0 ) = P  T ≥  = P(T ≥ 39.773)
 8.8 
∴ x = −0.54 z + 7.11 (3.s.f) = P(T≥40) = P(T>39.5) cc
= 0.328
8 80
(i) Unbiased estimate of µ is x = + 12 = 12.4
200 (iv) Let A be the r.v. denoting the number of orders for herbal chicken soup in 2 hours
200 1425 80 2  (lunch time)
Unbiased estimate of σ 2 is s2 = −( ) =7
199  200 200  A ~ P(2.3 × 4) i.e. A ~ P(9.2)
(ii) H0 : µ = 12 Let H be the r.v. denoting the number of days with more than 10 orders during lunch period
X − 12 from 12 to 2pm.
H1 : µ ≠ 12 Test Statistic : Z = ~ N(0,1) under H0 by CLT P ( A > 10 ) = 1 − P ( A ≤ 10 ) = 1 − 0.68202 = 0.31798
s / 200
Using GC, p -value = 0.0325 Given that H0 is not rejected, α < 3.25 H ~ B ( 25, P ( A > 10 ) ) i.e. H ~ B ( 25, 0.31798)
(iii) Since we will be using a one-tailed test in stead of a two-tailed test, P ( H < 14) = P ( H ≤ 13) = 0.98947 = 0.989 (3.s.f)
1
α < (3.250944) = 1.625472 ⇒ α < 1.63 Since the probability of having less than 14 days with more than 10 orders during the lunch
2 period is quite high, he should close down his business.
Page 3 of 5 Page 4 of 5
(iv) The use of the Poisson model is not suitable in this context as the number of orders during
lunch and dinner period will likely be higher than the rest of the hours in a day, thus it is
unlikely that the mean number of orders is the same for each 30 minute period.
11 (i) Let A be the r.v. denoting the amount of time taken by the machine to produce a large tub of
ice- cream. A ∼ N ( µ , σ 2 )
P( A < 60) = 0.88 P( A > 50) = 0.70
 60 − µ   50 − µ 
PZ < = 0.88 PZ > = 0.70
 σ   σ 
60 − µ 50 − µ
= 1.17499 = −0.52440
σ σ
µ = 60 − 1.17499σ − − − −(1) µ = 50 + 0.52440σ − − − −(2)

Equating (1) and (2), ∴σ = 5.8845 = 5.88 (3.s.f) and µ = 53.08583 = 53.1 (3.s.f)

(ii) Let B be the r.v. denoting the amount of time taken by the machine to produce a small tub of
ice- cream. B ∼ N (20, 22 )
(
Required prob = P A1 + A2 + A3 + A4 + A5 − 3 ( B1 + B2 + B3 ) > 60 )
Let T = A1 + A2 + A3 + A4 + A5 − 3 ( B1 + B2 + B3 ) .
T ∼ N ( 85.42915, 281.1367 )
( )
P A1 + A2 + A3 + A4 + A5 − 3 ( B1 + B2 + B3 ) > 60 = P ( T > 60 )
= 1 − P ( −60 < T < 60 ) = 0.935317 = 0.935 (3.s.f)

(iv) Let C be the number of times the machine takes at least 60 minutes to produce a large
tub of ice-cream.
C ∼ Bin ( n, P ( A ≥ 60 ) ) i.e. C ∼ Bin ( n,1 − 0.88 )
Since n is large, np > 5 and n(1-p) > 5, C ∼ N ( 0.12n, 0.1056n )
P ( C > 20 ) < 0.2
P ( C > 20 + 0.5 ) < 0.2 (cc)
 20.5 − 0.12n 
PZ >  < 0.2
 0.1056n 
Using GC, P ( Z > 0.84162 ) = 0.2
20.5 − 0.12n
Thus, > 0.84162 ⇒ 0.12n + 0.2735 n − 20.5 < 0
0.1056n
Solving -14.26 < n < 11.98 greatest n = 143.

Page 5 of 5
CATHOLIC JUNIOR COLLEGE
General Certificate of Education Advanced Level
Higher 2

MATHEMATICS 9740/01
Paper 1 31 August 2010
3 hours
Additional Materials: Answer Paper
Graph Paper
List of Formulae (MF 15)

READ THESE INSTRUCTIONS FIRST


Write your name and class on all the work you hand in.
Write in dark blue or black pen on both sides of the paper.
You may use a soft pencil for any diagrams or graphs.
Do not use paper clips, highlighters, glue or correction fluid.
Answer all the questions.
Give non-exact numerical answers correct to 3 significant figures, or 1 decimal place in the
case of angles in degrees, unless a different level of accuracy is specified in the question.
You are expected to use a graphic calculator.
Unsupported answers from a graphic calculator are allowed unless a question specifically
states otherwise.
Where unsupported answers from a graphic calculator are not allowed in a question, you are
required to present the mathematical steps using mathematical notations and not calculator
commands.
You are reminded of the need for clear presentation in your answers.

At the end of the examination, arrange your answers in NUMERICAL ORDER. Place this
cover sheet in front and fasten all your work securely together.
The number of marks is given in brackets [ ] at the end of each question or part question.

Name : __________________________________________ Class : ___________

Question No. Marks Question No. Marks

1 /4 7 /8

2 /7 8 / 10

3 /9 9 / 10

4 /8 10 / 15

5 /8 11 / 12

6 /9 TOTAL / 100
-------------------------------------------------------------------------------------------------------------------------
This document consists of 5 printed pages. [Turn Over]

1
1. Three sisters Audrey, Catherine and Gladys went on a shopping spree while on holiday.
They bought items for their sister Eliza, who could not go due to her ‘A’ level
examinations. After returning home, they could not recall the prices of the items they had
bought, but they had their credit card statements with the total amounts they each had
spent. The list of items that each sister had bought and the amount on their statements are
summarised in the table below:

Type of Item (Price) Catherine Audrey Gladys


High heels ($x per pair) 5 2 3
Facial Mask ($y per box) 10 7 15
Handbags ($z per piece) 3 8 5
Credit Card Statement ($) 1298.20 1158.30 1837.70

(i) Find the values of x, y and z. [3]


(ii) Hence evaluate the total cost of their gift to Eliza, which consists of 1 pair of high
heels, 5 boxes of facial mask and 2 handbags. [1]

2.  n + 1
A sequence is defined by u1 = 1 and u n +1 =  u n for n ≥ 1 .
 n2 
(i) Write down the values of u 2 , u3 , u 4 and u5 .
[2]
(ii) Hence make a conjecture for u n in terms of n. Prove your conjecture using
mathematical induction. [5]

3. In an East Asia ancient culture, there is a war monument which is formed by placing
concentric circular slabs of granite on top of each other. The slabs of granite are of
decreasing measurements.

(i) The first slab of the monument has diameter 200 cm and the diameter of each
subsequent slab is three-quarters the diameter of the previous slab. Show that the
total sum of the cross-sectional circular area of the slabs used will be less than
72 000 cm2, no matter how many slabs there are. [4]

(ii) The first slab of the monument has thickness 50 cm and each subsequent slab has
thickness d cm less than of that for the previous slab. Given that the maximum
possible number of slabs is 14, find the largest integer value of d. [2]

(iii) Given that d = 3, write down an expression in terms of n for the volume of the nth
slab and hence evaluate the total volume of the monument which consist of 14 slabs. [3]

[Turn Over]

2
4. The diagram below shows the graph of y = 2 ln x + 2 − x . The two roots of the equation
x − 2 ln x = 2 are denoted by α and β , where α < β .
y

x
α β

(i) Find the values of α and β , correct to 3 decimal places. [2]


A sequence of real numbers x1, x2, x3, … satisfies the recurrence relation
xn+1 = ln( xn ) 2 + 2 for n ≥ 1 .
(ii) Prove that if the sequence converges, it must converge to either α or β . [2]
(iii) By considering x n +1 − x n and the graph above, prove that
x n +1 > x n if α < x n < β ,
x n +1 < x n if x n < α or x n > β . [2]
(iv) Hence deduce the value that xn converges to for x1 = 2 , giving your answer correct to
3 decimal places. [2]

ax 2 + x
5. The curve C has equation y = where x ≠ 1 and a is a constant.
x −1
(i) Show that C does not have any stationary points when − 1 < a < 0 .
[3]
(ii) Sketch C when − 1 < a < 0 . Show on your diagram, the equations of the asymptotes
[5]
and the coordinates of any points of intersection with the axes.

[Turn Over]

3
y
6. (a)

x
−2 0 1

(−1, −3)

The diagram above shows the graph of y = f (x) . On separate diagrams, sketch
the graphs of
(i) y2 = f (x) [3]
(ii) y = f ′(x) [3]
showing clearly in each case the axial intercepts, the asymptotes and the coordinates
of the turning points, where possible.

(b) A graph with equation y = g(x) undergoes in succession, the following


transformations:
A: A reflection in the y-axis
B: A translation of 4 units in the direction of the positive x-axis
C: Scaling parallel to the y-axis by a factor of 2
The equation of the resulting curve is y = 2 x −3 . Determine the equation y = g(x). [3]

7. Relative to the origin O, the points A, B and C have position vectors


 1  2  − 1
     
 2  ,  1  and  2  respectively.
 1  3  3
     
The point P lies on the line AB produced such that AP:PB = 3: 1.

(i) Find the vector OP . [2]


0  2 
   
(ii) The line l has equation r =  1  + λ  − 1 , where λ ∈ ℜ .
 2  1 
   
Determine whether l and the line through A and B are intersecting. [3]

(iii) Find the shortest distance from C to AB. Hence or otherwise, find the area of triangle
ABC. [3]

[Turn Over]

4
8. The equation of a closed curve is (x + 2y)2 + 3(x − y)2 = 27.
(i) Show, by differentiation, that the gradient of the curve at the point (x, y) may be
dy y − 4x
expressed in the form dx =
7y − x [3]
(ii) The normal to the curve at the point (−2, 1) cuts the x-axis at P and the y-axis at Q.
Denoting the origin as O, find the area of the triangle OPQ. [4]
(iii) Find the equations of the tangents to the curve that are parallel to the y-axis. [3]

9. Let y = ln (1 − sin x).


y dy d2y dy2 −y
(i) Show that e dx = − cos x. Hence, show dx2 + dx = e − 1
  [3]
(ii) Find the Maclaurin’s series for y, up to and including the term in x3. [3]
cos x
(iii) Using the result in part (ii), find the Maclaurin’s expansion for up to and
sin x − 1
including the term in x2. [1]
cos x
(iv) Given that x is small, use binomial expansion to expand up to and
sin x − 1
including the term in x2. State the range of x for the expansion to be valid. [3]

10. (a) Find ∫ e 2 x sin x dx . [4]


(b) A curve C is defined by the parametric equations
x = 2(t − sin t ) , y = 2(1 − cos t ) for − π ≤ t ≤ π .
(i) Sketch the curve C. [2]
(ii) Find the exact area of the region bounded by C and the line y = 4. [6]
(c) The region R in the second quadrant is bounded by the y-axis, the line y = x + 5, and
the curve y = e x . Find the volume of the solid formed when R is rotated through 4
2

right angles about the y-axis. [3]

11. (a) (i) Solve the equation z 6 = −18 + (18 3 )i , expressing the solutions in the form

re iθ , where r > 0 and − π < θ ≤ π . [4]


(ii) Show all the roots on an Argand diagram and write down an equation of the
form x 2 + y 2 = k which contains all the roots. [3]

(b) The complex number z satisfies the relations



z − 3 + 2i ≤ − 2 3 + i and 0 < arg( z − 6 + 5i ) ≤ .
4
(i) Illustrate both of these relations on a single Argand diagram. [4]
(ii) Hence write down the exact area represented by the given relations. [1]

~End of Paper~

5
CATHOLIC JUNIOR COLLEGE
General Certificate of Education Advanced Level
Higher 2

MATHEMATICS 9740/02
Paper 2 15 September 2010
3 hours
Additional Materials: Answer Paper
Graph Paper
List of Formulae (MF 15)

READ THESE INSTRUCTIONS FIRST

Write your name and class on all the work you hand in.
Write in dark blue or black pen on both sides of the paper.
You may use a soft pencil for any diagrams or graphs.
Do not use paper clips, highlighters, glue or correction fluid.

Answer all the questions.


Give non-exact numerical answers correct to 3 significant figures, or 1 decimal place in the case of
angles in degrees, unless a different level of accuracy is specified in the question.
You are expected to use a graphic calculator.
Unsupported answers from a graphic calculator are allowed unless a question specifically states
otherwise.
Where unsupported answers from a graphic calculator are not allowed in a question, you are
required to present the mathematical steps using mathematical notations and not calculator
commands.
You are reminded of the need for clear presentation in your answers.

At the end of the examination, arrange your answers in NUMERICAL ORDER. Place this cover
sheet in front and fasten all your work securely together.
The number of marks is given in brackets [ ] at the end of each question or part question.

Name : ____________________________________________ Class : _______________

Question No. Marks Question No. Marks

1 /5 7 / 12

2 / 10 8 / 10

3 / 11 9 /6

4 / 14 10 /8

5 /7 11 /8

6 /9 TOTAL / 100
------------------------------------------------------------------------------------------------------------------------------------
This document consists of 6 printed pages. [Turn Over

Page 1
Section A: Pure Mathematics [40 marks]

1 Solve algebraically the inequality


2 x 2 + 4 x − 70
−1 ≥ 0 . [3]
x 2 + 4 x − 77
Hence, solve the inequality
2e 2 x + 4e x − 70
−1 ≥ 0 . [2]
e 2 x + 4e x − 77

2 The equations of the line l1 and the plane p1 are respectively,


1  − 1 1  2 
       
r =  0  + µ  1  and r = s1 + t  − 1 , where µ, s, t ∈ ℜ
 0 1 1  1 
       

(i) Find the acute angle between l1 and p1. [3]

α 
 
(ii) A second plane p2 has equation r ⋅  1  = 1 . Given that the two planes p1 and p2
β 
 
4 x − 15 5
intersect at the line l2: = y; z = , find the values of α and β. [3]
−2 2

(iii) The plane p3 with equation 2 x + by + z = 1 is parallel to l2. Find the value of b.
Hence find the distance between l2 and p3. [4]

3 The functions f and g are defined as follow:

2
f :x x+ , x ∈ ℜ \ {0}
x
g : x  ln( x − 3) , x>3

(i) Show that fg does not exist. [2]

Let the function h be defined as follows:

h : x  ln( x − 3) , for x > a

(ii) Find the least value of a such that fh exists. Define fh in similar form. [3]

(iii) Sketch the graph of f, indicating the turning points, asymptotes and axial intercepts, if
any. State the range of f and determine if f -1 exists. [4]

The function f has an inverse if its domain is restricted to x > k . Find the least value of k. [2]

[Turn Over

Page 2
4 (a) Verify that y = x is a particular solution of the differential equation
dy x 2 + y 2
= , x, y ≠ 0 [2]
dx 2 xy

(b) Show that the substitution y = ux reduces the differential equation


dy x 2 + y 2
=
dx 2 xy
to the differential equation
du 1 − u 2
x = . [3]
dx 2u
Hence find the general solution of the differential equation
dy x 2 + y 2
= . [4]
dx 2 xy

(c) Due to a rapid disease outbreak, the population of fish in a river, x (in thousands), is
believed to obey the differential equation
d2x
= 4ae − 2t
dt2
where t is the time in days, and a > 0 is a constant. Given that the entire population of
fish is wiped out by the disease eventually, show that the general solution of the
[3]
differential equation is x = ae −2t .
Explain the meaning of a, in the context of the question. Sketch the family of solution
curves of the differential equation for a = 1 and 2. [2]

Section B: Statistics [60 marks]

5 (a) Tourists visiting an amusement park in Singapore can purchase a day pass. Each day
pass allows one tourist to take any of the 6 designated rides only once.

(i) How many different ways can the order of the rides be chosen if a tourist goes on
all 6 rides? [1]

(ii) The attendants in the park keep track of the rides Amusement Park
each tourist has taken by marking on the ticket as  Crazy Monkey Swing
shown in the diagram on the right. Filling in a  Merry Go Bush
square indicates that the tourist has taken that  A Ride to Heaven
 Nightmare to Remember
particular ride. If a tourist can choose to go on a  Round about the Park
ride or not, how many different ways can the  Foodie Food
ticket be marked? [1]

(iii) If a tourist goes on at least one ride, find how many different selections he can
make? [1]

(b) From 10 students, including Vera and Daen, a group of 5 students from the
Mathematics Society will be selected to attend an Enrichment Camp. Daen will not
join the group without Vera, but Vera will join the group without Daen. In how many
ways can the group be formed? [4]

[Turn Over

Page 3
6 The fish-balls used by a popular noodle stall are supplied by two companies, supplier A
and supplier B. On average, 1 out of 50 fish-balls from supplier A is deformed and 3 out of
100 fish-balls from supplier B are deformed. It is known that the noodle stall gets a
proportion, denoted by p of their fish-balls from supplier A.

(i) Represent the situation using a probability tree. [2]

1
(ii) Given that p = , show that the probability that a randomly chosen fish-ball from the
3
2
noodle stall is deformed will be . [1]
75

(iii) For a general value of p, the probability that a fish-ball is supplied by B given that it is
3(1 − p )
deformed is denoted by f( p). Show that f( p) = . [3]
3− p
Prove by differentiation that f is a decreasing function for 0 ≤ p ≤ 1 , and explain what
this statement means in the context of the question. [3]

7 The occurrences of floods per year at a particular residential area in Singapore follow a
Poisson distribution with mean 4.

(i) Find the probability that in 4 months, this particular residential area is flooded at least
twice. [2]

(ii) A random sample of 12 periods of 4 months is taken. Find the probability that in at
most 5 of these 12 periods, this particular residential area is flooded at least twice. [2]

(iii) Using a suitable approximation, find the probability that in 5 years, this particular
residential area is flooded at least 11 times. [4]

(iv) In a long term study of the flooding problem in this particular residential area, it is
proposed that the residential area be observed for 40 years. The probability that in at
least n years out of these 40 years, this residential area is flooded at most thrice each
year is found to be less than 0.8. Using a suitable approximation, find the least value
of n. [4]

[Turn Over

Page 4
8 The masses of snapper fish and pomfret fish sold by a fishmonger are normally distributed
and independent of each other. The mean mass, standard deviation and selling price of
snapper fish and pomfret fish are given in the following table:

Snapper fish Pomfret fish


Mean mass in kg 1 0.6
Standard deviation in kg 0.1 0.05
Selling price per kg in $ 12 7

Find the probability that the

(i) total mass of 3 snapper fish and 2 pomfret fish is more than 4.5 kg [2]

(ii) mass of 3 snapper fish exceeds twice the mass of a pomfret fish by more than 1.85 kg. [2]

(iii) total selling price of a snapper fish and 2 pomfret fish is more than $21. [2]

A customer buys 15 fish, out of which n are snapper fish and the rest are pomfret fish. The
probability that the customer pays more than $150 is less than 0.7. Find the largest value of
n. [4]

9 CJC has 800 JC2 students in 2010 as follows:

Arts Science Total


Boys 75 320 395
Girls 145 260 405
Total 220 580 800

The Student Council (SC) wishes to conduct a survey on a sample of JC2 students to find
out their preferences for Graduation Night.

(a) Explain how stratified sampling can be used to select a sample of 30 students.
Suggest another set of strata that may be suitable for this survey. [3]

(b) The amount that a sample of 50 students are willing to pay for Graduation Night can
be summed up as follows:
∑ x = 4537, ∑ ( x − x ) 2 = 4825.62
Assume that the mean and variance for this sample are good estimators of the
population mean and variance. Another sample of 60 students is surveyed. Find the [3]
probability that the mean of this second sample lies between 90 and 100.

[Turn Over

Page 5
10 A manufacturer claims that the average length of its metal rods is 14 cm.
(i) A customer complains that the manufacturer understated the average length of its
metal rods. A random sample of 8 metal rods is taken and the length, x cm, of each
metal rod is summarised as follows:
∑ x = 113.40, ∑ x 2 = 1607.72
Test, at the 4% significance level, whether the customer’s complaint is valid. State
any assumptions made in carrying out the test. [4]
Explain, in the context of the question, the meaning of ‘at the 4% significance level’. [1]

(ii) Another random sample of 9 metal rods with standard deviation 0.200 cm is taken.
What range of values should the mean length of the sample be, in order for the
customer’s complaint to be not valid, at the 4% significance level? Give your answer
correct to 2 decimal places. [3]

11 The table below gives the proportion of people (in %) in an occupation earning more than
$5000 and the proportion of university graduates (in %) in that occupation.

Taxi
Occupation Teacher Chemist Accountant Lecturer Engineer Electrician Police Plumber
Driver
% of graduates,
97 87 75 84 52 36 22 10 8
x
% earning more
66 65 62 45 53 43 33 18 10
than $5000, y
A
(i) Calculate the product moment correlation coefficient between x and y. [1]

(ii) Draw a scatter diagram for the data. [1]

One of the values of y appears to be incorrect.

(iii) Indicate the corresponding point on your diagram by labelling it P and explain why
the scatter diagram for the remaining points may be consistent with a model of the
form y = a + b ln x . [2]

(iv) Omitting P, calculate the least squares estimates of a and b for the model
y = a + b ln x . [2]

(v) Assume that the value of x at P is correct. Estimate the value of y for this value of x. [1]

(vi) Comment on the use of the model in (iv) in predicting the value of y when x =100. [1]

~End of Paper~

Page 6
Reg.
Name Year 6C( )
No.

DUNMAN HIGH SCHOOL, SENIOR HIGH


PRELIMINARY EXAMINTION 2010
Higher 2

MATHEMATICS 9740/01
Paper 1 21 September 2010
3 hours
Additional Materials: Answer Paper
List of Formulae (MF15)

READ THESE INSTRUCTIONS FIRST

Write your Name, Class and Register Number on all the work you hand in.
Write in dark blue or black pen on both sides of the paper.
You may use a soft pencil for any diagrams or graphs.
Do not use paper clips, highlighters, glue or correction fluid.

Answer all the questions.


Give non-exact numerical answers correct to 3 significant figures, or 1 decimal place in the
case of angles in degrees, unless a different level of accuracy is specified in the question.
You are expected to use a graphic calculator.
Unsupported answers from a graphic calculator are allowed unless a question specifically
states otherwise.
Where unsupported answers from a graphic calculator are not allowed in a question, you are
required to present the mathematical steps using mathematical notations and not calculator
commands.
You are reminded of the need for clear presentation in your answers.

The number of marks is given in brackets [ ] at the end of each question or part question.

At the end of the examination, attach the question paper to the front of your answer
script.

The total number of marks for this paper is 100.

For teachers’ use:


Qn Q1 Q2 Q3 Q4 Q5 Q6 Q7 Q8 Q9 Q10 Q11 Total
Score
Max
4 6 6 8 8 9 9 10 12 14 14 100
Score

This question paper consists of 6 printed pages (including this cover page).
2

1− x
1 Expand in ascending powers of x up to and including the term in x 2 and state
2+ x
the range of values of x for which the expansion is valid. [4]

n
1 n
2 Prove by induction that ∑ (2r + 1)(2r + 3) = 3(2n + 3) .
r =1
[5]

1
Hence state the value of ∑ (2r + 1)(2r + 3) .
r =1
[1]

3 (i) Show that n 2 − 4n + 5 = (n − a )2 + b , where a and b are constants to be


determined. [1]

∑( )
N
(ii) Show that n 2 + 1 − n 2 − 4n + 5= N 2 + 1 + ( N − 1) 2 + 1 − 5 − 2. [3]
n =3

(iii) Without the use of a graphic calculator, deduce that the sum in (ii) is strictly less
than 2 N + 1. [2]

4
y
b
S
2
=
y x3 + 1

R
x
O

The diagram above shows the curve with equation = y x + 1. Given that the 3
(i)
two shaded areas R and S have the same value, find the value of b. [4]

(ii) Find the volume of the solid generated when S is rotated completely about the
x-axis. [4]

[Turn over

DHS 2010 Year 6 H2 Math Preliminary Examination


3

5 With respect to the origin O, the position vectors of three points A, B and C are given
→
 2   →
b →
 3
   
=
by OA = 3  , OB  7  = and OC  5  .
a  2 1 
     
(i) Find the values of a and b if A, B and C are collinear. [2]

→ 

(ii) If given instead that OA is perpendicular to OB , find a relationship between a
and b. Furthermore, if angle AOC is 60ο , find possible values for a and b,
giving your answers correct to the nearest integer.
A student claimed that since angle AOC is 60ο , angle BOC must be 30ο.
Without performing any calculation, state, with a reason, whether his claim is
necessarily true. [6]

et
6 (a) Find
∫ (1 + 3et ) 2
dt . [2]

( tan( x 2 ) ) . Hence find


∫ x sec ( x ) dx.
d 3 2 2
(b) Write down [4]
dx

4
(c) Evaluate
∫ 0
x 2 x − 3 dx without the use of the graphic calculator. [3]

7 (i) Explain whether the following statement is always true:


“If z =x + yi, x, y ∈  , is a root of the equation

z ) an z n + an −1 z n −1 + ⋅⋅⋅ + a1 z +=
P(= a0 0, n ∈  + , then z= x − yi is also a root.”
[1]

(ii) Solve the equation z 4 + 3 + i =0, expressing the roots in the form reiα , where
r > 0 and −π < α ≤ π . [5]
(iii) Show the roots on an Argand diagram. [1]
(iv) The points representing the roots in (iii) form a quadrilateral. Find the area of
this quadrilateral. [2]

[Turn over

DHS 2010 Year 6 H2 Math Preliminary Examination


4

8 The diagram shows a hexagon PQRSTU inscribed in a circle with radius 6 cm.
= UT
The sides QR and UT are parallel, and QR = 2 x cm .

Q 2x R

P • S
6 O 6

U 2x T

(i) Show that A, the area of the hexagon PQRSTU, is 2 ( 6 + x ) 36 − x 2 cm 2 . [3]

(ii) Using differentiation, find the value of x when A is maximum.


(You need not verify that it gives a maximum value.) [4]

Initially x = 6 cm, and the lengths of the parallel sides QR and UT are each decreasing
1
at a constant rate of cm s −1. Find the rate of change of A at the instant when
10
x = 2 cm. [3]
1
[ Area of a trapezium = × sum of the lengths of the parallel sides × height ]
2

9 (a) It is given that=


y ln(1 + e x ) .
d 2 y dy  dy 
(i) Show that = 1 −  . [3]
dx 2 dx  dx 
(ii) Find the Maclaurin’s series for y up to and including the term in x 2 . [2]

(iii) Verify that the same result is obtained if the standard series expansions for
e x and ln(1 + x) are used. [4]

(b) Given that x is sufficiently small for x3 and higher powers of x to be neglected,
and that 10 tan x − 3 = cos 2 x , form a quadratic equation in x and hence find the
value of x, leaving your answer in exact form. [3]

[Turn over

DHS 2010 Year 6 H2 Math Preliminary Examination


5

10 (a) The curve C has parametric equations


π
x eθ cos θ ,
= y sin θ + cos θ ,
= 0 ≤θ ≤ .
4
 2 π4 
(i) Show that the equation of the tangent at  e , 2  is given by
 2 
π
− 2
=y e 4x+
. [4]
2
(ii) Show that the area bounded by the curve C and the x-axis can be
π
expressed as ∫ 4
0
eθ cos 2θ dθ . Hence, evaluate the area, leaving your
answer correct to 2 decimal places. [4]

(b) The diagram shows the graph of y = f ( x ) , which has a turning point at
A ( −2, 2 ) .

y
x=2

A ( −2, 2 )

x
O

Sketch, on separate diagrams, the graphs of


(i) y = f '( x), [3]

1
(ii) y= , [3]
f(x)

showing clearly all relevant asymptotes, intercepts and turning point(s), where
possible.

[Turn over

DHS 2010 Year 6 H2 Math Preliminary Examination


6

11 (a) A geometric series has first term a. Find the range of values of a if the sum to
1
infinity of the series is . [3]
2
1
(b) The rth term of a series is given by =
Tr 2r
+ ln 32 r. Find the sum of the first N
3
terms. [4]

(c) Jon wants a gigantic cake prepared for his mother’s birthday. The cake is to
9
consist of 5 layers, where each layer has a square base with length that is of
10
the layer beneath it and a constant height of h units. The cost of the cake is
proportional to its volume and the largest layer costs $200.

(i) Given that the largest layer has length a units, find the cost of the whole
cake, rounded to the nearest dollar. [4]

(ii) Jon also wants candles to be placed at each layer such that the total number
of candles used is 75. If the largest layer has d2 candles and each
subsequent layer has d candles less than the one directly below, find the
number of candles placed at the top layer. [3]

END OF PAPER

DHS 2010 Year 6 H2 Math Preliminary Examination


Reg.
Name Year 6C( )
No.

DUNMAN HIGH SCHOOL, SENIOR HIGH


PRELIMINARY EXAMINTION 2010
Higher 2

MATHEMATICS 9740/02
Paper 2 23 September 2010
3 hours
Additional Materials: Answer Paper
List of Formulae (MF15)

READ THESE INSTRUCTIONS FIRST

Write your Name, Class and Register Number on all the work you hand in.
Write in dark blue or black pen on both sides of the paper.
You may use a soft pencil for any diagrams or graphs.
Do not use paper clips, highlighters, glue or correction fluid.

Answer all the questions.


Give non-exact numerical answers correct to 3 significant figures, or 1 decimal place in the
case of angles in degrees, unless a different level of accuracy is specified in the question.
You are expected to use a graphic calculator.
Unsupported answers from a graphic calculator are allowed unless a question specifically
states otherwise.
Where unsupported answers from a graphic calculator are not allowed in a question, you are
required to present the mathematical steps using mathematical notations and not calculator
commands.
You are reminded of the need for clear presentation in your answers.

The number of marks is given in brackets [ ] at the end of each question or part question.

At the end of the examination, attach the question paper to the front of your answer
script.

The total number of marks for this paper is 100.

For teachers’ use:


Qn Q1 Q2 Q3 Q4 Q5 Q6 Q7 Q8 Q9 Q10 Q11 Total
Score
Max
8 10 11 11 4 7 8 9 9 11 12 100
Score

This question paper consists of 7 printed pages (including this cover page).
2

Section A: Pure Mathematics [40 marks]

1 Show that the equation ( z − 1 + i)( z* − 1 − i) =2 represents a circle with centre C(1,–1)
and radius 2 and sketch the locus on an Argand diagram. On the same diagram,
sketch the locus represented by z + z* =
3. [3]

The points of intersection of these two loci are represented by A and B.

Find, in any order,

(i) the length AB, [1]


(ii) the complex numbers represented by A and B. [2]

State the cartesian equation of the perpendicular bisector of the line segment joining A
and B. Explain also why the perpendicular bisector of the line segment joining any
two distinct points on the circumference of the circle must pass through the centre of
the circle, C. [2]

2 For a > 0 , the functions f and g are defined as follows:

ax
f :x  , x ≠ −a 2 ,
x + a2
g : x  x2 − a2 , x∈ .

(i) Show that the inverse of f exists. [2]

(ii) Define f −1 in a similar form. [3]

(iii) Show that the composite function fg does not exist. [2]

(iv) Solve the equation f( x) = f −1( x) , expressing your answer(s) in terms of a. [3]

[Turn over

DHS 2010 Year 6 H2 Math Preliminary Examination


3

3 In an experiment, Andy and Bob attempt to devise a formula to describe how the
volume of water, V m3, in a tank, changes with time at t hours.

dV 1  8 
(i) Andy gives his formula as=  V − 2  . Given that V = 1 when t = 0,
dt 60  V 
t
3
=
show that V 8 − 7e 20 . Sketch this solution curve. [4]

d 2V
(ii) However, Bob believes that it is more likely to be = 12t 2 − 2. Given that
dt 2
dV
= 0 when t = 0 , show that the general solution for V can be expressed as
dt
1 1
V = (t 2 − ) 2 + C − , where C is a constant.
2 4
Hence, or otherwise, sketch on a single diagram, two distinct members of the
family of solution curves. [5]

(iii) It is also given that V = 1 when t = 0 in Bob’s model. Suppose that the water
in the tank does not overflow, explain, using your diagrams in parts (i) and
(ii), why Andy’s model is more appropriate compared to Bob’s model. [2]

4 The planes Π1 and Π2 are defined by

 2 1
   
Π1 : r  4 = 10, Π2 : r  3  = 8.
 
1 1
   

(i) Find the acute angle between the two planes. [3]
(ii) Obtain a vector equation of l1, the line of intersection of the two planes. [4]

x−2 7−z
The cartesian equation of another line, l2, is given by= = , y m,
6 3
where m is a real constant.
(iii) If the plane Π1 and line l2 intersect at the point (6, m, 5), find the value of m.
[2]
(iv) Show that the lines l1 and l2 are perpendicular for all values of m. [2]

[Turn over

DHS 2010 Year 6 H2 Math Preliminary Examination


4

Section B: Statistics [60 marks]

5 A group of students plan to collect data from students’ parents for a project.

(i) Describe how a quota sample of size 80 might be obtained based on the
parents’ educational qualifications. [2]

Subsequently, they decide to invite 80 parents to respond to an online survey. The


table below shows the profile of educational qualifications among the students’
parents:

“O” Level “A” Level University Total


Father 180 360 660 1200
Mother 300 420 480 1200
Total 480 780 1140 2400

(ii) Explain why stratified sampling is preferred over the method in (i). [1]
(iii) How many mothers with an “A” Level qualification should be included in the
stratified sample? [1]

6 To assess the level of work satisfaction in relation to the time spent at work, the
human resource department of an organisation polls nine of its officers. The number
of work hours per week (t) of each officer and the level of job satisfaction (x) are
recorded, where a higher value of x indicates a higher level of satisfaction. The data
are shown in the table below:

Work hours (per week), t 20.1 22.0 24.4 25.3 28.8 36.5 40.6 46.0 55.1
Satisfaction level, x 24.5 16.3 18.6 12.5 5.2 4.7 1.4 1.8 0.8

(i) Draw a scatter diagram for the data and find the product moment correlation
coefficient for the sample. [3]
b
(ii) A suggested model is of the form x= a + . Find a and b. Justify why this
t
model provides a better fit than a linear model between x and t. [2]
(iii) Use the model in (ii) to predict the satisfaction level when an officer works
5.0 hours per week. Comment on the reliability of your prediction. [2]

[Turn over

DHS 2010 Year 6 H2 Math Preliminary Examination


5

7 For every Monday of the week, the probability that Mylo wears a tie is 0.4. The
probability that he wears a jacket is 0.2. If he wears a jacket, the probability that he
wears a tie is 0.6.

Find the probability that, on a randomly selected Monday,


(i) Mylo wears a tie and a jacket, [2]
(ii) Mylo wears neither a tie nor a jacket. [2]

For Tuesday and Wednesday, the probability that Mylo wears a jacket is
• twice the probability he wears a jacket on the previous day if he wears a
jacket on the previous day,

• the same as the probability he wears a jacket on the previous day if he


does not wear a jacket on the previous day.

(iii) By constructing a tree diagram, find the probability that Mylo wears a jacket
on the third day given that he wears a jacket on exactly two of the three days.
[4]

8 A test consists of five Pure Mathematics questions, A, B, C, D and E, and six


Statistics questions, F, G, H, I, J and K.

(i) The examiner plans to arrange all eleven questions in a random order,
regardless of topic. Find the number of ways to arrange all eleven questions
such that
(a) the last question is a Pure Mathematics question, [2]
(b) a Pure Mathematics question must be separated from another with
exactly one Statistics question. [2]

(ii) Later, the examiner decides that the questions should be arranged in two
sections, Pure Mathematics followed by Statistics. Find the number of ways to
arrange all eleven questions such that
(a) question A is followed by question F, [2]
(b) questions B and K are separated by more than seven questions. [3]

[Turn over

DHS 2010 Year 6 H2 Math Preliminary Examination


6

9 A health food company claims that a breakthrough product that it launches,


Zenobrain, has the benefit of maintaining good mental health and will help its
consumers acquire a mean IQ score of not less than 115.
To verify the claim, a random sample of 12 consumers is taken and their IQ scores, x,
are recorded and summarised as follows:

∑ ( x − 100 ) =
50 and ∑ ( x − 100 ) =
2
4008 .

(i) Calculate unbiased estimates of the population mean and variance. [3]
(ii) State the null and alternative hypotheses and carry out an appropriate test at
5% level of significance. [4]
(iii) State, with a reason, whether
(a) any assumption is needed for the test in (ii) to be valid, [1]
(b) the conclusion would be the same if a two-tailed test is used with the
same level of significance. [1]

10 In a randomly chosen week, the numbers of unsolicited text messages and phone calls
received by a mobile line subscriber follow independent Poisson distributions with
means 5 and 3 respectively.

(i) Find the probability that the subscriber receives exactly 2 unsolicited text
messages in a day. [2]
(ii) Show the probability that the subscriber receives at most 10 unsolicited text
messages or phone calls in a week is 0.816. [2]

(iii) The subscriber decides to terminate his mobile line subscription if, in the next
10 weeks, there are more than 3 weeks where he receives more than 10
unsolicited text messages or phone calls in a week. Find the probability that he
terminates his mobile line subscription. [3]

(iv) Another subscriber intends to terminate her subscription if she receives a total
of 20 or more unsolicited text messages or phone calls in the next 2 weeks. By
using a suitable approximation, calculate the probability that she terminates
her subscription. [4]

[Turn over

DHS 2010 Year 6 H2 Math Preliminary Examination


7

11 (a) The distance, X km, covered by a school athlete during a regular training
session has mean 4 km. During competition season, training increases in
intensity and the distance, Y km, covered during the training session increases
to a mean of 6 km. Given that X and Y are independent normal distributions
with same variance σ 2 , and P(Y − X > 3) =0.4, find the probability that the
athlete covers a total distance between 8 km and 12 km in two randomly
chosen regular training sessions. [5]

(b) The amount of time that a student spends online each day has mean 120
minutes and standard deviation 45 minutes. A random sample of 60 students is
taken and they are surveyed on the amount of time that they spend online each
day. Find the probability that

(i) the total time spent online each day by the 60 students is at least 7000
minutes, [3]
(ii) the sample mean time spent online each day by the 60 students is
within 5 minutes of the population mean time of 120 minutes. [3]

Explain whether you need to assume that the amount of time spent online by a
student each day follows a normal distribution in your calculations above. [1]

END OF PAPER

DHS 2010 Year 6 H2 Math Preliminary Examination


Dunman High School
2010 Year 6 H2 Mathematics (9740) Preliminary Examination Paper 1
Suggested Solutions

Qn Suggested Solution
1 −1
1− x 1⎛ x⎞
= (1 − x) × ⎜1 + ⎟
2+ x 2⎝ 2⎠
⎛ 1 x ⎞⎛ x ⎛ x ⎞ ⎞
2

= ⎜ − ⎟ ⎜1 − + ⎜ ⎟ + ... ⎟
⎝ 2 2 ⎠ ⎜⎝ 2 ⎝ 2 ⎠ ⎟

1 1 1 1 1
= − x − x + x 2 + x 2 + ...
2 2 4 4 8
1 3 3 2
= − x + x + ...
2 4 8

x
Valid values of x: < 1 ⇒ −2 < x < 2
2

∑ (2r + 1)(2r + 3) = 3(2n + 3) for n ∈ Z


n
2 1 n +
Let Pn be the proposition .
r =1

When n = 1:
1

∑ (2r + 1)(2r + 3) = (2(1) + 1)(2(1) + 3) = 15 RHS = 3(2(1) + 3) = 15


1 1 1 1 1
LHS =
r =1

Since LHS = RHS, ∴ P1 is true.

Assume Pk is true for some k ∈ Z + ,

∑ (2r + 1)(2r + 3) = 3(2k + 3) ,


k
1 k
i.e.
r =1

to prove Pk +1 is true,
k +1
k +1
∑ (2r + 1)(2r + 3) = 3(2k + 5) .
1
i.e.
r =1

k +1

∑ (2r + 1)(2r + 3)
1
LHS =
r =1

∑ (2r + 1)(2r + 3) + (2k + 3)(2k + 5)


k
1 1
=
r =1

k 1
= +
3(2k + 3) (2k + 3)(2k + 5)

1
k (2k + 5) + 3
=
3(2k + 3)(2k + 5)
2k 2 + 5k + 3
=
3(2k + 3)(2k + 5)
(2k + 3)(k + 1)
=
3(2k + 3)(2k + 5)
k +1
= = RHS (shown)
3(2k + 5)

∴ Pk is true ⇒ Pk +1 is true

Since P1 is true, Pk is true ⇒ Pk +1 is true , by mathematical induction,


+
Pn is true for n ∈ .
n 1
=
3(2n + 3) ⎛ 3⎞
3⎜ 2 + ⎟
⎝ n⎠
1 1
∴ as n → ∞, →
⎛ 3 ⎞ 6
3⎜ 2 + ⎟
⎝ n ⎠

3(i) n2 − 4n + 5 = (n − 2)2 − 4 + 5
= (n − 2)2 + 1
3(ii) N
⎛ 2 2 ⎞
∑ ⎜ n + 1 − n − 4n + 5 ⎟
n =3 ⎝ ⎠
N
⎛ 2 2 ⎞
= ∑ ⎜ n + 1 − ( n − 2) + 1 ⎟
n =3 ⎝ ⎠

= 32 + 1 − 12 + 1

+ 42 + 1 − 22 + 1

+ 52 + 1 − 32 + 1
M M
M M

+ ( N − 2) 2 + 1 − ( N − 4) 2 + 1

+ ( N − 1) 2 + 1 − ( N − 3) 2 + 1

+ N 2 +1 − ( N − 2) 2 + 1

= N 2 + 1 + ( N − 1) 2 + 1 − 5 − 2

2
4(i) 0 0 3
Area of R = ∫ y dx = ∫ ⎡⎣ x3 + 1⎤⎦ dx =
−1 −1 4
1
y = 1 + x3 ⇒ x = ( y − 1) 3
1 b
3⎡ 4

( )
b b
Area of S = ∫ x dy = ∫ (y -1) 3 dy = ⎢ y − 1 3

2 2 4⎣ ⎦2
3⎡
( ) − 1⎤⎥
4
= ⎢ b − 1 3
4⎣ ⎦
Equating and solve for b:
3⎡ 4
⎤ 3
⎢ ( b − 1) 3 −1
⎥ =
4⎣ ⎦ 4
3
⇒ b = 1 + 2 4 = 2. 68 (3 s.f.)
4(ii) 1
For y =b , x = ( b − 1) 3 = 1. 1892 = k (say)

Volume required

⎡ 2 k

= π ⎢b k − 2 (1) − ∫1 ( x + 1) dx ⎥
2 3 2
⎣ ⎦

= 3. 53π (or 11.1) (unit cube)

5(i) If A, B and C are collinear, then


⎯⎯
→ ⎯⎯

AB = λ BC
⎛b − 2 ⎞ ⎛3− b ⎞
⎜ ⎟ ⎜ ⎟
⎜7 −3 ⎟ = λ ⎜5− 7⎟
⎜2− a⎟ ⎜1 − 2 ⎟
⎝ ⎠ ⎝ ⎠
i.e. λ = −2, a = 0, b = 4
5(ii) ⎯⎯
→ ⎯⎯

If OA is perpendicular to OB , then
⎯⎯
→ ⎯⎯

OA OB = 0
⎛2⎞ ⎛b⎞
⎜ ⎟⎜ ⎟
⎜3 ⎟ ⎜7⎟ = 0
⎜ a ⎟ ⎜ 2⎟
⎝ ⎠⎝ ⎠
i.e. 2b + 21 + 2a = 0

3
⎛ 2 ⎞ ⎛ 3⎞
⎜ ⎟⎜ ⎟
⎜ 3 ⎟ ⎜ 5⎟
⎜ a ⎟ ⎜1 ⎟
⎝ ⎠ ⎝ ⎠ = cos 60ο
13 + a 2 35
2(6 + 15 + a) = 13 + a 2 35
31a 2 − 168a − 1309 = 0
a = 10 (nearest int.) or a = −4 (nearest int.)
b = −20 (nearest int.) b = −6 (nearest int.)

His claim is not necessarily true since points O, A, B and C may not be coplanar.

6(a) et 1
∫ ∫ 3e (1 + 3e )
t −2
dt = t
dt
(1 + 3e )
t 2
3
(1 + 3et )−1 1
= +c = − +c
−3 3(1 + 3et )
1
∫ x sec ( x ) dx = 2 ∫ x ⎡⎣2 x sec ( x ) ⎤⎦ dx
6(b) 3 2 2 2 2 2

1
= ⎡ x tan ( x ) − 2 x tan ( x ) dx ⎤
( ) ∫
d
tan ( x 2 ) = 2 x sec 2 ( x 2 )
2 2 2

dx 2 ⎢⎣ ⎥⎦
1⎡ 2
= x tan ( x 2 ) − ln sec ( x 2 ) ⎤ + c
2⎣ ⎦
4


6(c)
x 2 x − 3 dx
0
3 4
=−
∫ 0
x 2 ( x − 3) dx +
∫ 3
x 2 ( x − 3) dx
3 4
⎡ x4 ⎤ ⎡ x4 ⎤
= − ⎢ − x3 ⎥ + ⎢ − x3 ⎥
⎣4 ⎦0 ⎣ 4 ⎦3
27
= or 13.5
2
7(i) No. The statement is not always true. It applies only for (polynomial) equation in z
with real coefficients.
7(ii) z 4 + 3+ i = 0 ⇒ z 4 = − 3 − i
⎛ 5π ⎞
− i⎜ ⎟
⇒ z = 2e
4 ⎝ 6 ⎠

1 1 5π 1 (12k −5)π
i ( − + 2 kπ ) i
z=2 4
e4 6 =2 e 4 24
, k = 0,1, 2,3

1 5π 1 7π 1 19π 1 17π
-i i i -i
∴z = 2 e 4 24 or 2 e
4 24 or 2 e 4 24 or 2 e 4 24

4
7(iii)   Im
Z2
Z3
L
Re
O
Z1
Z4 L

7(iv) The quadrilateral is a square.

Let the length of each side be L


1
Pythagoras Theorem: L = 2|z| =2(2 ) = 2 2
2 2 4 2

8(i) ON = 36 − x 2
⎡1 ⎤
A = 2 × ⎢ × (12 + 2 x ) 36 − x 2 ⎥
⎣2 ⎦
= 2 ( 6 + x ) 36 − x 2 2x
Q R
N
6 6

P 6 • 6 S
O

U 2x T

8(ii) dA ⎛ 1 ⎞⎛ 2x ⎞
= 2 36 − x 2 + 2 ( 6 + x ) ⎜ ⎟ ⎜ − ⎟
dx ⎝ 2 ⎠⎝ 36 − x 2 ⎠
72 − 2 x 2 − 12 x − 2 x 2
=
36 − x 2
4 (18 − 3x − x 2 )
=
36 − x 2
4 ( 6 + x )( 3 − x )
=
36 − x 2
dA
For maximum A , = 0 : x > 0 ⇒ x = 3 cm
dx
d d d 1 dx 1
( QR ) = ( 2 x ) = 2 ( x ) = − ⇒ =−
dt dt dt 10 dt 20
dA 4 ( 8 )(1)
When x = 2, = = 32 = 4 2
dx 32

5
dA dA dx
= ×
dt dx dt
⎛ 1 ⎞
= 4 2 ×⎜ − ⎟
⎝ 20 ⎠
2
=− cm2 s−1
5
2
A is decreasing at the rate of cm s −1.
5

9(a) y = ln(1 + e x )
(i)
⇒ e y = 1 + ex
d dy
: ey = ex
dx dx
dy
⇒ = e x− y
dx
d d2 y ⎛ dy ⎞
: 2
= e x− y ⎜1 − ⎟
dx dx ⎝ dx ⎠
2
d y dy ⎛ dy ⎞
⇒ 2 = ⎜ 1 − ⎟ (shown)
dx dx ⎝ dx ⎠
9(a) dy 1 d 2 y 1
(ii) When x = 0, y = ln 2, = , =
dx 2 dx 2 4
1 2
x
1
y = ln 2 + x + 4 + ...
2 2
1 1 2
= ln 2 + x + x + ...
2 8
9(a) ⎛ ⎛ x2 ⎞⎞
(iii) ln(1 + e x ) = ln ⎜ 1 + ⎜ 1 + x + + ... ⎟ ⎟
⎝ ⎝ 2 ⎠⎠
⎛ x2 ⎞
= ln ⎜ 2 + x + + ... ⎟
⎝ 2 ⎠
⎛ x x2 ⎞
= ln 2 + ln ⎜ 1 + + + ... ⎟
⎝ 2 4 ⎠
2
⎛ x x2 ⎞
⎜ + ⎟
⎛ x x2 ⎞ ⎝ 2 4 ⎠
= ln 2 + ⎜ + ⎟ − + ...
⎝2 4 ⎠ 2
x x2 ⎛ 1 ⎞ ⎛ x2 ⎞
= ln 2 + + − ⎜ ⎟ ⎜ ⎟ + ...
2 4 ⎝ 2 ⎠⎝ 4 ⎠
1 1
= ln 2 + x − x 2 + ... (verified)
2 8

6
9(b) 10 tan x − 3 = cos 2 x
( 2x)
2

10 x − 3 = 1 −
2
⇒ x + 5x − 2 = 0
2

−5 ± 52 − 4(−2) −5 + 33
∴x = = (rej -ve as x is small)
2 2
10(a) π
(i) x = eθ cos θ , y = sin θ + cos θ , 0 ≤θ ≤
4
dx dy
= eθ (cos θ − sin θ ), = cos θ − sin θ ,
dθ dθ
dy dy dx
= / = e-θ
dx dθ dθ
At (eθ cos θ ,sin θ + cos θ ), the equation of the tangent
is ( y − sin θ − cos θ ) = e-θ ( x − eθ cos θ ),
π
Set θ = ,
6
π π
π
3e 6 3 + 1 3 1 - 3e 6
at ( , ) , the equation of the tangent is ( y − − ) = e 6 (x − ),
2 2 2 2 2
π
− 1
y=e 6x+
2

10(a) Area under the curve C is


(ii) π
A = ∫ 4 (sin θ + cos θ ) eθ (cos θ − sin θ ) dθ
0
π
= ∫ 4 eθ (cos 2 θ − sin 2 θ ) dθ
0
π
= ∫ 4 eθ cos 2θ dθ ( shown)
0

= 0.68 (2 d.p.)
10(b)
(i) y

y = f '( x)

x
-2 O

x=2
7
10(b) y
1
(ii) y=
f(x )

A’(-2, 0.5)

2
x
O

11(a) a 1
=
1− r 2
2a = 1 − r
r = 1 − 2a
⇒ 1 − 2a < 1 ⇒ − 1 < 1 − 2a < 1
1
0 < a < 1, a ≠ (since r ≠ 0)
2
r
11(b) N N⎛1⎞ N
∑ Tr = ∑ ⎜ 2 ⎟ + ∑ 2r ln 3
r =1 r =1 ⎝ 3 ⎠ r =1

N
⎛1⎞
1− ⎜ ⎟
1
= × ⎝ ⎠ + N ( N + 1) ln 3
9
9 1
1−
9
1⎛ ⎛1⎞ ⎞
N

= ⎜ 1 − ⎜ ⎟ ⎟ + N ( N + 1) ln 3
8 ⎜⎝ ⎝ 9 ⎠ ⎟⎠
11(c) Volume of whole cake
(i) = a 2 h + (0.9a ) 2 h + (0.9 2 a ) 2 h + (0.93 a ) 2 h + (0.9 4 a ) 2 h
[1 − (0.9 2 )5 ] 2
= (1 + 0.9 2 + 0.9 4 + 0.96 + 0.98 )a 2 h = a h = 3.4280a 2 h
1 − 0.9 2
Cost of whole cake
= $3.4280 × 200 = $686 (nearest dollar)
11(c) 5
⎡⎣ 2(d 2 ) + (5 − 1)(−d ) ⎤⎦ = 75
(ii) 2
d 2 − 2d = 15
(d − 5)(d + 3) = 0
d =5 or d = −3 (rej. since d >0)

No. of candles at top layer = 52 + (5 − 1)(−5) = 5

8
Dunman High School
2010 Year 6 H2 Mathematics (9740)
Preliminary Examination Paper 2 Suggested Solutions

SECTION A

1 ( z − 1 + i)( z * − 1 − i) = 2
2
z −1+ i = 2
z −1+ i = 2
i.e. A circle with centre (1,–1) and radius 2. (shown)
Im

A
O Re
2
D
C(1,-1)

x=1.5

(i) AB = 2 AD
2
( 2) ⎛1⎞
2
       = 2 −⎜ ⎟
⎝2⎠
       = 7
(ii) Complex numbers represented are
3 ⎛ 7 ⎞ 3 ⎛ 7 ⎞
+ ⎜⎜ − 1⎟⎟ i and − ⎜⎜ + 1⎟⎟ i.
2 ⎝ 2 ⎠ 2 ⎝ 2 ⎠
Cartesian equation of the perpendicular bisector of the line segment joining A and B is y = − 1.

Since any two distinct points on the circumference of the circle are equidistance from the
centre C, hence perpendicular bisector of the line segment joining these points must pass
through C.

1
2(i)
y

y=a

O x
x = −a 2

Recommended
(1)From the graph above, any horizontal line y = b, b ∈ cuts the graph of f at most once,
therefore f is one-one and the inverse of f exists.

OR

(2)From the graph above, any horizontal line y = b, b ≠ a , cuts the graph of f exactly once,
therefore f is one-one and the inverse of f exists.
(ii) ax
y= , x ≠ −a 2 ,
x+a 2

xy + a 2 y = ax
a2 y
x= , y ≠ a,
a− y
a2 x a3
f −1 : x a , x ≠ a, or f −1 : x a − a 2 , x ≠ a,
a−x a−x
(iii) Rg = [− a 2 , ∞), Df = \{−a 2 },
∴ Rg ⊄ Df ,
thus fg does not exist

2
(iv) Method 1(Recommended)
f( x) = f −1( x)
⇒ f( x) = x
ax
⇒ =x
x + a2
⇒ x 2 + a 2 x − ax = 0
⇒ x( x + a 2 − a) = 0
⇒ x = 0 or x = a − a 2 .

Method 2 (not preferred)


f( x) = f −1( x)
ax a2 x
⇒ =
x + a2 a − x
⇒ ax(a − x) = a 2 x( x + a 2 )
⇒ ax[(a + 1) x + (a 3 − a )] = 0
⇒ ax[(a + 1) x + (a 3 − a )] = 0
a − a 3 −a (a 2 − 1)
⇒ x = 0 or x = = = − a (a − 1) = a − a 2 .
a +1 a +1

3(i) dV 1 8 1 V 3 −8
= (V − 2 ) = ( 2 )
dt 60 V 60 V
2
V dV 1
⇒ =
V − 8 dt 60
3

1 1
⇒ ln | V 3 − 8 |= t + C '
3 60
t
+ C ''
⇒ | V 3 − 8 |= e 20 , C '' = 3C '
t
⇒ V 3 − 8 = Ae , 20
A = ± eC ''

When t = 0, V = 1,
⇒ A = −7,
t
3
⇒ V = 8 − 7e 20
V
1

8 t
20ln( ) or 2.67
7

3
(ii) d 2V
2
= 12t 2 − 2
dt
dV
⇒ = 4t 3 − 2t + C1
dt
dV
When t = 0, = 0, ∴ C1 = 0
dt
dV
⇒ = 4t 3 − 2t
dt
⇒ V = t 4 − t 2 + C , C is a constant.

V = t4 − t2 + C
1 1
= (t 2 − ) 2 + (C − )
2 4
V 1
(I)C >
4
C
C (II)C ≤
1
4
t

(iii)
1
When t = 0, V = 1, then C2 = 1 > .
4
Therefore given the above initial condition, Bob’s model corresponds to solution curve type
(I) in part (ii).

Therefore in Bob’s model, the volume of water approaches infinity in the long run (not
realistic) whereas in Andy’s model, the volume of water reasonably diminishes to zero in the
long run/after some time.

Thus, Andy’s model is more appropriate than Bob’s model.

4(i) ⎛ 2 ⎞ ⎛1 ⎞
⎜ ⎟⎜ ⎟
⎜ 4 ⎟ .⎜ 3 ⎟
⎜1 ⎟ ⎜1 ⎟
n1 .n1 ⎝ ⎠⎝ ⎠ 15
cos θ = = =
| n1 || n 2 | 21 11 21 11
∴θ = 9.3o.

4
(ii) ⎛1 ⎞
⎜ ⎟
d = n1 × n 2 = ⎜ −1⎟
⎜2 ⎟
⎝ ⎠
Set z=0,
2 x + 4 y = 10
x + 3y = 8

⇒ x = −1, y = 3

⎛ −1⎞ ⎛1 ⎞
⎜ ⎟ ⎜ ⎟
∴ l1: r = a1 +α d1 = ⎜ 3 ⎟ + α ⎜ −1⎟ , α ∈ .
⎜0 ⎟ ⎜2 ⎟
⎝ ⎠ ⎝ ⎠
Alternative

2 x + 4 y + z = 10
x + 3y + z = 8
Let z = t ∈ ,
⇒ 2 x + 4 y = 10 − t
x + 3y = 8 − t
t t
⇒ x = −1 + , y = 3− ,
2 2

⎛ ⎞1 ⎛ ⎞
1
⎜ ⎟ ⎜ ⎟ t
∴ l1: r = ⎜ 3 ⎟ + α ⎜ −1⎟ , α= ∈
⎜0 ⎟ ⎜2 ⎟ 2
⎝ ⎠ ⎝ ⎠
(iii) Since the point with co-ordinates (6,m.5) lies on the first plane,
a d1 = D1
⎛6 ⎞ ⎛ 2⎞
⎜ ⎟⎜ ⎟
⇒ ⎜ m ⎟ ⎜ 4 ⎟ = 10
⎜ 5 ⎟ ⎜1 ⎟
⎝ ⎠⎝ ⎠
⇒ 12 + 4m + 5 = 10
7
⇒m=− .
4

(iv) ⎛2 ⎞ ⎛2 ⎞
⎜ ⎟ ⎜ ⎟
l2 : r = a 2 +β d 2 = ⎜ m ⎟ + β ⎜ 0 ⎟ , β∈ .
⎜7 ⎟ ⎜ −1⎟
⎝ ⎠ ⎝ ⎠
⎛1 ⎞ ⎛ 2 ⎞
⎜ ⎟⎜ ⎟
d1 d 2 = ⎜ −1⎟ ⎜ 0 ⎟ = 2 − 2 = 0 (independent of the value of m)
⎜ 2 ⎟ ⎜ −1 ⎟
⎝ ⎠⎝ ⎠

Therefore lines l1 and l2 are perpendicular for all real values of m.

5
SECTION B

5(i) To obtain a quota sample of size 80:


Identify and categorise the parents into mutually exclusive sub-groups according to
education levels. Set a quota, i.e. a target number of respondents for each group where the
total adds up to 80.
Poll respondents on a first-come-first-serve basis, say, when the parents arrive at school in the
morning with their children, until the number for each category is filled.
 
(ii) Stratified sampling is more representative in terms of the proportion of parents’ educational
qualifications in each category.

(iii) 420
× 80 = 14
2400

6(i)
x

From GC, r = −0.860


(ii) 260.56
From GC, regression line x = −37.612 +
t
ie, a = −37.6 , b = 261 ( 3 sig fig )
1
Suggested model between x and is a better fit with |r |= 0.930 > |r|= 0.860 for the linear
t
model between x and t.

260.56
(iii) x = −37.612 + = 78.9
5.0

t = 5 lies outside the data range of t , thus model may not be valid and estimate not likely to be
reliable.

6
7(i) Let J be the event where Mylo wears a jacket and T be the event where Mylo wears a tie.

P (T J ) = 0.6
P (T ∩ J )
= 0.6
P( J )
P (T ∩ J )
= 0.6
0.2
P (T ∩ J ) = 0.6 × 0.2 = 0.12
(ii) P (T ∪ J ) '
= 1 − P (T ∪ J )
= 1 − [ P(T ) + P ( J ) − P (T ∩ J ) ]
= 1 − ( 0.4 + 0.2 − 0.12 )
= 1 − 0.48
= 0.52
(iii)
0.8 J
J
0.4 0.2 J'

J 0.4 J
0.2 0.6
J'
0.6 J'
0.4 J
0.8 0.2 J
J' 0.6 J'
0.2 J
0.8
J'
0.8 J'
Mon Tue Wed

Let J n be the nth day where Mylo wears a jacket.


Required Probability
P ( J 3 ∩ J1 ) + P ( J 3 ∩ J 2 )
=
P ( J 3 ∩ J1 ) + P ( J 3 ∩ J 2 ) + P ( J1 ∩ J 2 )
(0.2)(0.6)(0.4) + (0.8)(0.2)(0.4)
=
(0.2)(0.6)(0.4) + (0.8)(0.2)(0.4) + (0.2)(0.4)(0.2)
0.048 + 0.064
=
0.048 + 0.064 + 0.016
0.112
=
0.128
= 0.875

7
8(i) Number of ways = 10!(5) = 18144000
(a)
(b)
S S S S S S

Number of ways or
= ( 6!)( 5!)( 3) = 259200 =(5!)6 ⋅ 5 ⋅ 4 ⋅ 3 ⋅ (3!) = 259200

(ii) Number of ways


(a) = ( 4!)( 5!) = 2880
(ii) Case One: 8 Questions (4M and 4S) between B and K
(b) B K
⎛5⎞
⎜ ⎟ ( 4!)
4!
⎝ 4⎠

Case Two: 8 Questions (3M and 5S) between B and K


B K
⎛ 4⎞ 5!
⎜ ⎟ ( 3!)
⎝ 3⎠

Case Three: 9 Questions between B and K


B K

4! 5!

Pure Mathematics Questions Statistics Questions

Number of ways
⎛ 4⎞ ⎛5⎞
= ⎜ ⎟ ( 3!)( 5!) + ⎜ ⎟ ( 4!)( 4!) + ( 5!)( 4!)
⎝ 3⎠ ⎝ 4⎠
= 2880 + 2880 + 2880
= 8640

8
9(i) Let u = x − 100
∴ ∑ u = 50 , ∑ u 2
= 4008

Unbiased estimate of population mean:


x = u + 100 = 4.16667 + 100 = 104.17 ≈ 104 (3 s.f .)

Unbiased estimate of population variance:


1⎛ 50 2 ⎞
s2 = ⎜ 4008 − ⎟ = 345.42 ≈ 345 (3 s.f .)
11 ⎝ 12 ⎠
(ii) To test H0: μ = 115
against H1: μ < 115

One-tail test at 5% level (α = 0.05)


Use t-test since σ2 is unknown and sample size of 12 is small
X − 115
under H0, T = ~ t (11).
345.42/12
From GC, p-value = 0.0342

Since p-value =0.0342< 0.05, there is sufficient evidence to reject H0 at the 5% level of
significance and conclude that the mean IQ score is less than 115, hence the manufacturer’s
claim is disputable.

(iii) (a) The IQ score of customers is normally distributed.

(b) For 2-tailed test, p-value =2(0.0324) =0.0684 > 0.05. H0 will not be rejected. The
conclusion would be different.
10 Let X be the number of unsolicited text messages received in a day.
(i) X ~ Po 5( ) 7
P( X = 2) = 0.125 (3 s.f.)
(ii) Let Y be the number of unsolicited text messages or phone calls received in a week.
Y ~ Po ( 8 )
P(Y ≤ 10) = 0.816 (3 s.f.) (shown)
(iii) Let W be the number of weeks where receives more than 10 unsolicited text messages or
phone calls in a week out of 10 weeks.
W ~ B (10, 0.184 )
P(W > 3) = 1 − P(W ≤ 3)
= 0.0944 (3 s.f.)
(iv) Let T be the total number of unsolicited text messages or phone calls received in the next 2
weeks.

9
T ~ Po (16 )
Since λ =16>10, ∴ T ~ N(16,16) approximately.

P(T ≥ 20) = P(T > 19.5) (apply c.c.)


= 0.191 (3 s.f.)
11 Y − X ~ N(2, 2σ 2 )
(a)
P(Y − X > 3) = 0.4
3− 2
P(Z > ) = 0.4

From GC,
1
= 0.25335

σ = 2.7910

Var( X1 + X 2 ) = 2 × 2.79102 = 3.94712


X1 + X 2 ~ N (8,3.94712 )
P(8 < X1 + X 2 < 12) = 0.345 (3 s.f.)
b(i) Let X min be the amount of time spent by a student online each day.

E( X1 + X 2 L + X 60 ) = 60 × 120 = 7200

( )
2
Var( X1 + X 2 L + X 60 ) = 60 × 452 = 90 15

Since n=60 is large, by Central Limit Theorem,



( )
2⎞
X1 + X 2 L + X 60 ~ N ⎜ 7200, 90 15 ⎟ approximately.
⎝ ⎠

P(X1 + X 2 L + X 60 ≥ 7000)
= 0.717 (3 s.f.)
(ii) Since n=60 is large, by Central Limit Theorem,
⎛ 452 ⎞
X ~ N ⎜ 120, ⎟ approximately.
⎜ 60 ⎟⎠

P( X − 120 < 5)
= P(−5 < X − 120 < 5)
= P(115 < X < 125)
= 0.611 (3 s.f.)
We do not need to assume that the amount of time spent online follows a normal distribution
since by the Central Limit Theorem, the sample mean follows a normal distribution
approximately when n is large.

10
1 Solve the inequality
x 4
≤ ,
x − 2 ( x − 2) 2

giving your answer in an exact form. [3]

ex 4
Hence solve ≤ x . [2]
e + 2 (e + 2
x 2
)

2 The sequence of numbers un , where n = 0, 1, 2, 3, …, is such that u0 = −2 and


(n + 2)un −1
un = .
2un −1 + n + 1

Proof by induction that, for n ≥ 0,


n+2
un = . [5]
2n − 1

3 The functions f and g are defined as follows:


f : x  (2 x − 1) − 2, x < −1 ,
2

 ln ( x + a ) ,
g:x  x > −1 .

(a) Define f −1 in a similar form. [3]

(b) State the value of a such that the range of g is (0, ∞) . [1]

(c) Show that the composite function gf exists, and find the range of gf, giving
your answer in terms of a. [2]

4 A curve is defined by the parametric equations


t t
x= , y= , where t ≠ −1, 1.
1+ t 2
1− t2

(i) Show that the tangent to the curve at any point with parameter t has equation
(1 − t )
2 3
(1 + t 2 ) x − 4t 3.
3
y= [3]

1
(ii) Find the gradient of the tangent to the curve at t = . Hence determine the
2
acute angle between this tangent and the line y= x + 3 . [3]

2 [Turn Over
5 Robert took a study loan of $100 000 from a bank on 1st January 2010. The bank
charges an annual interest rate of 10% on the outstanding loan at the end of each
year. After his graduation, Robert pays the bank $x at the beginning of each month.
The first payment is made on 1st January 2014. Let un denote the amount owed by
Robert at the end of nth year after 2013, where n ∈  +0 .

(i) Find u0 . [1]

(ii) Show that un = 1.1n u0 − kx(1.1n − 1) , where k is a constant to be determined. [4]

(iii) Given that Robert owes the bank less than $1000 at the end of 2020, find the
minimum value of x, giving your answer to the nearest dollar. [3]


1
6 (a) Find dt . [3]
3 − 4t 2
(b) Use the substitution u = 5 x to find ∫ 5 x cos 2 ( 5 x ) dx . [5]

7 It is given that the function y = f ( x ) has the Maclaurin’s series 1 + 4 x + ax 2 + ... and

(
satisfies 1 + x 2 ) ddyx =
b (1 + y ) , where a and b are real constants.
2

(i) Show that b = 2 and find the value of a. [4]

f( x)
(ii) Find the series expansion of in ascending powers of x, up to and
4+ x
including the term in x 2 . [3]

f( x)
(iii) State the equation of the normal to the curve y = at x = 0. [1]
4+ x

3 [Turn Over
4−r A B C
8 (i) Express in the form + + . [2]
(r − 1)r (r + 2) r −1 r r + 2

n
4−r
(ii) Hence find ∑ (r − 1)r (r + 2)
r =2
. [3]

Give a reason why the series is convergent, and state its limit. [2]

n
3− r
(iii) Use your answer to part (ii) to find ∑ r (r + 1)(r + 3) .
r =2
[2]

9 On a single Argand diagram, sketch the loci given by


z −1 − i ≥ 2 ,
2
(i)
 z +1  π
(ii) arg  ≥ ,
 3 + i  12
(iii) z > z − 1 . [7]

Hence, or otherwise, find the range of values of z − i and arg ( z − i). [3]

10 A file is downloaded at r kilobytes per second from the internet via a broadband
connection. The rate of change of r is proportional to the difference between r and a
constant. The initial value of r is 348. If r is 43, it remains at this constant value.
dr
(i) Show that = k (r − 43) . [2]
dt

(ii) Hence obtain an expression for r in terms of k and t. [4]

The total amount of data downloaded, I kilobytes, in time t seconds, is given by


dI
=r .
dt
(iii) Given that there is no data downloaded initially, find I in terms of k and t. [2]

(iv) It is given that a file with a size of 5700 kilobytes takes 90 seconds to
download. Find the value of k . [2]

(v) Explain what happens to the value of r in the long run. [1]

4 [Turn Over
D
11
C

j
k
i A
O

B
The diagram above shows part of the structure of a modern art museum designed by
Marcus, with a horizontal base OAB and vertical wall OADC. Perpendicular unit
vectors i, j, k are such that i and k are parallel to OA and OC respectively.

The walls of the museum BCD and ABD can be described respectively by the
equations
 −1  14  5  −1
       
r ⋅  −5  =36 and r = 0  + λ  4  + µ  0  , where λ , µ ∈  .
6 0 0 4
       

(i) Write down the distance of A from O. [1]

(ii) Find the vector equation of the intersection line of the two walls BCD and
ABD. [3]

(iii) Marcus wishes to repaint the inner wall ABD. Find the area of this wall. [3]

Suppose Marcus wishes to divide the structure into two by adding a partition such
that it intersects with the walls BCD and ABD at a line. This partition can be
β , where α , β ∈  .
described by the equation 2 x − 7 y + α z =

(iv) Find the values of α and β . [2]

(v) Another designer, Jenny, wishes to construct another partition which is


described by the equation 2 x − 7 y + α z = γ , where γ ≠ β . State the
relationship between Jenny’s and Marcus’ partitions. [1]

Deduce the number of intersection point(s) between the walls BCD, ABD, and
Jenny’s partition. [1]

5 [Turn Over
x2 − 4
12 The curves C1 and C2 have equations ( x − 2) 2 = a 2 (1 − y 2 ) and y = , where
x +1
1 < a < 2, respectively. Describe the geometrical shape of C1. [1]

(a) State a sequence of transformations which transforms the graph of x 2 + y 2 =1


to the graph of C1 . [3]

(b) (i) Sketch C1 and C2 on the same diagram, stating the coordinates of any
points of intersection with the axes and the equations of any
asymptotes. [6]

(ii) Show algebraically that the x-coordinates of the points of intersection


of C1 and C2 satisfy the equation
( x + 1) ( x − 2 ) = a 2 ( x + 1) 2 − a 2 ( x 2 − 4) 2 .
2 2
[2]

(iii) Deduce the number of real roots of the equation in part (ii). [1]

6 [Turn Over
Section A: Pure Mathematics [40 marks]

1 A tin has a fitting cylindrical lid which overlaps its cylindrical body by 5 cm. When
completely closed, it has base radius x cm and height y cm, as shown in the diagram.
The body and the lid are made from thin metal sheet such that the difference in their
radii is negligible. The total area of metal sheet used to make the tin and its lid is
400π cm2.

Show that the volume V cm3 of the tin is given by

V π x ( 200 − x 2 − 5 x ) .
=

If x varies, find the values of x and y for which V has its maximum value. [6]

2 (i) Solve the equation


z 5 − 32 =
0,

expressing the answers in the form reiθ , where r > 0 and −π < θ ≤ π . [2]

 2w + 1 
5

(ii) Explain why the equation   − 32 =


0 has four roots. [1]
 w 

1
The roots of the equation are denoted by w1, w2, w3 and w4. By finding ,
w
4
1
show that ∑w
i =1
is a real number. [4]
i

2 [Turn Over
3 The nth term and the sum of the first n terms of a sequence are denoted by un and Sn
respectively. Given that Sn is a quadratic polynomial in n and u1 = 100 , u2 = 90 and
10

∑u
r =3
r = 360 , find Sn in terms of n. [4]

Hence show that this sequence is an arithmetic progression. [3]

4 The points A, B, C have position vectors i, 2j − tk, tk relative to an origin O


respectively, where t is a fixed constant. The points X and Y lie on AB and BC
respectively such that
AX BY µ
= = ,
XB YC 1 − µ

where µ is a parameter such that 0 < µ < 1.



(i) Find the vector XY in terms of t and µ . [2]

(ii) Prove that O, X, Y are non-collinear. [2]

(iii) Determine if  XOY can be 90ο , justify your answer. [3]



(iv) Find the projection vector of XY onto 4i + j. [2]

5 A point P(x, y) moves in the x-y plane such that the distance from the line x = −3 is
always equal to the distance from the point ( 6, −2 ) .

(i) Prove that the locus of P can be represented by a curve C with equation
( y + 2) = 9 ( 2 x − 3) .
2
[2]

(ii) Sketch the curve C, making clear the main relevant features of C. [2]

The region R is bounded by the curve C, the lines y = 7 and x = 2 . Find


(iii) the exact area of R , showing your working clearly, [4]

(iv) the volume of revolution formed when R is rotated through 2π radians about
the x-axis. [3]

3 [Turn Over
Section B: Statistics [60 marks]

6 An airline wishes to assess its in-flight service for a specific flight and employs a
marketing research company to administer a survey. The seats on this flight are
divided into classes as follows:

First Class Business Class Economy Class Total


No. of Seats 20 80 300 400

An employee of the company proposes using stratified sampling method to select 80


seats and ask the passengers occupying these seats to complete a questionnaire.
Describe how this sample can be obtained. Suggest a practical difficulty which may
be encountered in carrying out this proposal. [3]

Another employee suggests using simple random sampling method to choose a


sample of 80 passengers from the list of passengers who have checked in. Explain
why it may not be appropriate to use this sampling method. [1]

7 A boy intends to arrange a set of coloured square tiles flat on the floor as shown
below with each row being labelled.

First row

Second row

Third row

Fourth row

There are four identical blue tiles, three identical yellow tiles, two identical green tiles
and one red tile.
Find the number of ways to arrange the tiles if

(i) there are no restrictions, [1]

(ii) exactly one yellow tile and exactly two blue tiles are in the same row, [3]

(iii) there are less than 3 yellow tiles in the fourth row . [2]

All the ten tiles are now placed in a row. Find the number of ways he can arrange the
tiles such that all the blue tiles are separated. [2]

4 [Turn Over
8 The life span, x (in hours), of a certain electronic component is known to follow a
normal distribution with mean 9000 hours and standard deviation σ hours. Following
a change in the manufacturing procedure, a batch of components is produced and a
random sample of 10 components is taken from this batch.

(a) The life spans of the 10 components are summarized by


Σ(x – 9000) = 2010, Σ(x – 9000)2 = 911 157.
Test at the 5% level of significance whether the mean life span of the
components has increased after the change in the manufacturing procedure. [3]

(b) Let x denote the mean life spans (in hours) of the 10 components in the sample.
If σ = 25, find the set of values of x so that we can conclude at the 1% level of
significance that the population mean life span of the components has increased.
State an assumption for the above test to be valid. [5]

9 An orchard owned by Mark produces oranges which have masses (in grams) that
follow a normal distribution N(190, 576). Visitors to this orchard can buy the oranges
at $0.10 per 100 g.

Find the probability that the payment made by a visitor buying 20 oranges will differ
from 2 times the payment made by another visitor buying 10 oranges by at most
$0.15. [4]

The orchard produces apples which are graded according to their mass. Apples with a
mass exceeding 150 g are graded as 'large' while apples with a mass less than 70 g are
graded as 'small'. Mark finds that the proportion of apples graded as 'large' is the same
as that of the apples graded as 'small'. It is given that the mass of a randomly chosen
apple from Mark's orchard follows a normal distribution with mean µ g and standard
deviation 30 g.

(i) Find the value of µ . [1]

(ii) What is the probability that Mark will get at least 5 apples graded as 'large'
when he randomly selects 65 apples from his orchard? [3]

5 [Turn Over
10 A public opinion poll surveyed a sample of 1000 voters. The table below shows the
number of males and females supporting Party A, Party B and Party C.

Party A Party B Party C


Male 200 130 70
Female 250 300 50

(a) One of the voters is chosen at random. Events A, C and M are defined as
follows:
A : The voter chosen supports Party A.
C : The voter chosen supports Party C.
M : The voter chosen is a male.
Find
(i) P(A M ),

(ii) P(M ' ∩ C ').

Determine whether A and M are independent. [4]

(b) It is given that in the sample, 20% of Party A supporters, 30% of Party B
supporters and 5% of Party C supporters are immigrants.

(i) One of the voters selected from the sample at random is an immigrant.
What is the probability that this voter supports Party A? [2]

(ii) Three voters are chosen from the sample at random.


Find the probability that there is exactly one immigrant voter who
supports Party C or exactly one female who supports Party A (or both). [4]

11 At the counselling centre CareforSociety, the average number of call-ins received in a


month regarding alcohol abuse problem is denoted by λ . The probability of receiving
at most 9 such calls in a week is 0.701. Assuming that there are 4 weeks in a month,

(i) show that the value of λ is 32.5, correct to 3 significant figures. State a
condition under which the distribution used is valid. [3]

(ii) by using a suitable approximation, find the probability that in a month, the
number of call-ins received is more than 25 but not more than 40. [4]

The centre also has 70 support groups, each consisting of n people, which help one
another to deal with alcohol abuse problems. It is known that, on average, 3 in 20
people in such support groups will be successful in correcting their alcohol abuse
problem. Given that in the 70 support groups, the probability of having an average of
at least 4 people per group successfully correcting their alcohol abuse problem is
more than 0.7, determine the minimum value of n. [4]

6 [Turn Over
12 With the implementation of the new bus fare system, Jasmine wanted to know how
the new system would affect her. She identified 12 common locations and used a map
to measure the straight line distance, x km, of each location from her home. She also
measured the road distance, y km, of each location from her home and the
corresponding bus fare, s cents. The data are shown below.

Location A B C D E F G H I J K L
Straight line
7.7 3.0 24.1 13.2 9.3 9.0 10.4 3.5 17.6 4.5 2.0 2.5
distance, x
Road
8.8 3.3 28.0 16.1 9.4 8.9 12.5 15.8 22.5 5.0 2.2 2.8
distance, y
Bus fare, s 121 81 181 149 125 121 137 149 173 91 71 71

(i) By considering the values of x and y, explain why Location F should be omitted
from any further analysis. State, with a reason, another location that should be
omitted. [2]

Omit the data for the two locations in (i).

(ii) Use a suitable regression line to give an estimate of the straight line distance
when the road distance is 20.0 km. [2]

(iii) Draw a scatter diagram of s against y. State, with a reason, which of the
following models is more appropriate to describe the relationship between y and
s:
Model I: s= a + by 2 ,
Model II: s= a + b ln y , where a and b are positive constants. [3]

(iv) Using the more appropriate model found in (iii), calculate the equation of the
corresponding regression line. [2]

(v) Comment on the use of the regression line found in (iv) to estimate the road
distance travelled if the bus fare is 170 cents. [2]

7 [Turn Over
2010 HCI Prelim Paper 1 Solutions

Qtn Solutions
1. x 4
− ≤0
x − 2 ( x − 2) 2
( x − 1) − 5 ≤ 0
2
x2 − 2 x − 4
⇒ ≤0⇒
( x − 2) 2
( x − 2) 2
⇒ ( x − 1) 2 − 5 ≤ 0, , ( x − 2) 2 is always positive for all real values of x.
⇒ 1− 5 ≤ x ≤ 1+ 5 , x ≠ 2
ex 4
For x ≤ x
e + 2 (e + 2) 2
Replace x by −e x ,

⇒ 1 − 5 ≤ −e x ≤ 1 + 5
⇒ x ≤ ln ( 5 −1 )
2. n+2
Let P(n) be the proposition un = .
2n − 1
When n = 0,
LHS of P(0) = u0 = −2 (given)
2
RHS of P(0) = = −2
−1
∴ P(0) is true.
Assume P(k) is true for some k ∈ +
∪ {0}
k +2
i.e. uk = .
2k − 1

Show that P(k+1) is true


k +3
i.e. uk +1 = .
2k + 1
When n = k + 1,
(k + 3)uk
LHS of P(k+1) = uk +1 =
2uk + k + 2
( k + 3)( k + 2 )
= 2k − 1
⎛ k + 2⎞
2⎜ ⎟+k +2
⎝ 2k − 1 ⎠
⎡ (k + 3)(k + 2) ⎤
= ⎢ ⎥
⎣ (2k + 1)(k + 2) ⎦

2 [Turn Over
k +3
= = RHS of P(k+1)
2k + 1
Since P(0) is true & P(k) is true ⇒ P(k + 1) is also true, hence by mathematical
induction P(n) is true for all n ∈ + ∪ {0} .
3

y = f ( x)

7
-1
x = −a y = g ( x)

1− a

y = (2 x − 1) − 2
2
(a)
± y + 2 = 2x − 1
1 1
x= − y + 2 Q x < −1
2 2
1 1
∴ f −1 : x a − x + 2, x>7
2 2
(b) a=2

(c) Rf = (7, ∞ ) , Dg = (− 1, ∞ ) .
Since Rf ⊆ Dg , gf exists.

( −∞, −1) ⎯⎯
f
→ ( 7, ∞ ) ⎯⎯
g
→ ( ln ( 7 + a ) , ∞ )
Df Rf Rgf

4(i) dx 1− t2
=
dt (1 + t 2 )2

dy 1+ t2
=
dt (1 − t 2 )2

3 [Turn Over
3
dy ⎛ 1 + t 2 ⎞
=⎜ ⎟
dx ⎝ 1 − t 2 ⎠
Equation of tangent:
3
t ⎛ 1+ t2 ⎞ ⎛ t ⎞
y− = ⎜ 2 ⎟ ⎜
x− 2 ⎟
1− t 2
⎝ 1− t ⎠ ⎝ 1+ t ⎠
(1 − t )
2 3
y = (1 + t 2 ) x − t (1 + t 2 ) + t (1 − t 2 )
3 2 2

= (1 + t 2 ) x − 4t 3
3

1 dy
(ii) When t = , = 27
2 dx

Let α be the acute angle between the two lines.

α
B A

Note: α = A - B
tan A = 27 , tan B = 1

A = tan −1 27 = 87.879°
B = tan −1 (1) = 45°
α = A − B = 42.9°

Alternative Solution

27 − 1 26
tan α = =
1 + ( 27 )(1) 28
26
α = tan −1 = 42.9°
28

5(i) u0 = $1.14 (100 000) = $146 410


(ii) u1 = 1.1(u0 − 12 x)
u2 = 1.1[1.1(u0 − 12 x) − 12 x ]
= 1.12 u0 − 1.12 (12 x) − 1.1(12 x)

4 [Turn Over
u3 = 1.1 ⎡⎣1.12 u0 − 1.12 (12 x) − 1.1(12 x) − 12 x ⎤⎦
= 1.13 u0 − 1.13 (12 x) − 1.12 (12 x) − 1.1(12 x)
:
:
n −1
un = 1.1 u0 − 1.1 (12 x) − 1.1 (12 x) − ... − 1.1(12 x)
n n

= 1.1n u0 − 12 x (1.1n + 1.1n −1 + ... + 1.1)


⎛ 1.1(1.1n − 1) ⎞
= 1.1 u0 − 12 x ⎜
n

⎜ 0.1 ⎟
⎝ ⎠
= 1.1n u0 − 132 x (1.1n − 1)
(iii) n = 7 at end of 2020
1.17 u0 − 132 x(1.17 − 1) < 1000
x > $2270.30
Least x to the nearest dollar = $2271

6(a) 1
∫ 3 − 4t 2
dt

1 1
=− ∫
4 t2 − 3
dt

4
1 1
=−
4 ∫ ⎛ 3 ⎞
2
dt
t2 − ⎜ ⎟
⎝ 2 ⎠
⎡ 3 ⎤
t−
1⎢ 1 ⎥
=− ⎢ ln 2 ⎥ +C
4⎢ 3 3 ⎥
⎢⎣ t+ ⎥⎦
2
3 2t − 3
=− ln + C
12 2t + 3
(b) u = 5x
du
= 5 x ln 5
dx
dx 1 1
∴ = x =
du 5 ln 5 u ln 5

5 [Turn Over
∫ 5 x cos 2 (5 x ) dx
1
= ∫ u cos 2 u ⋅ du
u ln 5
1
=
ln 5 ∫ cos 2 u du

1
2 ln 5 ∫
= (1 + cos 2u ) du

1 ⎡ sin 2u ⎤
= ⎢ u+ +C
2 ln 5 ⎣ 2 ⎥⎦
1 ⎡ x sin 2(5x ) ⎤
= ⎢5 + ⎥+C
2 ln 5 ⎣ 2 ⎦
7i dy
When x = 0, y = 1, = 4.
dx

(1 + x ) ddyx = b (1 + y )
2 2

dy
⇒ = b(2) = 4
dx
⇒ b = 2 (Shown)
d2 y dy ⎛ dy ⎞
(1 + x )2

dx 2
+ 2x
dx
= 2⎜ 2y ⎟
⎝ dx ⎠
d2 y
= 16
dx 2
16
a= =8
2!

1
ii −
f( x)(4 + x) 2

1 x −1
= (1 + 4 x + 8 x 2 + ...)(1 + ) 2
2 4
1 3
(− )(− )
1 1 x 2 ( x ) 2 + ...)
= (1 + 4 x + 8 x + ...)(1 + (− )( ) + 2
2

2 2 4 2! 4
1 x 3 2
= (1 + 4 x + 8 x 2 + ...)(1 − + x + ...)
2 8 128
1 31 963 2
= (1 + x + x + ...)
2 8 128
iii 16
Gradient of normal = −
31
Equation of normal:

6 [Turn Over
1 16
y= − x
2 31

8(i) 4−r A B C
= + +
(r − 1)r (r + 2) r − 1 r r + 2
4 − r = Ar (r + 2) + B(r − 1)(r + 2) + C (r − 1)r
A = 1, B = −2, C = 1
(ii) n
4−r

r = 2 ( r − 1) r ( r + 2)
n
1 2 1
=∑ − +
r =2 r − 1 r r+2
1
= 1−1+
4
1 2 1
+ − +
2 3 5
1 2 1
+ − +
3 4 6
1 2 1
+ − +
4 5 7
+ M
1 2 1
+ − +
n − 4 n − 3 n −1
1 2 1
+ − +
n−3 n−2 n
1 2 1
+ − +
n − 2 n −1 n +1
1 2 1
+ − +
n −1 n n + 2
1 2 1 1 1 2 1
= − + + + − +
2 3 3 n n +1 n n + 2
1 1 1 1
= − + +
6 n n +1 n + 2
(iii) ⎛ 1 1 1 ⎞
lim ⎜ − + + ⎟ = 0 , hence the series in (ii) converges.
n →∞
⎝ n n +1 n + 2 ⎠


4−r 1
∑ (r − 1)r (r + 2) = 6
r =2

7 [Turn Over
(iv) n
3− r
∑ r (r + 1)(r + 3)
r =2

1 3 − (n − 1) 3− n
= + ... + +
(2)(3)(5) (n − 1)(n)(n + 2) n(n + 1)(n + 3)
n +1
4−r 2
=∑ −
r = 2 ( r − 1) r ( r + 2) (1)(2)(4)
1 1 1 1 1
= − + + −
6 n + 1 n + 2 n + 3 12
1 1 1 1
=− − + +
12 n + 1 n + 2 n + 3
9.
y
Q

P R (1,1)
1
π
4 x
-1 O 1
π
6

z −1− i 2 ≥ 2
⇒ z − (1 + i) ≥ 2

π ⎛ z +1 ⎞
≤ arg ⎜ ⎟≤π
12 ⎝ 3+i⎠
π π π
⇒ + ≤ arg( z + 1) ≤ π +
12 6 6
π 7π
⇒ ≤ arg( z + 1) ≤
4 6

1 7 7 1
Method 1: QR = 2 − = ; PQ = + = 2
4 2 4 4
Method 2: QR is the perpendicular bisector, so PQ = 2 (radius)
⇒ z −i > 2
π π
≤ arg ( z − i ) <
4 2

8 [Turn Over
10(i) Let the constant be a.
dr
∴ = k (r − a) , where k is a constant.
dt
dr
Given r = 43 when =0,
dt
∴ 0 = k (43 − a )
Since k ≠ 0, then a = 43
dr
∴ = k (r − 43) (shown)
dt
(ii) 1
∫ r − 43 dr = k ∫ dt
ln r − 43 = kt + C1
r − 43 = e kt + C1
r = 43 + Ae kt where A = eC1
When t = 0 , r = 348 .
∴ A = 305 .
∴ r = 43 + 305e kt
(iii) I = ∫ r dt

= ∫ (43 + 305e kt ) dt
305 kt
= 43t + e + C2
k
When t = 0 , I = 0 .
305
∴ C2 = −
k
305 kt
∴ I = 43t + (e − 1)
k
(iv) Given I = 5700 and t = 90 ,
305 90 k
∴ 5700 = 43(90) + (e − 1)
k
305 90 k
1830 = (e − 1)
k
6k = e90 k − 1

Solving using GC,


k = − 0.167 or k = 0 (NA)
1
(v) − t
r = 43 + 305e 6

1
− t
If t becomes larger, 305e → 0 , r → 43
6

Hence r would be reduced to a steady 43 kilobytes per second in the long run.

9 [Turn Over
11i OA = 14
ii Plane ABD OR
⎛ 5 ⎞ ⎛ −1⎞ ⎛ 16 ⎞ ⎛14 + 5λ − μ ⎞ ⎛ −1 ⎞
⎜ ⎟ ⎜ ⎟ ⎜ ⎟ ⎜ ⎟⎜ ⎟
4λ ⎟ . ⎜ −5 ⎟ = 36
⎜ 4 ⎟ × ⎜ 0 ⎟ = ⎜ −20 ⎟ ⎜
⎜0⎟ ⎜ 4 ⎟ ⎜ 4 ⎟ ⎜ 4μ ⎟⎜6⎟
⎝ ⎠ ⎝ ⎠ ⎝ ⎠ ⎝ ⎠⎝ ⎠
⎛ 4 ⎞ ⎛14 ⎞ ⎛ 4 ⎞ −14 − 5λ + μ − 20λ + 24μ = 36
⎜ ⎟ ⎜ ⎟⎜ ⎟ 25μ = 25λ + 50
r. ⎜ −5 ⎟ = ⎜ 0 ⎟ . ⎜ −5 ⎟ = 56
%⎜ ⎟ ⎜ ⎟⎜ ⎟
⎝1⎠ ⎝0⎠⎝1⎠ μ =λ+2
⎧ 4 x − 5 y + z = 56 ⎛14 ⎞ ⎛ 5⎞ ⎛ −1⎞
⇒⎨ ⎜ ⎟ ⎜ ⎟ ⎜ ⎟
r = ⎜ 0 ⎟ + λ ⎜ 4 ⎟ + (λ + 2) ⎜ 0 ⎟
⎩− x − 5 y + 6 z = 36
⎜0⎟ ⎜ 0⎟ ⎜4⎟
Using GC to solve: ⎝ ⎠ ⎝ ⎠ ⎝ ⎠
⎛ 4⎞ ⎛ 1⎞ ⎛12 ⎞ ⎛ 4⎞
⎜ ⎟ ⎜ ⎟ ⎜ ⎟ ⎜ ⎟
∴ r = ⎜ −8 ⎟ + γ ⎜ 1⎟ , γ ∈ = ⎜ 0 ⎟ + λ ⎜ 4⎟
% ⎜ ⎟ ⎜ 1⎟ ⎜8⎟ ⎜ 4⎟
⎝0⎠ ⎝ ⎠ ⎝ ⎠ ⎝ ⎠
⎛12 ⎞ ⎛ 1⎞
⎜ ⎟ ⎜ ⎟
= ⎜ 0 ⎟ + γ ⎜ 1⎟ , γ ∈
⎜8⎟ ⎜ 1⎟
⎝ ⎠ ⎝ ⎠
iii ⎛ 4+γ ⎞ ⎛ 12 ⎞
uuuv ⎜ ⎟ uuuv ⎜ ⎟
OD = ⎜ −8 + γ ⎟ ⇒ γ = 8 ⇒ OD = ⎜ 0 ⎟
⎜ γ ⎟ ⎜8⎟
⎝ ⎠ ⎝ ⎠
(Reason: j is zero.)
%
⎛ 4⎞
uuuv ⎜ ⎟
OB = ⎜ −8 ⎟
⎜0⎟
⎝ ⎠
⎛12 ⎞ ⎛14 ⎞ ⎛ −2 ⎞
uuuv ⎜ ⎟ ⎜ ⎟ ⎜ ⎟
AD = ⎜ 0 ⎟ − ⎜ 0 ⎟ = ⎜ 0 ⎟
⎜8⎟ ⎜0⎟ ⎜ 8⎟
⎝ ⎠ ⎝ ⎠ ⎝ ⎠
⎛12 ⎞ ⎛ 4 ⎞ ⎛ 8 ⎞
uuuv ⎜ ⎟ ⎜ ⎟ ⎜ ⎟
BD = ⎜ 0 ⎟ − ⎜ −8 ⎟ = ⎜ 8 ⎟
⎜ 8 ⎟ ⎜ 0 ⎟ ⎜8⎟
⎝ ⎠ ⎝ ⎠ ⎝ ⎠

10 [Turn Over
1 uuuv uuuv
Area ABD = BD × AD
2
⎛ 8 ⎞ ⎛ −2 ⎞ ⎛ 1 ⎞ ⎛ −1 ⎞ ⎛ 4⎞
1⎜ ⎟ ⎜ ⎟ ⎜ ⎟ ⎜ ⎟ ⎜ ⎟
= ⎜ 8 ⎟ × ⎜ 0 ⎟ = 8 ⎜ 1 ⎟ × ⎜ 0 ⎟ = 8 ⎜ −5 ⎟
2⎜ ⎟ ⎜ ⎟ ⎜ 1⎟ ⎜ 4 ⎟ ⎜1⎟
⎝8⎠ ⎝ 8 ⎠ ⎝ ⎠ ⎝ ⎠ ⎝ ⎠
= 8 42 = 51.8 (3 s.f.)
iv 2(4) − 7(−8) + α (0) = β
2(12) − 7(0) + 8α = β
⇒ β = 64, α = 5

OR

The 3 planes intersect at the line


⎛ 4⎞ ⎛ 1⎞
⎜ ⎟ ⎜ ⎟
r = ⎜ −8 ⎟ + γ ⎜ 1⎟ , γ ∈
% ⎜ ⎟ ⎜ 1⎟
⎝0⎠ ⎝ ⎠
⎛ 2 ⎞ ⎛ 1⎞
⎜ ⎟⎜ ⎟
⎜ −7 ⎟ . ⎜ 1 ⎟ = 0
⎜ α ⎟ ⎜ 1⎟
⎝ ⎠⎝ ⎠
2 − 7 +α = 0
α =5
⎛ 4⎞⎛ 2⎞
⎜ ⎟⎜ ⎟
⎜ −8 ⎟ . ⎜ −7 ⎟ = 8 + 56 = 64
⎜0⎟⎜ 5⎟
⎝ ⎠⎝ ⎠
β = 64

v Their partitions are parallel to each other.

There is no intersection point.


12 Ellipse

(a) ( x − 2) 2 = a 2 (1 − y 2 )
( x − 2) 2
⇒ + y2 = 1
a2

Method 1:
Sequence of transformations:
1) Scale // to x-axis by factor a.
2) Translate in the positive x-direction by 2 units.

11 [Turn Over
Method 2:
Sequence of transformations:
2
1) Translate in the positive x-direction by units.
a
2) Scale // to x-axis by factor a.

(bi) x −1
x +1 x 2
−4
−( x 2 + x)
−x−4
− (− x − 1)
−3
x = −1 y y = x −1

−2 2−a 2 2+a
−4 ( x − 2) 2
+ y2 = 1
a2

x2 − 4
y=
x +1

(bii) x2 − 4
Sub y = into ( x − 2) 2 = a 2 (1 − y 2 ) :
x +1
⎛ ⎛ x 2 − 4 ⎞2 ⎞
( x − 2) = a ⎜1 − ⎜
2 2

⎜ ⎝ x + 1 ⎟⎠ ⎟
⎝ ⎠
⇒ ( x + 1) ( x − 2) 2 = a 2 ( x + 1) − a 2 ( x 2 − 4 ) --- (*)
2 2 2

(shown)

Hence the x-coordinate of the points of intersection of


C1 and C2 satisfy equation (*).
(b) From (ii), number of intersection points between C1 and C2 gives the number of
(iii) real roots of the equation (*).

From the graphs, there are 2 points of intersection between C1 and C2 . Hence 2
real roots.

12 [Turn Over
HCI Prelim H2 Mathematics P2 Solutions

2010 HCI H2 Mathematics Preliminary Examination Paper 2 Solution


Qn Solutions
1 Surface area of the tin and lid
= 2π x 2 + 2π xy + 10π x = 400π

200 − x 2 − 5 x
y=
x

Volume of the container


⎛ 200 − x 2 − 5 x ⎞
= π x2 ⎜ ⎟
⎝ x ⎠
= π ( 200 x − x3 − 5 x 2 )

dV
= π ( 200 − 3 x 2 − 10 x )
dx
dV 20
=0⇒ x= or x = −10 (rejected)
dx 3

d 2V 20
2
= π ( −6 x − 10 ) < 0 when x =
dx 3
20
V is maximum when x = .
3
20 55
When x = , y=
3 3
(or x = 6.67, y = 18.3).
2(i) z 5 − 32 = 0 ⇒ z 5 = 32 ei0 = 32ei2k π
2 kπ i
⇒ z = 2e 5
where k = 0, ± 1, ± 2.
(ii) ⎛ 2w + 1 ⎞
5
5
The highest power in the equation ⎜ ⎟ = 32 is four since the terms with w are
⎝ w ⎠
canceled out. Hence the equation has only four roots.

5
⎛ 1⎞
⎜ 2 + ⎟ = 32
⎝ w⎠
1 2 kπ i
⇒ 2 + = z = 2e 5
w
1 2 kπ i
⇒ = 2e 5 − 2 = 2 e 5 − 1
w
2 kπ i
( )
1 1 1 1
⇒ + + +
w1 w2 w3 w4

⎢⎣(
= 2⎡ e
2π i
5
−1 + e )( −2π i
5
)(
−1 + e
4π i
5
)(
−1 + e
−4π i
5
)
−1 ⎤
⎥⎦

1
HCI Prelim H2 Mathematics P2 Solutions

⎡ 2π 4π ⎤
= 2 ⎢ 2 cos + 2 cos − 4⎥
⎣ 5 5 ⎦
⎡ 2 π 4π ⎤
= 4 ⎢cos + cos − 2⎥ ∈ .
⎣ 5 5 ⎦

1 1 1 1
Or use GC, + + + = −10.
w1 w2 w3 w4
3 S n = an 2 + bn + c
U1 = S1 = a + b + c = 100
S 2 = 4a + 2b + c = 190
S10 = 100a + 10b + c = 360 + 100 + 90 = 550
Using GC,
a = −5 , b = 105 , c = 0
Thus Sn = −5n 2 + 105n

U n = Sn − Sn−1
(
= −5n 2 + 105n − −5 ( n − 1) + 105 ( n − 1)
2
)
= 110 − 10n
U n − U n −1
= 110 − 10n − (110 − 10n + 10 )
= −10 (a constant)
Hence sequence is an AP.
4i ⎛0⎞ ⎛ 1 ⎞ ⎛1 − μ ⎞
uuuv ⎜ ⎟ ⎜ ⎟ ⎜ ⎟
OX = μ ⎜ 2 ⎟ + (1 − μ ) ⎜ 0 ⎟ = ⎜ 2 μ ⎟
⎜ −t ⎟ ⎜ 0 ⎟ ⎜ −t μ ⎟
⎝ ⎠ ⎝ ⎠ ⎝ ⎠
⎛0⎞ ⎛0⎞ ⎛ 0 ⎞
uuuv ⎜ ⎟ ⎜ ⎟ ⎜ ⎟
OY = μ ⎜ 0 ⎟ + (1 − μ ) ⎜ 2 ⎟ = ⎜ 2 − 2 μ ⎟
⎜t⎟ ⎜ −t ⎟ ⎜ −t + 2t μ ⎟
⎝ ⎠ ⎝ ⎠ ⎝ ⎠
⎛ 0 ⎞ ⎛1 − μ ⎞ ⎛ μ − 1 ⎞
uuuv ⎜ ⎟ ⎜ ⎟ ⎜ ⎟
XY = ⎜ 2 − 2 μ ⎟ − ⎜ 2 μ ⎟ = ⎜ 2 − 4 μ ⎟
⎜ −t + 2t μ ⎟ ⎜ −t μ ⎟ ⎜ −t + 3t μ ⎟
⎝ ⎠ ⎝ ⎠ ⎝ ⎠

OR
⎛ −1⎞ ⎛ 0⎞
uuur ⎜ ⎟ uuur ⎜ ⎟
AB = ⎜ 2 ⎟ BC = ⎜ −2 ⎟
⎜ −t ⎟ ⎜ 2t ⎟
⎝ ⎠ ⎝ ⎠

2
HCI Prelim H2 Mathematics P2 Solutions

⎛ −1 ⎞ ⎛ μ − 1 ⎞
uuur ⎜ ⎟ ⎜ ⎟
XB = (1 − μ ) ⎜ 2 ⎟ = ⎜ 2 − 2 μ ⎟
⎜ −t ⎟ ⎜ t μ − t ⎟
⎝ ⎠ ⎝ ⎠
⎛ 0⎞ ⎛ 0 ⎞
uuur ⎜ ⎟ ⎜ ⎟
BY = μ ⎜ −2 ⎟ = ⎜ −2 μ ⎟
⎜ 2t ⎟ ⎜ 2t μ ⎟
⎝ ⎠ ⎝ ⎠
⎛ μ −1 ⎞ ⎛ 0 ⎞ ⎛ μ −1 ⎞
uuur uuur uuur ⎜ ⎟ ⎜ ⎟ ⎜ ⎟
XY = XB + BY = ⎜ 2 − 2 μ ⎟ + ⎜ −2 μ ⎟ = ⎜ 2 − 4 μ ⎟
⎜ t μ − t ⎟ ⎜ 2t μ ⎟ ⎜ 3t μ − t ⎟
⎝ ⎠ ⎝ ⎠ ⎝ ⎠
ii Suppose O, X, Y are collinear.
Then
uuuv uuuv
OX = kOY
⎛1 − μ ⎞ ⎛ 0 ⎞
⎜ ⎟ ⎜ ⎟
⎜ 2μ ⎟ = k ⎜ 2 − 2μ ⎟
⎜ −t μ ⎟ ⎜ −t + 2t μ ⎟
⎝ ⎠ ⎝ ⎠
1 − μ = 0 ⇒ μ = 1 (Out of range)
Thus O, X, Y are not collinear.

iii ⎛1 − μ ⎞ ⎛ 0 ⎞
uuuv uuuv ⎜ ⎟⎜ ⎟
OX OY = ⎜ 2 μ ⎟ ⎜ 2 − 2 μ ⎟
⎜ −t μ ⎟ ⎜ −t + 2t μ ⎟
⎝ ⎠⎝ ⎠
= μ(4 – 4μ + t – 2μt )
2 2

=0
4 + t2 1 1
⇒ μ = 0 (reject) or μ = = + 2
4 + 2t 2 t + 2
2

For all t ∈ℝ\{0}, 0 < μ < 1.


Thus ∠XOY can be 90° when t ≠ 0 .

3
HCI Prelim H2 Mathematics P2 Solutions

iv ⎛ μ −1 ⎞
uuuv ⎜ ⎟
XY = ⎜ 2 − 4 μ ⎟
⎜ −t + 3t μ ⎟
⎝ ⎠
projection vector
⎛ μ −1 ⎞ ⎛ 4 ⎞ ⎛ 4 ⎞
⎜ ⎟⎜ ⎟⎜ ⎟
⎜ 2 − 4μ ⎟ . ⎜ 1 ⎟ . ⎜ 1 ⎟
⎜ −t + 3t μ ⎟ ⎜ 0 ⎟ ⎜ 0 ⎟
=⎝ ⎠⎝ ⎠⎝ ⎠
17
⎛ 4⎞
4μ − 4 + 2 − 4μ ⎜ ⎟
= ⎜1⎟
17 ⎜0⎟
⎝ ⎠
⎛ 4⎞
2⎜ ⎟
= − ⎜1⎟
17 ⎜ ⎟
⎝0⎠
5(i)
( x − 6) + ( y + 2)
2 2
= x+3

( y + 2) = ( x + 3) − ( x − 6 )
2 2 2

= 9 ( 2 x − 3)

(ii)

(iii)

For the equation ( y + 2 ) = 9 ( 2 x − 3) ,


2

When x = 2 , y = 1 .

4
HCI Prelim H2 Mathematics P2 Solutions

When y = 7 , x = 6 .

Method 1: Using ∫ x dy
( y + 2) = 9 ( 2 x − 3)
2

3 ( y + 2) 1⎛ ( y + 2) ⎞
2 2

x= + = ⎜3+ ⎟
2 18 2 ⎜⎝ 9 ⎟

⎛ ( y + 2)
⎞ 2
1 7
R=
2 ∫
1
⎜3+


⎟ d y − 2(6)

9

⎪⎧ 1 ⎡ ⎪⎫
7
( y + 2)3 ⎤
= ⎨ ⎢3 y + ⎥ − 12 ⎬
⎪⎩ 2 ⎣ 27 ⎦ 1 ⎪⎭
⎧1 ⎫
= ⎨ ⎡⎣( 21 + 27 ) − ( 3 + 1) ⎤⎦ − 12 ⎬
⎩2 ⎭
= 10 units 2

Method 2: Using ∫ y dx
( y + 2) = 9 ( 2 x − 3)
2

y = −2 + 3 2 x − 3 [ y = −2 − 3 2 x − 3 N.A.]

∫ ( −2 + 3 )
6
R = 4(7) − 2 x − 3 dx
2

⎧⎪ ⎡ 3 6⎫
⎤ ⎪
= ⎨ 28 − ⎢ −2 x + (2 x − 3) 2 ⎥ ⎬
⎪⎩ ⎣ ⎦ 2 ⎪⎭
= { 28 – [(–12 + 27) – (–4 + 1)]}
= 10 units2

(iv) Volume required


= vol. of cylinder – (vol. generated by curve from y = –2 to y = 1)

∫( )
⎡ 6 2 ⎤
= ⎢π (7) 2 (4) − π −2 + 3 2 x − 3 d x ⎥
⎣ 2 ⎦
= 196π – 92π
= 327 unit3 (3 s.f.)
6 Use random sampling method to select a sample from each class. The number of seats from
each class would be proportional to the size of each stratum.
First Class Business Class Economy Class
Any 1 of the answers below:
4 16 60
Some passengers have
booked a flight ticket but did not turn up or changed flight so some of the seats in the sample
may not have a passenger.

OR

5
HCI Prelim H2 Mathematics P2 Solutions

The flight is not fully booked so the chosen seat could be empty.

OR The passenger may ignore the questionnaire.

It is not appropriate to use simple random sampling as passengers from different classes may
have different opinions on the service. The number of passengers in the first class is very
small, so the passengers from the first class may not be chosen at all using the simple random
sampling method.

7(i) 10!
No. of ways = = 12600
4!3!2!

7(ii) Case 1: The 2 blue tiles and 1 yellow tile are in the 4th row with the 4th tile being red or
green.
No. of ways
= no. of ways with B, B, Y, G in 4th row + no. of ways with B, B, Y, R in 4th row
4! 6! 4! 6!
= × + × = 3240
2! 2!2! 2! 2!2!2!
Case 2: The 2 blue tiles and 1 yellow tile are in the third row.
3! 7!
No. of ways = × = 1890
2 2!2!2!
3! 3! 4! 3! 4!
Total no. of ways = 3240 + 1890 – 2! 2! 2! – 2! 2! 3!
= 5130 – 108 – 216 = 4806

7(iii) No. of ways such that less than 3 yellow tiles are in the fourth row
7!
= 12600 − 4C3 =12600 − 420 =12180
4!2!
7 last No. of ways
part 6!
= × 7C4 = 2100
3!2!
8(i) 2010 + 10(9000)
x= = 9201,
10
1 ⎡⎢ ( ∑ ( x − 9000 ) ) ⎤⎥ 507147
2

s = ∑ ( x − 9000 ) −
2
2
=
9⎢ 10 ⎥ 9
⎣ ⎦
H0 : μ = 9000
H1 : μ > 9000
x − 9000
Test Stat: = ~ t (9)
507147
9 × 10
p–value = 0.01265 < 0.05

Since the p –value = 0.01265 < 0.05, we reject H 0 and conclude that there is sufficient
evidence, at 5% level of significance, that the mean life span of the electronic component has
increased.

6
HCI Prelim H2 Mathematics P2 Solutions

8(ii) H0: μ = 9000 vs H1: μ > 9000


252
Under H0, X ~ N(9000, 10 ) = N(9000, 62.5).

X – 9000
Test Statistic = ~ N(0, 1).
62.5
Level of significance = 1%
P(Z > 2.326347877) = 0.01
At the 1% significance level, reject H 0 if z ≥ 2.326347877.
x – 9000
z= ≥ 2.326347877
62.5
x ≥ 9018.391395 = 9020.

Assumptions: The standard deviation of the life span remains unchanged after the change in
process.
9
First X ~ N(190, 576)
part T = 0.001( X1 + ... + X20 ) − 0.001(2)( X21 + ... + X30 ) ~ N(0, 0.03456)

P( | T |≤ 0.15) = P( − 0.15 ≤ T ≤ 0.15)


= 0.580

OR

A = X1 +... + X20 – 2(X21 +... + X30) ~ N(0, 34560)

0.15
P( | A |≤ ) = P( − 150 ≤ A ≤ 150)
0.001
= 0.580
9(i) Let Y be the r.v. denoting the mass of a randomly chosen apple from Mark's orchard.
Y ~ N(μ , 302 )

Since the shaded area is the same, using the symmetric property of the normal curve,
μ = 110

9(ii) Probability that Mark will get an apple graded as 'large' chosen at random = P(Y > 150) =

7
HCI Prelim H2 Mathematics P2 Solutions

0.09121128

Let A be the r.v. denoting the number of apples graded as large out of 65 randomly chosen
apples.
A ~ B(65, 0.09121128)

P ( A ≥ 5) = 1 − P ( A ≤ 4)
= 0.718
10(a) 200 1
(i) P(A ⎜M ) = =
400 2
250 + 300 11
(ii) P(M ' ∩ C ') = =
1000 20

9 1
P ( A) = , P ( A M ) = ≠ P ( A)
20 2
A and M are not independent.
10(b) (i) No. of immigrants in the sample
= 0.2 ( 200 + 250 ) + 0.3 (130 + 300 ) + 0.05 (120 ) = 225
0.2 × 450
P(voter supports Party A given voter is an immigrant) = = 0.4
225

(ii) Number of immigrants supporting Party C = 0.05 (120 ) =6


P(exactly one immigrant voter supporting Party C or
exactly one female voter supporting Party A (or both))
= P ( exactly 1 immigrant voted for C )
+ P ( exactly 1 female voted for A ) − P ( both )
C1 994C2 + 250C1 750C2 − 250C1 6C1 744C1
6
= 1000
= 0.434
C3

Alternative method:
6 994 993 250 750 749 6 250 744
Required Probability = ×3+ ×3− × 3!
1000 999 998 1000 999 998 1000 999 998
= 0.434

8
HCI Prelim H2 Mathematics P2 Solutions

11(i) ⎛λ⎞
Let X be the r.v. denoting the number of call–ins in a week. Hence X ~ Po ⎜ ⎟ .
⎝4⎠

We are looking for the λ such that P(X ≤ 9) = 0.701


P( X ≤ 9)

From graph, the value of λ = 32.5 (to 3 sig.fig).

The condition is that the rate of call–ins received by the centre is constant throughout a
month / the call–in occurs randomly / The call–ins occur in a month are independent of one
another

11(ii) Let Y be the r.v. denoting the number of call–ins in a week.


Y ~ Po(32.5)
Since the mean is bigger than 10, hence
Y ~ N (32.5, 32.5) approximately.

c.c
P (25 < Y ≤ 40) ⎯⎯ → P (25.5 < Y < 40.5) = 0.810

11(iii Let S be the r.v. denoting the number of successful cases out of the n people in a support
) group.
3
S ~ B (n, )
20

Since the number of groups concerned, which is 70, is large, therefore by applying CLT,
⎛ 3 ⎞⎛ 3 ⎞
n ⎜ ⎟⎜ 1 − ⎟
3 20 20 ⎠
S ~ N ( n, ⎝ ⎠⎝ ) approximately.
20 70

EITHER
n P ( S ≥ 4)
27 0.589
28 0.812
Hence minimum value of n is 28.

OR

P ( S ≥ 4) > 0.7

9
HCI Prelim H2 Mathematics P2 Solutions

P ( S < 4) < 0.3


4 − 0.15n
P(Z < ) < 0.3
0.1275n
70
4 − 0.15n
< −0.5244
0.1275n
70
0.1275
4 − 0.15n < (−0.5244 ) n
70
0.1275
0.15n − (0.5244 ) n −4>0
70
n > 5.23912 or n < −5.0899(reject)
n > 27.45
Least n = 28.

12(i) Location F should be omitted as the road distance cannot be smaller than the straight line
distance, indicating that it is an incorrect data entry.

From the scatter diagram, another location that should be omitted is location H, as it is an
outlier based on the scatter diagram.

12(ii) The suitable regression line is the regression x on y:


x = 0.3936554 + 0.81702935 y
When y = 20.0,
x = 16.7 km

12(iii s
)
180

70
y
2 30

10
HCI Prelim H2 Mathematics P2 Solutions

Since the graph of s = a + b ln y is concave downwards whereas the graph of s = a + by 2 is


concave upwards, the graph of s = a + b ln y will be more suitable to describe the scatter
diagram of s and y. Hence model II is more suitable.

12(iv The appropriate regression line of s on ln y is s = 25.9499647 + (45.24427905) ln y ,


) i.e. s = 25.9 + (45.2) ln y (to 3 s.f.)

12(v) Since r for s and ln y is 0.992 close to 1, the linear correlation is strong between s and ln y.
Furthermore, 170 cents is within the data range of the sample. Therefore the estimation using
the line in (iv) is reliable.

Since y is the independent variable, the line found in (iv) is also suitable for the estimation.

11
INNOVA JUNIOR COLLEGE
JC 2 PRELIMINARY EXAMINATION 1
in preparation for General Certificate of Education Advanced Level
Higher 2

CANDIDATE
NAME

CLASS INDEX NUMBER

MATHEMATICS 9740/01
Paper 1 15 September 2010
3 hours

Additional Materials: Answer Paper


Graph Paper
List of Formulae (MF15)

READ THESE INSTRUCTIONS FIRST

Do not open this booklet until you are told to do so.

Write your name, class and index number on all the work you hand in.

Write in dark blue or black pen on both sides of the paper. You may use a soft pencil for any
diagrams or graphs.
Do not use staples, paper clips, highlighters, glue or correction fluid.

Answer all the questions.


Give non-exact numerical answers correct to 3 significant figures, or 1 decimal place in the
case of angles in degrees, unless a different level of accuracy is specified in the question.
You are expected to use a graphic calculator.
Unsupported answers from a graphic calculator are allowed unless a question specifically
states otherwise.
Where unsupported answers from a graphic calculator are not allowed in a question, you
are required to present the mathematical steps using mathematical notations and not
calculator commands.

You are reminded of the need for clear presentation in your answers.

At the end of the examination, fasten all your work securely together.

The number of marks is given in brackets [ ] at the end of each question or part question.

This document consists of 6 printed pages.

Innova Junior College [Turn over


2

1 For any given mass of gas, the volume V cm3 and pressure p (in suitable units) satisfy
the relationship
1 n
V p ,
k
where k and n are constants.
For a particular type of gas, n  2.3 . At an instant when volume is 32 cm3, the
pressure is 105 units and the pressure is increasing at a rate of 0.2 units s1.
Calculate the rate of decrease of volume at this instant. [4]

2
2 Given that the coefficient of x 2 in the series expansion of is 108, find the
1  3x  n

value of n, where n is a positive integer. [4]

3 The sequence of numbers un , where n  1, 2, 3,..., is such that


9
u1  and 8un1  un  7 n  8 .
8
Use the method of mathematical induction to show that
un  n  23n for n  1 . [5]
(i) Determine if the sequence converges. [1]
n
(ii) Find  ur in terms of n. [2]
r 1

IJC/2010/JC2 9740/01/S/10
3

C
4

O
M

A B

The diagram shows a quadrilateral OABC with OA  AB and OC  BC .


1  1 
   
Points A and B have position vectors    and  0  respectively.
2 1
   
(a) Find cosine of angle OAB in terms of  . [2]
 
(b) M is the midpoint of OB and 4AM  MC . By considering the area of the
quadrilateral OABC, show that
  5  
OA  OC  OA  OB . [4]
2

5 Illustrate, on a single Argand diagram, the locus of the point representing the complex
number z that satisfies both the inequalities
3 
  arg  z  3  3i   and z  3  3i  2 . [4]
4 2
(i) Find the greatest and least values of z  3i . [2]

(ii) Find the least possible value of arg z  , giving your answer in radians. [3]

IJC/2010/JC2 9740/01/S/10 [Turn over


4

6 Show, by means of the substitution w  x 2 y , that the differential equation


dy
x  2 y  3 xy  0
dx
can be reduced to the form
dw
  3w . [2]
dx
1
Hence find y in terms of x, given that y   when x  2 . [4]
2

7 A curve is defined parametrically by


x  2 cos t , y  2t  1 ,
where 0  t   .

(i) Find the equation of the normal to the curve at the point P where t  . [5]
3
(ii) The normal at P meets the y-axis at N and the x-axis at M. Given that the curve
meets the y-axis at Q, find the area of triangle MNQ, correct to 1 decimal place.
[5]

x 2 ( y  4) 2
8 A curve has equation  1.
4 9
(i) Sketch the curve, stating the equations of the asymptotes and the coordinates
of the vertices. [3]

x 2 ( y  4) 2
(ii) The region enclosed by the curve   1 , the x-axis and the line
4 9
x  2 is rotated through 4 right angles about the y-axis to form a solid of
revolution of volume V. Find the exact value of V, giving your answer in terms
of  . [4]

IJC/2010/JC2 9740/01/S/10
5

9 In a medical research centre, a particular species of insect is grown for treatment of


open wounds. The insects are grown in a dry and cool container, and they are left to
multiply. The increase in the number of insects at the end of each week is at a
constant rate of 4% of the number at the beginning of that week. At the end of each
week, 10 of the insects would die due to space constraint and are removed from the
container.
A researcher puts y insects at the beginning of the first week and then a further y at the
beginning of the second and each subsequent week. He also decides that he will not
take any insect out of the container.
(i) How many insects will there be in the container at the end of the first week?
Leave your answer in terms of y. [1]

(ii) Show that, at the end of n weeks, the total number of insects in the container is

 26 y  250  1.04 n  1 . [4]

(iii) Find the minimum number of complete weeks for the population of the insects
to exceed 13 y  125 . [4]

10 The functions f and g are defined as


f : x  1 x for x  1
g : x  e x  1 for x  0

(i) Define f 1 in a similar form, including its domain. [3]


(ii) State the relationship between f and f 1 , and sketch the graphs of f and f 1 on
the same diagram. [3]
(iii) Find the exact solutions of the equation
f ( x)  f 1 ( x) . [2]
(iv) Show that the composite function fg exists. [2]
(v) Given that h ( x)  fg( x) for x  0 , show that h is an increasing function for
x  0. [2]

IJC/2010/JC2 9740/01/S/10 [Turn over


6

11 (a) Write 2 cos 3x cos x in the form cos  px   cos  qx  , where p and q are
positive integers. [1]
Hence find
(i)  cos 3x cos x dx , [2]

(ii) the exact value of  0
4 x cos 3 x cos x dx . [4]

(b) State a sequence of transformations which transform the graph of y  sin x to


 3 
the graph of y  sin  2 x    . [2]
 2 

(c) Find the numerical value of the area of the region bounded by the curves
 3  
y  cos 3x cos x and y  sin  2 x    for 0  x  . [3]
 2  2

2  3  4
     
12 A plane 1 has equation r.  1  9 and a line 1 has equation r   0     1  .
3    
  0  2 

(i) Find the coordinates of P, the point of intersection of 1 and 1 . [4]


Hence, or otherwise, find the shortest distance from point A (3, 0, 0) to 1 .
[2]
The equations of planes 2 and 3 are given as
1  2   2 
     
2 : r  1  s  0   t  2  , and
1  3   1 
     
3 : x y  z   , where  ,   .

(ii) Find the equation of plane 2 in the form r n = d . Explain why the planes
1 and 2 intersect. [4]
(iii) The line of intersection of planes 1 and 2 is  2 . The line  2 has equation
 3 2
   
r =  3   1  .
 2  1 
   
Given that the three planes 1 , 2 and 3 do not have any points in common,
find the conditions satisfied by  and  . [3]

IJC/2010/JC2 9740/01/S/10
1

INNOVA JUNIOR COLLEGE


JC 2 PRELIMINARY EXAMINATION 2
in preparation for General Certificate of Education Advanced Level
Higher 2

CANDIDATE
NAME

Civics Group INDEX NUMBER

Mathematics 9740/02
Paper 2
16 September 2010
Additional materials: Answer Paper
Graph paper 3 hours
List of Formulae (MF15)

READ THESE INSTRUCTIONS FIRST

Do not open this booklet until you are told to do so.

Write your name, class and index number on all the work you hand in.

Write in dark blue or black pen on both sides of the paper. You may use a soft pencil for any
diagrams or graphs.
Do not use staples, paper clips, highlighters, glue or correction fluid.

Answer all the questions.


Give non-exact numerical answers correct to 3 significant figures, or 1 decimal place in the
case of angles in degrees, unless a different level of accuracy is specified in the question.
You are expected to use a graphic calculator.
Unsupported answers from a graphic calculator are allowed unless a question specifically
states otherwise.
Where unsupported answers from a graphic calculator are not allowed in a question, you are
required to present the mathematical steps using mathematical notations and not calculator
commands.

You are reminded of the need for clear presentation in your answers.

The number of marks is given in brackets [ ] at the end of each question or part question.

At the end of the examination, fasten all your work securely together.

This document consists of 6 printed pages.

Innova Junior College [Turn over


2

Section A: Pure Mathematics [40 marks]

1 Three space probes, Probe A, Probe B and Probe C with mass 600 kg, 630 kg and 900 kg
respectively were sent into space to find the gravitational pull on the planets Venus, Mars and
Saturn. The sum of the weights of Probe B on the three planets is 13860 N. The weight of Probe
C on Saturn is 2880 N more than the weight of Probe A on Venus. The average weight of
Probe A on Saturn, Probe B on Venus and Probe C on Mars is 4870 N.
Find the gravitational pull on each of the planets. [3]
Hence find the weight of Probe D, which has mass 500 kg, on planet Saturn. [1]
[Weight (in N) = Mass (in kg)  gravitational pull ms 2 ]  
x2  6 x  1
2 Express f ( x)  in the form
 3x  1  x 2  3
A Bx  C
 2 ,
3x  1 x  3
where A, B and C are constants. [3]
1
Hence find the exact value of
 3
f  x  dx . [4]

3 Given that y  cos 1  x 2  , show that


dy
 sin y   2x  0 .
dx
d3 y
Show that  0 when x  0 . Hence write down the first two non-zero terms in the Maclaurin
dx3
 
series for cos 1 x 2 . [8]

4a  5
4 The curve C has equation y   ax   2a  4   , where a is a constant.
2 x
5
(i) Given that curve C has turning points, show that a   or a  0 . [3]
4
(ii) Sketch curve C for the case when a  1 , indicating clearly the coordinates of the turning
points and the equations of any asymptotes. [4]
(iii) Hence by sketching an appropriate graph on the same diagram, solve

9
x  x6 . [3]
2x

IJC/2010/JC2 9740/02/S/10
3

 
3
3 i
5 (a) The complex number z is given by z  , where p  0 .
1  pi 2
Given that z  2 , find the value of p. [3]
5
Show that arg  z   . [3]
6

(b) Find the exact roots of the equation


z5  2  0
in the form rei , where r  0 and      . [5]

Section B: Statistics [60 marks]

6 A company’s director wants to obtain his employees’ views on flexible working


hours. The company has 600 employees. Describe clearly how you would choose a systematic
random sample of 30 employees. Describe briefly one disadvantage of this sampling method in
this context. [3]

7 Box A contains 6 balls numbered 1, 2, 2, 2, 5, 7.


Box B contains 4 balls numbered 1, 4, 4, 7.
Box C contains 3 balls numbered 3, 4, 6.

One ball is removed at random from each box.


1
(i) Show that the probability that each of the three numbers obtained is greater than 3 is .
6
[1]
(ii) Find the probability that each of the three numbers obtained is greater than 3 or the sum
of the three numbers obtained is 13 (or both). [4]

All the balls are now placed in a single container.


A game is played by a single player. The player takes balls, one by one and with replacement,
from the container, continuing until either a number 1 results or a prime number results. The
player wins if the number on the last ball chosen is 1 and loses otherwise.

Find the probability that a player wins a particular game. [3]

IJC/2010/JC2 9740/02/S/10 [Turn over


4

8 Magnolia cows are milked by hand and Daisy cows are milked by machine. The time taken to
milk a randomly chosen Magnolia cow may be taken to have a normal distribution with mean
30 minutes and standard deviation 2 minutes. The time taken to milk a randomly chosen Daisy
cow may be taken to have an independent normal distribution with mean 5.5 minutes and
standard deviation 0.5 minutes.

(i) The probability that it will take less than a minutes to milk a randomly chosen Magnolia
cow is 0.85. Find a. [1]

(ii) Using an appropriate approximation, find the probability that out of 50 randomly chosen
Magnolia cows, there are more than 10 but at most 40 which take less than a minutes to
milk. [4]

(iii) Find the probability that the total time taken to milk two randomly chosen Magnolia cows
exceeds eleven times the time taken to milk a randomly chosen Daisy cow by at least 3
minutes. [3]

9 An electronic game called ‘Wishful Thinking’ is played with 5 boxes arranged in a row. When a
button is pressed, each of the five boxes displays a picture of a fruit – either an apple, orange or
pear. A possible result of the game is shown below.

Box 1 Box 2 Box 3 Box 4 Box 5

Events ‘Success’ and ‘Windfall’ are defined as follows.

‘Success’ : Exactly 3 boxes display the same fruit.


‘Windfall’ : All 5 boxes display the same fruit.

Find
(i) the total number of possible results, [1]
(ii) the number of ways of obtaining ‘Windfall’, [1]
(iii) the total number of ways of not obtaining ‘Success’. [3]

Ten electronic game machines with distinct serial numbers are sent to 3 different game centres.
In how many ways can the game machines be distributed if each centre must have at least 3
machines? [3]

IJC/2010/JC2 9740/02/S/10
5

10 Observations of a cactus graft were made under controlled environmental conditions. The table
gives the observed heights x cm of the graft at t weeks after grafting.

t 1 2 3 4 5 6 8 10
x 2.0 2.4 2.5 5.1 6.7 9.4 18.3 35.1

(i) Calculate the product moment correlation coefficient between t and x. [1]
(ii) Draw a scatter diagram for the data. [2]
(iii) Using your answer in part (i) and the scatter diagram in part (ii), explain why it is
advisable to draw a scatter diagram first before interpreting the value of the product
moment correlation coefficient. [1]

(iv) Explain why the scatter diagram may be consistent with a model of the form x  eat b . [1]

(v) For the model x  eat b , show that the relation between ln x and t is linear. Hence
calculate the equation of the appropriate regression line. [3]
(vi) Use the regression line in part (v) to predict the height of the cactus graft 20 weeks after
grafting. Hence explain in the context of the question why it is unwise to extrapolate. [2]

11 A company claims that the guitar strings that the company manufactures have a tensile strength
of 430 kpsi (kilo-pounds per square inch) on average. An engineer obtained a sample of 8 guitar
strings and the tensile strength of each guitar string, x kpsi, is measured. The data is summarised
by
  x  430   23,   x  430   211 .
2

Test, at the 2% significance level, whether the company has overstated the average tensile
strength of a guitar string. State any assumptions that you have made. [7]

The engineer will be able to conclude that the company has overstated the average tensile
strength of a guitar string if he conducts the same test at  % significance level. State the
smallest possible integer value of  . [1]

The engineer takes a sample of 20 guitar strings manufactured by a rival company, whose guitar
strings have tensile strength that is normally distributed with mean  kpsi and standard
deviation 4.7 kpsi. The null hypothesis   430 is being tested against the alternative hypothesis
  430 at 5% level of significance. Find the range of values of the sample mean for which the
null hypothesis is rejected, giving 2 decimal places in your answer. [3]

IJC/2010/JC2 9740/02/S/10 [Turn over


6

12 A roller-coaster ride has a safety system to detect faults on the track.


State a condition under which a Poisson distribution would be a suitable probability model for
the number of faults detected on the track on a randomly chosen day. [1]

Faults on the track are detected at an average rate of 0.16 per day. Find the probability that on a
randomly chosen day, the number of faults detected on the track is between 2 and 6 inclusive. [2]

Find the probability that in a randomly chosen period of 20 days, there are not more than 4 faults
detected on the track. [2]

There is a probability of at least 0.15 that the mean number of faults detected on the track per day
over a randomly chosen long period of n days is at least 0.2. Find the greatest value of n. [3]

There is also a separate safety system to detect faults on the roller-coaster train itself. Faults are
detected by this system at an average rate of 0.05 per day, independently of the faults detected on
the track.
Find the probability that in a randomly chosen period of 20 days, the number of faults detected
on the track is at most 1 given that the total number of faults detected is 5. [4]

IJC/2010/JC2 9740/02/S/10
2010 IJC JC 2 PRELIMINARY EXAMINATION 2 Paper 1 (Solutions)

1 1 n
Given V = p
k
dV 1 n −1
= np
dp k
1 ⎛ pn ⎞
= n⎜ ⎟
k ⎝ p ⎠
n⎛1 ⎞
= ⎜ pn ⎟
p⎝k ⎠
nV
=
p
dp
Given n = −2.3 , when V = 32, p = 105, = 0.2
dt
dV dV d p
= ×
dt d p dt
dV nV dV nV dp
Since = , = ×
dp p dt p dt

=
( −2.3)( 32 ) × 0.2
105
= − 0.140 (to 3 s.f.)
Thus, the rate of decrease of volume at the instant is 0.140cm3s −1 .
2 2
= 2 (1 + 3 x )
−n

(1 + 3x )
n

− n ( − n − 1)
Consider (1 + 3 x ) = 1 + ( − n )( 3 x ) + ( 3x ) + ...
−n 2

2!
n ( n + 1)
= 1 − 3nx +
2
( )
9 x 2 + ...

⎛ n ( n + 1) ⎞
2
= 2 ⎜ 1 − 3nx + ( )
9 x 2 + ... ⎟
(1 + 3x )
n
⎝ 2 ⎠
⎛ n ( n + 1) 2 ⎞
= ... + 2 ⎜ 9 x ⎟ + ...
⎝ 2 ⎠
Given: coefficient of x 2 = 108
⇒ 9n ( n + 1) = 108
n 2 + n − 12 = 0
( n + 4 )( n − 3) = 0
+
n = −4 (rejected since n ∈ ) or n = 3
Thus, value of n = 3
3 Let Pn denote the statement un = n + 2−3n , for n ≥ 1 .
When n = 1 ,
9
LHS = u1 = (given)
8
1 9
RHS = 1 + 2−3 = 1 + = = LHS
8 8
Thus P1 is true.
Assume that Pk is true for some k ≥ 1 , i.e.,
u k = k + 2 −3 k

Want to show that Pk +1 is true, i.e.,


uk +1 = k + 1 + 2 ( )
−3 k +1

LHS = uk +1
1
= ( uk + 7 k + 8 )
8
=
1
8
(
k + 2−3k + 7 k + 8 )
1
(
= 8k + 8 + 2−3k
8
)
1 1
= ( 8k + 8 ) + 3 2−3k
8 2
( )
= k + 1 + 2 −3 k − 3
= k + 1 + 2 ( ) = RHS
−3 k +1

Thus Pk true ⇒ Pk +1 is true


Since P(1) is true,
and Pk true ⇒ Pk +1 is true,
by mathematical induction, un = n + 2−3n , for n ≥ 1 .
3i un = n + 2−3n
The sequence does not converge because as n → ∞ , un → ∞ .

∑ ( r + 2−3r )
3ii n n
∑ ur =
r =1 r =1
n n
= ∑ r + ∑ 2−3r
r =1 r =1
n
(
= (1 + n ) + 2−3 + 2−6 + ... + 2−3n
2
)
( )
2−3 ⎛⎜1 − 2−3 ⎞⎟
n

n
= (1 + n ) + ⎝ ⎠

2 1− 2 3
1⎛ 1 ⎞
⎜1 − n ⎟
= (1 + n ) + ⎝ 8 ⎠
n 8
2 1
1−
8
n 1⎛ 1 ⎞
= (1 + n ) + ⎜1 − n ⎟
2 7⎝ 8 ⎠
4a ⎛1⎞
uuur ⎜ ⎟ O
OA = ⎜ α ⎟
⎜2⎟
⎝ ⎠
⎛ 1 ⎞ ⎛ −1⎞ ⎛ 2 ⎞
uuur ⎜ ⎟ ⎜ ⎟ ⎜ ⎟ B
BA = ⎜ α ⎟ − ⎜ 0 ⎟ = ⎜ α ⎟ A
⎜2⎟ ⎜ 1 ⎟ ⎜1⎟
⎝ ⎠ ⎝ ⎠ ⎝ ⎠
uuur uuur uuur uuur
OA • BA = OA BA cos OAB
ˆ

⎛1⎞ ⎛2⎞
⇒ ⎜ ⎟ ⎜ ⎟
⎜ α ⎟ • ⎜ α ⎟ = 1 + α + 4 4 + α + 1 cos OAB
2 2 ˆ
⎜2⎟ ⎜1⎟
⎝ ⎠ ⎝ ⎠

ˆ =α +4
2
⇒ cos OAB
α2 +5
4b Area of quadrilateral OABC C
= 2 × Area of ΔAOC
1 uuur uuur
= 2 × OA × OC
2
uuur uuur
= OA × OC
4

Area of ΔAOB O
1 uuur uuur
= OA × OB M
2 1

A B
1 uuur uuuur
Area of ΔBOC = OB MC
2
1 uuur
( )
uuuur uuuur uuuur
= OB 4 AM since 4AM = MC
2
⎛ 1 uuur uuuur ⎞
= 4 ⎜ OB AM ⎟
⎝2 ⎠
= 4 ( area of AOB )
⎛ 1 uuur uuur ⎞ uuur uuur
= 4 ⎜ OA × OB ⎟ = 2 OA × OB
⎝2 ⎠

Thus, area of quadrilateral OABC


= Area of ΔAOB + Area of ΔBOC
1 uuur uuur uuur uuur
= OA × OB + 2 OA × OB
2
5 uuur uuur
= OA × OB
2
uuur uuur 5 uuur uuur
⇒ OA × OC = OA × OB (Shown)
2

5
Im

(3,3)
3

Re
0 1 2 3

5i

Im

(3,3)
3

2 2

Re
0 1 2 3

The least value of z + 3i = 45 − 2

−3 The greatest value of z + 3i = 45 + 2


5ii

Im

(3,3)
3

2
2
18

1
θ
Re
0 1 2 3

2
sin θ =
18
−1 2
θ = sin = 0.49088
18
π
arg( z ) = − 0.49088
4
= 0.295 rad (to 3 s.f.)

6 Given: w = x2 y
Differentiate w.r.t x
dw dy
= 2 xy + x 2
dx dx
dy
x + 2 y + 3 = 0 ------- (1)
dx
dy
(1) × x : x2 + 2 xy + 3 x 2 y = 0
dx
dw
+ 3w = 0
dx
dw
= −3w (shown)
dx
dw
= −3w
dx
1 1
− ∫ dw = ∫ 1 dx
3 w
1
− ln w = x + c
3
ln w = −3 x − 3c
w = e −3 x − 3 c
w = ±e−3c e−3 x
w = Ae−3 x
x 2 y = Ae−3 x
1
Given that y = − when x = 2 ,
2
⎛ 1⎞
22 ⎜ − ⎟ = Ae−6
⎝ 2⎠
⇒ A = −2e6

Thus, x 2 y = −2e6 e −3 x
2
⇒ y = − 2 e 6 −3 x
x
7i Given: x = 2 cos t y = 2t − 1
dx dy
= −2 sin t =2
dt dt
dy 2 1
∴ = =−
dx −2 sin t sin t

Gradient of tangent at the point P is


1 1 2
− =− =−
π 3 3
sin
3 2
Gradient of normal at the point P is
1 3
− =
⎛ 2 ⎞ 2
⎜− ⎟
⎝ 3⎠
π
When t = ,
3
⇒ ⎛π ⎞ 2π
y = 2⎜ ⎟ −1 = −1
⎝ ⎠
3 3
⎛π ⎞
and x = 2 cos ⎜ ⎟ = 1
⎝3⎠
Hence equation of normal at the point
⎛ 2π ⎞ 3
y −⎜ − 1⎟ = ( x − 1)
⎝ 3 ⎠ 2
3 3 2π
y= x− −1+
2 2 3
y = 0.86603 x + 0.22837
y = 0.866 x + 0.228
ii 3 3 2π
Equation of normal at P: y = x− −1+
2 2 3
To find y-intercept:
3 2π
When x = 0, y = − −1+
2 3
or y = 0.22837
To find x-intercept:
2 ⎛ 3 2π ⎞
When y = 0, x = ⎜⎜ +1− ⎟⎟
3⎝ 2 3 ⎠
or x = −0.26370
To find y-intercept of the curve
π
When x = 0, ⇒ t = , y = π −1
2
1⎛ ⎡ 3 2π ⎤ ⎞ 2 ⎛ 3 2π ⎞
Area of triangle MNQ = ⎜ π − 1 − ⎢ − −1+ ⎥ ⎟⎟ ⎜⎜ +1− ⎟⎟
2 ⎝⎜ ⎣ 2 3 ⎦⎠ 3 ⎝ 2 3 ⎠
= 0.252257
= 0.3 (to 1 d.p.)
8
y 3
y = 4+ x
2

centre ( 0, 4 )

vertex ( −2, 4 ) vertex ( 2, 4 )


0

x
−2 2
3
y = 4− x
2

x
−2 0 2
V = π ∫ x 2 dy − π ( 2 2 ) ( 4 )
4

4⎛ 4 2 ⎞
V = π ∫ ⎜ 4 + ( y − 4 ) ⎟ dy − 16π
0
⎝ 9 ⎠
4
⎡ 4 3⎤
= π ⎢ 4 y + ( y − 4 ) ⎥ − 16π
⎣ 9 ⎦0
⎡ 4 3 ⎛ 4 ⎛ 64 ⎞ ⎞ ⎤
= π ⎢16 + ( 0 ) − ⎜ 0 + ⎜ − ⎟ ⎟ ⎥ − 16π
⎣ 9 ⎝ 9 ⎝ 3 ⎠ ⎠⎦
⎡ 688 ⎤ 256
=π ⎢ ⎥ − 16π = π
⎣ 27 ⎦ 27

9i Number of insects in the container at the end of 1st week =1.04y − 10

Wk Start End
1 y 1.04y -10

2 1.04 y − 10 + y 1.04 (1.04 y − 10 + y ) − 10


= 1.042 y + 1.04 y − 1.04(10) − 10

1.04 (1.042 y + 1.04 y − 1.04(10) − 10 + y ) − 10


3 1.042 y + 1.04 y − 1.04(10) − 10 + y
= 1.043 y + 1.042 y + 1.04 y
−1.042 (10) − 1.04(10) − 10

1.04n y + ..... + 1.043 y + 1.042 y + 1.04 y


n
−10 ⎡⎣1.04n −1 + 1.04n − 2 + ......... + 1.04 + 1⎤⎦

ii The total number of insects in the container are


= 1.04n y + ..... + 1.043 y + 1.042 y + 1.04 y
−10 ⎡⎣1.04n −1 + 1.04n − 2 + ......... + 1.04 + 1⎤⎦

1.04 (1.04n − 1) (1.04 n


− 1)
=y − 10
1.04 − 1 1.04 − 1

= 26 y (1.04n − 1) − 250 (1.04n − 1)

= ( 26 y − 250 ) ⎡⎣1.04n − 1⎤⎦

iii ( 26 y − 250 ) (1.04n − 1) > 13 y − 125


1
1.04n − 1 >
2
3
1.04n >
2
3
n ln1.04 > ln
2
n > 10.3

Therefore the minimum number of complete weeks is 11.


10i Let y = f ( x) = 1 − x , x ≤1
⇒ 1 − x = y2
⇒ x = 1 − y2
⇒ f −1 ( x ) = 1 − x 2
Df −1 = Rf = [ 0, ∞ )
Thus, f −1 : x → 1 − x 2 , x ≥ 0

ii The graph of f −1 is the reflection of the graph of f in the line y = x .


y

x
1 −1
f

11a 2 cos 3 x cos x = cos ( 4 x ) + cos ( 2 x )

1
i
∫ cos 3x cos x dx = 2 ∫ 2 cos 3x cos x dx
1
( cos 4 x + cos 2 x ) dx
2∫
=

1⎛1 1 ⎞
= ⎜ sin 4 x + sin 2 x ⎟ + c
2⎝4 2 ⎠
1 1
= sin 4 x + sin 2 x + c
8 4
π
ii
∫0
4 x cos 3 x cos x dx
π
π
= x ⎡⎢ sin 4 x + sin 2 x ⎤⎥ − ∫ 4 ⎛⎜ sin 4 x + sin 2 x ⎞⎟ dx
1 1 4 1 1
⎣8 4 ⎦0 0
⎝8 4 ⎠
π
⎛ π ⎞ ⎡1 ⎛π ⎞ 1 ⎛ π ⎞⎤ ⎡ 1 1 ⎤4
= ⎜ ⎟ ⎢ sin 4 ⎜ ⎟ + sin 2 ⎜ ⎟ ⎥ − ⎢ − cos 4 x − cos 2 x ⎥
⎝ 4 ⎠ ⎣8 ⎝4⎠ 4 ⎝ 4 ⎠ ⎦ ⎣ 32 8 ⎦0
⎛ π ⎞ ⎡1 1 ⎛ π ⎞⎤ ⎡ 1 ⎛π ⎞ 1 ⎛ π ⎞ ⎛ 1 1 ⎞⎤
= ⎜ ⎟ ⎢ sin π + sin ⎜ ⎟ ⎥ + ⎢ cos 4 ⎜ ⎟ + cos 2 ⎜ ⎟ − ⎜ + ⎟ ⎥
⎝ 4 ⎠ ⎣8 4 ⎝ 2 ⎠ ⎦ ⎣ 32 ⎝4⎠ 8 ⎝ 4 ⎠ ⎝ 32 8 ⎠ ⎦
π ⎡ 1 1 1⎤ π − 3
= + ⎢− − − ⎥ =
16 ⎣ 32 32 8 ⎦ 16
b Method 1: (Translation – Stretching)
3
1: Translation by π in the direction of the x-axis.
2
1
2: Stretching parallel to the x-axis with scale factor of .
2

Method 2: (Stretching − Translation)


1
1: Stretching parallel to the x-axis with scale factor of .
2
3
2: Translation by π in the direction of the x-axis.
4
c Area bounded by the curves
1.0471976 ⎡ ⎛ 3 ⎞ ⎤
=∫ ⎢sin ⎜ 2 x − 2 π ⎟ − cos 3 x cos x ⎥ dx
0
⎣ ⎝ ⎠ ⎦
= 0.32476

= 0.325 units2 (to 3 sf)

12i ⎛2⎞
π1 : ⎜ ⎟
r . ⎜ −1⎟ = 9
%⎜ ⎟
⎝3⎠
⎛ 3⎞ ⎛ 4⎞
⎜ ⎟ ⎜ ⎟
l1 : r = ⎜0⎟ + λ⎜ 1 ⎟
% ⎜ ⎟ ⎜ −2 ⎟
⎝0⎠ ⎝ ⎠
For point of intersection,
⎡⎛ 3 ⎞ ⎛ 4 ⎞⎤ ⎛ 2 ⎞
⎢⎜ ⎟ ⎜ ⎟⎥ ⎜ ⎟
⎢ ⎜ 0 ⎟ + λ ⎜ 1 ⎟ ⎥ . ⎜ −1 ⎟ = 9
⎜ ⎟ ⎜ −2 ⎟ ⎥ ⎜ 3 ⎟
⎣⎢⎝ 0 ⎠ ⎝ ⎠⎦ ⎝ ⎠
⎛ 3⎞ ⎛ 2 ⎞ ⎛ 4⎞ ⎛2⎞
⎜ ⎟ ⎜ ⎟ ⎜ ⎟ ⎜ ⎟
⇒ ⎜ 0 ⎟ . ⎜ −1 ⎟ + λ ⎜ 1 ⎟ . ⎜ − 1 ⎟ = 9
⎜ ⎟ ⎜ ⎟ ⎜ ⎟ ⎜ ⎟
⎝0⎠ ⎝ 3 ⎠ ⎝ −2 ⎠ ⎝ 3 ⎠
9−6
⇒ λ= =3
8 −1− 6
⎛ 3 ⎞ ⎛ 4 ⎞ ⎛ 15 ⎞
uuur ⎜ ⎟ ⎜ ⎟ ⎜ ⎟
⇒ OP = ⎜ 0 ⎟ + 3 ⎜ 1 ⎟ = ⎜ 3 ⎟
⎜ 0 ⎟ ⎜ −2 ⎟ ⎜ − 6 ⎟
⎝ ⎠ ⎝ ⎠ ⎝ ⎠
Thus, coordinates of the P are (15, 3, − 6 ) .

⎛ 15 ⎞ ⎛ 3 ⎞ ⎛ 12 ⎞
uuur ⎜ ⎟ ⎜ ⎟ ⎜ ⎟
AP = ⎜ 3 ⎟ − ⎜ 0 ⎟ = ⎜ 3 ⎟ l1
⎜ −6 ⎟ ⎜ 0 ⎟ ⎜ −6 ⎟
⎝ ⎠ ⎝ ⎠ ⎝ ⎠
A (3, 0, 0)
Shortest distance from A to π1
⎛2⎞
⎛2⎞ ⎜ ⎟
uuur ⎜ ⎟ ⎜ −1⎟
⎜3⎟
AP. ⎜ −1⎟ ⎝ ⎠
⎜3⎟
= ⎝ ⎠ P (15, 3, −6) π1
⎛2⎞
⎜ ⎟
⎜ −1 ⎟
⎜3⎟
⎝ ⎠
⎛ 12 ⎞ ⎛ 2 ⎞
⎜ ⎟ ⎜ ⎟
⎜ 3 ⎟ . ⎜ −1⎟
⎜ −6 ⎟ ⎜ 3 ⎟ 24 − 3 − 18
⎝ ⎠ ⎝ ⎠ 3 3 14
= = = = =
4 +1+ 9 14 14 14
Alternative solution for shortest distance:

Line AN: l1

⎛ 3⎞ ⎛2⎞
⎜ ⎟ ⎜ ⎟ A (3, 0, 0)
r = ⎜ 0 ⎟ + λ ⎜ −1 ⎟ ⎛2⎞
% ⎜ ⎟ ⎜3⎟ ⎜ ⎟
⎝0⎠ ⎝ ⎠ ⎜ −1 ⎟
⎜3⎟
⎝ ⎠

N is the point of intersection P N π1


of line AN and plane π1 .
⎡⎛ 3 ⎞ ⎛ 2 ⎞⎤ ⎛ 2 ⎞
⎢⎜ ⎟ ⎜ ⎟⎥ ⎜ ⎟
⎢ ⎜ 0 ⎟ + λ ⎜ −1 ⎟ ⎥ . ⎜ − 1 ⎟ = 9
⎢⎣⎜⎝ 0 ⎟⎠ ⎜ 3 ⎟⎥ ⎜ 3 ⎟
⎝ ⎠⎦ ⎝ ⎠
⎛ 3⎞ ⎛ 2 ⎞ ⎛2⎞ ⎛2⎞
⎜ ⎟ ⎜ ⎟ ⎜ ⎟ ⎜ ⎟
⇒ ⎜ 0 ⎟ . ⎜ −1 ⎟ + λ ⎜ − 1 ⎟ . ⎜ − 1 ⎟ = 9
⎜0⎟ ⎜ 3 ⎟ ⎜3⎟ ⎜3⎟
⎝ ⎠ ⎝ ⎠ ⎝ ⎠ ⎝ ⎠
⇒ 6 + (4 + 1 + 9)λ = 9
3
⇒ λ=
14
⎛ 3⎞ ⎛2⎞ ⎛ 48 ⎞
uuur ⎜ ⎟ 3 ⎜ ⎟ 1 ⎜ ⎟
Thus, ON = ⎜ 0 ⎟ + ⎜ −1⎟ = ⎜ −3 ⎟
⎜ 0 ⎟ 14 ⎜ 3 ⎟ 14 ⎜ 9 ⎟
⎝ ⎠ ⎝ ⎠ ⎝ ⎠
⎛ 48 ⎞ ⎛ 3 ⎞ ⎛2⎞
uuur 1 ⎜ ⎟ ⎜ ⎟ 3 ⎜ ⎟
AN = ⎜ −3 ⎟ − ⎜ 0 ⎟ = ⎜ −1⎟
14 ⎜ ⎟ ⎜ ⎟ 14 ⎜ ⎟
⎝ 9 ⎠ ⎝ 0⎠ ⎝3⎠
Thus, shortest distance AN
uuur 3 3 14
= AN = 4 +1+ 9 =
14 14
ii ⎛2⎞
⎜ ⎟
π1 : r . ⎜ −1⎟ = 9
%⎜ ⎟
⎝3⎠
⎛ 1⎞ ⎛ 2 ⎞ ⎛ 2 ⎞
π2 : r = ⎜⎜1⎟⎟ + s ⎜⎜ 0 ⎟⎟ + t ⎜⎜ 2 ⎟⎟
% ⎜ ⎟ ⎜ ⎟ ⎜ ⎟
⎝1⎠ ⎝ −3 ⎠ ⎝ 1 ⎠
⎛ 2 ⎞ ⎛ 2⎞ ⎛ 6 ⎞ ⎛ 3⎞
A normal to plane π2 = ⎜ 0 ⎟ × ⎜ 2 ⎟ = ⎜ −(8) ⎟ = 2 ⎜⎜ −4 ⎟⎟
⎜ ⎟ ⎜ ⎟ ⎜ ⎟
⎜ −3 ⎟ ⎜ 1 ⎟ ⎜ 4 ⎟ ⎜ 2⎟
⎝ ⎠ ⎝ ⎠ ⎝ ⎠ ⎝ ⎠
⎛ 3 ⎞ ⎛ 1⎞ ⎛ 3 ⎞
⎜ ⎟ ⎜ ⎟ ⎜ ⎟
π2 : r . ⎜ −4 ⎟ = ⎜1⎟ . ⎜ −4 ⎟ = 3 − 4 + 2 = 1
%⎜ ⎟ ⎜ ⎟ ⎜ ⎟
⎝ 2 ⎠ ⎝ 1⎠ ⎝ 2 ⎠
⎛ 3⎞
⇒ ⎜ ⎟
r . ⎜ −4 ⎟ = 1
% ⎜ ⎟
⎝ 2⎠
⎛ 3⎞ ⎛2⎞
Since ⎜ 4 ⎟ ≠ k ⎜⎜ −1⎟⎟ for any k ∈ ,
⎜ ⎟
⎜ 2⎟ ⎜3⎟
⎝ ⎠ ⎝ ⎠
the normal of planes π1 and π2 are not parallel to each other
⇒ The planes are not parallel to each other.
⇒ The planes will intersect in a line.
iii ⎛ 3⎞ ⎛2⎞
⎜ ⎟ ⎜ ⎟
l2 : r = ⎜ 3⎟ + μ⎜ 1 ⎟
⎛2⎞
⎜ 2⎟ ⎜ −1⎟ ⎜ ⎟
⎝ ⎠ ⎝ ⎠ ⎜1⎟
⎜ −1 ⎟
π3 : αx + y − z = β ⎝ ⎠ l2
⎛α⎞ (3, 3, 2)
⎛α⎞
⎜ ⎟ ⎜ ⎟
⇒ r. ⎜ 1 ⎟ = β ⎜1⎟
⎜ −1 ⎟
⎜ −1 ⎟ ⎝ ⎠
⎝ ⎠ π3
π1 , π2 and π3 do not have any points in common
⇒ l 2 is parallel to π3 and does not lie on π3
⎛α⎞ ⎛ 3⎞ ⎛ α ⎞
⇒ l 2 is perpendicular to ⎜ 1 ⎟ and ⎜⎜ 3 ⎟⎟ . ⎜⎜ 1 ⎟⎟ ≠ β
⎜ ⎟
⎜ −1 ⎟ ⎜ 2 ⎟ ⎜ −1 ⎟
⎝ ⎠ ⎝ ⎠ ⎝ ⎠
⎛2⎞ ⎛α⎞
Thus, ⎜⎜ 1 ⎟⎟ . ⎜⎜ 1 ⎟⎟ = 0
⎜ −1⎟ ⎜ −1⎟
⎝ ⎠ ⎝ ⎠
⇒ 2 α + 1 + 1 = 0 ⇒ α = −1
⎛ 3⎞ ⎛ α ⎞
and ⎜⎜ 3 ⎟⎟ . ⎜⎜ 1 ⎟⎟ ≠ β ⇒ 3(−1) + 3 − 2 ≠ β
⎜ 2 ⎟ ⎜ −1⎟
⎝ ⎠ ⎝ ⎠
⇒ β ≠ −2
2010 IJC JC 2 PRELIMINARY EXAMINATION 2 Paper 2 (Solutions)

1 Let V ms −2 , M ms −2 , S ms −2 be the gravitational pull on each planet Venus, Mars


and Saturn respectively.
630S + 630V + 630M = 13860 ---------- (1)
900S − 600V = 2880 ---------- (2)
600S + 630V + 900M = 3(4870) = 14610 --------- (3)
⎛ 630 630 630 ⎞ ⎛ S ⎞ ⎛ 13860 ⎞
⎜ ⎟⎜ ⎟ ⎜ ⎟
⎜ 900 −600 0 ⎟ ⎜ V ⎟ = ⎜ 2880 ⎟
⎜ 600 630 900 ⎟ ⎜ M ⎟ ⎜ 14610 ⎟
⎝ ⎠⎝ ⎠ ⎝ ⎠
From GC,
M = 3.8, V = 9, S = 9.2
Hence the weight of Probe D on Saturn is
( 500 × 9.2 ) N = 4600 N
2 x2 − 6x + 1
f ( x) =
(
( 3x + 1) x 2 + 3 )
A Bx + C
= + 2
3x + 1 x + 3

=
( )
A x 2 + 3 + ( Bx + C )( 3 x + 1)
( 3x + 1) ( x 2 + 3)
x 2 − 6 x + 1 = Ax 2 + 3 A + 3Bx 2 + 3Cx + Bx + C
= ( A + 3B ) x 2 + ( 3C + B ) x + 3 A + C

Comparing coefficients,
A + 3B =1
B + 3C = −6
3A +C=1
From GC, A = 1, B = 0, C = −2
x2 − 6x + 1 1 2
∴ f ( x) = = − 2
( 3x + 1) x + 3 ( 3x + 1) x + 3
2
( ) ( )
−1 −1
⎡ 1 2 ⎤
∫−3
f ( x ) dx = ∫−3 ⎢⎣ 3 x + 1 − x 2 + 3 ⎥⎦ dx
−1
⎡1 2 x ⎤
= ⎢ ln 3 x + 1 − tan −1 ⎥
⎣3 3 3 ⎦ −3
1 2 ⎛ 1 ⎞ ⎡1 2 ⎛ 3 ⎞⎤
= ln 3 ( −1) + 1 − tan −1 ⎜ − ⎟ − ⎢ ln 3 ( −3) + 1 − tan −1 ⎜ − ⎟⎥
3 3 ⎝ 3 ⎠ ⎣3 3 ⎝ 3 ⎠⎦
1 2 ⎛ π ⎞ ⎡1 2 ⎛ π ⎞⎤
= ln 2 − ⎜ − ⎟ − ⎢ ln 8 + ⎜ ⎟⎥
3 3 ⎝ 6 ⎠ ⎣3 3 ⎝ 3 ⎠⎦
1 1 2 ⎛ π ⎞ 2 ⎛π ⎞
= ln − ⎜− ⎟− ⎜ ⎟
3 4 3⎝ 6⎠ 3⎝3⎠
1 1 π
= ln −
3 4 3 3
3 y = cos −1 x 2( )
cos y = x 2

dy
− ( sin y ) = 2 x
dx
dy
( sin y ) + 2 x = 0 (shown)
dx

2
d2 y ⎛ dy ⎞
( sin y ) + cos y ⎜ ⎟ = −2
dx 2
⎝ dx ⎠

( sin y ) 3 + ( cos y ) ⎛⎜ ⎞⎟ 2
d3 y dy d 2 y
dx ⎝ dx ⎠ dx
⎛ ⎛ dy ⎞ d 2 y ⎞ ⎛ dy ⎞⎛ dy ⎞
2
+ cos y ⎜⎜ 2 ⎜ ⎟ 2 ⎟⎟ − ( sin y ) ⎜ ⎟⎜ ⎟ = 0
⎝ ⎝ dx ⎠ dx ⎠ ⎝ dx ⎠⎝ dx ⎠
3
d3 y ⎛ dy ⎞ d y ⎛ dy ⎞
2
( sin y ) + 3 ( cos y ) ⎜ ⎟ 2 − sin y ⎜ ⎟ = 0
dx 3
⎝ dx ⎠ dx ⎝ dx ⎠

When x = 0 ,
π
y = cos −1 0 =
2
dy
=0
dx
d2 y
= −2
dx 2
d3 y
=0
dx3
4 4a + 5
y = − ax − ( 2a + 4 ) +
2− x
dy (4a + 5)(−1)
⇒ = −a −
dx ( 2 − x )2
dy
For turning points, =0
dx
4a + 5
⇒ −a + =0
( 2 − x )2
a ( 2 − x ) = 4a + 5
2

⇒ ( )
a 4 − 4 x + x 2 − 4a − 5 = 0
⇒ ax 2 − 4ax − 5 = 0
Since there are turning points, there are two distinct real roots.
Thus, D > 0
⇒ ( −4a )2 − 4(a)(−5) > 0
⇒ 4 a 2 + 5a > 0
⇒ a ( 4a + 5 ) > 0

5 0

4

5
Thus, a < − or a > 0 . (Shown)
4
Alternatively
dy
For turning points, =0
dx
4a + 5
⇒ −a + =0
( 2 − x )2
4a + 5
⇒ ( 2 − x )2 =
a
Since there are turning points, there are two distinct real roots.
4a + 5 −
Thus, >0 + +
a 5
− 0
4

5
Thus, a < − or a > 0 . (Shown)
4
4(ii) 9
When a = 1 , y = − x − 6 +
2− x
y x=2

9
y = −x − 6 +
2− x

−1 5 x
−6 −2

−6

−14
y = −x − 6
4(iii) 9
x + x+6>
2− x
9
⇒ x > −x − 6 +
2− x
y x=2
y= x

9
y = −x − 6 +
2− x

−1 5 x
−6 −2

−6

−14
y = −x − 6

From GC, the curves intersect at x = 0.823 (to 3 s.f)

From graph, solution is x > 2 or x < 0.823 .

5(a)
( )
3 3
3 −i 3 −i
z = =
(1 + pi ) 1 + pi
2 2

( )
3
3 +1
=
( )
2
1 + p2
8
=
1 + p2

z =2
8
⇒ =2
1 + p2

4 = 1 + p2
p = 3 or − 3 (rejected)
(
⎛ 3 −i 3 ⎞
arg z = arg ⎜⎜ ⎟ )
2 ⎟
⎜ (1 + pi ) ⎟
⎝ ⎠
( ) ( )
3 2
= arg 3 −i − arg 1 + 3i
= 3arg ( )
3 − i − 2 arg 1 + 3i ( )
⎛ π⎞ ⎛π ⎞ 7π
= 3⎜ − ⎟ − 2 ⎜ ⎟ = −
⎝ 6⎠ ⎝3⎠ 6
For −π < arg( z ) ≤ π , 5π

7π 5π
6
arg( z ) = 2π − = (Shown)
6 6 7π

6

5(b) z5 = − 2
= 2eπ i
= 2e(
π + 2 kπ )i
, k = 0, ±1, ±2

By De Moivre’s Theorem
1 ⎛ π + 2 kπ ⎞
⎜ ⎟i
z = 210 e⎝ 5 ⎠
, k = 0, ±1, ±2

1 ⎛π ⎞ 1 ⎛ 3π ⎞ 1 1 ⎛ π⎞ 1 ⎛ 3π ⎞
⎜ ⎟i ⎜ ⎟i ⎜ − ⎟i ⎜ − ⎟i
, 210 e( ) , 210 e⎝ 5 ⎠ , 210 e⎝ 5 ⎠
π i
z = 210 e⎝ 5 ⎠ , 210 e⎝ 5 ⎠

1 π 1 3π 1 1 π 1 3π
i i − i − i
z = 210 e 5 , 210 e 5 , − 210 , 210 e 5 , 210 e 5
6 - Use a list with all the employees’ names arranged in alphabetical order and
number them from 1 to 600
600
- Determine the sampling interval : k = = 20
30
- Randomly select the first person from the first 20
people on the list, then select every 20th person subsequently until a sample of 30
employees is obtained.

The sample obtained might be over-represented by a particular department (or


gender or ethnic group) of the company which has the greatest proportion of
employees, hence the systematic random sample obtained is not a good
representative of the population.

7(i) P(all are greater than 3)


2 3 2 1
= . . =
6 4 3 6
7(ii) Let X be the event : “each of the 3 numbers is greater than 3”
Y be the event : “sum of the 3 numbers is equal to 13”

P(Y) = P((2,7,4), (5,4,4))


3 1 1 1 2 1
= . . + . .
6 4 3 6 4 3
5
=
72

Required probability = P( X ∪ Y )
= P( X ) + P(Y ) − P( X ∩ Y )
1 5 1 2 1
= + − . .
6 72 6 4 3
5
=
24
P(a player wins a particular game)
2
2 4 2 ⎛4⎞ 2
= + . + ⎜ ⎟ . + ......
13 13 13 ⎝ 13 ⎠ 13
2⎛ ⎞ 2 ⎛ 1 ⎞
2
4 ⎛4⎞
= ⎜⎜ 1 + + ⎜ ⎟ + ...... ⎟⎟ = .⎜
⎜ 1 − 4 ⎟⎟
13 ⎝ 13 ⎝ 13 ⎠ ⎠ 13 ⎝ 13 ⎠
2
=
9
8 Let M minutes be the time taken to milk a Magnolia cow,
and D minutes be the time taken to milk a Daisy cow.

( ) (
M ~ N 30, 22 , D ~ N 5.5, 0.52 )
8(i) P ( M < a ) = 0.85
From GC, a = 32.073 = 32.1 (3 s.f.)

8(ii) Let X be the number of cows which take less than a minutes to milk, out of 50
Magnolia cows.
X ~ B ( 50, 0.85 )
Since n = 50 is large, np = 42.5 > 5, nq = 7.5 > 5 ,

∴ X ~ N ( 42.5, 6.375 ) approx.

Reqd prob = P (10 < X ≤ 40 )


= P (10.5 < X ≤ 40.5 ) (using c.c.)
= 0.21415
= 0.214 (3 s.f.)
8(iii) E ( M1 + M 2 − 11D ) = 2 × 30 − 11× 5.5 = −0.5
Var ( M1 + M 2 − 11D ) = 2 × 22 + 112 × 0.52 = 38.25
M1 + M 2 − 11D ~ N ( − 0.5, 38.25 )
Reqd prob = P ( M 1 + M 2 − 11D ≥ 3)
= 0.28573
= 0.286 (3 s.f.)
9(i) Total number of possible results = 35 = 243
9(ii) Number of ways of obtaining ‘Windfall’ = 3
9(iii) No. of ways to obtain a success
⎛ 5! 5 !⎞
=⎜ × 2 + ⎟ × 3 = 120
⎝ 3!2 ! 3! ⎠
Total number of ways = 243 − 120 = 123
9 Reqd no. of ways
3!
= 10
C3 × 7C3 × 4C4 ×
= 12600
2!
10(i) From GC, r = 0.92378 = 0.924 (3 s.f.)
10(ii) x
35.1

2.0 t
1 10

10(iii) From (i) and (ii), we see that though r is close to +1, the scatter diagram indicates a
curvilinear relationship between t and x, instead of a positive linear relationship.
Thus it is advisable to draw a scatter diagram first before interpreting the value of the
product moment correlation coefficient.

10(iv) As t increases, x increases but by increasing amounts. Thus the scatter diagram may
be consistent with a model of the form x = e at +b .

10(v) x = e at +b
ln x = at + b , where a and b are constants
Thus the relation between ln x and t is linear.

Find regression line of ln x on t.

t 1 2 3 4 5 6 8 10
ln x 0.69315 0.87547 0.91629 1.6292 1.9021 2.2407 2.9069 3.5582

From GC, eqn of regression line of ln x on t is:


ln x = 0.20723 + 0.33498t
ln x = 0.207 + 0.335t (3 s.f.)
10(vi) When t = 20, ln x = 0.20723 + ( 0.33498 )( 20 ) = 6.90683
x = 999

Height of the cactus graft 20 weeks after grafting is 999 cm.

It is impossible that the cactus can grow to that height after 20 weeks. Thus it is
unwise to extrapolate.
11 The t-test will be used.
Assume a normal distribution for tensile strength of guitar strings.

1⎡ ( −23) ⎤
2
−23
s = ⎢ 211 −
2
⎥ x= + 430
7 ⎢⎣ 8 ⎥⎦ 8
= 20.696 = 427.125

H 0 : μ = 430
H1 : μ < 430
Level of significance = 2%

X − 430
Under H 0 : T= ~ t7
S n

From GC, t = −1.79


p-value = 0.0585 > 0.02

Do not reject H 0 and conclude that there is insignificant evidence at the 2%


significance level that the manufacturer has overstated its claim.

11 α =6

11 σ = 4.7

H 0 : μ = 430
H1 : μ ≠ 430
Level of significance = 5%

X − 430
Under H 0 : Z= ~ N (0,1)
4.7 20
To reject H 0 ,
x − 430 x − 430
< −1.96 or > 1.96
4.7 20 4.7 20

x < 427.94 or x > 432.06

12 Suitable condition
• The mean number of faults detected is a constant from day to day.
• The faults are detected independently of one another.
Let X be the number of faults detected on the track in a day.
X ~ Po ( 0.16 )
P ( 2 ≤ X ≤ 6 ) = P ( X ≤ 6 ) − P ( X ≤ 1)
= 0.011513 = 0.0115 (3 s.f.)
Let Y be the number of faults detected on the track in 20 days.
Y ~ Po ( 3.2 )
P (Y ≤ 4 ) = 0.78061 = 0.781 (3 s.f.)
X1 + X 2 + L X n
X=
n
⎛ 0.16 ⎞
Since n is large, by Central Limit Theorem, X ~ N ⎜ 0.16, ⎟ approx.
⎝ n ⎠
⎛ ⎞
⎜ 0.2 − 0.16 ⎟
P ( X ≥ 0.2 ) ≥ 0.15 ⇒ P ⎜ Z ≥ ⎟ ≥ 0.15
⎜ 0.16 ⎟
⎜ ⎟
⎝ n ⎠
(
⇒ P Z ≤ 0.1 n ≤ 0.85 )
From GC, 0.1 n ≤ 1.0364
⇒ n ≤ 107.41
Greatest value of n is 107.
12 Let W be the number of faults detected on the train in 20 days.
W ~ Po (1)

Y + W ~ Po ( 4.2 )

P (Y = 0 ) P (W = 5 ) + P (Y = 1) P (W = 4 )
P (Y ≤ 1| Y + W = 5 ) =
P (Y + W = 5 )

0.00212437
=
0.163316
= 0.0130 (3 s.f.)
JURONG JUNIOR COLLEGE
J2 Preliminary Examination

MATHEMATICS 9740/01
Higher 2 20 August 2010

Paper 1 3 hours

Additional materials: Answer Paper


Graph paper
List of Formulae (MF15)
Cover Page

READ THESE INSTRUCTIONS FIRST

Write your name and civics class on all the work you hand in.
Write in dark blue or black pen on both sides of the paper.
You may use a soft pencil for any diagrams or graphs.
Do not use staples, paper clips, highlighters, glue or correction fluid.

Answer all the questions.


Give non-exact numerical answers correct to 3 significant figures, or 1 decimal place in the case of angles in
degrees, unless a different level of accuracy is specified in the question.
You are expected to use a graphic calculator.
Unsupported answers from a graphic calculator are allowed unless a question specifically states otherwise.
Where unsupported answers from a graphic calculator are not allowed in a question, you are required to
present the mathematical steps using mathematical notations and not calculator commands.
You are reminded of the need for clear presentation in your answers.

The number of marks is given in brackets [ ] at the end of each question or part question.
At the end of the examination, fasten all your work securely together, with the cover page in front.

This document consists of 6 printed pages.

[Turn over
2

1 A Bx + C
1 (i) Express in the form + , where A, B, and C are constants to be
1− x 3
1 − x 1 + x + x2
found. [2]
x+2
(ii) Hence find the coefficient of x r in the expansion of , in ascending powers
1 + x + x2
of x. [4]

 k   k + 1  k  +
2 (i) Show that=    −   , where k ∈  , k ≥ 3 . [2]
 2  3   3
n
r
(ii) Hence find ∑   . [4]
r =3  2 

3 A curve is defined by the parametric equations

=x 3(sin 2t − cos 2t ), =y 3(2t − cos 2t ).

π
Find the equations of the tangent and the normal to the curve at the point P, where t = .
4
The tangent and normal to the curve at P meet the y-axis at R and S respectively.
Find the area of the triangle PRS. [6]

4 A sequence u1 , u2 , u3 ,  is such that u1 = 0 and

un +1 =3un + 4n ( n − 1) , for all n ∈  + .

(i) Prove by induction that un =3n − 2n 2 − 1 for all n ∈  + . [4]

2N
(ii) Given u0 = 0 , find ∑ un in terms of N . [4]
n =0
 n 

1
Given r 2 = n ( n + 1)( 2n + 1) 
 r =1
6 
3

5 Given that y = 1 + ln(1 + x) , show that


2
d 2 y  dy  1
(i) y 2 +  + =
0. [2]
dx  dx  2(1 + x) 2

(ii) the Maclaurin’s series for y in ascending powers of x, up to and including the
term in x3 , is
1 3 17
1+ x − x 2 + x3 . [3]
2 8 48
2
 1 3 17 
(iii) Expand 1 + x − x 2 + x 3  in powers of x up to and including x3 , simplifying
 2 8 48 
your answer. Explain briefly how the result can be used as a check on the correctness
of the first four terms in the series for y. [3]

6 (a) A geometric progression has first term 1 and common ratio r. The sum of the first four
terms is less than the sum to infinity of the remaining terms. Without the use of a
graphic calculator, find the range of values of r. [4]

(b) Adam decided to save some money each day to buy his favorite toys. On the first day,
he saved one twenty-cent coin; on the second day, he saved two twenty-cent coins;
and so on. Find the total amount of money saved at the end of one year.
(Assume 365 days in a year). [3]
After spending all his savings in the first year to buy his favorite toys, Adam started
saving in the same manner again in the following year. As an encouragement, his
mother contributed the same amount that he saved every Saturday and Sunday.
Assuming the second year started on a Monday, find the total amount of money saved
at the end of the second year. [3]

7 The line l passes through the points A and B with coordinates ( −1, 2, 3) and
( 5, 14, 11) respectively. The plane p has equation 2x + 3y − 6z =
−7.

(i) Show that the line l is parallel but not contained on the plane p. [3]

(ii) Find the distance of the line l from the plane p. [3]

(iii) Find a cartesian equation of the plane which contains l and is perpendicular to p. [2]

[Turn over
4

8 (a) The complex numbers p and q are such that

p= 2 + ia , q= b − i ,
where a and b are real numbers.

Given that pq= 13 + 13i , find the possible values of a and b . [4]

(b) Sketch the locus of z which satisfies

=
z − 4 − 3i 2 and Re(z) ≥ 4 . [2]

(i) Find the least and greatest value of z . [2]


π
(ii) Show that the greatest value of arg( z − i) is . [1]
4

x
9 The function f is defined by f:x  for x ∈  , − λ < x < 0, where λ is a
λ − x2
positive constant.

(i) Show by differentiation that f ( x ) increases as x increases. [2]

(ii) Find f −1 ( x ) , stating the domain of f −1. [4]

 1 1
(iii) Find the value of λ such that f −1f −1  −  =
− . [3]
 2 2

x −1
10 (i) Find the integral ∫ 1+ 4x 2
dx. [3]

(ii) By sketching the graphs of y = 3e x and y= x + 3, or otherwise, solve the inequality


3e x > x + 3. [3]
Hence find
∫−2 3e − x − 3 dx,
2
x

giving your answer in an exact form. [4]


5

11 (a) y
y = ( ln x )
2

x
O A

The function f is defined by f(x) = (ln x)2 for x > 0. The diagram shows a sketch of the graph of
y = f(x). The minimum point of the graph is A.
(i) State the x-coordinate of A. [1]
(ii) Use the substitution x = eu to show that the area of the region bounded by the
x-axis, the line x = e and the curve is given by
1 2 u
∫ 0
u e du.

Hence, find the exact value of this area. [5]

(b) y

3
y=
− x+2
2

y =1
R

x
O
y = cos x
3
Find the volume of the solid formed when the region R, bounded by the lines y = − x+ 2,
2
y = 1 and the curve y = cos x, is rotated 2π radians about the y-axis, giving your answer
correct to 3 decimal places. [4]

[Turn over
6

12 The curve C has equation


ax − k 2
y= 2 ,
x − kx + k
where a, k are constants and a > 0 .

(i) Find the values of k such that C has two asymptotes. [2]

5x − k 2
(ii) The diagram shows the graph of y = for some k > 0.
x 2 − kx + k
y

O x
k2
−k 5

On separate diagrams, draw sketches of the graphs of


25 x − k 2
(a) y= ,
25 x 2 − 5kx + k

k 2 − 5x
(b) y2 = ,
x 2 − kx + k

x 2 − kx + k
(c) y= ,
5x − k 2

including the coordinates of the points where the graphs cross the axes and the
equations of any asymptotes. [8]
JURONG JUNIOR COLLEGE
J2 Preliminary Examination

MATHEMATICS 9740/02
Higher 2 26 August 2010

Paper 2 3 hours

Additional materials: Answer Paper


Graph paper
List of Formulae (MF15)
Cover Page

READ THESE INSTRUCTIONS FIRST

Write your name and civics class on all the work you hand in.
Write in dark blue or black pen on both sides of the paper.
You may use a soft pencil for any diagrams or graphs.
Do not use staples, paper clips, highlighters, glue or correction fluid.

Answer all the questions.


Give non-exact numerical answers correct to 3 significant figures, or 1 decimal place in the case of angles in
degrees, unless a different level of accuracy is specified in the question.
You are expected to use a graphic calculator.
Unsupported answers from a graphic calculator are allowed unless a question specifically states otherwise.
Where unsupported answers from a graphic calculator are not allowed in a question, you are required to
present the mathematical steps using mathematical notations and not calculator commands.
You are reminded of the need for clear presentation in your answers.

The number of marks is given in brackets [ ] at the end of each question or part question.
At the end of the examination, fasten all your work securely together, with the cover page in front.

This document consists of 6 printed pages.

[Turn over
2

Section A: Pure Mathematics [40 marks]

1 (i)

O x P
The diagram shows a rectangle inscribed in a semicircle of centre, O, and fixed radius a.
The length OP is denoted by x. Show that, as x varies, the perimeter of the rectangle is a
maximum when its sides are in the ratio 4 : 1. [6]

(ii) Variables x and y are related by the equation


2
y 2 + xy = x 2 − + 3 , where y > 0 .
x

1
Given that x is increasing at the rate of units s −1 , find the rate of increase of y when x is 1.
5
[4]

2 3
2 A particular solution of a differential equation is given by ( x + y ) 2 = 2 xy − y . Show that
3
dy
( y 2 + y) =
−x [2]
dx

A second, related, family of curves is given by the differential equation

dy
x = y2 + y
dx

By means of the substitution y = ux , show that the general solution for y, in terms of x, is
−x
y= ,
x+c

where c is an arbitrary constant. [3]

Sketch, on a single diagram, three distinct members of the second family of solution curves,
stating clearly the coordinates of the points where the curves cross the axes and the equations of
any asymptotes. [5]
3

3 Relative to the origin O, the position vectors of A, B and C are

α i + 3j + 4k, 2i – 2k and 4i + β j – k respectively.

(i) Point M lies on the line segment AB such that AM : MB = 1: 2 . Given that the position
vector of M is 2j + 2k, find α . [2]

 3
(ii) Given that the length of projection of BC onto the line OM is units, find β , where β is
2
a positive constant. [3]

 2 a
   
=
(iii) The line l has vector equation r  0  + µ  0  , µ ∈  , a ∈  . Using the values of α and β
 −2  1
   
π
found above, determine the value of a if l makes an angle of radians with the plane ABC.
6
[4]

π
4 (i) Given that z 2 = 2 and arg( −iz ) = , find w in the form a + bi , where a, b ∈  , if
4
z 
2
5
wz = 2 2 and arg   = − π . [4]
 w 6
 

(ii) Solve the equation


z 4 + 1 − 3 i =0 ,

giving the roots in the form re iθ where r > 0 and − π < θ ≤ π. [3]

Show the roots on an Argand diagram. [2]

Describe the geometrical shape formed by the points representing the roots and justify your
answer. [2]

[Turn over
4

Section B: Statistics [60 marks]

5 From the letters of the word DISTRIBUTION, find

(i) the number of 4-letter code-words that can be formed if the code-word contains exactly
three ‘I’s. [2]

(ii) the number of code-words that can be formed using all the letters such that all the three ‘I’s
are separated. [2]

6 In a badminton team of 8 players, 5 are boys and 3 are girls. Boy A and Girl B are the only 2
left-handed players in the team. In a particular practice, 4 players are chosen to play doubles.
Find the probability that
(i) exactly 1 left-handed player is chosen, [2]

(ii) 2 girls are chosen given that exactly 1 left-handed player is chosen, [3]

(iii) either Boy A or Girl B is chosen (or both). [2]

7 The number of guitars sold by a music shop per day follows a Poisson distribution with mean λ .
It is known that on 2 in 7 days, there are no guitars sold. Show that λ = 1.253 , correct to 3
decimal places. [2]

(i) Calculate the probability that less than 4 guitars are sold in a day. [2]

(ii) Using a suitable approximation, find the probability that, in a random sample of 100 days,
there will be more than 95 days in which less than 4 guitars are sold per day. [4]

(iii) Calculate the probability that in a period of 90 days, the mean number of guitars sold per
day is more than 1.5. [3]
5

8 The random variable X has a normal distribution with mean 15 and variance 5. The random
variable T is the sum of 2 independent observations of X.

(i) Find P (T > 2 + 3 X ) . [3]

(ii) Three independent observations of X are obtained. Find the probability that exactly two of
the observations have value less than 20. [3]

The random variable Y has a normal distribution with mean µ and variance σ 2 .
(iii) If σ = 22.5 , find the greatest probability of P(15.1 < Y < 29.9) , stating the value of µ . [2]

0.25 , calculate the value of σ and state an assumption


(iv) If µ = 10 and P( X + Y > 27) =
needed to carry out the calculation. [4]

9 A sample of 60 customers is to be chosen to take part in a survey conducted by a restaurant


owner.

(i) Explain briefly how the restaurant owner could use quota sampling. [1]

(ii) The purpose of the survey is to investigate customers’ opinions about the different lunch
and dinner menus.

Give a reason why a stratified sample might be preferable in this context. [1]

Explain clearly how the restaurant owner could use stratified sampling from his list of
regular customers if the ratio of regular customers for lunch and dinner is 2 : 3. [2]

[Turn over
6

10 Water in a reservoir undergoes a purification process before it can be consumed. The


effectiveness, y %, of the process for various flow rates, x m3 s-1, is shown below.

x 1 2 4 6 8 10 20 30 40
y 80 60 45 40 30 25 18 15 10

The variables x and y are thought to be related by the equation e y = axb , where a and b are
constants.

(i) Give a sketch of the scatter diagram of y against ln x . Comment on whether a linear model
would be appropriate referring to the scatter diagram. [2]

(ii) Find the value of the product moment correlation coefficient between y and ln x and explain
whether it supports your comment in part (i). [2]

(iii) Find the least squares regression line of y on ln x and estimate the values of a and b . [3]

(iv) Predict the effectiveness of the process when water flows at 0.5 m3s-1. Comment on the
reliability of your prediction. [2]
(v) Explain why in this context, the above model would not be appropriate for large values of x .
[1]

11 The past records of a supermarket show that the mean amount spent per customer was $59 with
standard deviation $8. The supermarket’s management suspects that the mean amount spent per
customer has decreased. A random sample of 8 customers was taken and the amount spent per
customer , $x, was recorded. The following result was obtained.
∑ x = 432
Stating a necessary assumption about the population, test the supermarket’s management’s
suspicion at the 5% significance level. [6]

To encourage customers to spend more at the supermarket, the management initiated a


promotional campaign whereby each customer will receive a voucher which can be used to
redeem products at the supermarket. A week after the start of the campaign, the manager of the
supermarket took a sample of 9 customers and the amount spent per customer, $y, is
summarised by
∑ ( y − 70) =
−72, ∑ ( y − 70) =
2
1234 .

The actual mean amount spent per customer is $ µ . In a test at the 5% level of significance, the
hypotheses are:

Null hypothesis : µ = µ0
Alternative hypothesis : µ ≠ µ0 .

Given that the null hypothesis is rejected in favour of the alternative hypothesis, find the set of
possible values of µ0 . [6]
2010 JJC H2 Mathematics Prelim Exam P1 Solutions
1(i) 1 A Bx + C
Let ≡ +
1 − x 1 − x 1 + x + x2
3

Hence, 1 ≡ A (1 + x + x 2 ) + ( Bx + C )(1 − x )
1
Using cover-up rule, A =
3
Let x = 0 , 1= A+C ,
2
C=
3
1
Comparing coefficients of x , 0 = A + B − C , B =
3
1 1 x+2
∴ ≡ + .
1− x 3
3 (1 − x ) 3 (1 + x + x 2 )
1(ii) x+2 3 1
= −
1+ x + x 2
1− x 1− x
3

= 3 (1 − x 3 ) − (1 − x )
−1 −1

= 3 (1 + x 3 + x 6 + K) − (1 + x + x 2 + K)
= 2 − x − x 2 + 2 x3 − x 4 − x5 + K
⎧2 if r is a multiple of 3
Hence, coefficients of x r = ⎨
⎩ −1 otherwise

2. n
⎛r⎞ ⎛ n ⎞ ⎛ n − 1⎞ ⎛ n − 2 ⎞ ⎛ 4⎞ ⎛ 3⎞
∑⎜ 2⎟ = ⎜ 2⎟ + ⎜ ⎟+⎜ ⎟ +K + ⎜ ⎟ + ⎜ ⎟
r =3 ⎝ ⎠ ⎝ ⎠ ⎝ 2 ⎠ ⎝ 2 ⎠ ⎝ 2⎠ ⎝ 2⎠
⎛ n + 1⎞ ⎛ n ⎞
=⎜ ⎟−⎜ ⎟
⎝ 3 ⎠ ⎝ 3⎠
⎛ n ⎞ ⎛ n − 1⎞
+⎜ ⎟−⎜ ⎟
⎝3⎠ ⎝ 3 ⎠
⎛ n − 1⎞ ⎛ n − 2 ⎞
+⎜ ⎟−⎜ ⎟
⎝ 3 ⎠ ⎝ 3 ⎠
M
⎛5⎞ ⎛ 4⎞
+⎜ ⎟−⎜ ⎟
⎝ 3⎠ ⎝ 3 ⎠
⎛ 4 ⎞ ⎛ 3⎞
+⎜ ⎟−⎜ ⎟
⎝ 3 ⎠ ⎝ 3⎠
⎛ n + 1⎞ (n + 1)!
=⎜ ⎟ −1 or −1
⎝ 3 ⎠ 3!(n − 2)!

1
3. x = 3(sin 2t − cos 2t ) , y = 3(2t − cos 2t )
dx
= 3(2 cos 2t + 2sin 2t )
dt
dy
= 3(2 + 2sin 2t )
dt
dy 3(2 + 2sin 2t ) 1 + sin 2t
= =
dx 3(2 cos 2t + 2sin 2t ) cos 2t + sin 2t
π 3π dy
When t = , at point P , x = 3 , y = and =2
4 2 dx

Equation of tangent at P is y − = 2( x − 3)
2
3π 3π
When x = 0, y = − 6 . R is (0 , −6 )
2 2
3π −1
Equation of normal at P is y − = ( x − 3)
2 2
3π 3 3π 3
When x = 0, y = + . S is (0 , + )
2 2 2 2
As PR ⊥ PS,
1 1 3 1
Area of the triangle PRS = RS .(3) = ( + 6)(3) = 11
2 2 2 4

4. (i) Let Pn be the statement un = 3n − 2n 2 − 1 for all n ≥ 1 .

When n = 1, LHS = u1 = 0 (By definition)


RHS = 3 2 −2 ( 2 ) − 1 = 0
2

∴ P1 is true.

+
Assume that Pk is true for some k ∈ , k ≥ 1.
That is, uk = 3k − 2k 2 − 1 -----------------------(1)
We want to prove Pk +1 , ie uk +1 = 3k +1 − 2 ( k + 1) − 1.
2

LHS = 3uk + 4 ( k )( k − 1)
= 3 ( 3k − 2k 2 − 1) + 4k ( k − 1)
= 3k +1 − 6k 2 − 3 + 4k 2 − 4k
= 3k +1 − 2 ( k 2 + 2k + 1) − 1
= 3k +1 − 2 ( k + 1) − 1
2

∴ Pk is true ⇒ Pk +1 is true.

Since P1 is true, and Pk is true ⇒ Pk +1 is true. By Mathematical


Induction, un = 3n − 2n 2 − 1 is true for all n ≥ 1 .

2
4.
∑ un = ∑ (3n − 2n2 − 1)
2N 2N
(ii)
n=0 n =0

=
(
1 32 N +1 − 1) − 2 2N ( 2N + 1) ⎡2 ( 2N ) + 1⎤ − ( 2N + 1)
2 6 ⎣ ⎦

32 N +1 − 1 2 N + 1
= − ⎡ 2 N ( 4 N + 1) + 3⎤⎦
2 3 ⎣
32 N +1 − 1 ( 2 N + 1)
=
2

3
(
8N 2 + 2 N + 3 )
5(i) Method 1
y 2 = 1 + ln(1 + x)
dy 1
2y =
dx 1 + x
dy 1
=
dx 2 y (1 + x)
dy 1
y =
dx 2(1 + x)
Method 2
− ⎛ 1 ⎞
1
dy 1
= (1 + ln(1 + x)) 2 ⎜ ⎟
dx 2 ⎝ 1+ x ⎠
1
=
2 y (1 + x)
dy 1
y =
dx 2(1 + x)

−1
2
d 2 y ⎛ dy ⎞ 1
y 2 + ⎜ ⎟ = (−1)(1 + x) −2 =
dx ⎝ dx ⎠ 2 2(1 + x) 2
2
d 2 y ⎛ dy ⎞ 1
y +⎜ ⎟ + =0
dx ⎝ dx ⎠ 2(1 + x) 2
2

5(ii) d 3 y dy d 2 y dy d 2 y 1
y 3 + . 2 +2 − =0
dx dx dx dx d x 2
(1 + x)3

when x = 0,
dy 1 d 2 y 3 d 3 y 17
y = 1, = , 2 = − , 3 =
dx 2 dx 4 dx 8

⎛ 3 ⎞ ⎛ x ⎞ ⎛ 17 ⎞ ⎛ x ⎞
2 3
1
∴ y ≈ 1+ x + ⎜ − ⎟⎜ ⎟ + ⎜ ⎟⎜ ⎟
2 ⎝ 4 ⎠ ⎝ 2! ⎠ ⎝ 8 ⎠ ⎝ 3! ⎠
1 3 17
= 1+ x − x 2 + x3
2 8 48

3
5(iii) ⎛ 1 3 2 17 3 ⎞
2

⎜1 + x − x + x ⎟
⎝ 2 8 48 ⎠
⎛ 1 3 17 ⎞ ⎛ 1 3 17 3 ⎞
= ⎜1 + x − x 2 + x3 ⎟ ⎜1 + x − x 2 + x ⎟
⎝ 2 8 48 ⎠ ⎝ 2 8 48 ⎠
1 1
= 1 + x − x 2 + x 3 + ...
2 3
Given y = 1 + ln(1 + x) ⇒ y 2 = 1 + ln(1 + x)
1 1
LHS = y 2 = 1 + x − x 2 + x3 + ...
2 3
RHS = 1 + ln(1 + x)
⎛ 1 1 ⎞
= 1 + ⎜ x − x 2 + x 3 + ... ⎟
⎝ 2 3 ⎠
Since LHS = RHS, the first four terms in the series for y is correct.
6(a) S 4 < S∞ − S 4
2S 4 < S∞
2 (1 − r 4 ) 1
<
1− r 1− r
2 (1 − r ) < 1
4
(Since 1 − r > 0 )
1
r4 >
2
⎛ 2 1 ⎞⎛ 2 1⎞
⎜⎜ r − ⎟⎟ ⎜⎜ r + ⎟>0
⎝ 2 ⎠⎝ 2 ⎟⎠
1 1
r2 < − (no solution) or r2 >
2 2
⎛ 4 ⎞⎛
1
4 ⎞
1
⎛ 1 ⎞ ⎛ 1 ⎞
⎜ r − ⎜ ⎟ ⎟⎜ r + ⎜ ⎟ ⎟ > 0
⎜ ⎝ 2 ⎠ ⎟⎜ ⎝ 2 ⎠ ⎟
⎝ ⎠⎝ ⎠
1 1
⎛ 1 ⎞4 ⎛ 1 ⎞4
−1 < r < − ⎜ ⎟ or ⎜ ⎟ < r < 1 .
⎝2⎠ ⎝2⎠
6(b) Total sum = 20 + 2 ( 20 ) + 3 ( 20 ) +K 365 ( 20 )
⎛ ( 365 )(1 + 365 ) ⎞
= ( 20 ) ⎜ ⎟
⎝ 2 ⎠
= 1335900 cents

Total sum = 1335900 + 6 ( 20 ) + 7 ( 20 ) + 13 ( 20 ) + 14 ( 20 ) + ... + 363 ( 20 ) + 364 ( 20 )


(
= 1335900 + 20 13 + (13 + 14 ) + (13 + 2 (14 ) ) + K (13 + 51(14 ) ) )
⎛ 52 ⎞
= 1335900 + 20 ⎜ ⎟ (13 + 727 )
⎝ 2 ⎠
= 1720700 cents

4
7(i) ⎛ 5 ⎞ ⎛ −1⎞
uuur ⎜ ⎟ ⎜ ⎟
AB = ⎜14 ⎟ − ⎜ 2 ⎟
⎜ 11 ⎟ ⎜ 3 ⎟
⎝ ⎠ ⎝ ⎠
⎛6⎞ ⎛ 3⎞
⎜ ⎟ ⎜ ⎟
= ⎜12 ⎟ = 2 ⎜ 6 ⎟
⎜8⎟ ⎜ 4⎟
⎝ ⎠ ⎝ ⎠
⎛ 3⎞ ⎛ 2 ⎞
⎜ ⎟ ⎜ ⎟
⎜ 6 ⎟ • ⎜ 3 ⎟ = 6 + 18 − 24
⎜ 4 ⎟ ⎜ −6 ⎟
⎝ ⎠ ⎝ ⎠
=0
⇒ l is parallel to p.

⎛ −1⎞ ⎛ 2 ⎞
⎜ ⎟ ⎜ ⎟
⎜ 2 ⎟ • ⎜ 3 ⎟ = −2 + 6 − 18
⎜ 3 ⎟ ⎜ −6 ⎟
⎝ ⎠ ⎝ ⎠
= −14 ≠ −7
∴ l is parallel but not contained on the plane p.
7(ii) Method 1
A
l

π
C
Let C(c,0,0) be a point on π .
⎛c⎞ ⎛ 2 ⎞
⎜ ⎟ ⎜ ⎟
⎜ 0 ⎟ • ⎜ 3 ⎟ = −7
⎜ 0 ⎟ ⎜ −6 ⎟
⎝ ⎠ ⎝ ⎠
7
c=−
2
⎛ 2⎞
⎜ ⎟
3
⎛ ⎛ −7 / 2 ⎞ ⎛ −1⎞ ⎞ ⎜⎜ ⎟⎟
⎜⎜ ⎟ ⎜ ⎟ ⎟ −6
d = ⎜⎜ 0 ⎟ − ⎜ 2 ⎟⎟• ⎝ ⎠
⎜⎜ 0 ⎟ ⎜ 3 ⎟⎟ 49
⎝⎝ ⎠ ⎝ ⎠⎠

⎛ −5 / 2 ⎞ ⎛ 2 ⎞
1⎜ ⎟ ⎜ ⎟
= ⎜ −2 ⎟ • ⎜ 3 ⎟ = 1
7⎜ ⎟ ⎜ ⎟
⎝ −3 ⎠ ⎝ −6 ⎠

5
Method 2
Let m be the line perpendicular to p and passing through A.
⎛ −1⎞ ⎛ 2⎞
Vector equation of line m : r = ⎜ 2 ⎟ + μ ⎜⎜ 3 ⎟⎟ , μ ∈
⎜ ⎟
⎜3⎟ ⎜ −6 ⎟
⎝ ⎠ ⎝ ⎠
Let F be the foot of perpendicular of A to p.
⎛ −1 + 2μ ⎞ ⎛ 2 ⎞
⎜ ⎟ ⎜ ⎟
⎜ 2 + 3μ ⎟ • ⎜ 3 ⎟ = −7
⎜ 3 − 6μ ⎟ ⎜ −6 ⎟
⎝ ⎠ ⎝ ⎠
2 ( −1 + 2μ ) + 3(2 + 3μ ) − 6(3 − 6μ ) = −7
1
μ=
7
⎛ −1⎞ ⎛ 2⎞
uuur ⎜ ⎟ 1 ⎜ ⎟
OF = ⎜ 2 ⎟ + ⎜ 3 ⎟
⎜ 3 ⎟ 7 ⎜ −6 ⎟
⎝ ⎠ ⎝ ⎠
⎛ −1⎞ ⎛ 2 ⎞ ⎛ −1 ⎞
uuur ⎜ ⎟ 1 ⎜ ⎟ ⎜ ⎟
AF = ⎜ 2 ⎟ + ⎜ 3 ⎟ − ⎜ 2 ⎟
⎜ 3 ⎟ 7 ⎜ −6 ⎟ ⎜ 3 ⎟
⎝ ⎠ ⎝ ⎠ ⎝ ⎠
⎛ 2⎞
1⎜ ⎟
= ⎜ 3⎟
7⎜ ⎟
⎝ −6 ⎠
uuur 1 2 2
AF = 2 + 3 + (−6) 2
7
=1

∴ Distance of the line l from the plane p=1 unit


7(ii)
Let the plane required be p1 .

⎛ 3 ⎞ ⎛ 2 ⎞ ⎛ −48 ⎞
⎜ ⎟ ⎜ ⎟ ⎜ ⎟
Normal of p1 = ⎜ 6 ⎟ × ⎜ 3 ⎟ = ⎜ 26 ⎟
⎜ ⎟ ⎜ ⎟ ⎜ ⎟
⎝ 4 ⎠ ⎝ −6 ⎠ ⎝ −3 ⎠
⎛ −1⎞ ⎛ −48 ⎞
⎜ ⎟ ⎜ ⎟
⎜ 2 ⎟ • ⎜ 26 ⎟ = 48 + 52 − 9 = 91
⎜ 3 ⎟ ⎜ −3 ⎟
⎝ ⎠ ⎝ ⎠
⎛ −48 ⎞
⎜ ⎟
∴ Equation of p1 : r • ⎜ 26 ⎟ = 91 ⇒ −48 x + 26 y − 3z = 91
⎜ −3 ⎟
⎝ ⎠

6
8(a) pq = 13 + 13i
( 2 + ia )( b − i ) = 13 + 13i
2b − 2i + abi + a = 13 + 13i
(a + 2b) + i (ab − 2) = 13 + 13i

Comparing real and imaginary parts,


a + 2b = 13 - (1)
ab − 2 = 13 - (2)
15
(2): b =
a
15
Subst. b = into (1):
a
⎛ 15 ⎞
a + 2 ⎜ ⎟ = 13
⎝a⎠
a 2 − 13a + 30 = 0
( a − 3)( a − 10 ) = 0
a = 3 or 10
3
b = 5 or
2
8b(i)
y
Locus of z
R (4,5)
Q

(4, 3)
3

A(0, 1) P (4, 1)

x
O 4

Least z = OP Greatest z = OQ

= 42 + 12 = 42 + 32 + 2
= 17 =7
8b(ii) Max arg( z − i ) = ∠PAR
⎛4⎞
= tan −1 ⎜ ⎟
⎝4⎠
π
=
4
7
9(i) x
f ( x) =
λ − x2
⎛1⎞
λ − x2 − x ⎜ ⎟ ( λ − x2 )
−1
2
( −2 x ) λ
f ' ( x) = ⎝2⎠ = >0
λ − x2 (λ − x ) 2 32

f is strictly increasing, thus f(x) increases as x increases.

9(ii) domain of f −1 = range of f = ( −∞, 0 ) .


x
y =
λ − x2
y 2 ( λ − x2 ) = x2
x 2 ( y 2 + 1) = λ y2
λ y2
x = ±
1+ y2
λy
Since − λ < x < 0, that is, y < 0, x = .
1+ y2
λx
That is, f −1 ( x) = , for x < 0.
1 + x2
9(iii) λx
f −1f −1 ( x) =
1 + ( λ + 1) x 2

λ ( −0.5 )
= −0.5
1 + ( λ + 1)( −0.5 )
2

4λ 2 − λ − 5 = 0
( 4λ − 5)( λ + 1) = 0
5
Since λ > 0, λ = .
4

Alternatively:
f ( −0.5 ) = −0.5
( −0.5) = 0.5
λ − ( −0.5 )
2

1 = λ − 0.25

5
λ=
4

8
10(i) x −1
∫ 1 + 4x 2
dx

1 8x 1
= ∫
8 1 + 4x 2
dx −∫
1 + 4 x2
dx

= ln (1 + 4x2 ) − tan −1 (2x) + c


1 1
8 2
(ii) y y = x+3

y = 3e x
x
-2.82 O

From the graphs, x < −2.82 or x > 0 .

For x < −2.82 or x > 0 , 3e x > x + 3


i.e. 3e x − x − 3 > 0 if x < −2.82 or x > 0

For −2.82 < x < 0 , 3e x < x + 3


i.e. 3e x − x − 3 < 0 if −2.82 < x < 0

∫−2 3e − x − 3 dx
2 x

= ∫−2 3e x − x − 3 dx + ∫0 3e x − x − 3 dx
0 2

x2 x2
= [3e x − − 3 x]0−2 + [3e x − − 3 x]02
2 2
= −1 − 3e−2 + 3e2 − 11 (need to check positive or negative within each modulus to
remove the modulus sign)
= 1 + 3e−2 + 3e2 − 11
= 3e−2 + 3e2 − 10

9
11a(i) x-coordinate of A = 1
(ii) e
Area of region bounded = ∫1 (ln x) 2 dx
dx
Let x = eu ⇒ ln x = u, = eu
du
When x = 1, u = 0 & When x = e, u = 1
1
∴ Area of region bounded = ∫0 u
2 u
e du
1
∫0 u
2 u
Area of region bounded = e du

= [u 2 eu ] 0 − ∫ 0 2ueu du
1 1

1 1
= e − [ ⎡ 2ueu ⎤ − ∫ 2eu du ]
⎣ ⎦0 0
1
= e – 2e + ⎡ 2eu ⎤
⎣ ⎦0
= − e + 2e − 2 = e − 2
11(b) 3
Let P be the point of intersection of y = − x + 2 and y = cos x .
2
Using GC, the coordinates of point of intersection = (0.94031, 0.58954)
Volume formed about the y –axis
1 2
⎡2 ⎤

−1
=π ⎢⎣ 3 (2 − y )⎥⎦ − (cos y ) dy
2

0.58954
= π (0.0907) = 0.285 (3 d.p)

12(i) Horizontal asymptote is y = 0 .


Discriminant of x 2 − kx + k = 0 ⇒ k 2 − 4k = 0 ⇒ k ( k − 4 ) = 0 .
Therefore when k = 0 or 4, C has two asymptotes.

(ii)(a)
y

x
O k 2 25
−k

10
(ii)(b)
y

x
O k2 5

− k

(c)

O k2 5
−1 k
1 k2 k
y = x+ −
5 25 5

11
2010 JJC Prelim P2 Solutions

1(i)

a
y

Let the length and the width of the rectangle be 2x and y

The perimeter S = 2y + 4x ……(1)


y 2 = a 2 − x 2 …………………(2)

Subst (2) into (1)


S = 2 a2 − x2 + 4 x
dS 2x
=− +4=0
dx a2 − x2
2x
− +4=0
a − x2
2

x
=2
a2 − x2

4 2
x2 = a
5
1
⇒ y2 = a 2
5
4 1
⇒ x= a and y = a
5 5
d 2s 2a 2
= − 3 < 0
dx 2 (a 2 − x 2 ) 2

Length : width = 2x : y = 4:1

1
1(ii) 2
y 2 + xy = x 2 − + 3 ……(1)
x
Differentiate w.r.t x
dy dy 2
2 y + x + y = 2x + 2
dx dx x
dy 2
(2 y + x) = 2 x + 2 − y …….(2)
dx x
Subst. x = 1 into
(1): y 2 + y = 2 ⇒ y= 1 and y ≠ − 2 (rejected)
dy
(2): =1
dx
dy dy dx
= .
dt dx dt
1
= (1)( )
5
1
= units s −1
5
2 2
( x + y ) 2 = 2 xy − y 3
3
2
( x + y ) 2 − 2 xy = − y 3
3
2
y 2 + x2 = − y3
3
dy dy
2 y + 2 x = −2 y 2
dx dx
dy
( y + y 2 ) = − x (shown)
dx
Alternative:
⎛ dy ⎞ dy dy
2( x + y ) ⎜ 1 + ⎟ = 2 x + 2 y − 2 y 2
⎝ dx ⎠ dx dx
dy
( y + y 2 ) = − x (shown)
dx

2
2 dy
x = y2 + y
dx
use y = ux
dy d u
= x+u
dx dx
⎛ du ⎞
∴ x ⎜ x + u ⎟ = ( ux ) + ux
2

⎝ dx ⎠
du
x + u = u2 x + u
dx
1 du
=1
u 2 dx
1
∫ u 2 du = ∫ 1 dx
1
− = x + c , where c is an arbitrary constant
u
x
− = x+c
y
x c
y=− = −1 +
x+c x+c
2 Family of solution curves:

1
c = −1, y = −1 −
x −1
c = 0, y = −1
c =1
1
c = 1, y = −1 +
x +1
y
c = −1
x
y = −1 c=0

x = −1 x =1

3
3(i) Using ratio theorem,
uuur uuur
uuuur 2OA + OB 1 2
OM = A M C
3
uuuur uuur
uuur 3OM − OB
OA =
2
⎡ ⎛ 0 ⎞ ⎛ 2 ⎞⎤
1 ⎢ ⎜ ⎟ ⎜ ⎟⎥
= ⎢3 ⎜ 2 ⎟ − ⎜ 0 ⎟ ⎥
2 ⎜ ⎟ ⎜ ⎟
⎢⎣ ⎝ 2 ⎠ ⎝ −2 ⎠ ⎥⎦
⎛ −1 ⎞
⎜ ⎟
=⎜ 3 ⎟
⎜4⎟
⎝ ⎠
α = −1
3(ii) ⎛4⎞ ⎛ 2⎞
uuur ⎜ ⎟ ⎜ ⎟
BC = ⎜ β ⎟ − ⎜ 0 ⎟
⎜ −1⎟ ⎜ −2 ⎟
⎝ ⎠ ⎝ ⎠
⎛2⎞
⎜ ⎟
= ⎜β ⎟
⎜1⎟
⎝ ⎠
uuur 3
Length of projection of BC onto the line OM =
2
uuur uuuur
BC • OM 3
uuuur =
OM 2

⎛ 2 ⎞ ⎛0⎞
⎜ ⎟ ⎜ ⎟
⎜ β ⎟•⎜1⎟
⎜ 1 ⎟ ⎜1⎟
⎝ ⎠ ⎝ ⎠ 3
=
12 + 12 2
β +1 = 3
β 2 + 2β − 8 = 0
β = 2 or − 4 (rejected)
∴β = 2
Alternatively, β + 1 = 3
β + 1 = 3 or β + 1 = −3
β = 2 or − 4 (rejected)
∴β = 2

4
3(iii) ⎛ 2 ⎞ ⎛ −1⎞ ⎛1⎞
uuur ⎜ ⎟ ⎜ ⎟ ⎜ ⎟
AB = ⎜ 0 ⎟ − ⎜ 3 ⎟ = 3 ⎜ −1 ⎟
⎜ −2 ⎟ ⎜ 4 ⎟ ⎜ −2 ⎟
⎝ ⎠ ⎝ ⎠ ⎝ ⎠
⎛ 4 ⎞ ⎛ −1⎞ ⎛ 5 ⎞
uuur ⎜ ⎟ ⎜ ⎟ ⎜ ⎟
AC = ⎜ 2 ⎟ − ⎜ 3 ⎟ = ⎜ −1 ⎟
⎜ −1⎟ ⎜ 4 ⎟ ⎜ −5 ⎟
⎝ ⎠ ⎝ ⎠ ⎝ ⎠
⎛1⎞ ⎛5⎞
⎜ ⎟ ⎜ ⎟
Normal of plane ABC = ⎜ −1 ⎟ × ⎜ −1 ⎟
⎜ −2 ⎟ ⎜ −5 ⎟
⎝ ⎠ ⎝ ⎠
⎛ 3⎞
⎜ ⎟
= ⎜ −5 ⎟
⎜ 4⎟
⎝ ⎠
⎛a⎞ ⎛ 3 ⎞ ⎛a⎞ ⎛ 3⎞
⎜ ⎟ ⎜ ⎟ ⎜ ⎟ ⎜ ⎟ π
⎜ 0 ⎟ • ⎜ −5 ⎟ = ⎜0⎟ −5
⎜ ⎟ sin
6
⎜1⎟ ⎜ 4 ⎟ ⎜1⎟ ⎜ 4⎟
⎝ ⎠ ⎝ ⎠ ⎝ ⎠ ⎝ ⎠
⎛1⎞
3a + 4 = a 2 + 1 50 ⎜ ⎟
⎝2⎠
( 6a + 8 )2 = 50 ( a 2 + 1)
7 a 2 − 48a − 7 = 0
1
a = 7 or - (rejected since angle is acute, 3a +4>0)
7
∴a = 7

5
4(i) z2 = 2 ⇒ z = 2
π π
arg ( −iz ) = ⇒ arg ( −i ) + arg( z ) =
4 4
π⎛ π⎞
arg( z ) = −⎜− ⎟
4 ⎝ 2⎠

=
4

wz = 2 2
w z =2 2
∴w =2

⎛ z2 ⎞ 5
arg ⎜ ⎟ = − π
⎜ w⎟ 6
⎝ ⎠
5
2 arg( z ) − arg( w) = − π
6
⎛3 ⎞ 5
arg( w) = 2 ⎜ π ⎟ + π
⎝4 ⎠ 6
7
= π
3
π
= (principle value)
3
⎡ ⎛π ⎞ ⎛ π ⎞⎤
w = 2 ⎢cos ⎜ ⎟ + i sin ⎜ ⎟ ⎥
⎣ ⎝3⎠ ⎝ 3 ⎠⎦
= 1 + 3i

6
4(ii)
4
z + 1 − 3i = 0
4
z = −1 + 3i
⎛ 2π ⎞
⎜ + 2 kπ ⎟ i
= 2e⎝ 3 ⎠

1 ⎛ π + kπ ⎞ i
⎜ ⎟
z = 2 4 e⎝ 6 2 ⎠
1 π
(1+ 3k )i
= 24 e 6 , k = 0, ±1, −2
1 π 1 2π 1 π 1 5π
i i − i − i
= 24 e 6 , 24 e 3 , 24 e 3 , 24 e 6

z3
z2
π
6 x
π

3
z4

z1

The points form a square since the diagonals are perpendicular and of equal
length.

7
5(i) The different letters left are D S T R B U O N
4!
No of different code words = 8C1 = 32
3!

5(ii) 9!
To permute the remaining 9 letters 1st, ways.
2
To slot in the 3Is, 10C3 ways.
Thus number of code words that can be formed is
9!
= 10C3
2
= 21772800
6(i) 2
C 6C 4
P(1 left-handed player) = 81 3 =
C4 7
6(ii) P(G R G L BR BR )+P(G R G R BL BR )
P(2 G given exactly 1 left-handed)=
P(exactly 1 left-handed)
2
C1 4 C2 + 2 C2 4 C1
8
C4
=
4
7
8
2
= 35 =
4 5
7
6(iii) P(Boy A or Girl B is chosen) = 1 – P (Both Boy A and Girl B are not chosen)
6
C 11
= 1− 8 4 =
C4 14

Alternative:
P(Boy A or Girl B is chosen) = P(Boy A is chosen) + P(Girl B is chosen)
−P(Boy A and Girl B are chosen)
7 7
C C 6C
= 8 3+8 3−8 2
C4 C4 C 4
1 1 3 11
= + − =
2 2 14 14

8
7 Let X denote the number of guitars sold by a music shop in a day.
X P0 ( λ )
2
Given that P( X = 0) =
7
2
⇒ e− λ =
7
2
⇒ λ = − ln = 1.253
7
7(i) P ( X < 4) = P( X ≤ 3)
= 0.96145 ≈ 0.961
7(ii) Let Y be the number of days out of 100 in which at least 4 guitars were sold per day.
Y B(100,1 − 0.96145) = B(100, 0.03855)
Since n is large and np < 5 , we use a Poisson approximation.
Y P0 (3.855) approx
P(more than 95 days in which less than 4 guitars were sold per day)
= P( at most 4 days in which at least 4 guitars were sold per day )
= P(Y ≤ 4)
= 0.657 (to 3 sf)

7(iii) X P0 (1.253)
For a large sample of size 90, by Central Limit theorem,
⎛ 1.253 ⎞
X N ⎜ 1.253, ⎟ approx .
⎝ 90 ⎠
(
P X > 1.5 ).
= 0.0182

Alternative : X 1 + K + X 90 P0 (1.253 × 90 ) = P0 (112.77 )

( )
P X > 1.5 = P ( X 1 + K + X 90 > 135 )
= 1 − P ( X 1 + K + X 90 ≤ 135 )
= 0.0183

8(i) X N (15,5)
Let T = X 1 + X 2
E (T − 3 X ) = E ( X 1 + X 2 ) − 3E ( X ) = 15 + 15 − 3(15) = −15
Var (T − 3 X ) = Var ( X 1 + X 2 ) + 9Var ( X ) = 5 + 5 + 32 (5) = 55
T − 3 X N (−15,55)
P (T > 2 + 3 X ) = P(T − 3 X > 2)
= 0.0109

9
8(ii) P ( X < 20) = 0.9873 ≈ 0.987
Probability = 3[ P( X < 20) ] P( X > 20)
2

= 3(0.9873) 2 (1 − 0.9873)
= 0.0371
Alternatively, use binomial distribution.
Let W be r.v. “no. of observations with value less than 20 out of 3”
W B(3, 0.987)
P (W = 2) = 0.0371
8(iii) Y N ( μ , 22.52 )
15.1 + 29.9 45
For greatest probability, μ = = = 22.5
2 2
Greatest P(15.1 < Y < 29.9) = 0.258
8(iv) (iv) X N (15,5) Y N (10, σ 2 )
X + Y N (15 + 10, 5 + σ 2 ) = N (25, 5 + σ 2 )
P ( X + Y > 27) = 0.25
⎛ 27 − 25 ⎞
P⎜Z > ⎟ = 0.25
⎝ 5 +σ 2 ⎠
⎛ 2 ⎞
P⎜Z > ⎟ = 0.25
⎝ 5 +σ 2 ⎠
⎛ 2 ⎞
P⎜Z < ⎟ = 0.75
⎝ 5 +σ 2 ⎠
2
= 0.6745
5 +σ 2
σ = 1.95

Assumption : The random variables X and Y are independent of each other.

10
9(i) To obtain a quota sampling of 60, divide the diners into two subgroups : male and
female. Select the first 30 males and 30 females who leaves the restaurant.
Or any other relevant answers.
9(ii) By drawing random samples according to the proportion in each stratum, lunch and
dinner, the sample will be a better representation of the population.

Draw random samples from each stratum with sample size proportional to the size of the
strata as follows :
Lunch Dinner
Number of 2 3
customers to be × 60 = 24 × 60 = 36
5 5
sampled
10(i) e y = axb
y = ln a + b ln x
The scatter diagram is plotted with y against ln x .
y

ln x

From the scatter diagram, the points lie close to a straight line, so the linear model is
appropriate.
10(ii) From GC, since r ≈ −0.982 which is very close to -1, it supports the claim in part (i).

10(iii) y = 73.3 − 18.4 ln x


a = 6.89 × 1031 (3sf )

b = −18.4 (3sf)
10(iv)x = 0.5, y = 73.3 − 18.4 ln(0.5) = 86.0
86.0% (3sf)
Since x = 0.5 is out of the given data range of 1 ≤ x ≤ 40 , the prediction is unreliable.
10(v) For large values of x , the model gives y < 0 . So the model is not valid for large values
of x .

11
11 Assume that the amount spent per customer follows a normal distribution.
Let X be the actual amount spent per customer per visit.
and μ be the actual mean amount spent per customer per visit.
H 0 : μ = 59
H1 : μ < 59
82 X −μ
Under H 0 , X ~ N(59, ) and test statistic Z = ~ N(0, 1)
8 σ
n
432
where x = = 54, μ = 59, σ = 8, n = 8.
8
At 5% level of significance, we use a left-tailed z- test and reject H0 if p-value < 0.05.
From the GC, p − value = 0.0385 .
Since p − value = 0.0385 < 0.05 , we reject H 0 and conclude that at 5% level of
significance there is sufficient evidence to suggest that the mean amount spent per
customer per visit has decreased in recent months.

11 H 0 : μ = μ0
H1 : μ ≠ μ0
∑ ( y −70)
y= + 70 = 62
9
1 ⎡ ( −72 ) ⎤
2

s =
2
⎢1234 − ⎥ = 82.25
9 − 1 ⎣⎢ 9 ⎦⎥
Y − μ0
Under H 0 , test statistic T = ~ t(8)
s
n
where y = 62, μ = μ0 , s = 82.25, n = 9
At 5% level of significance, we use a 2-tailed t- test and reject H0 if T ≥ 2.306
Since H 0 is rejected at 5% level of significance,
y − μ0 y − μ0
< −2.306 or > 2.306
s s
n n
82.25 82.25
μ0 > 62 + 2.306 or μ0 < 62 − 2.306
9 9
{μ0 ∈ : μ0 < 55.0 or μ0 >69.0 }

12
Class Adm No

Candidate Name:

2010 Preliminary Examination II


Pre-university 3

MATHEMATICS 9740

PAPER 1 9740/1
Thursday 16 September 3 hours
Additional materials:

Cover page
Answer papers
List of Formulae (MF15)

READ THESE INSTRUCTIONS FIRST

Write your name, admission no. and class in the spaces at the top of this page and on all the work you
hand in.
Write in dark blue or black pen on both sides of the paper.
You may use a soft pencil for any diagrams or graphs.
Do not use staples, paper clips, highlighters, glue or correction fluid.

Answer all the questions.


Give non-exact numerical answers correct to 3 significant figures, or 1 decimal place in the case of
angles in degrees, unless a different level of accuracy is specified in the question.
You are expected to use a graphic calculator.
Unsupported answers from a graphic calculator are allowed unless a question specifically states
otherwise.
Where unsupported answers from a graphic calculator are not allowed in a question, you are required to
present the mathematical steps using mathematical notations and not calculator commands.
You are reminded of the need for clear presentation in your answers.

The number of marks is given in brackets [ ] at the end of each question or part question.
At the end of the examination, fasten all your work securely together.

This question paper consists of 6 printed pages.

[Turn over
2

1
1 (i) Express f ( r ) = in partial fractions. [2]
r − 2r
2

n
2
(ii) Hence find ∑r
r =3
2
− 2r
in terms of n. [3]

(There is no need to express your answer as a single algebraic fraction.)



2
(iii) Deduce the value of ∑r
r =3
2
− 2r
. [2]

2 Functions f and g are defined by

f : x → e x + 10, for x ∈ℜ,


g : x → x 2 − 3 x, for x ∈ℜ.

(i) Sketch the graphs of f and g on the same diagram, indicating clearly the equations of
any asymptotes and the coordinates of any turning points. Hence, or otherwise, solve
g ( x) = f ( x) . [4]

(ii) Show that the composite function gf exists and find an expression for gf ( x ) . [2]

(iii) If the domain of g is restricted to the set { x ∈ℜ : x ≥ a} , find the least value of a for
which g −1 exists. Hence, find g −1 and state its domain. [4]

3 The fourth, ninth and nineteenth term of an arithmetic progression are consecutive terms
of a geometric progression.

(i) Show that the common ratio of the geometric progression is 2. [3]

(ii) The twentieth term of the arithmetic progression is 63. Find its first term and
common difference. [3]

(iii) The sum of the first n terms of the arithmetic progression is denoted by S n . Using
the results in (ii), find the least value of n for which S n exceeds 200. [3]
3

4 The curve C has equation

a 2 ( x + 1) − b 2 y 2 =
2
1

where a and b are positive constants.

 3 
Given that the curve passes through the point  − , 0  and the equations of its
 2 
asymptotes are =
y 2 x + 2 and y =−2 x − 2 , show that a = 2 and b = 1 . [4]

Hence sketch C, stating the equations of any asymptotes and the coordinates of any
points of intersection with the axes. [3]

dy
5 The equation of a curve C is 2 x 3 − 3 xy + y 3 =
p , where p is a constant. Find . [2]
dx

It is given that C has a tangent which is parallel to the y-axis. Show that the y-coordinate
of the point of contact of the tangent with C must satisfy

2 y6 − 2 y3 − p =
0.

1
Hence show that p ≥ − . [3]
2

Find the values of p in the case where the line x = 4 is a tangent to C. [3]

1
It is given instead that C has a tangent which is parallel to the x-axis. Show that p ≥ −
2
in this case also. [2]

6 A disease is spreading through a population of N individuals. It is given that the rate of


increase of the number of infected individuals at any time is proportional to the product
of the number of infected individuals and the number of uninfected individuals at that
time. At any time t, x is the number of infected individuals.

N eN k t
Given that initially only one person is infected, show that x = , where k is a
N −1 + eN k t
positive constant. [7]

[Turn over
4

7 f ( x ) ln ( 2 x + 1) , find f ( 0 ) , f ′ ( 0 ) , f ′′ ( 0 ) and f ′′′ ( 0 ) . Hence obtain the


(i) Given that =
first three non-zero terms in the Maclaurin’s series for f ( x ) . [5]

(ii) Hence, or otherwise, show that the first three non-zero terms in the expansion of
ln ( 2 x 2 + 3 x + 1) are ax + bx 2 + ax 3 , where a and b are constants to be found. [3]

1
8 The diagram below shows the graph of y = f ( x ) with a vertical asymptote x = − . The
2
points A ( −2, 0 ) and B ( 0, 2 ) are the point of inflexion and the minimum point
respectively.

y = f (x)

B
A x
O

1
x= −
2

Sketch, on separate diagrams, the graphs of

x
(i) =y f   +1, [3]
2

1
(ii) y= . [3]
f ( x)
5

9 Given that the plane π : r ⋅ n =d and the line l : r = a + λb intersect at a point,


d − a ⋅n
show that λ = . [2]
b ⋅n

Find

(i) the value of λ when l lies in π, [2]

(ii) the position vector of the point of intersection of l and π. [2]

10 (a) Given that the equation z 4 − z 3 − 9 z 2 + 29 z − 60 =


0 has a root of the form z = 1 + ki ,
where k is a non-zero real number, find the possible values of k.

Hence solve the equation z 4 − z 3 − 9 z 2 + 29 z − 60 =


0. [5]

(b) In an Argand diagram, the point P represents the complex number z such that

π 3π
z − 2 − 2i ≤ 2 and ≤ arg ( z − 2 − 2i ) ≤ .
4 4

(i) Sketch the locus of P. [3]

π  2+ 2 
(ii) Hence, or otherwise, show that ≤ arg ( z ) ≤ tan −1   . [3]
4  2− 2 

11 (a) Use the substitution x = cos θ to find the exact value of

π sin θ
∫ π /2 1 + cos 2 θ
dθ . [5]

(b) (i) Find ∫ e x sin 2 x dx . [4]

π /2
(ii) Hence find the exact value of 5∫ e x sin 2 x dx . [2]
0

[Turn over
6

12
y

P (3, 1) C

S
Q ( 8 , 0) R (4, 0)
x
O

The diagram shows the curve C with parametric equations

1 1
x =+
2t , y =−
2t , t > 0 .
t t

(i) Find gradient of the curve at the point where t = 1. [3]

(ii) Show that the cartesian equation of C is x 2 − y 2 =


8. [2]

Two points, P and Q, lie on the curve C with coordinates ( 3,1) and ( 8, 0 )
respectively. Point R ( 4, 0 ) lie on the x-axis. The region S is bounded by the lines QR
and PR and the arc PQ of the curve C.

(iii) Find the exact value of the volume of revolution when S is rotated completely
about the x–axis. [3]

End of Paper
Class Adm No

Candidate Name:

2010 Preliminary Examination II


Pre-University 3
MATHEMATICS
Higher 2 9740/02

Friday 17 September 2010 3 hours

Additional materials:
Cover Page, Writing paper
List of Formulae (MF 15)

READ THESE INSTRUCTIONS FIRST

Write your name, admission number and class in the spaces at the top of this page and
on all the work you hand in.
Write in dark blue or black pen on both sides of the paper.
You may use a soft pencil for any diagrams or graphs.
Do not use staples, paper clips, highlighters, glue or correction fluid.

Answer all the questions.


Give non-exact numerical answers correct to 3 significant figures, or 1 decimal place in
the case of angles in degrees, unless a different level of accuracy is specified in the
question.
You are expected to use a graphic calculator.
Unsupported answers from a graphic calculator are allowed unless a question
specifically states otherwise.
Where unsupported answers from a graphic calculator are not allowed in a question, you
are required to present the mathematical steps using mathematical notations and not
calculator commands.
You are reminded of the need for clear presentation in your answers.

The number of marks is given in brackets [ ] at the end of each question or part question.
At the end of the examination, fasten all your work securely together.

This question paper consists of 6 printed pages.

[Turn over]

1
Section A: Pure Mathematics [40 marks]

1 The rth term of a sequence is given by ur = r(3r + 1), r = 1, 2, 3, ….

n
(i) Write down the values of ∑u
r =1
r for n = 1, 2, 3, and 4. [2]
n
(ii) Make a conjecture for a formula for ∑ u r , giving your answer in the form
r =1
nf(n), where f(n) is a function of n. [1]
n
(iii) Use the method of mathematical induction to prove your conjecture for ∑ u r .
r =1
[4]

2 (i) Solve the equation z 5 = 32 , expressing the solutions in the form re iθ , where
r > 0 and − π < θ ≤ π . [3]

(ii) Show all the solutions on an Argand diagram. [2]


5
w 
(iii) Show that the roots of the equation w5 − 32  − 1 = 0 are
2 

1 − i cot where k = 0, ±1, ±2 . [4]
5

9 12 
   
3 (a) The points M and N have position vectors  6  and  p  respectively, where p
3 q 
   
and q are constants.
(i) The straight line which passes through M and N has equation
9 1 
   
r=  6  + λ  3  where λ ∈ R . Find the values of p and q. [3]
3 0
   

(ii) The line MN is extended to point S such that N divides MS in the ratio 3:2.
Find the position vector of S. [2]

[Turn over]

2
1  2 
   
(b) The planes p1 and p2 have equations r  −3  =
7 and r  −1 = 4 respectively,
a  1 
   
where a is a constant. Both planes pass through the point (1, b,3) and meet in the
line l where b is a constant.

(i) Find the values of a and b. [3]


(ii) Hence find the vector equation of line, l. [3]

4 (a) A conical water tank with its axis vertical and downwards has a radius of
5
r m at the top and is r m high (see diagram). If water flows into the tank
2

at a rate of 25 m3s −1 , find the rate at which the depth of water is increasing
when the water is 15 m high. [3]
r

5
r
2

(b) A chocolate maker is interested in using containers in the shape shown


below to package her chocolates. The container is made up of an open
cylinder of height h cm and radius r cm, with a hollow hemispherical cover
of radius r cm.
In order to minimise production cost in this economic recession, the
chocolate maker wants to use containers with the least surface area while
maintaining the volume of each container at 500 cm3. The material used to
construct the container costs $0.02 per cm2.
1000 5 2
(i) Show that the surface area, S is given by = S + πr . [4]
r 3
(ii) Find the radius that gives the minimum surface area. [4]
(iii) Find the minimum cost of one container, leaving your answer to 2
decimal places. [2]
4
[Volume of sphere, V = π r 3 ; Surface area of sphere, S = 4π r ]
2

3
Section B: Statistics [60 marks]

5 The Millennia Institute Library is planning to organize a campaign to promote reading


among students. The librarian conducted a survey on 50 randomly chosen students to
collect ideas for the campaign. However, she realized that most of these students
happen to be from a certain level.
(i) Advise the librarian on a more appropriate sampling method. [3]
(ii) Give an advantage and disadvantage of the sampling method selected in part (i).
[2]

6 Two players M and N regularly play each other in chess. When M has the first move
in a game, the probability of M winning the game is 0.4 and the probability of N
winning the game is 0.2. When N has the first move in a game, the probability of N
winning the game is 0.3 and the probability of M winning the game is 0.2. Any game
that is not won by either player ends in a draw.

(a) To decide who has the first move in a game, a fair coin is tossed. The player
who gets heads will make the first move. Find the probability that the game
ends in a draw. [2]
(b) Find the probability that N had made the first move given that the game ends in
a draw. [3]
(c) To make the game more interesting, M and N change the procedure for
deciding who has the first move in the game. As a result of the new procedure,
the probability of M making the first move in a game is p. Find the value of p
such that M and N have an equal chance of winning the game. [2]

7 Two families are invited to a party. The first family consists of a man and both his
parents while the second family consists of a woman and both her parents. The two
families sit at a round table with two other men and two other women.
Find the number of possible arrangements if
(i) there is no restriction, [1]

(ii) the men and women are seated alternately, [2]

(iii) members of the same family are seated together and the two other women must
be seated separately, [3]

(iv) members of the same family are seated together and the seats are numbered.
[2]

4
8 In a particular study, a medical student recorded the age and blood pressure of 8 men.
The regression line of y on x is determined to=be y 1.144 x + 79.914 .
Unfortunately, the student then lost the record of one of the men. He urgently needs
the missing data, b in his report.
Age (x) 49 58 44 75 37 57 63 66
Blood Pressure (y) 133 148 b 166 123 147 154 152

(i) Show that b is 130. [3]

(ii) Draw a scatter diagram for the data in the table. [2]

(iii) Find the linear product moment correlation coefficient between y and x. [1]

(iv) Explain why a linear product moment correlation coefficient that is close to 1
alone is not sufficient to conclude a strong linear relationship between two
variables. [1]

(v) Use the given regression line to estimate the age of a man when his blood
pressure is 150, giving your answer to the nearest integer. [1]

(vi) Explain whether this choice of the regression line is appropriate [1]

9 Durians and mangoes are sold by mass. The masses, in kg, of durians and mangoes
are modelled as having independent normal distributions with means and standard
deviations as shown in the table.

Mean Standard Deviation


Durians 1.6 0.2
Mangoes 0.3 0.05

Durians are sold at $8 per kg and mangoes at $3 per kg.

(i) Find the mass, m that will be exceeded by 80% of the durians. [1]

(ii) Find the probability that the total mass of 3 randomly chosen durians and 4
randomly chosen mangoes exceeds 6.5 kg. [3]

(iii) The mean mass of n randomly chosen durians is D kg. Given


that P( D < 1.45) =
0.0122 , find the value of n. [3]

(iv) Find the probability that the total selling price of 3 randomly chosen durians
and 4 randomly chosen mangoes is less than $45. [3]

5
10 There is an outbreak of an infection caused by a new strain of a virus in City X. The
probability, p, of a randomly chosen person being infected is 0.2. Find the
probability that, in a random sample of 12 people chosen from City X, at least 4 are
infected. [3]
After a certain time, the virus mutates and the value of p increases to 0.6. By using a
suitable approximation, find the probability that, in a random sample of 100 people,
there are at least 60 but not more than 80 who are infected. [4]

The virus then mutates to a much more deadly form and the value of p is now 0.97.
By using a suitable approximation, find the probability that, in a random sample of
100 people, at most 5 are not infected. [3]

11 A beverage producer claims that each packet of soya bean milk he produces contains
250 ml of the drink. A consumer group took a sample of 50 packets and recorded the
volume, x in ml of each packet. The results are summarized by

∑ x =12349 , ∑ x 2
=3054283 .

(i) Find the unbiased estimates of the population mean and variance. [2]

(ii) Test, at 5% significance level, whether the producer is overstating the mean
volume. [4]

(iii) In conducting the test in part (ii), explain if there is a need to assume that the
volume of a packet of soya bean milk follows a normal distribution. [1]

(iv) In conducting the test in part (ii), explain the meaning of ‘a significance level of
5%’ in the context of the question. [1]

(v) Given that the population variance of the volumes of packets of soya bean milk is
85, find the range of values of the sample size for which the producer’s claim
will not be rejected at a significance level of 5%. [3]

End of Paper

6
2010 MI PU3 Prelim Exam II
9740/02 H2 Mathematics Paper 1 Suggested Solutions

1 (i) f ( r ) = 1 1 A B
= = + (ii)
r − 2r r ( r − 2 ) r r − 2
2

n 2 n 1 n ⎡ 1 1⎤
By cover-up rule, A = − 1 , B = 1 ∑ = 2 × ∑ f (r ) = 2 × ∑ ⎢ −
2 2 r =3 r − 2 r
2
r =3 2 r = 3 ⎣ r − 2 r ⎥⎦
1 1 1
∴ f (r ) = − + = 1−
2r 2 ( r − 2 ) 3
1 1
+ −
2 4
∞ 2 ⎡3 1 1⎤
∑ = lim ⎢ − − ⎥ 1 1
+ −
r =3 r − 2r ⎣ 2 n −1 n ⎦
2 n →∞
(iii) 3 5
3 1 1
= + −
2 4 6
+ ...
1 1
+ −
n−4 n−2
1 1
+ −
n − 3 n −1
1 1
+ −
n−2 n
3 1 1
= − −
2 n −1 n
2 (i)

y f ( x) = e x + 10

g ( x) = x 2 − 3 x  

A 11 
y = 10

0  3 x

(3/2, -9/4)

For g ( x) = f ( x) , by GC, x ≈ −2.02

(ii) Rf = (10, ∞) ⊆ Dg = ℜ
∴ gf exists. (shown)
gf ( x ) = ( e x + 10 ) − 3 ( e x + 10 )
2

  1
3
(iii) From the graph of g above, g −1 exits if x ≥ .
2
3
Hence, the least value of a = .
2
2
⎛ 3⎞ 9
Let y = g ( x) = ⎜ x − ⎟ −
⎝ 2⎠ 4
3 9 3
∴ x = + y + , since x ≥
2 4 2
3 9 ⎡ 9 ⎞
∴ g −1 ( x ) = + x + , Dg −1 = ⎢ − , ∞ ⎟
2 4 ⎣ 4 ⎠
3 (i) Let the AP with T1 = a, common difference = d.

T4, T9, T19 are consecutive terms of a GP :

T19 T9 a + 18d a + 8d
⇒r= = ⇒r= =
T9 T4 a + 8d a + 3d
⇒ ( a + 18d )( a + 3d ) = ( a + 8d ) ⇒ 10d 2 = 5ad
2

5a 1
Since d ≠ 0, d = = a ---------(1)
10 2
⎛1 ⎞
a + 8⎜ a ⎟
a + 8d ⎝ 2 ⎠ = 2 (shown)
∴r = =
a + 3d ⎛1 ⎞
a + 3⎜ a ⎟
⎝2 ⎠

(ii) Given T20 = a + 19d = 63 ---------(2)

⎛1 ⎞
Substitute (1) into (2), a + 19 ⎜ a ⎟ = 63
⎝2 ⎠

∴a = 6 & d = 3

n
(iii) Sn = [ 2(6) + (n − 1)3] > 200
2

3n 2 + 9n − 400 > 0

n < −13.14 (NA) or n > 10.14

Hence, least n = 11

  2
4 ( x + 1)
2
y2
( x + 1)
2
a 2
−b y
2 2
=1⇒ 2
− 2
=1
⎛1⎞ ⎛1⎞
⎜ ⎟ ⎜ ⎟
⎝a⎠ ⎝b⎠

a
Asymptotes : y = ± ( x + 1)
b

Comparing with y = 2 x + 2 and y = −2 x − 2 ,

a
= 2 ⇒ a = 2b ------(1)
b

And the curve passes through the point ⎛ 3 ⎞ ,


⎜ − ,0 ⎟
⎝ 2 ⎠

2
⎛ 3 ⎞
⎜ − + 1⎟ − 0 = 1
2
a
⎝ 2 ⎠

⇒ a = 2 (shown), since a is positive.

Substitute into (1), b = 1 (shown)

y
y = − 2x −2
(0, √3)

(−1.5, 0) (−0.5, 0) x

(0, −√3)
y = 2x + 2

5 2 x 3 − 3 xy + y 3 = p -------(1)
dy dy
⇒ 6 x 2 − 3x − 3y + 3y2 =0
dx dx
dy y − 2 x 2
⇒ = 2
dx y −x
If tangent is parallel to the y-axis, y 2 = x .
2 ( y 2 ) − 3 ( y 2 ) y + y3 − p = 0
3
Substitute into (1),
⇒ 2 y 6 − 2 y 3 − p = 0 (shown)
For the point of contact of the tangent with C,

  3
b 2 − 4ac = ( −2 ) − 4 ( 2 )( − p ) ≥ 0
2

1
∴ p ≥ − (shown)
2
When x = 4 is a tangent to C, y 2 = 4 ⇒ y = ±2
When y = 2, p = 112 .

When y = −2, p = 144

If the tangent is parallel to the x-axis, y = 2 x 2 .


Substitute into (1),
2 x3 − 3x ( 2 x 2 ) + ( 2 x 2 ) − p = 0 ⇒ 8 x 6 − 4 x3 − p = 0
3

For the point of contact of the tangent with C,


b 2 − 4ac = ( −4 ) − 4 ( 8 )( − p ) ≥ 0
2

1
∴ p ≥ − (shown)
2
 
6 dx
= kx ( N − x ) , k is a positive constant.
dt

1
∫ x( N − x) dx = ∫ kdt
1 ⎡1 1 ⎤
∫ +
N ⎣ x N − x ⎥⎦
⎢ dx = kt + C

ln x − ln( N − x) = Nkt + NC

⎛ x ⎞
ln ⎜ ⎟ = Nkt + NC
⎝N−x⎠
x
= Ae Nkt , A = e NC
N−x

1
When t = 0, x = 1, =A
N −1

x 1 Nkt
= e
N − x N −1

⇒ x ( N − 1 + e Nkt ) = Ne Nkt

Ne Nkt
∴x=
N − 1 + e Nkt (shown)

  4
7 (i) f ( x ) = ln ( 2 x + 1)

2 ⎫
= 2 ( 2 x + 1)
−1
f ′( x) = ⎪
2x +1
⎪⎪
f ′′( x) = −2 ( 2 x + 1) (2) = −4 ( 2 x + 1)
−2 −2

−3 ⎪
f ′′′( x) = −4(−2) ( 2 x + 1) (2) = 16 ( 2 x + 1) ⎪
−3

⎪⎭

f (0) = 0 ⎫
f ′(0) = 2 ⎪⎪

f ′′(0) = −4 ⎪
f ′′′(0) = 16 ⎪⎭

1 2 1
f ( x) = f (0) + xf ′(0) +
x f ′′(0) + x3 f ′′′(0) + ...
2 3!
1 2 1 3
= 0 + 2 x + x (−4) + x (16) + ...
2 6
8
∴ f ( x) = 2 x − 2 x 2 + x3 + ...
3

(ii)

ln ( 2 x 2 + 3 x + 1) = ln(2 x + 1)( x + 1)
= ln(2 x + 1) + ln( x + 1)
8 1 1
= 2 x − 2 x 2 + x 3 + x − x 2 + x 3 + ...
3 2 3
5
= 3 x − x 2 + 3 x 3 + ...
2

5
⇒ a = 3, b = −
2

8 (i)


A ( −2, 0 ) → ( −4, 0 ) → A′ ( −4,1) ⎪

B ( 0, 2 ) → ( 0, 2 ) → B′ ( 0,3) ⎬
1 ⎪
Asymptote : x = − → x = −1 ⎪
2 ⎭

  5
y

⎛x⎞
y = f ⎜ ⎟ +1
⎝2⎠

B′
A′

x = −1

(ii)

A ( −2, 0 ) → asymptote at x = −2 ⎫

1 1 ⎪
Asymptote at x = − → x − intercept at x = −
2 2 ⎪

⎛ 1⎞ ⎪
B ( 0, 2 ) , min. pt. → ⎜ 0, ⎟ , max. pt.
⎝ 2⎠ ⎪

"New" asymptote : y = 0 ⎭
y

(0, ½)

(- ½, 0) 0 x

x=-2

  6
 

9 r ⋅n = d ------ (1)
r = a + λb ------(2)

Substitute (2) into (1),

( a + λb ) ⋅ n = d
⇒ a ⋅ n + λb ⋅ n = d

d − a⋅n
∴λ = (Shown)
b ⋅n

(i) If l is in π , a is on π , a ⋅ n = d .  b . n = 0  

  Hence, λ has infinitely many solutions.

d − a ⋅n
(ii) When l and π intersect at one point, λ = , from above.
b ⋅n
Substitute this into the equation of the line l, the position vector
⎛ d − a⋅n ⎞
required is r = a + ⎜ ⎟b
⎝ b ⋅n ⎠

10 (a) Given that z = 1 + ki is a root, so substitute into the given equation


(1 + ki ) − (1 + ki ) − 9 (1 + ki ) + 29 (1 + ki ) − 60 = 0
4 3 2

1 + 4ki − 6k 2 − 4k 3i + k 4 − (1 + 3ki − 3k 2 − k 3i ) − ( 9 + 18ki − 9k 2 ) + 29 + 29ki − 60 = 0

Comparing the real or imaginary parts on both sides,


4k − 4k 3 − 3k + k 3 − 18k + 29k = −3k 3 + 12k = 0
⇒ k = ±2 or k = 0 (N.A.)
−6k 2 + k 4 + 3k 2 − 9 + 9k 2 − 31 = k 4 + 6k 2 − 40 = 0
OR,
By GC, ⇒ k = ±2

Hence, ( z − (1 − 2i ) ) ( z − (1 + 2i ) ) = z 2 − 2 z + 5 is a factor of the given equation


z 4 − z 3 − 9 z 2 + 29 z − 60 = ( z 2 − 2 z + 5 )( z 2 + z − 12 ) = 0
(z 2
− 2 z + 5 ) = 0 or (z 2
+ z − 12 ) = 0
∴ z = 1 ± 2i , z = −4 or z = 3

  7
(b) (i)


arg ( z − 2 − 2i ) =
4 π
y arg ( z − 2 − 2i ) =
4
A D

α C α Note : α = π/4
2

x
0 B 2

(ii) min. (arg z) = argument of any complex numbers along CD


2
= tan −1
             
2
π
=
4

At A, x = 2 − 2 and y = 2 + 2 . -----(1)

max. (arg z) = argument of a, where A ≡ a

AB
= tan −1 -----(2)
OB
⎛ 2+ 2 ⎞
= tan −1 ⎜⎜ ⎟⎟
⎝ 2− 2 ⎠
π ⎛ 2+ 2 ⎞
Hence, ≤ arg ( z ) ≤ tan −1 ⎜⎜ ⎟⎟ (Shown) 
4 ⎝ 2− 2 ⎠

11 dx
(a) Let x = c o s θ ⇒ = −s i nθ

π
When θ = , x = 0 . When θ = π , x=−1.
2
π sin θ −1 −1

∫ π /2 1 + cos 2 θ
dθ =
0 1+ x
2
dx

−1
= − ⎡⎣ t a n − 1 x ⎤⎦
0
π
=
4

  8
(b)(i)

∫e sin 2 x dx = e x sin 2 x − ∫ e x .2 cos 2x dx


x

= e x sin 2 x − 2 ⎡ e x cos 2 x + ∫ e x .2sin 2 x dx ⎤


⎣ ⎦
5∫ e x sin 2 x dx = e x sin 2 x − 2e x cos 2 x + C
1
∴ ∫ e x sin 2 x dx = e x [sin 2 x − 2 cos 2 x ] + C
5

(b)(ii)

1
(e sin 2 x ) dx = 5 × ⎡⎣e x ( sin 2 x − 2cos 2 x ) ⎤⎦
π /2 π /2
5∫ x
0 5 0

= 2 ( eπ /2 + 1)

12 dy 1 dx 1
(i) = 2+ , = 2−
2
dt t dt t2
1
2+ 2
dy dy dx t
= ÷ =
dx dt dt 2 − 1
t2
When t = 1, d y 2 +1
= =3
dx 2 −1
1 1
(ii) x = 2t + ------ (1) ; y = 2t − ------ ( 2)
t t
(1) + (2), x + y = 4t ------ ( 3)
(1) – (2), x − y = 2 ------ ( 4 )
t
(3) × (4), ( x + y )( x − y ) = 8
⇒ x 2 − y 2 = 8 (Shown) 
3
(iii) Volume of the solid = π ∫ y 2 dx + volume of cone
8
3
  = π ∫ ( x2 − 8) dx + 13 π (12 ) (1)
8
= 4 ( 4 8 − 1 1) π or 4
( )
8 2 − 1 1 π (exact value)
3 3

  9
2010 MI PU3 Prelim Exam II
9740/02 H2 Mathematics Paper 2 Suggested Solutions

1 (i)
1 2 3 4

∑u
r =1
r = 1× 22 = 4, ∑ ur = 2 × 32 = 18, ∑ ur = 3 × 42 = 48, ∑ ur = 4 × 52 = 100
r =1 r =1 r =1

(ii)
n

∑u
r =1
r = n(n + 1) 2

(iii)
n
Let Pn be the statement that ∑u r =1
r = n(n + 1) 2 for n ∈ Z +

Prove P1 is true:
LHS = (1)(3 ×1 + 1) = 4
RHS = (1)(1 + 1) 2 = 4
P1 is true

Assume Pk is true for some k ∈ Z + i.e.


k

∑u
r =1
r = k (k + 1) 2

k +1
Prove Pk +1 is true i.e. ∑u
r =1
r = (k + 1)(k + 2) 2

LHS
k
= ∑ ur + (k + 1)(3k + 3 + 1)
r =1

= k (k + 1) 2 + (k + 1)(3k + 4)
= ( k + 1) ( k 2 + k + 3k + 4 )
= ( k + 1) ( k 2 + 4k + 4 )

= ( k + 1)( k + 2 )
2

= RHS
Since P1 is true and Pk is true ⇒ Pk +1 is true,
by the Principle of Mathematical Induction, Pn is true for n ∈ Z +

1
2 (i)
z 5 = 32 = 32ei (2 kπ )
⎛ 2 kπ ⎞
i⎜ ⎟
z = 2e ⎝ 5 ⎠
where k = ±2, ±1, 0

Im (z)
Z2
(ii)
Z3

5 2π
2π 5 Z1
5 Re (z)


5
Z4 2π
5
Z5

(iii)
5
⎛w ⎞
w − 32 ⎜ − 1⎟ = 0
5

⎝2 ⎠
5
⎛ ⎞
⎜ w ⎟
⎜ w ⎟ = 32
⎜ −1 ⎟
⎝2 ⎠
⎛ 2 kπ ⎞
w i⎜ ⎟
= 2e ⎝ 5 ⎠
w
−1
2
⎛ 2 kπ ⎞ ⎛ 2 kπ ⎞
i⎜ ⎟ i⎜ ⎟
w = we ⎝ 5 ⎠
− 2e ⎝ 5 ⎠

⎛ 2 kπ ⎞
i⎜ ⎟
2e ⎝ 5 ⎠
w= ⎛ 2 kπ ⎞
i⎜ ⎟
e ⎝ 5 ⎠
−1
⎛ 2 kπ ⎞
i⎜ ⎟
⎝ 5 ⎠
2e
w= ⎛ kπ ⎞
i⎜ ⎟ ⎛ i⎛⎜ k5π ⎞⎟ i⎛⎜ − k5π ⎞⎟ ⎞
e ⎝ 5 ⎠
⎜e ⎝ ⎠ −e ⎝ ⎠ ⎟
⎜ ⎟
⎝ ⎠
⎛ kπ ⎞
i⎜ ⎟
⎝ 5 ⎠
2e
w=
⎛ i⎛⎜ k5π ⎞⎟ ⎞
2 Im ⎜ e ⎝ ⎠ ⎟
⎜ ⎟
⎝ ⎠

2
⎛ kπ ⎞ ⎛ kπ ⎞
cos ⎜ ⎟ + i sin ⎜ ⎟
w= ⎝ 5 ⎠ ⎝ 5 ⎠
⎛ kπ ⎞
i sin ⎜ ⎟
⎝ 5 ⎠
⎛ kπ ⎞
cot ⎜ ⎟
w = 1+ ⎝ 5 ⎠
i
⎛ kπ ⎞
w = 1 − i cot ⎜ ⎟ (shown)
⎝ 5 ⎠

3 (a)(i)
⎛12 ⎞ ⎛ 9 ⎞ ⎛1 ⎞
⎜ ⎟ ⎜ ⎟ ⎜ ⎟
⎜ p ⎟ − ⎜6⎟ = λ ⎜3⎟
⎜q ⎟ ⎜3⎟ ⎜ 0⎟
⎝ ⎠ ⎝ ⎠ ⎝ ⎠
⎛ 3 ⎞ ⎛1 ⎞
⎜ ⎟ ⎜ ⎟
⎜ p − 6⎟ = λ ⎜3⎟
⎜q −3 ⎟ ⎜0⎟
⎝ ⎠ ⎝ ⎠
λ =3
q=3
p − 6 = (3)(3) p = 15

(a)(ii)
uuuur uuur
uuur 2OM + 3OP
ON =
5
uuur 2 uuuur 5 uuur
OP = − OM + ON
3 3
⎛9⎞ ⎛12 ⎞ ⎛ 14 ⎞
uuur 2⎜ ⎟ 5⎜ ⎟ ⎜ ⎟
OP = − ⎜ 6 ⎟ + ⎜ 15 ⎟ = ⎜ 21⎟
3⎜ ⎟ 3⎜ ⎟ ⎜ ⎟
⎝ 3⎠ ⎝3⎠ ⎝3⎠
(b)(i)
⎛ 1 ⎞ ⎛1 ⎞
Point (1, b,3) is on both planes: ⎜ ⎟⎜ ⎟
⎜ 1 ⎟ ⎜ −3 ⎟ = 7
⎛1⎞ ⎛2 ⎞ ⎜ 3⎟ ⎜ a ⎟
⎜ ⎟⎜ ⎟ ⎝ ⎠⎝ ⎠
⎜ b ⎟ ⎜ −1 ⎟ = 4 1 − 3 + 3a = 7
⎜ 3 ⎟ ⎜1 ⎟
⎝ ⎠⎝ ⎠ a=3
2−b+3= 4
b =1

(b)(ii)

3
Line l is perpendicular to the normals to both planes:
⎛ 1 ⎞ ⎛ 2 ⎞ ⎛ 0⎞
⎜ ⎟ ⎜ ⎟ ⎜ ⎟
⎜ −3 ⎟ × ⎜ −1⎟ = ⎜ 5 ⎟
⎜ 3 ⎟ ⎜1 ⎟ ⎜ 5 ⎟
⎝ ⎠ ⎝ ⎠ ⎝ ⎠
⎛1⎞ ⎛0⎞ ⎛1⎞ ⎛ 0⎞
⎜ ⎟ ⎜ ⎟ ⎜ ⎟ ⎜ ⎟
l : r = ⎜ 1 ⎟ + α ⎜ 1 ⎟ where α ∈ or r = ⎜ 1 ⎟ + α ⎜ 5 ⎟ where α ∈
⎜ 3⎟ ⎜1⎟ ⎜ 3⎟ ⎜ 5⎟
⎝ ⎠ ⎝ ⎠ ⎝ ⎠ ⎝ ⎠

4 (a)
2
1 1 ⎛2 ⎞ 4
V = π r 2 h = π ⎜ h ⎟ h = π h3
3 3 ⎝5 ⎠ 75
dV 4
= π h2
dh 25

dV dV dh
=
dt dh dt
At h = 15m:
4 dh
25= π (15) 2
25 dt
dh 25
= ≈ 0.221 ms -1
dt 36π

(b)(i)
2
V = π r 2 h + π r 3 = 500
3
1500 − 2π r 3
h=
3π r 2
500 2
h= 2 − r
πr 3

S = 2π rh + π r 2 + 2π r 2
⎛ 500 2 ⎞
= 2π r ⎜ 2 − r ⎟ + 3π r 2
⎝πr 3 ⎠
1000 4 2
= − π r + 3π r 2
r 3
1000 5 2
= + πr
r 3

4
(b)(ii)

1000 5 2
S= + πr
r 3
dS 1000 10
= − 2 + πr
dr r 3
dS 1000 10
For minimum surface area: = − 2 + πr = 0
dr r 3
1000 10
= πr
r2 3
300
r3 =
π
r = 4.570781497
r ≈ 4.57 cm
d 2 S 1000 10
= 3 + π > 0 (verified r = 4.57 gives a minimum surface area)
dr 2 r 3

(b)(iii)
1000 5
S min = + π (4.570781497) 2 = 328.1714518
4.570781497 3
Costminimum = (328.1714518)(0.02) ≈ $6.56

5 (i) Use stratified sampling. Divide the student population into different strata
for example PU1, PU2 and PU3 (or any other logical strata) and select
students randomly from each strata according to the strata’s proportion to
the population until 50 students have been selected.

(ii) Advantage: Good representation of student population.


Disadvantage: Time-consuming

6 0.4 Win
0.4
M starts Draw
0.5 first 0.2
Lose

0.2 Win
0.5 0.5
M starts Draw
second 0.3
Lose

(a) P (game ends in draw) = (0.5)(0.4) + (0.5)(0.5) = 0.45

5
(b)
P ( N made the 1st move game ends in draw )
P ( N made the 1st move ∩ game ends in draw)
=
P(game ends in draw)
0.5 × 0.5
=
0.45
= 0.556

(c)
P ( M wins game) = P ( M loses game)
0.4 p + 0.2(1 − p) = 0.2 p + 0.3(1 − p)
1
p = ≈ 0.333
3

7 (i) Number of arrangements = (10 − 1)! = 362880

(ii) Number of arrangements = (5 − 1)!× 5! = 2880

(iii) Number of arrangements = (4 − 1)!× 3!× 3!× 4 P2 = 2592

(iv) Number of arrangements = (6 − 1)!× 3!× 3!× 10 = 43200


8 (i)

x=
∑ x = 449 = 56.125
n 8
y = 1.144(56.125) + 79.914 = 144.121
b + 1023
= 144.121
8
b ≈ 130

(ii)

6
(iii) Using GC, r = 0.9888 ≈ 0.989

(iv) A r value that is close to 1 alone does not lead to the conclusion
that there is a strong linear relationship between 2 variables
because a r value close to 1 could be a result of outliers.
(v)
150 = 1.144 x + 79.914
x ≈ 61 .
The age of the man is 61.

(vi) Age is the independent variable (can be precisely controlled) and the
estimate from the regression line of y on x is appropriate.
(Also accept r is close to 1, using any of the 2 regression lines will result
in a close estimate. Thus, the estimate is appropriate).

9 Let D be the mass of a durian


D ~ N (1.6, 0.22 )
Let M be the mass of a durian
M ~ N (0.3, 0.052 )

(i)
Let P( D > m) = 0.8
Using GC, m = 1.43167 ≈ 1.43 kg.

(ii)
D(3) + M (4) ~ N (3 × 1.6 + 4 × 0.3, 3 × 0.22 + 4 × 0.052 ) = N (6, 0.13)
P ( D(3) + M (4) > 6.5) = 0.08275892 ≈ 0.0828

⎛ 0.22 ⎞
(iii) D ~ N ⎜ 1.6, ⎟
⎝ n ⎠
P ( D < 1.45) = 0.0122
⎛ ⎞
⎜ 1.45 − 1.6 ⎟
P⎜Z < ⎟ = 0.0122
⎜ 0.2 ⎟
⎜ ⎟
⎝ n ⎠
1.45 − 1.6
= −2.2507717
0.2
n
n≈9

7
(iii)
8 D(3) + 3M (4) ~ N (8 × 4.8 + 3 ×1.2, 82 × 3 × 0.22 + 32 × 4 × 0.052 )
= N (42, 7.77)
P ( 8 D(3) + 3M (4) < 45 ) ≈ 0.859
10 Let X be the number of people infected with the disease
X B(12, 0.2)
P ( X ≥ 4)
= 1 − P( X ≤ 3)
= 1 − 0.794568
≈ 0.205

Let X be the number of people infected with the disease


X B(100, 0.6)
np = 60 > 5; n(1 − p) = 40 > 5

X N (60, 24) approximately

P (60 ≤ X ≤ 80)
cc
= P(59.5 ≤ X ≤ 80.5)
= 0.541

Let Y be the number of people NOT infected with the disease


Y B(100, 0.03)
np = 3 < 5; n > 50, p < 0.1,

Y Po(3) approximately
P (Y ≤ 5)
= 0.916

11 (i)

Unbiased estimates of the population mean , X = ∑X =


12349
= 246.98 ≈ 247
n 50
Unbiased estimates of the population variance,

8
1 ⎡ ⎛ ∑ ( x) ⎞ ⎤
2

s = ⎢∑ ( x ) − n ⎜ ⎥
⎜ n ⎟⎟ ⎥ = 88.30571429 ≈ 88.3
2 2

n −1 ⎢ ⎝ ⎠
⎣ ⎦

(ii) H 0 : μ = 250
H1 : μ < 250
Use z-test, p-value = 0.01153 (z = -2.272)
p-value = 0.01153 < 0.05, reject H 0
At 5% significance level, the producer is overstating the mean volume.

(iii) n is large (n = 50). By Central Limit Theorem, x-bar will


approximately follow a normal distribution. The z-test can still be
conducted. There is no need to assume that the volume of a packet
of soya bean follow a normal distribution.

(iv) The significance level of 5% means that 0.05 is the probability that
we conclude the producer has overstated the mean volume when the
mean volume is actually 250 ml.

(v) σ 2 = 85 (given)
Level of significance = 0.05.
Use z-test.
246.98 − 250
Do not reject H 0 if z = > −1.6448536
85 / n

n < 25.215
n ≤ 25

9
2

1 Given that
r −1 r 2
f (r ) = − + ,
r ! ( r − 1) ! ( r − 2 ) !
use the method of differences to prove that
N
N +1
∑ f (r )=
r =2
2−
N!
. [3]

Hence, give a reason why the series is convergent and state the sum to infinity. [2]

2 (i) Find
dx
( )
d x2 +1
e . [1]

∫x e
3 x 2 +1
(ii) Hence, find dx . [3]

(x e )
1

3 x 2 +1
(iii) Find the exact value of + e 2 dx . [2]
0

3 A sequence of positive real numbers x1 , x2 , x3 ,... satisfies the recurrence relation

3 xn
xn +1 =
2 + xn
for n ≥ 1.
(i) Given that as n → ∞, xn → α, find the exact value of α . [2]

(ii) Show that this sequence is

(a) strictly increasing for 0 < xn < α, [2]

(b) strictly decreasing for xn > α . [2]

[Turn over

MJC/2010 JC2 Preliminary Examination/9740/01


3

x−4
4 (i) Express f ( x ) = in partial fractions.
( x + 1)( 3x + 2 )
Hence, expand f ( x ) in ascending powers of x, up to and including the

term in x3 . [4]

(ii) State the range of values of x for which this expansion is valid. [1]

(iii) Find the coefficient of x n in this expansion. [2]

5 On a single Argand diagram, sketch the following loci.

(i) z − 5 = 3 + 7i , [1]

(ii) z − 6 − 3i = z − 4 + 3i . [1]

Two complex numbers that satisfy the above equations are represented by p and q,
where Re( p ) < Re(q ) . By using the cartesian equations of the loci, find p and q.
Hence, determine the value of arg( p − q ) . [5]

6 Newton’s Law of Cooling states that the rate at which the temperature of a body
falls is proportional to the amount by which its temperature exceeds that of its
surroundings. At time t minutes after cooling commences, the temperature of the
body is θ o C . Assuming that the room temperature remains constant at 30 o C
and the body has an initial temperature of 90 o C , show that θ= 30 + 60e − kt ,
where k is an arbitrary constant. [5]

Given that it takes 8 minutes for the temperature of the body to drop from 90 o C
to 55 o C , determine how much more time is needed for the body to cool to 35 o C ,
leaving your answer to one decimal place. [3]

MJC/2010 JC2 Preliminary Examination/9740/01


4

1
7 (a) A geometric progression has first term a and common ratio − . The first
2
two terms of the geometric progression are the first and fourth terms
respectively of an arithmetic progression. Find the sum of the first n even-
numbered terms of the arithmetic progression in terms of a and n. [4]

(b) A customer borrows $50000 from a bank at the beginning of a month. In


the middle of the month, the customer pays $x to the bank. On the last day
of every month, the bank adds interest at a rate of 3.5% of the remaining
amount owed after payment has been made. This repayment process is
repeated every month until the loan is repaid in full.
(i) Find, in terms of x, the amount owed at the beginning of the third
month. [1]
(ii) Show that, if the repayment of the loan is completed upon the nth
1750(1.035n −1 )
payment, then x ≥ . [4]
1.035n − 1

8 The diagram shows the graph of y = f ( x ) , where f ( x ) is a cubic polynomial and

C is a maximum point.

A ( −2, 75 )

C (1, 12 )

B ( 0, 3)
0 x

[Turn over

MJC/2010 JC2 Preliminary Examination/9740/01


5

It is also given that when y = 3 , x = 0, α or β , where α < 0 < β . On separate


diagrams, sketch the graphs of
(i) y f (1 − 2 x ) ,
= [3]

(ii) y2 f ( x) − 3 .
= [3]

Determine the equation of the curve. [4]

9 A line l passes through the points A and B with coordinates ( 0, −1, 2 ) and (1, 0,1)

respectively.

(i) Find the angle between OA and the line l. [2]

(ii) Hence, find the shortest distance from the origin to the line l. [1]

A plane π 1 has equation r  ( i + 2 j + 3k ) =


4.

(iii) Show that the line l lies on the plane π 1. [2]

A second plane π 2 contains the line l and is perpendicular to the plane π 1.

(iv) Find a vector equation of π 2 . [2]

A third plane π 3 is perpendicular to both the planes π 1 and π 2 , and is at a

perpendicular distance of 3 units from the origin.

(v) Find possible vector equations of π 3 . [3]

MJC/2010 JC2 Preliminary Examination/9740/01


6

10 The functions f, g and h are defined by


f : x  cos x for x∈ ,

 1 
g : x  ln   for − 1 < x ≤ 1,
 1+ x 
 1 
ln   for −1 < x < 1,
h:x  1+ x 
−1 for x= 1.
(i) Show that the composite function fg exists. [2]

(ii) Find the series expansion for fg( x) up to and including the term in x3 . [4]

(iii) Write down the value of h −1 (−1) . Hence, find h −1 ( x) , expressing your
answer in the form
p( x) for x > a,
h −1 : x 
c for x = −1,
where a and c are constants, and p is a function of x. [4]

11 The complex numbers z and w are given by =


z 3 + i and w =−1 + i 3 .

Find the set of values of the positive integer n for which z n − ( z * ) =


n
(a) 0 . [4]

(b) Sketch an Argand diagram, with origin O, showing the points Z, W and P
representing the complex numbers z, w and z + w respectively. Show that
OWPZ is a square. [3]

By considering the argument of z + w , deduce that tan = 2+ 3 . [3]
12

[Turn over
MJC/2010 JC2 Preliminary Examination/9740/01
7

2 x 2 − 3x + 1
12 The curve C has equation y = .
x2 − 4
(i) Prove, using an algebraic method, that C cannot lie between

7−3 5 7+3 5
and . [3]
8 8

(ii) Sketch the curve C, showing clearly all asymptotes, axial intercepts and
turning points. [4]

(iii) R is the point on C where x = −5 . The tangent and normal to the curve at
R cut the y-axis at P and Q respectively. Show, to 3 significant figures,
that the coordinates of P and Q are ( 0,5.15 ) and ( 0, −9.31) respectively.

Hence, determine the area of triangle PQR. [5]

MJC/2010 JC2 Preliminary Examination/9740/01


Section A: Pure Mathematics [40 marks]

x+7
1 Without using a calculator, solve the inequality ≤ 1. [4]
4 + 3x − x 2

2 Three non-zero and non-parallel vectors p, q and r are such that p × q = 3p × r .


λp , where λ is a scalar.
Show that q − 3r = [2]

5
It is also given that p is a unit vector, q = 5 , r = 2 and the angle between q and r is cos −1 .
6
By considering ( q − 3r ) ⋅ ( q − 3r ) , find the exact values of λ . [4]

∑ 3( 2 r ) 3 ( 2n +1 ) − n ( n + 1) − 6 for n ∈  + .
n
3
3 Prove by induction r
−= [5]
r =1 2

∑ 3( 2 − r ) > 5800 .
n
r
Hence, find the least value of n for [3]
r =5

4 (a) The variables x and y are related by

= x + ln ( xy )
3
y
dy
Find the value of when both x and y are equal to 1. [4]
dx

(b)

45

Water is poured at a constant rate of 1 cm3s -1 into a cone of semi-vertical angle 45
with its axis vertical and vertex downwards (see diagram). At the beginning, the cone is
partially filled with 30 cm3 of water. Find the rate at which the depth of water is
increasing after 2 minutes. [6]

[Turn Over
3

x2 y 2 y x
5 (i) The region R is bounded by the ellipse 2 + 2 =1 and the line − =1 as shown in
a b b a
the diagram below.
y
y x
− =1
b b a
R

−a a x
O
x2 y 2
−b + =
1
a 2 b2

1 0 b
Show that the area of R is − ab + ∫ a 2 − x 2 dx . Hence, by substituting x = a sin θ ,
2 − a a
find the exact area of R in terms of a and b. [8]

(ii) Find the volume of revolution formed when R is rotated completely about the y-axis.
Give your answer in the form ka 2b , where k is a constant to be determined. [4]

Section B: Statistics [60 marks]

6 There are 800 students in PPQZ Secondary School where 65% of the students are boys. A
random sample of 200 students is selected to find out their preferred canteen stalls.

(a) Student A suggests selecting the sample by taking the first 120 boys and the first 80
girls who visit the canteen during recess time. Identify the sampling method used.
State, in the context of the question, one disadvantage of this method used. [2]

(b) Student B suggests using systematic sampling using the following steps:
(1) Label all the students from 1 to 800.
(2) Using a sampling interval of 5, he randomly selects an integer from 1 to 5 to
determine the first student to be chosen and selects every 5th student thereafter until
200 students are chosen.

Identify the mistake that student B made and write down the correct step. [2]

MJC/2010 JC2 Preliminary Examination/9740/02


7 In a computer game played by a single player, the player has to find, within a fixed time, the
path through a maze shown on the computer screen. On the first occasion that a particular
player plays the game, the computer shows a simple maze, and the probability that the player
5
succeeds in finding the path in the time allowed is . On subsequent occasions, the maze
7
shown depends on the result of the previous game. If the player succeeded on the previous
occasion, the next maze is harder, and the probability that the player succeeds is one third of
the probability of success on the previous occasion. If the player failed on the previous
occasion, another simple maze is shown and the probability of the player succeeding is
5
again .
7

The player plays three games. Find the probability that

(i) the player succeeds in exactly one of the games, [2]

(ii) the player succeeds in at least one of the games, [2]

(iii) the player has exactly two failures given that the player has at least one success. [2]

8 (a) Find the number of teams of 8 players that can be selected from a group of 13 players if
at least two of the three tallest players and at most one of the two shortest players are to
be included. [4]

(b) Secret codes can be sent using 6 available letters X, Y, Y, Z, Z, Z.

(i) How many different 5-letter secret codes can be sent? [3]

(ii) How many different 5-letter secret codes will begin and end with Y? [1]

[Turn Over
5

9 The time t minutes spent queuing at an ice-cream bar by each customer in a random sample of
∑ t = 213.5 and ∑ ( t − t ) =
2
50 customers was measured and it was found that 44.105 where
t is the mean time spent at the ice-cream bar by each customer in the sample.

Find unbiased estimates of the population mean and variance. [2]

Test, at 1% level of significance, whether the mean time spent is less than 4.5 minutes. [4]

Determine the smallest level of significance of the test where the null hypothesis is rejected. [1]

State, giving a reason, whether any assumptions about the population are needed in order for
the test to be valid. [1]

10 On average, a hospital and a police station receive 36 and 15 calls respectively in a three-week
period. Calls are received at random times. The number of calls received by the hospital may
be assumed to be independent of the number of calls received by the police station.

(i) Find the probability that the hospital and the police station receive a total of more than 11
calls in a randomly chosen week. [3]

(ii) Using a suitable approximation, find the probability that out of 50 randomly chosen weeks,
the number of weeks in which the hospital and police station receive a total of more than
11 calls is greater than 45. [4]

(iii) A nurse records the number of calls received by the hospital per week for 100 randomly
chosen weeks. She then calculates the average number of calls received per week based on
her data. Another nurse independently goes through the same procedure. Find the
probability that the sum of the two averages obtained by the nurses is at most 23. [3]

MJC/2010 JC2 Preliminary Examination/9740/02


11 (a) King Crabs and Snow Crabs are sold by weight. The masses, in kg, of King Crabs and
Snow Crabs are modelled as having normal distributions with means and variances as
shown in the table.
Mean Mass Variance
King Crabs 1.65 0.71
Snow Crabs 1.10 0.34
King Crabs and Snow Crabs are sold at $40 and $45 per kg respectively.

(i) Find the probability that the average mass of 3 King Crabs and 2 Snow Crabs is
less than 1.5 kg. [3]

(ii) Find the probability that the total selling price of a King Crab and a Snow Crab
exceeds $140. [3]

(iii) State an assumption for your working in (i) and (ii) to be valid. [1]

(b) The masses of bars of chocolate are normally distributed with mean µ kg and standard
deviation σ kg. It is known that 20% of the bars of chocolate have masses which differ
from µ kg by at most m kg. Find the probability that a randomly chosen bar of
chocolate has a mass which exceeds µ kg by at least 3m kg. [5]

[Turn Over
7

12 The table below shows the number of monthly new car licences, x, issued by the government
and the selling price of a car, $y in year 2009. A student wants to investigate how the selling
price of a car depends on the number of monthly new car licences issued by the government.

Month Jan Feb Mar Apr May June July Aug Sep Oct Nov Dec
x 1432 1339 1664 1774 2055 2275 2221 2173 1749 1854 1360 2012

y 42000 48000 37000 36000 32850 31800 31800 32050 36000 34500 46000 33000

(i) Plot a scatter diagram for the data and explain, in the context of the question, if a linear
model is appropriate in the long run. [3]

(ii) State, with a reason, which of the following model is most appropriate to fit the data points.

(A)=
y ae x + b
(B)=
y ax 2 + b
b
(C) y= a + [2]
x

(iii) For the model chosen in (ii), calculate the product moment correlation and comment on its
value. [2]

(iv) Use an appropriate regression line to estimate the selling price of the car when the monthly
number of new car licences issued is (a) 1300 and (b) 2000. Comment on the reliability of
your answers. [4]

(v) The student concluded that the decrease in the number of new car licences issued causes the
selling price of the car to rise. State, with a reason, whether you agree with this
conclusion. [1]

MJC/2010 JC2 Preliminary Examination/9740/02


H2 MATHS (9740) JC2 PRELIMINARY EXAM 2010
PAPER 1 SUGGESTED SOLUTIONS

Qn Solution
1 Method of Differences
N ⎛
N
r −1 r 2 ⎞

r =2
f (r ) = ∑ ⎜⎜
r =2 ⎝ r !
− + ⎟
( r − 1)! ( r − 2 )! ⎟⎠
⎡1 2 2
=⎢ − +
⎣ 2! 1! 0!
2 3 2
+ − +
3! 2! 1!
3 4 2
+ − +
4! 3! 2!
4 5 2
+ − +
5! 4! 3!
+ M
N −3 N −2 2
+ − +
( N − 2 ) ! ( N − 3) ! ( N − 4 ) !
N −2 N −1 2
+ − +
( N − 1)! ( N − 2 )! ( N − 3)!
N −1 N 2 ⎤
+ − + ⎥
N ! ( N − 1) ! ( N − 2 ) !⎦
N −2 N N −1
= −2 + 2 + 2 + − +
( N − 1)! ( N − 1)! N !
N ( N − 2 ) − N 2 + ( N − 1)
= 2+
N!
N +1
= 2−
N!

N +1
As N → ∞, → 0 , hence the series is convergent.
N!

∑ f (r ) = 2
r =2

Qn Solution
2 Integration (by part)

(i)
dx
e( )
d x2 +1
= 2 xe x +1
2

(ii)
∫ x e dx
3 x 2 +1

= ∫ ( 2 xe ) ( x ) dx
1 x 2 +1 2

2
=
2
(
1 2 x2 +1
x e − ∫ 2 xe x +1dx
2

)
( e x +1 2
)
2

=
1 2 x2 +1 x2 +1
2
x e −e +c =
2
( x − 1) + c

(iiii)
(x e )
1

3 x 2 +1
+ e 2 dx
0
1
⎡1 ⎤
= ⎢ ⎡ x 2 e x +1 − e x +1 ⎤ + e 2 x ⎥
2 2

⎣2 ⎣ ⎦ ⎦0
⎡ ⎤ ⎡ ⎤
= ⎢ ( e 2 − e 2 ) + e 2 ⎥ − ⎢ ( −e ) ⎥
1 1
⎣ 2 ⎦ ⎣ 2 ⎦
= e 2 + 0.5e
0

Qnn Solution
3 Recurrrence Relatioon
(i)) As n → ∞, xn → α & xn +1 → α
3α 9α 2
⇒α= ⇒ α2 =
2+α 2+α

⇒ α 2 ( 2 + α ) − 9α 2 = 0
⇒ α 2 ( α − 7 ) = 0 ⇒ α = 0 or α = 7.
Thus α = 7 since α > 0.

(ii)) Metho od 1 [Graph hical]


3xn
xn +1 − xn = − xn
2 + xn
3xn
h y = xn +1 − xn =
Sketch − xn
2 + xn

From the
t graph:
For 0 < xn < α : xn +1 − xn > 0 ⇒ xn +1 > xn
Thereffore the sequuence is stricctly increasinng.

For xn > α : xn +1 − xn < 0 ⇒ xn +1 < xn


Thereffore the sequuence is stricctly decreasing. [Shownn]
Methood 2 [Graph
hical]
3x
hy=x &y=
Sketch
2+ x

xn
xn +1
α
xn +1
xn From the
t graph:
For 0 < xn < α : xn +1 > xn
Thereffore the sequuence is stricctly increasinng.

For xn > α : xn +1 < xn


Thereffore the sequuence is stricctly decreasing.
[Shown]
Metho od 3 [Algebrraic]
3 xn 3x − x 2 + xn
xn +1 − xn = − xn = n n
2 + xn 2 + xn

=
(
xn 3 − 2 + xn )
2 + xn
For 0 < xn < α :
2 + xn > 0, xn > 0 & 3 − 2 + xn > 0 sincee 2 + xn < 9 = 3
Thus: xn +1 − xn > 0 ⇒ xn +1 > xn
Thereffore the sequuence is stricctly increasinng.

For xn > α
2 + xn > 0, xn > 0 & 3 − 2 + xn < 0 sincee 2 + xn > 9 = 3
Thus: xn +1 − xn < 0 ⇒ xn +1 < xn
Thereffore the sequuence is stricctly decreasing.
[Showwn]

Qnn Solutioon
4 Binomiial Expansioon
x−4 A B
Let f ( x ) = = +
(i) ( x + 1)(( 3x + 2 ) x + 1 3x + 2
⇒ x − 4 = A ( 3 x + 2 ) + B ( x + 1)

Using cover
c up rulee,
2 2 ⎛1⎞
For x = − , − − 4 = B ⎜ ⎟ ⇒ B = −14
3 3 ⎝3⎠
For x = −1 , −1 − 4 = A ( −1) ⇒ A = 5
5 14
∴f ( x) = −
x + 1 3x + 2

5 14
f ( x) = −
x + 1 3x + 2
= 5 (1 + x ) − 14 ( 3 x + 2 )
−1 −1

−1
⎡ ⎛ 3x ⎞ ⎤
= 5 (1 + x ) − 14 ⎢ 2 ⎜1 + ⎟ ⎥
−1

⎣ ⎝ 2 ⎠⎦
−1
14 ⎛ 3 x ⎞
= 5 (1 + x ) −
−1
⎜1 + ⎟
2⎝ 2 ⎠
= 5 (1 − x + x 2 − x 3 +K)
⎛ ⎛ 3x ⎞ ( −1)( −2 ) ⎛ 3x ⎞ ( −1)( −2 )( −3) ⎛ 3x ⎞ ⎞
2 3

−7 ⎜1 + ( −1) ⎜ ⎟ + ⎜ ⎟ + ⎜ ⎟ + K ⎟⎟
⎜ ⎝ 2 ⎠ 2! ⎝ 2 ⎠ 3! ⎝ 2 ⎠
⎝ ⎠
⎛ 3x 9 x 27 x2 3

= 5 (1 − x + x 2 − x 3 +K) −7 ⎜1 − + − + K⎟
⎝ 2 4 8 ⎠
11 43 149 3
= −2 + x − x 2 + x +K
2 4 8
(ii)
Expansion of (1 + x )
−1
is valid for −1 < x < 1
−1
⎛ 3x ⎞ 2 2
Expansion of ⎜1 + ⎟ is valid for − <x<
⎝ 2 ⎠ 3 3

Therefore, the range of values of x for the expansion of f(x) to be valid is


2 2
− <x< .
3 3
(iii)
n ⎡ ⎛3⎞ ⎤
n n
⎛3⎞
Coefficient of x = ( −1) 5 + ( −1) 7 ⎜ ⎟ = ( −1) ⎢5 − 7 ⎜ ⎟ ⎥
n n n +1

⎝2⎠ ⎣⎢ ⎝ 2 ⎠ ⎦⎥

Qn Solution
5 Complex Numbers (Loci)

Im

z − 5 = 3 + 7i
p
6 + 3i

Re
5
4 − 3i
q
z − 6 − 3i = z − 4 + 3i
Cartesian equation of (i):

( x − 5) 2 + y 2 = 42 ---- (1)

Gradient of line segment joining 6 + 3i and 4 − 3i

=
3− − 3( )= 3
6−4
Cartesian equation of (ii):
1
y=− ( x − 5) ---- (2)
3

Using GC to solve (1) & (2), we get


p = 1.5359 + 2i
q = 8.4641 − 2i

⎛ 4 ⎞ 5π
∴ arg( p − q ) = π − sin −1 ⎜ ⎟ = = 2.62 (3 s.f.)
⎝8⎠ 6
Alternative:
p − q = −6.9282 + 4i
4
∴ arg ( p − q ) = π − tan −1 = 2.62 (3s.f.)
6.9282
Or using GC,
∴ arg ( p − q ) = 2.62 (3s.f.)

Qn Solution
6 Differential Equations

= − k (θ − 30 ) , k > 0
dt
1
∫ θ − 30 dθ = − ∫ k dt
ln θ − 30 = − kt + C
θ − 30 = e − kt +C
θ − 30 = ±e− kt eC = Ae− kt (where A = ±eC )
θ = 30 + Ae− kt
when t = 0, θ = 90
90 = 30 + A
A = 60
∴θ = 30 + 60e− kt (shown)

when t = 8, θ = 55
55 = 30 + 60e −8 k
5
e −8 k =
12
1 5
k = − ln
8 12
when θ = 35,
⎛1 5 ⎞
⎜ ln ⎟t
35 = 30 + 60e⎝ 8 12 ⎠

⎛1 5 ⎞
⎜ ln ⎟t 1
e⎝ 8 12 ⎠
=
12
t = 22.707
∴ additional time needed = 22.707 − 8 = 14.7 min (1d.p.)

Qn Solution
7 AP and GP
(a) 1
ar = a + 3d ⇒ d = − a
2
Sum of first n even-numbered terms
n⎡ ⎛1 ⎞ ⎤
= ⎢ 2 ⎜ a ⎟ + (n − 1) ( −a ) ⎥
2⎣ ⎝2 ⎠ ⎦
n
= [ 2a − an]
2
an ( 2 − n )
=
2
(b)(i) Amount owed at the beginning of the third month
= ⎣⎡( 50000 − x ) (1.035) − x ⎦⎤ (1.035)
= 50000(1.0352 ) − (1.035 + 1.0352 ) x
(b)(ii) Amount owed at the beginning of the nth month
= 50000(1.035n −1 ) − (1.035 + 1.0352 + L + 1.035n−1 ) x

For the repayment to be completed during the nth payment,

50000(1.035n −1 ) − (1.035 + 1.0352 + L + 1.035n −1 ) x ≤ x


50000(1.035n−1 ) ≤ (1 + 1.035 + 1.0352 + L + 1.035n−1 ) x
1.035n − 1
≤ x
1.035 − 1
1.035n − 1
≤ x
0.035

1750(1.035n−1 )
Thus x ≥ .
1.035n − 1
Qn Solution
8 Transformation of Graphs + System of Linear Equations
(i) y ⎛3 ⎞
A ' ⎜ , 75 ⎟
⎝2 ⎠

C ' ( 0, 12 ) ⎛1 ⎞
B '⎜ , 3⎟
⎝2 ⎠
0 x

y = f (1 − 2 x )

(ii) (
A" −2, 72 )
C " (1, 3)

B " ( 0, 0 )
α β
0 x

C "' (1, − 3 )

(
A"' −2, − 72 )
Let the curve be y = ax3 + bx 2 + cx + d .
Since the points ( −2, 75 ) , ( 0,3) and (1,12 ) lie on the curve.
Using ( 0,3) , d = 3
Using ( −2, 75 ) , a ( −2 ) + b ( −2 ) + c ( −2 ) + d = 75
3 2

−8a + 4b − 2c = 72 LLL (1)


Using (1,12 ) , a + b + c + d = 12
a + b + c = 9 LLL (2)
dy
Since (1,12 ) is a maximum point, = 0.
dx
3ax 2 + 2bx + c = 0
3a + 2b + c = 0 LLL (3)

Using GC to solve (1),(2) and (3),


a = −8, b = 7, c = 10

Thus the equation of the curve is y = −8 x3 + 7 x 2 + 10 x + 3 .


Qn Solution
9 Vectors (line and planes)

(i) Vector equation of the line l


⎛1⎞ ⎛1⎞
⎜ ⎟ ⎜ ⎟
r = ⎜0⎟ + λ ⎜ 1 ⎟, λ∈R
⎜1⎟ ⎜ −1 ⎟
⎝ ⎠ ⎝ ⎠

Angle between OA and the line l


⎛0⎞⎛1⎞
⎜ ⎟⎜ ⎟
⎜ −1 ⎟ . ⎜ 1 ⎟
⎜ 2 ⎟ ⎜ −1 ⎟ −3
⎝ ⎠⎝ ⎠ 3
cos θ = = =
5 3 15 15
θ = 39.232 ≈ 39.2 (1 d.p.)
o o

(ii) Let the shortest distance from the origin to the line l be x.
x
sin θ =
5
x = 1.41( 3 s.f.)
(iii) Since
⎛1⎞ ⎛1⎞
⎜ ⎟ ⎜ ⎟
⎜0⎟ .⎜ 2 ⎟ = 4 ⇒ A point on l lies on π 1
⎜1⎟ ⎜ 3⎟
⎝ ⎠ ⎝ ⎠
⎛ 1 ⎞ ⎛1⎞
⎜ ⎟ ⎜ ⎟
and ⎜ 1 ⎟ . ⎜ 2 ⎟ = 0 ⇒ l is parallel to π 1
⎜ −1 ⎟ ⎜ 3 ⎟
⎝ ⎠ ⎝ ⎠

The line l lies on the plane (shown)

Alternative Solution (1):


Since
⎛ 0 ⎞ ⎛1⎞
⎜ ⎟ ⎜ ⎟
⎜ -1⎟ . ⎜ 2 ⎟ = 4 ⇒ Point A on l lies on π 1
⎜ 2 ⎟ ⎜ 3⎟
⎝ ⎠ ⎝ ⎠
⎛1⎞ ⎛1⎞
⎜ ⎟ ⎜ ⎟
⎜ 0 ⎟ . ⎜ 2 ⎟ = 4 ⇒ Point B on l lies on π 1
⎜1⎟ ⎜ 3⎟
⎝ ⎠ ⎝ ⎠
∴ line l lies on π 1
Alternative Solution (2):
Since
⎡ ⎛1⎞ ⎛ 1 ⎞ ⎤ ⎛ 1 ⎞ ⎛1 + λ ⎞ ⎛ 1 ⎞
⎢ ⎜ ⎟ ⎜ ⎟⎥ ⎜ ⎟ ⎜ ⎟⎜ ⎟
⎢ ⎜ 0 ⎟ + λ ⎜ 1 ⎟ ⎥ . ⎜ 2 ⎟ = ⎜ λ ⎟ . ⎜ 2 ⎟ = 1 + λ + 2λ + 3 − 3λ = 4
⎢⎣ ⎜⎝ 1 ⎟⎠ ⎜ −1⎟ ⎥ ⎜ 3 ⎟ ⎜ 1 − λ ⎟ ⎜ 3 ⎟
⎝ ⎠⎦ ⎝ ⎠ ⎝ ⎠⎝ ⎠
∴ line l lies on π 1
⎛ 1 ⎞ ⎛1⎞ ⎛ 5 ⎞
(iv) ⎜ ⎟ ⎜ ⎟ ⎜ ⎟
normal of π 2 = ⎜ 1 ⎟ × ⎜ 2 ⎟ = ⎜ −4 ⎟
⎜ −1 ⎟ ⎜ 3 ⎟ ⎜ 1 ⎟
⎝ ⎠ ⎝ ⎠ ⎝ ⎠
⎛ 5 ⎞ ⎛1⎞ ⎛ 5 ⎞
π 2 : r . ⎜⎜ −4 ⎟⎟ = ⎜⎜ 0 ⎟⎟ . ⎜⎜ −4 ⎟⎟
⎜ 1 ⎟ ⎜1⎟ ⎜ 1 ⎟
⎝ ⎠ ⎝ ⎠ ⎝ ⎠
⎛ 5⎞
π 2 : r . ⎜⎜ −4 ⎟⎟ = 6
⎜1⎟
⎝ ⎠
⎛ 1 ⎞ ⎛ 5 ⎞ ⎛ 14 ⎞ ⎛1⎞
(v) ⎜ ⎟ ⎜ ⎟ ⎜ ⎟ ⎜ ⎟
normal of π 3 = ⎜ 2 ⎟ × ⎜ −4 ⎟ = ⎜ 14 ⎟ = 14 ⎜ 1 ⎟ OR
⎜ 3 ⎟ ⎜ 1 ⎟ ⎜ −14 ⎟ ⎜ −1 ⎟
⎝ ⎠ ⎝ ⎠ ⎝ ⎠ ⎝ ⎠
⎛ ⎞
1
⎜ ⎟
normal of π 3 = director vector of l = ⎜ 1 ⎟
⎜ −1 ⎟
⎝ ⎠
⎛1⎞
π 3 : r . ⎜⎜ 1 ⎟⎟ = d
⎜ −1 ⎟
⎝ ⎠
d d
perpendicular distance from origin to π 3 = =
⎛1⎞ 3
⎜ ⎟
⎜1⎟
⎜ −1 ⎟
⎝ ⎠
d
=± 3 ∴ d = ±3
3
⎛1⎞ ⎛1⎞
π 3 : r . ⎜⎜ 1 ⎟⎟ = 3 or π 3 : r . ⎜⎜ 1 ⎟⎟ = −3
⎜ −1 ⎟ ⎜ −1 ⎟
⎝ ⎠ ⎝ ⎠

Qn Solution
10 Functions + Maclaurin’s Series
(i) R g = [− ln 2, ∞) and D f =
Since R g ⊆ D f , the composite function fg exists.
(ii) ⎛ 1 ⎞
fg( x) = cos ⎜ ln ⎟
⎝ 1+ x ⎠
= cos ( − ln (1 + x ) )
⎛ x 2 x3 ⎞
≈ cos ⎜ − x + − ⎟
⎝ 2 3⎠
2 4
⎛ x 2 x3 ⎞ ⎛ x 2 x3 ⎞
⎜ − x + − ⎟ ⎜ − x + − ⎟
≈ 1− ⎝ 2 3⎠ ⎝
+
2 3⎠
−L
2! 4!
1 ⎡⎛ ⎤
2 2
x2 ⎞ ⎛ x 2 ⎞ ⎛ x3 ⎞ ⎛ x3 ⎞
= 1 − ⎢⎜ − x + ⎟ − 2 ⎜ − x + ⎟ ⎜ ⎟ + ⎜ ⎟ ⎥ +L
2 ⎢⎝ 2⎠ ⎝ 2 ⎠⎝ 3 ⎠ ⎝ 3 ⎠ ⎥⎦

= 1 − ( x 2 − x3 ) + L
1
2
1 1
= 1 − x 2 + x3 + L
2 2

(iii) h(1) = −1 so h −1 (−1) = 1 .

Let y = − ln(1 + x ) . Then x = e − y − 1 .


For −1 < x < 1 , h( x ) = − ln(1 + x ) so h( x ) > − ln 2 .
Thus h −1 ( x) = e− x − 1 for x > − ln 2 .
Thus
−1 e− x − 1 for x > − ln 2,
h :xa
1 for x = −1.

Qn Solution
11 Complex Numbers
(a) ⎛ π π⎞
z = 3 + i = 2 ⎜ cos + i sin ⎟
⎝ 6 6⎠
⎛ nπ nπ ⎞
z n = 2 n ⎜ cos + i sin ⎟
⎝ 6 6 ⎠
⎛ nπ ⎞
z n − ( z * ) = 2i Im( z n ) = 2i ⎜ 2n sin
n
⎟=0
⎝ 6 ⎠

sin =0
6

= kπ , k ∈ Z +
6
The set of values of n is {n : n = 6k , k ∈ Z + }
(b) z+w= ( ) (
3 −1 + i 1+ 3 )
Im

(
i 1+ 3 ) P

W i 3

i Z

Re
−1 O 3 −1 3

Note that OWPZ is a parallelogram,


3 2π
arg( w) = π − tan −1 =
1 3
2π π π
∠WOZ = − =
3 6 2
OZ= z = 2
OW= w = −1 + i 3 = 2
π
Since ∠WOZ = and OZ = OW , OWPZ is a square. (shown)
2

π π 5π
arg( z + w) = arg( z ) + ∠POZ = + =
6 4 12

3 +1
Also, arg( z + w) = tan −1
3 −1

5π 3 +1
∴ = tan −1
12 3 −1
( )
2
5π 3 +1 3 +1 4+2 3
∴ tan = = = = 2 + 3 (deduced)
12 3 −1 3 −1 2

Qn Solution
12 Graphing + tangent/normal
(i) 2 x 2 − 3x + 1
Let y =
x2 − 4
yx 2 − 4 y = 2 x 2 − 3 x + 1
( 2 − y ) x 2 − 3x + ( 4 y + 1) = 0
For real roots of x,
Discriminant ≥ 0
( −3) − 4 ( 2 − y )( 4 y + 1) ≥ 0
2

(
9 − 4 8 y + 2 − 4 y2 − y ≥ 0 )
16 y 2 − 28 y + 1 ≥ 0
For 16 y 2 − 28 y + 1 = 0
( −28 ) − 4 (16 )
2
28 ±
y=
2 (16 )
28 ± 720
=
32
7±3 5
=
8
∴16 y 2 − 28 y + 1 ≥ 0
7−3 5 7+3 5
y≤ or y≥
8 8

7−3 5 7+3 5
Hence, C cannot lie between and
8 8
(ii)

y
2 x 2 − 3x + 1
y=
x2 − 4

y=2
( 0.764, 0.0365) ( 5.23,1.71)
1 1
O x
1 2

4

x = −2 x=2

(iii) 22
At x = −5, y=
7
dy
Using GC, = 0.40136
dx x =−5
Equation of tangent at x = −5 is

22
y− = 0.40136 ( x + 5 )
7
y = 0.40136 x + 5.149657
When x = 0, y = 5.149657 = 5.15 (to 3 s.f)
The coordinates of P are ( 0,5.15 ) (shown)

Equation of normal at x = −5 is

22 −1
y− = ( x + 5)
7 0.40136
y = −2.4915 x − 9.3148
When x = 0, y = −9.3148 = −9.31 (to 3 s.f)
The coordinates of Q are ( 0, −9.31) (shown)

1
Area of PQR = ( 5.15 + 9.31)( 5 )
2
= 36.2 units2
H2 MATHS (9740) JC2 PRELIMINARY EXAM 2010
PAPER 2 SUGGESTED SOLUTIONS

Qn Solution
1 Equations and Inequalities
x+7
≤1
4 + 3x − x 2
x+7
−1 ≤ 0
4 + 3x − x 2
(
x + 7 − 4 + 3x − x 2
≤0
)
4 + 3x − x 2
x + 7 − 4 − 3x + x2
≤0
− ( x2 − 3x − 4 )
x2 − 2 x + 3
≥0
x 2 − 3x − 4
Since x 2 − 2 x + 3 = ( x − 1)2 + 2 > 0 for x ∈ ,
x 2 − 3x − 4 > 0
( x − 4 )( x + 1) > 0 + − +
x < −1 or x > 4 −1 4

Qn Solution
2 Vectors
p × q = 3p × r
p × ( q − 3r ) = 0
p // ( q − 3r )
q − 3r = λ p , where λ is a scalar.

( q − 3r ) . ( q − 3r ) = λ p.λ p
q − 6q.r + 9 r = λ 2 p
2 2 2

⎛5⎞
52 − 6 × 5 × 2 ⎜ ⎟ + 9 ( 2 ) = λ 2 (1)
2 2

⎝6⎠
λ 2 = 11
λ = ± 11

Qn Solution
3 Mathematical Induction
(i) Let Pn be the statement:

∑ 3( 2− r ) = 3 ( 2n +1 ) − n ( n + 1) − 6, ∀n ∈ + .
n
r 3
r =1 2
Check P1 ,

∑ 3( 2 − r ) = 3 ( 21 − 1) = 3
1
r
LHS of P1 :
r =1
RHS of P1 : 3 ( 21+1 ) −
3
(1)(1 + 1) − 6 = 12 − 3 − 6 = 3 = LHS
2
∴ P1 is true.
Assume Pk true for some k ∈ + .

∑ 3(2
− r ) = 3 ( 2k +1 ) − k ( k + 1) − 6 ----(*)
k
r 3
r =1 2
k +1
Prove Pk +1 true, i.e. ∑ 3 ( 2r − r ) = 3 ( 2k + 2 ) − ( k + 1)( k + 2 ) − 6
3
r =1 2
LHS of Pk +1 :

( ) ⎢∑ 3 ( 2r − r )⎤⎥ + 3 ⎡⎣2k +1 − ( k + 1)⎤⎦


⎡ k
k +1


r =1
3 2 r
− r =
⎣ r =1 ⎦
= 3 ( 2k +1 ) − k ( k + 1) − 6 + 3 ⎡⎣ 2k +1 − ( k + 1) ⎤⎦
3
2
= 3 ( 2k +1 ) − k ( k + 1) − 6 + 3 ( 2k +1 ) − 3 ( k + 1)
3
2
= 3 ( 2k +1 ) + 3 ( 2k +1 ) − k ( k + 1) − 3 ( k + 1)
3
2
⎛1 ⎞
= 2 ⎡⎣3 ( 2k +1 ) ⎤⎦ − 3 ( k + 1) ⎜ k + 1⎟
⎝2 ⎠
= 3 ( 2k + 2 ) − ( k + 1)( k + 2 ) − 6 = RHS
3
2
∴ Pk true ⇒ Pk +1 true
Since P1 is true and Pk true ⇒ Pk +1 true , therefore, by Mathematical
Induction, Pn is true ∀n ∈ + .

∑ 3(2 − r ) > 5800


n
(iii) r

r =5

∑ 3(2 − r ) − ∑ 3 ( 2r − r ) > 5800


n 4
r

r =1 r =1

⎡ ⎤ ⎡ ⎤
⎢⎣3 ( 2 ) − 2 ( n )( n + 1) − 6 ⎥⎦ − ⎢⎣3 ( 2 ) − 2 ( 4 )( 5 ) − 6 ⎥⎦ > 5800
n +1 3 4 +1 3

⎡ ⎤
⎢⎣3 ( 2 ) − 2 ( n )( n + 1) − 6 ⎥⎦ − 60 > 5800
n +1 3

3 ( 2 n +1 ) − ( n )( n + 1) − 66 > 5800
3
2
3 ( 2 n +1 ) − ( n )( n + 1) > 5866
3
2
Using GC, using table method
⎡ ⎤
When n = 9, ⎢3 ( 2n +1 ) − ( n )( n + 1) ⎥ = 2737<5866
3
⎣ 2 ⎦
⎡ ⎤
When n = 10, ⎢3 ( 2n +1 ) − ( n )( n + 1) ⎥ = 5979>5866
3
⎣ 2 ⎦
∴ Least n = 10
Qn Solution
4 Application of Differentiation (Rate of change)
(a) y = x + ln ( xy )
3

= x + 3ln ( x ) + 3ln ( y )
dy 1 3 3 dy
= + +
dx 2 x x y dx
when x = 1 and y = 1,
dy 7
=−
dx 4

(b) 1
V = π r 2h
3
1 3
= π h (from diagram r = h)
3
dV
= π h2
dh
dh dh dV
= .
dt dV dt
1
= 2 (1)
πh
After 2 min, amt of water poured into cone = 2 × 1× 60
= 120 cm3
Amt of water in cone after 2 min = 120 + 30
= 150 cm3
1
∴ π r 2 h = 150
3
1 3
π h = 150
3
450
h= 3
π
dh 1
= 2
dt ⎛ 450 ⎞
π⎜3 ⎟
⎝ π ⎠
= 0.0116

Qn Solution
5 Integration (area and volume)
y
(i)
b
R

-a a x

-b
Area of R = Area of quadrant-Area of triangle
0 b2 x2 1
=∫ b2 − 2
dx − ab
−a a 2
1 0 b
= − ab + ∫ a 2 − x 2 dx (shown)
2 − a a

x = a sin θ
dx
= a cos θ

⎛ 0 = a sin θ ⇒ θ = 0 ⎞
⎜ ⎟
⎜ −a = a sin θ ⇒ θ = − π ⎟
⎝ 2⎠
0 b 1
∫ −a a a − x dx − 2 ab
2 2

b 0 1
= ∫ π a 2 − ( a sin θ ) ( a cos θ ) dθ − ab
2

a 2 − 2
a 2b 0 1
= ∫ π 1 − sin 2 θ ( cos θ ) dθ − ab
a − 2
2
0 1
= ab ∫ π cos 2 θ dθ − ab
− 2
2
0 1 + cos 2θ 1
= ab ∫ π dθ − ab
− 2 2
2

sin 2θ ⎤
0
⎡1 1
= ab ⎢ θ + ⎥ − ab
⎣2 4 ⎦ −π 2
2

⎡ ⎛ 1 ⎛ π ⎞ sin ( −π ) ⎞ ⎤ 1
= ab ⎢0 + 0 − ⎜ ⎜ − ⎟ + ⎟ ⎥ − ab
⎢⎣ ⎝2⎝ 2 ⎠ 4 ⎠ ⎥⎦ 2
⎛π ⎞ 1
= ab ⎜ ⎟ − ab
⎝4⎠ 2
ab
= (π − 2 ) units 2
4
(ii) Volume generated
b 1
= π ∫ x 2 dy − π a 2b
0 3
2 2
b a y 1
= π ∫ a 2 − 2 dy − π a 2b
0 b 3
b
⎡ a2 y3 ⎤ 1
= π ⎢ a 2 y − 2 ⎥ − π a 2b
⎣ 3b ⎦ 0 3
⎡ 2 a 2b3 ⎤ 1 2
= π ⎢a b − 2 ⎥ − π a b
⎣ 3b ⎦ 3
⎡ a 2b ⎤ 1 2
= π ⎢ a 2b − − πa b
⎣ 3 ⎥⎦ 3
⎛2 ⎞ 1
= π ⎜ a 2 b ⎟ − π a 2b
⎝3 ⎠ 3
1
= π a 2b
3

Qn Solution
6 Sampling Methods
(a) Quota sampling.
It will not be representative of the cohort as it misses out those who do not visit
the canteen.
OR
It is not random.

(b) Step (2) is incorrect.


The interval size should be 4 students. He should pick the random start student
from the 1st-4th student and select every 4th student thereafter.
Qn Solution
7 Probability
(i) Set: S = Success; F = Failure
5. 5 2
7 7

S F

5 16 5 2
21 21
7 7
S F S F

5 58 5 2 5 16 5 2
63 63 7 7 21 21 7 7

S F F S S F S F
⎛ 5 ⎞ ⎛ 16 ⎞ ⎛ 2 ⎞ ⎛ 2 ⎞ ⎛ 5 ⎞ ⎛ 16 ⎞ ⎛ 2 ⎞ ⎛ 2 ⎞ ⎛ 5 ⎞
Required Prob = ⎜ ⎟ ⎜ ⎟ ⎜ ⎟ + ⎜ ⎟ ⎜ ⎟ ⎜ ⎟ + ⎜ ⎟ ⎜ ⎟ ⎜ ⎟
⎝ 7 ⎠ ⎝ 21 ⎠ ⎝ 7 ⎠ ⎝ 7 ⎠ ⎝ 7 ⎠ ⎝ 21 ⎠ ⎝ 7 ⎠ ⎝ 7 ⎠ ⎝ 7 ⎠
380
=
1029
(ii) Required Prob = 1 − P ( Player fails in all three games )
⎛ 2 ⎞ ⎛ 2 ⎞ ⎛ 2 ⎞ 335
= 1− ⎜ ⎟⎜ ⎟⎜ ⎟ =
⎝ 7 ⎠ ⎝ 7 ⎠ ⎝ 7 ⎠ 343

(iii) Let Event A = Player has exactly two failures


Let Event B = Player has at least one success
P ( A ∩ B)
Required Prob = P ( A | B ) =
P ( B)
⎛ 380 ⎞
(i) ⎜⎝ 1029 ⎟⎠
= =
(ii) ⎛ 335 ⎞
⎜ ⎟
⎝ 343 ⎠
76
=
201

Qn Solution
8 Permutations and Combinations
(a) No of teams
3 tallest, no shortest ⎛ 3 ⎞⎛ 8 ⎞
⎜ ⎟⎜ ⎟ = 56
⎝ 3 ⎠⎝ 5 ⎠
3 tallest, 1 shortest ⎛ 3 ⎞⎛ 2 ⎞⎛ 8 ⎞
⎜ ⎟⎜ ⎟⎜ ⎟ = 140
⎝ 3 ⎠⎝ 1 ⎠⎝ 4 ⎠
2 tallest, no shortest ⎛ 3 ⎞⎛ 8 ⎞
⎜ ⎟⎜ ⎟ = 84
⎝ 2 ⎠⎝ 6 ⎠
2 tallest, 1 shortest ⎛ 3 ⎞⎛ 2 ⎞⎛ 8 ⎞
⎜ ⎟⎜ ⎟⎜ ⎟ = 336
⎝ 2 ⎠⎝ 1 ⎠⎝ 5 ⎠

Total no of teams= 56 + 140 + 84 + 336 = 616

b(i) No of different 5-letter secret codes


= No of codes using {X,Y,Z,Z,Z}+ No of codes using {X,Y,Y,Z,Z}+No of codes
using {Y,Y,Z,Z,Z}
5! 5! 5!
= + +
3! 2!2! 2!3!
= 20 + 30 + 10 = 60

(ii) Y {Z, Z, Z} Y or Y {X, Z, Z} Y

No of 5-letter secret codes that will begin and end with Y


= 1+ 3 = 4

Qn Solution
9 Hypothesis Testing

Unbiased estimate of μ is t =
∑ t = 213.5 = 4.27
n 50
1 ⎡
( )
∑ t −t ⎤
2
Unbiased estimate of σ 2 is s 2 =
n − 1 ⎣⎢ ⎦⎥
1
= [ 44.105]
49
= 0.90010 ≈ 0.900 (3 s.f)

⎛ σ2 ⎞
Since n = 50 is large, by Central Limit Theorem, X N ⎜ μ , ⎟
⎝ 50 ⎠
approximately.
H0: μ = 4.5
H1: μ < 4.5
X −μ
Test Statistic: Z =
S
n
Level of significance: 1%
Critical Region: Reject H0 if p -value < 0.01

Assuming H0 is true, from the GC, p -value = 0.0432439 .


Since p -value = 0.0432 > 0.01 , we do not reject H0 and conclude that there is
no significant evidence, at 1% level, that the population mean time spent is less
than 4.5 minutes.

In order for H0 to be rejected in favour of H1, we require 0.0432439 < 0.01α


⇒ α > 4.3244
4.33% is the smallest level of significance resulting in the rejection of H0.
It is not necessary to assume that the population follows a normal distribution
for the test to be valid because since n = 50 is large, by Central Limit Theorem,
⎛ σ2 ⎞
X N ⎜ μ , ⎟ approximately.
⎝ 50 ⎠

Qn Solution
10 Binomial + Poisson (incl. CLT)
(i) Let H be the number of calls received by the hospital in a week.
Let S be the number of calls received by the police station in a week.
Then H ~ Po (12 ) , S ~ Po ( 5 ) .
Also, H + S ~ Po (17 ) .
P ( H + S > 11) = 0.91533
≈ 0.915

(ii) Let W be the number of weeks, out of 50, in which the hospital and police
station receive a total of at most 11 calls.
Then W ~ B ( 50,1 − 0.91533 ) . i.e. W ~ B ( 50, 0.08467 ) .

Since n = 50 is large, p = 0.08467 is small, and np = 4.2335 < 5 ,


W ~ Po ( 4.2335 ) approximately.
From GC,
P (W ≤ 4 ) = 0.58332
≈ 0.583

(iii) Since n = 100 is large, by the Central Limit Theorem,


⎛ 12 ⎞
H N ⎜ 12, ⎟ approximately
⎝ 100 ⎠
⎛ 12 12 ⎞
H1 + H 2 N ⎜12 + 12, + ⎟ ( approximately )
⎝ 100 100 ⎠
P ( H1 + H 2 ≤ 23) = 0.020613
≈ 0.0206

Qn Solution
11 Normal Distribution
(a)(i) Let the weight of 1 King Crab and 1 Snow Crab be K and S respectively.
K ~ N (1.65, 0.71)
S ~ N (1.10, 0.34 )
K1 + K 2 + K3 + S1 + S2
Let T =
5
1
E (T ) = ⎡⎣3E ( K ) + 2E ( S ) ⎤⎦
5
1
= ⎣⎡3 (1.65 ) + 2 (1.10 ) ⎦⎤
5
= 1.43
1
Var (T ) = ⎡3Var ( K ) + 2Var ( S ) ⎤⎦
52 ⎣
1
= ⎡3 ( 0.71) + 2 ( 0.34 ) ⎤⎦
25 ⎣
= 0.1124
∴ T ~ N (1.43, 0.1124 )
P ( T < 1.5 ) = 0.583

(ii) Let the selling price of 1 King Crab and 1 Snow Crab be X and Y respectively.
( )
X = 40 K ~ N 40 × 1.65, 402 × 0.71 ⇒ X ~ N ( 66,1136 )
( )
Y = 45S ~ N 45 ×1.10, 45 × 0.34 ⇒ Y ~ N ( 49.5, 688.5 )
2

Let the total selling price of an King Crab and a Snow Crab be C = X + Y
C ~ N ( 66 + 49.5,1136 + 688.5 ) ⇒ C ~ N (115.5,1824.5 )
P ( C > 140 ) = 0.283

(iii) The weight of all crabs are independent of one another.

NOTE: Weight of snow crab and weight of king crab is independent of each other
is insufficient, students need to bring out that the weight of all crabs are
independent of one another.

(b) Let X be the mass of a bar of chocolate, X ~ N ( μ , σ 2 ) .


P( X − μ ≤ m) = 0.2
P( Z ≤ m ) = 0.2
σ

P( − m ≤ Z ≤ m ) = 0.2
σ σ
P( Z ≤ − m ) = 0.4
σ
m = 0.25335
σ
P( X − μ ≥ 3m)
= P( Z ≥ 3m )
σ
= P( Z ≥ 0.76004)
= 0.224

Qn Solution
12 Correlation & Regression
(i) y
48000

31800
1339 2275 x

The linear model is not valid in the long run, as it is impossible to have the
selling price of the car to be $0 or negative when the number of new car licenses
increases to a certain number.
(ii) Model C is the most appropriate, as when x increases, y decreases at a
decreasing rate.

(iii) r = 0.97817 = 0.978 (3 s.f.)


It indicates a strong positive linear correlation between y and 1 / x .

Regression line:
(iv) 50012000
y = 8452.3 + (5.s.f)
x

(a) When x =1300,


50012000
y = 8452.3 + = 46923.07 = $46900 (3s.f.)
1300
Since x =1300 lies outside the data range, the linear relation may no longer hold,
hence, the estimate is unreliable.

(b) When x =2000,


50012000
y = 8452.3 + = 33458.30 = $33500 (3s.f.)
2000
Since x =2000, lies within the data range, and r = 0.978 is close to 1, the
estimate is reliable.

(v) No, I do not agree, as there is no causal effect between the two variables. The
rise in the selling price of the car could be due to other factors like the
production cost of the car.
  NATIONAL JUNIOR COLLEGE

PRELIMINARY EXAMINATIONS
  Higher 2
 

MATHEMATICS 9740/01
Paper 1 13 September 2010
3 hours
Additional Materials: Answer Paper
List of Formulae (MF15)
Cover Sheet 0815 – 1115 hours

READ THESE INSTRUCTIONS FIRST


Write your name, registration number, subject tutorial group, on all the work you hand in.
Write in dark blue or black pen on both sides of the paper.
You may use a soft pencil for diagrams or graphs.
Do not use paper clips, highlighters, glue or correction fluid.

Answer all the questions.


Give non-exact numerical answers correct to 3 significant figures, or 1 decimal place in the case of
angles in degrees, unless a different level of accuracy is specified in the question.
You are expected to use a graphic calculator.
Unsupported answers from a graphic calculator are allowed unless a question specifically states
otherwise. Where unsupported answers from a graphic calculator are not allowed in a question,
you are required to present the mathematical steps using mathematical notations and not calculator
commands.
You are reminded of the need for clear presentation in your answers.

At the end of the examination, fasten all your work securely together.
The number of marks is given in the brackets [ ] at the end of each question or part question.

This document consists of 9 printed pages.

  National Junior College

[Turn Over

 
2

1 The sum of the digits in a three-digit-number is 15. Reversing the digits in that number
decreases its value by 594. Also, the sum of the tenth digit and four times the unit digit is
five more than the hundredth digit. Find the number. [4]

p 2
2 Find the value of p such that  0
3x dx 
ln 3
. [2]

The graph of y  3x for 0  x  1 , is shown in the diagram below. Rectangles, each of width

1
, where n is an integer, are drawn under the curve.
n

y  3x

x
0 1  2  3

n3 n  2 n 1
1
n n n n n n

Show that the total area of all n rectangles, A, is given by


2 3n  1

. [2]
 
(i) 1
n 3n  1

(ii) State the limit of A as n   . [1]

NJC 2010 9740/01/2010 [Turn Over


3

3 The points A and B have position vectors a and b respectively, relative to the origin O such
that a = b . The point P with position vector p lies on AB such that b • p = a • p .

(i) Show that AB is perpendicular to OP. [2]

(ii) Determine the position vector of the point D in terms of a and b, where D is the
reflection of O about the line AB. [2]

(iii) Give the geometrical meaning of a  b . [1]

π
4 The diagram shows the graph of y  sin x for 0  x  . P is a fixed point on the curve
2
with coordinates  x1 ,sin x1  and Q is a point on the curve with coordinates
 x   ,sin  x     , where 
1 1 is measured in radians.

y  sin x
Q

x
O

sin x1 (cos   1)  cos x1 sin 


(i) Prove that the gradient of PQ is . [2]

(ii) Given that  is sufficiently small for  3 and higher powers of  to be neglected,
express the gradient of PQ as a linear expression in terms of  . [2]

Verify that the gradient of PQ approximates to the gradient of the tangent at P when
 tends to zero. [2]

NJC 2010 9740/01/2010 [Turn Over


4

5 Given that z  3  4i  5 , illustrate the locus of the point P representing the complex

number z in an Argand diagram. [1]

Hence, find the least exact value of z  1 . [2]

The locus of point Q, representing the complex number w, satisfies the relation
w  2i  w  ai , where a is a real number, a  2 . Find the range of values of a such that

the loci of P and Q meet more than once. [3]

6 By using the substitution y  vx , find the general solution of the differential equation

dy
x  3x  y  2 . [4]
dx

(i) State the equation of the locus where the stationary points of the solution curves lie.
[1]

(ii) Sketch, on a single diagram, the graph of the locus found in part (i) and two
members of the family of solution curves where the arbitrary constant in the solution
is non-zero. [3]

7(a) By considering x  2  A(2  2 x)  B where A and B are real constants, or otherwise, find

x2
  x2  2x  8
dx. [5]

1
(b) Show that  e x cos 2 x dx  e x (cos 2 x  2 sin 2 x)  c , where c is an arbitrary constant.
5

Hence, find e cos 2 x dx. [5]


x

NJC 2010 9740/01/2010 [Turn Over


5

8 (a) The sum, Sn , of the first n terms of an arithmetic progression is given by S n  n 2  2n .

Write down the expression for S n  Sn 1 . Hence, find the value of the common difference.
[3]

(b) A metal screw of length L (measured in millimetre) is driven into a concrete wall by an
electrical screwdriver, such that its distance driven into the wall is proportional to the angle
turned by the screwdriver.

Wall
L

Due to some reasons, every subsequent turn by this electrical screwdriver can only achieve
an 80% of the angle turned previously.

(i) Given that the initial angle turned by the screwdriver is  radians, write down the
expressions for the first 3 distances driven into the concrete wall, leaving your
answers in terms of  and k, where k is the constant of proportionality. [2]

(ii) Find the total distance driven into the concrete walls after n turns, leaving your

 
answer in the form ak 1  b n , where a and b are constants to be determined. [3]

(iii) Given that k  2 and assuming that the total distance driven could never exceed the
length of the screw, find the minimum length of the metal screw required, giving
your answer in terms of  . [2]

NJC 2010 9740/01/2010 [Turn Over


6

0
  
9 Relative to the origin O, the point A has position vector given by OA   1  and A lies on
0
 

1
 
the plane 1 with equation defined by r •  3   3 . Another plane  2 has equation y  x .
 2
 
The planes 1 and  2 intersect at line l.

(i) Find the vector equation of the line l. [1]

7
(ii) Show that the cosine of the acute angle between the planes 1 and  2 is . [2]
7

(iii) Find the position vector of the foot of perpendicular, OF , from point A to the line l.

Hence, find the exact length of projection of AF onto the plane  2 . [5]

(iv) Another plane  3 has equation px  qy  1 , where p and q are real constants. Find

the condition in which p and q must satisfy such that the planes 1 ,  2 and  3
intersect at exactly one point. [2]

10(a) Given that 2  3i is a solution to the equation

z 2  (a  i) z *  16  bi  0 ,

where z * is the conjugate of the complex number z, find the values of a and b, where a and
b are real constants [3]

(b) (i) Solve the equation z 5  1  0 , expressing the solutions in the form rei , where r  0

and  π    π . Show the roots of the equation on an Argand diagram. [4]


i   i
(ii) For  π    π , show that 1  e  2 cos   e 2 . [2]
2

Deduce the roots of  w  1  1  0 in the exponential form.


5
(iii) [3]

NJC 2010 9740/01/2010 [Turn Over


7

11 The curve C has equation

ax 2  bx  1
f(x)  , where a, b and c are real constants.
xc

Given that the line y  2 x  1 is an asymptote of C, find the value of a and show that
b  2c 1 . [3]

(i) For c  1 , using algebraic method, prove that the curve C cannot lie between 2

values, which are to be determined. [3]

2 x2  x  1
(ii) Sketch the graph of f(x)  , showing clearly its asymptotes, the coordinates of
x 1
the axial intercepts, and turning point(s) (if any). [3]

Hence, state the range of x for which f(x) is concaving downwards. [1]

(iii) Given that the line y  kx  k  3 ,where k is a real constant, passes through the
intersection of the asymptotes of C, deduce the range of k where

2 x 2  x  1  (kx  k  3)( x  1)

has 2 real solutions. [1]

NJC 2010 9740/01/2010 [Turn Over


8

12 A curve C has parametric equations

x  2 2 and y   3  2 , where  is a real parameter.

Sketch the curve C for 0    3 . [1]

The tangent to C at point P  2 2 ,  3  2  cuts the y-axis at point Q. Show that the equation

of the tangent at P may be written as

4 y  3 x  2  4   3  . [2]

(a) (i) C cuts the y-axis at the point R. Find the area of triangle PQR, A, in terms of  . [2]

(ii) If x increases at a rate of 4 units per second when   2 , find the rate of change of
A at that instant. [3]

(b) Calculate the exact area of the region bounded by the curve C, the tangent to C at
point P when   2 and the y-axis. [5]

End of Paper

NJC 2010 9740/01/2010 [Turn Over


9

  NATIONAL JUNIOR COLLEGE

PRELIMINARY EXAMINATIONS
 
Higher 2

MATHEMATICS 9740/01
Higher 2 Paper 1
13 September 2010, Monday 0815 – 1115 hours

Candidate Name: ______________________ Registration No.:_____________

Subject Class: 2ma______/ 2IPma2_______ Subject Tutor: _______________


  For office use
Question No. Marks TOTAL
  over Page Obtained MARKS

1 4
INSTRUCTIONS TO CANDIDATES
2 5
Write your name, registration number, subject
tutorial group, subject tutor’s name and calculator 3 5
model in the spaces provided on the cover sheet
and attached it on top of your answer paper. 4 6

Circle the questions you have attempted and 5 6


arrange your answers in NUMERICAL ORDER.
6 8
Write your calculator’s model number(s) in the
box below. 7 10

Scientific Calculator Model: 8 10

9 10

Graphic Calculator Model: 10 12


  
11 11

12 13

Presentation –1 / –2

TOTAL 100

GRADE

NJC 2010 9740/01/2010 [Turn Over


 
NATIONAL JUNIOR COLLEGE

PRELIMINARY EXAMINATIONS
  Higher 2

MATHEMATICS 9740/02
Paper 2 17 September 2010
3 hours
Additional Materials: Answer Paper
List of Formulae (MF15)
Cover Sheet 0815 – 1115 hours

READ THESE INSTRUCTIONS FIRST


Write your name, registration number, subject tutorial group, on all the work you hand in.
Write in dark blue or black pen on both sides of the paper.
You may use a soft pencil for diagrams or graphs.
Do not use paper clips, highlighters, glue or correction fluid.

Answer all the questions.


Give non-exact numerical answers correct to 3 significant figures, or 1 decimal place in the case of
angles in degrees, unless a different level of accuracy is specified in the question.
You are expected to use a graphic calculator.
Unsupported answers from a graphic calculator are allowed unless a question specifically states
otherwise. Where unsupported answers from a graphic calculator are not allowed in a question,
you are required to present the mathematical steps using mathematical notations and not calculator
commands.
You are reminded of the need for clear presentation in your answers.

At the end of the examination, fasten all your work securely together.
The number of marks is given in the brackets [ ] at the end of each question or part question.

This document consists of 9 printed pages.

  National Junior College

[Turn Over
2

Section A: Pure Mathematics [40 marks]

1 The functions g and h are defined as follows:

1
g : x  2 x 1  , x  0,
2
h : x  x  3x  2, x  0.
2

(i) Justify why hg exists and find the range of hg. [3]

(ii) The function h has an inverse if its domain is restricted to x  b . Find the value of b
for this domain to be maximal. Sketch the graph of h and its inverse on the same
diagram. [2]

(iii) Solve h 1 (x)  x exactly. [2]

2 (a) A sequence of positive real numbers x1 , x2 , x3 , satisfies the recurrence relation


xn 1  3   ln  xn  1  for n  1 . Given that the sequence converges to L, find the value of L.
2

[3]

r 2  7r  11 A B
(b) (i) By expressing in the form  where A and B are
 r  4 !  r  2 !  r  4 !

n
r 2  7 r  11 5 n5
real constants, show that 
r 1
 
 r  4 ! 4!  n  4 !
. [3]

(ii) Use the method of mathematical induction to prove your result in (i). [5]

 r 2  9r  19
(iii) Hence, find   r  5!
r 1
. [3]

NJC 2010 9740/02/2010 [Turn Over


3

dy
3 Given that y  2 esin x , show that 2  y cos x .
dx
By further differentiation of this result, find the Maclaurin’s series for y, up to and
including the term in x3 . [5]

Deduce the Maclaurin’s series for e  sin x up to and including the term in x 2 . [3]

4  x2 x
4 (i) Use the substitution x  2 tan  to show  dx   C , where C is an
4  x 
2 2 4  x2

arbitrary constant. [5]


y
(ii)

 3 15y  3  x  2   
1, 
 5 

x
–2 O 2

4  x2
The diagram above shows the curve with equation y 2  with stationary
4  x  2 2

points at x  0 . The line 15y  3  x  2  intersects the curve at  2, 0  and

 3
1,  .
 5 

4  x2
(a) The region bounded by the curve y and the line
 4  x2 
2

15y  3  x  2  is rotated through 4 right angles about the x-axis to form a

solid of revolution of volume V. Find the exact value of V, giving your


answer in the form bπ. [4]

4  x2
(b) Sketch the gradient graph of y   . [2]
4  x  2 2

NJC 2010 9740/02/2010 [Turn Over


4

Section B: Statistics [60 marks]

5 The Head of Mathematics department of Holistics Junior College decides to take a survey
of opinions of 700 graduating students regarding the quality of teaching of the subject.

(i) What is the sampling frame in the context of the question? [1]

(ii) Describe clearly how a systematic sample of size 140 can be obtained. [3]

6 The data in the table below were obtained in an experiment to estimate the relation between
d, the duration of a television commercial (in seconds) and s, the average sales of a
particular brand of detergent (in thousands of bottles):

Duration, d 15 18 22 25 26 29 34 39
Average Sales, s 0.43 1.12 1.75 1.98 2.11 2.26 2.40 2.44

(i) Draw a scatter diagram to illustrate the data. [1]

(ii) Fit a model of the form s     ln d to the data above and find the least squares
estimates of  and  . [1]

Hence state the product moment correlation coefficient between ln d and s, and explain
whether your answer suggests that a linear model is appropriate for the transformed
variables. [2]

(iii) Using the regression model in part (ii), predict the average sales if the duration of a
television commercial is 28 seconds. Comment on the reliability of your answer.
[3]

NJC 2010 9740/02/2010 [Turn Over


5

7 Four married couples attend a wedding dinner. One of the couples brought along two

children. Find the number of ways in which these ten people can be seated round a table if

(i) there are no restrictions, [1]


(ii) each couple must sit together. [3]

They are to take a photo with the bride and bridegroom. The twelve people are to arrange in
two rows of six and the bride and bridegroom must be together in the middle of the front
row. How many ways can the photographer arrange the twelve people such that the two
children must also be in the front row? [3]

In this question, give each of your answers as an exact fraction in its lowest term.

8 (a) In a certain sample space, it is known that events A and B are independent. Given that
3 2
P  A  B  and P  A ' B   , where A '  is the complement of event A, find
4 15

(i) P  B , [3]

(ii) P  A  B A  B . [2]

(b) A teacher is to form two groups of 4 students from a class of 3 boys and 5 girls for
Mathematics consultation session. Three of the girls, Ivy, Tamie and Cassy are good friends
from the class. Find the probability that

(i) the boys are together in the same group, [2]

(ii) either Ivy or Tamie is in the same group as Cassy. [3]

NJC 2010 9740/02/2010 [Turn Over


6

9 Suppose that the arrival and departure of aircrafts at a domestic airport follow two
independent Poisson distributions. In a one-hour period, it is expected that there are 4
arrivals and 3 departures.

(i) Show that the probability that there are at least 13 arrivals in a two-hour period is
0.0638, correct to 3 significant figures. [1]

(ii) Find the probability that, in a randomly selected one-hour period, there are less than
2 departures given that the airport handles a total of exactly 9 arrivals and departures.
[3]

A study of the domestic airport arrivals for 60 randomly selected two-hour periods is being
conducted to see if there are at least 13 arrivals for each two-hour period.

(iii) Giving two reasons, in this context, explain why the binomial distribution is a
suitable model for the study of the domestic airport arrivals. [2]

(iv) Using a suitable approximation, find the probability that there are at most 50 two-
hour periods with less than 13 arrivals each, explaining clearly why the
approximation is appropriate. [3]

NJC 2010 9740/02/2010 [Turn Over


7

10 The national average for monthly electricity usage measured in kilowatts hour (kWh), of
Housing Development Authority (HDA) units is 380 kWh. The monthly electricity usage of
3-room units follows a normal distribution with mean of 290 kWh and variance,  2 ,
whereas the monthly electricity usage of 5-room units follows an independent normal
distribution with mean of 450 kWh and variance 105 kWh2.

(i) Calculate the probability that the monthly electricity usage of two randomly chosen
3-room units exceeds 290 kWh each and a randomly chosen 5-room unit’s monthly
electricity usage is less than 450 kWh. [2]

(ii) Given that the probability of the total monthly electricity usage of four randomly
chosen 3-room units exceeds thrice the national average is 0.868, find  2 , correct to
the nearest integer, [3]

With effect from 1 July 2010, the monthly electricity bill is charged at 24 cents per
kilowatts hour.

(iii) Determine the value of a, correct to 2 decimal places, such that the probability of the
monthly electricity bill of a randomly chosen 5-room unit exceeding $a is 0.9. [2]

(iv) Eighty 5-room units are randomly selected. Using a suitable approximation, find the
probability that there are not less than seventy 5-room units with monthly electricity
bill exceeding $a. [3]

NJC 2010 9740/02/2010 [Turn Over


8

11 The drying time, X minutes, of Noppin brand paint under specified test conditions is known
to have mean value 75 minutes.

(a) On one occasion, in the manufacture of a large batch of Noppin paint, it was suspected that
an accidental chemical contamination had resulted in a change in the drying time of the
paint. Due to the high costs involved in discarding the entire batch of paint, the
manufacturer decided he will only do so if there was strong evidence from a test at 5% level
of significance to suggest that the drying time has changed. 50 random and independent
specimens of paint samples were taken from the batch and the drying time is summarised as
follows

 x  3791 x
2
and  287959 .

(i) Determine whether the manufacturer will discard the entire affected batch of paint.
[6]

(ii) Assuming that the unbiased estimate of the population variance is the same as the
one found in the above sample, find the probability such that the total drying time of
another 60 randomly selected specimens of paint samples obtained from the same
batch is between 72 hours and 76 hours. [3]

(b) On another occasion, chemists proposed a new additive designed to decrease the drying
time. The mean and standard deviation from 20 random and independent specimens of paint
samples with the new additive are x minutes and 7.5 minutes respectively. A test is to be
carried out at the 5% level of significance to determine whether the new additive had been
effective.

(i) Determine the largest value of x for which the chemists can claim that their new
additive had been effective in decreasing the drying time of the paint. [3]

(ii) State a necessary assumption for validity of the test. [1]

End of Paper
NJC 2010 9740/02/2010 [Turn Over
9

  NATIONAL JUNIOR COLLEGE

PRELIMINARY EXAMINATIONS
 
Higher 2

MATHEMATICS 9740/02
Higher 2 Paper 2
17 September 2010, Friday 0815 – 1115 hours

Name: _______________________________ Registration No. :_____________

Subject Class: 2ma______/ 2IPma2_______ Subject Tutor: _______________


 
For office use
Question No. Marks TOTAL
  over Page Obtained MARKS

INSTRUCTIONS TO CANDIDATES 1 7

Write your name, registration number, subject 2 14


tutorial group, subject tutor’s name and calculator
model in the spaces provided on the cover sheet 3 8
and attached it on top of your answer paper.
4 11
Circle the questions you have attempted and
arrange your answers in NUMERICAL ORDER. 5 4

Write your calculator’s model number(s) in the 6 7


box below.
7 7
Scientific Calculator Model:
8 10

9 9
Graphic Calculator Model:
10 10
  
11 13

Presentation –1 / –2

TOTAL 100

GRADE

NJC 2010 9740/02/2010 [Turn Over


Suggested Solutions to 2010 NJC SH2 H2 Math Prelim Paper 1

1 Let the unit, tenth and hundredth digits be z, y, x respectively.


x + y + z = 15 − (1)

(100 x + 10 y + z ) − (100 z + 10 y + x) = 594


⇒ 99 x − 99 z = 594 − (2)

y + 4z = x + 5
⇒ −x + y + 4z = 5 − (3)

Using GC to solve the equations simultaneously,


x = 8, y = 5, z = 2 .

Thus the number is 852.

2 p 2
∫ 0
3x dx =
ln 3
p
⎡ 3x ⎤ 2
⎢ ln 3 ⎥ = ln 3
⎣ ⎦0
1 2
ln 3
( 3 p − 30 ) =
ln 3
3 −1 = 2
p

3p = 3
∴ p =1
2(i) Total area of all the n rectangles, A

1 ⎛ 1n ⎞ 1 ⎛ n2 ⎞ 1 ⎛ n3 ⎞ 1 ⎛ nn ⎞
= 3
⎜ ⎟ + 3
⎜ ⎟ + 3
⎜ ⎟ + L + ⎜3 ⎟
n⎝ ⎠ n⎝ ⎠ n⎝ ⎠ n⎝ ⎠
1 ⎛ 1n 2 3 n

= ⎜ 3 + 3 + 3 +L + 3 ⎟
n n n
n⎝ ⎠
⎛ ⎛ 1 ⎞n ⎞
1⎜ ⎜ 3n ⎟ − 1 ⎟
3 ⎜⎝ ⎠
n

= ⎜ 1 ⎟
n ⎜ n
3 −1 ⎟
⎜ ⎟
⎝ ⎠
⎛ ⎞ 1
2 ⎜ 3n ⎟
= ⎝ ⎠
⎛ 1 ⎞
n ⎜ 3 n − 1⎟
⎝ ⎠

Page 1 of 16 

 
Suggested Solutions to 2010 NJC SH2 H2 Math Prelim Paper 1

2(ii) As n → ∞ ,
limit of A = area under the curve y = 3x for 0 ≤ x ≤ 1
1
= ∫ 3x dx
0

2
=
ln 3
uuuv uuuv
3(i) AB ⋅ OP = ( b − a ) • p
=b•p–a•p
= a • p – a • p (since b • p = a • p)
=0
Hence, AB is perpendicular to OP.

3(ii) Since a = b , then P must be the midpoint of AB.


uuur 1
Using ratio theorem, OP = ( a + b )
2
uuuv uuuv
Thus, OD = 2OP
⎛1 ⎞
= 2 ⎜ (a + b ) ⎟
⎝ 2 ⎠
=a+b

3(iii) a × b represents the

(1) area of rhombus OADB or OBDA. (or)


(2) magnitude of a vector which is perpendicular to a and b.

4(i) Gradient of PQ

sin( x1 + θ ) − sin x1
=
( x1 + θ ) − x1
sin x1 cos θ + cos x1 sin θ − sin x1
=
θ
sin x1 (cos θ − 1) + cos x1 sin θ
= (shown)
θ

4(ii) When θ is small,

Gradient of PQ

2                                                                                                                 
Suggested Solutions to 2010 NJC SH2 H2 Math Prelim Paper 1

sin x1 (cos θ − 1) + cos x1 sin θ


=
θ
⎡⎛ 1 ⎞ ⎤
sin x1 ⎢⎜ 1 − θ 2 ⎟ − 1⎥ + θ ( cos x1 )
≈ ⎣⎝ 2 ⎠ ⎦
θ
1
= cos x1 − θ sin x1
2

4(iii) As θ tends to zero,

⎛ 1 ⎞
lim ⎜ cos x1 − θ sin x1 ⎟
gradient of PQ = θ →0 ⎝ 2 ⎠
= cos x1

gradient of tangent at point P


d
= ( sin x )
dx x = x1

= cos x1

5 Im (z)

×  ( −3, 4 )  

Re (z)
O
–1

Least exact value of z − 1

= (−3 − 1) 2 + (4 − 0) 2 − 5
= 32 − 5 or 4 2 −5

3                                                                                                                 
Suggested Solutions to 2010 NJC SH2 H2 Math Prelim Paper 1

a+2
w − 2i = w − ai represents the perpendicular bisector y =
2
Im (z)

a+2
y=
2

×  ( −3, 4 )  

Re (z)
O
–1

For the line to meet the circle more than once,


a+2
−1 < <9
2
−2 < a + 2 < 18
∴−4 < a < 16 (ans)

6 dy dv
y = vx ⇒ =v+ x  
dx dx
dy
x = 3x + y − 2
dx
dv
⇒ x(v + x ) = 3 x + vx − 2  
dx
dv 3 x − 2
⇒ =
dx x2
3 2
⇒ ∫ dv = ∫ − dx
x x2
2
⇒ v = 3 ln | x | + + C  
x
⇒ y = 3 x ln | x | +2 + Cx

4                                                                                                                 
Suggested Solutions to 2010 NJC SH2 H2 Math Prelim Paper 1

6 (i) dy
= 0 ⇒ y = −3 x + 2
dx

6 (ii) (ii)

C > 0 
C <0 

y = −3 x + 2  

Note: (0,2) satisfies the differential equation. (0,2) is a singular solution.

7(a) x − 2 = A(2 − 2 x) + B

By comparing coefficient of

1
x :1 = −2 A ⇒ A = −
2

constant: − 2 = 2 A + B ⇒ B = −1

x−2
∫ −x + 2x + 8
2
dx

1
− (−2 x + 2) − 1
=∫ 2 dx
− x2 + 2x + 8
1 −2 x + 2 1
=− ∫ dx − ∫ dx
2 −x + 2x + 8
2
3 − ( x − 1) 2
2

1 ⎛ x −1 ⎞
=− (2 − x 2 + 2 x + 8) − sin −1 ⎜ ⎟+C
2 ⎝ 3 ⎠
⎛ x −1 ⎞
= − − x 2 + 2 x + 8 − sin −1 ⎜ ⎟+C
⎝ 3 ⎠

In absence of C, deduct from presentation marks.

5                                                                                                                 
Suggested Solutions to 2010 NJC SH2 H2 Math Prelim Paper 1

7 (b)
∫e
x
cos 2 x dx = e x cos 2 x − ∫ e x ( −2 sin 2 x ) dx

= e x cos 2 x + 2 ⎡⎣ e x sin 2 x − 2 ∫ e x cos 2 x dx ⎤⎦

= e x cos 2 x + 2e x sin 2 x − 4 ∫ e x cos 2 x dx

∴ 5∫ e x cos 2 x dx = e x cos 2 x + 2e x sin 2 x

1 x
∫e cos 2 x dx = e (cos 2 x + 2 sin 2 x) + c (shown)
x

∫e
x
cos 2 x dx
1 x
2∫
= e (1 + cos 2 x) dx

1 1
= ∫ e x dx + ∫ e x cos 2 x dx
2 2
1 x 1 x
= e + e (cos 2 x + 2 sin 2 x) + c (ans)
2 10
8(a) Given Sn = n 2 − 2n .

(
Sn − Sn−1 = n 2 − 2n − ( n − 1) − 2 ( n − 1)
2
)
= n 2 − 2n − ( n − 1) + 2n − 2
2

= 2n − 3

Since the progression is AP,

common difference, d = Tn − Tn−1


= 2n − 3 − ( 2 ( n − 1) − 3)
= 2.

OR
d = T2 − T1
= 1 − ( −1)
=2

8(b) Let d be the distance driven for every turn.


(i) n = 1 , d = kθ
⎛8⎞
n = 2 , d = ⎜ ⎟ kθ or 0.8kθ
⎝ 10 ⎠
2
⎛ 8⎞
n = 3 , d = ⎜ ⎟ kθ or 0.64kθ
⎝ 10 ⎠

6                                                                                                                 
Suggested Solutions to 2010 NJC SH2 H2 Math Prelim Paper 1

(ii) Total distance driven into the wall after n turns


2 n −1
⎛ 8⎞ ⎛ 8⎞ ⎛ 8⎞
= kθ + ⎜ ⎟ kθ + ⎜ ⎟ kθ + ... + ⎜ ⎟ kθ
⎝ 10 ⎠ ⎝ 10 ⎠ ⎝ 10 ⎠
⎛ ⎛ 8⎞ ⎛ 8⎞ 2
⎛ 8⎞
n −1

= kθ ⎜ 1 + ⎜ ⎟ + ⎜ ⎟ + ... + ⎜ ⎟ ⎟⎟
⎜ ⎝ 10 ⎠ ⎝ 10 ⎠ ⎝ 10 ⎠
⎝ ⎠
⎛ ⎛ 8 ⎞n ⎞
⎜ 1− ⎜ ⎟ ⎟
10
= kθ ⎜ ⎝ ⎠ ⎟
⎜ ⎛ 8⎞ ⎟
⎜ 1 − ⎜ 10 ⎟ ⎟
⎝ ⎝ ⎠ ⎠
⎛ ⎛ 4 ⎞n ⎞
= 5kθ ⎜1 − ⎜ ⎟ ⎟
⎜ ⎝5⎠ ⎟
⎝ ⎠
4
a =5, b =
5
(iii) Distance driven in the long run
⎛ ⎛ 4 ⎞n ⎞
= lim 5kθ ⎜1 − ⎜ ⎟ ⎟
n →∞ ⎜ ⎝5⎠ ⎟
⎝ ⎠
= 5kθ

Given k = 2 , minimum length of the metal screw is 10θ unit.

9 (i) ⎛1⎞
⎜ ⎟
r • ⎜ 3 ⎟ = 3 ⇒ x + 3y + 2z = 3
⎜ 2⎟
⎝ ⎠
Π2 : −x + y + 0z = 0
⎛3⎞ ⎛ 1⎞
⎜4⎟ ⎜− 2 ⎟
⎜ ⎟ ⎜ ⎟
⎜ 3⎟ ⎜ 1⎟
Using GC: l: r = + λ − , λ ∈R
⎜4⎟ ⎜ 2⎟
⎜ ⎟ ⎜ ⎟
⎜⎜ 0 ⎟⎟ ⎜⎜ 1 ⎟⎟
⎝ ⎠ ⎝ ⎠
OR
⎛3⎞
⎜4⎟
⎜ ⎟ ⎛1⎞
⎜ 3⎟ ⎜ ⎟
r= + λ ⎜ 1 ⎟, λ ∈
⎜4⎟
⎜ ⎟ ⎜ −2 ⎟
⎝ ⎠
⎜⎜ 0 ⎟⎟
⎝ ⎠

7                                                                                                                 
Suggested Solutions to 2010 NJC SH2 H2 Math Prelim Paper 1

9 (ii) For the plane Π 2 :


y = x ⇒ −x + y + 0z = 0
⎛ −1⎞
⎜ ⎟
Let n 2 be normal vector to plane Π 2 , then n 2 = ⎜ 1 ⎟
⎜0⎟
⎝ ⎠
⎛ 1 ⎞ ⎛ −1⎞
⎜ ⎟⎜ ⎟
⎜ 3⎟ ⎜ 1 ⎟
⎜ 2⎟ ⎜ 0 ⎟ 2 1 7
∴ cos θ = ⎝ ⎠ ⎝ ⎠ = = = (shown)
14 2 28 7 7

9 (iii) Since point F lies on line l,

⎛3⎞ ⎛3 ⎞
⎜4⎟ ⎜ 4 +λ⎟
⎛1⎞ ⎜
uuur ⎜⎜ 3 ⎟⎟ ⎜ ⎟ ⎜3

Let OF = +λ⎜ 1 ⎟ = + λ ⎟ for some λ .
⎜4⎟ ⎜4 ⎟
⎜ ⎟ ⎜ −2 ⎟ ⎜ ⎟
⎝ ⎠
⎜⎜ 0 ⎟⎟ ⎜⎜ −2λ ⎟⎟
⎝ ⎠ ⎝ ⎠

⎛3 ⎞ ⎛ 3 ⎞
⎜ 4 +λ⎟ ⎜ 4 +λ ⎟
uuur ⎜⎜ 3 ⎟ ⎛0⎞ ⎜ ⎟
⎜ ⎟ ⎜ 1
Then AF = + λ −⎜1⎟ = − + λ ⎟

⎜4 ⎟ ⎜ 4 ⎟
⎜ ⎟ ⎜⎝ 0 ⎟⎠ ⎜ ⎟
⎜⎜ −2λ ⎟⎟ ⎜⎜ −2λ ⎟⎟
⎝ ⎠ ⎝ ⎠
⎛1⎞
uuur uuur ⎜ ⎟
Now, AF ⊥ l ⇒ AF • ⎜ 1 ⎟ = 0
⎜ −2 ⎟
⎝ ⎠
⎛ 3 ⎞
⎜ 4 +λ ⎟
⎜ ⎟ ⎛1⎞
⎜− 1 + λ⎟ • ⎜ 1 ⎟ = 0
⎜ 4 ⎟ ⎜ ⎟
⎜ ⎟ ⎜⎝ −2 ⎟⎠
⎜⎜ −2λ ⎟⎟
⎝ ⎠
1
λ=−
12

8                                                                                                                 
Suggested Solutions to 2010 NJC SH2 H2 Math Prelim Paper 1

⎛3 1 ⎞ ⎛2⎞
⎜ 4 − 12 ⎟ ⎜ 3 ⎟
uuur ⎜⎜ 3 1 ⎟⎟ ⎜⎜ 2 ⎟⎟
OF = − =  
⎜ 4 12 ⎟ ⎜ 3 ⎟
⎜ ⎟ ⎜ ⎟
⎜⎜ 1 ⎟⎟ ⎜⎜ 1 ⎟⎟
⎝ 6 ⎠ ⎝6⎠

⎛ 3 1 ⎞ ⎛ 2 ⎞
⎜ 4 − 12 ⎟ ⎜ 3 ⎟
uuur ⎜⎜ 1 1 ⎟⎟ ⎜⎜ 1 ⎟⎟
AF = − − = −  
⎜ 4 12 ⎟ ⎜ 3 ⎟
⎜ ⎟ ⎜ ⎟
⎜⎜ 1 ⎟⎟ ⎜⎜ 1 ⎟⎟
⎝ 6 ⎠ ⎝ 6 ⎠

2 2 2
uuur ⎛ 2⎞ ⎛ 1⎞ ⎛1⎞ 7
AF = ⎜ ⎟ + ⎜ − ⎟ + ⎜ ⎟ =  
⎝ 3⎠ ⎝ 3⎠ ⎝ 6⎠ 12
Π1  
 

  l 
  θ 
F  Π2  
 

uuur
Hence, exact length of projection from AF to the plane Π 2
uuur
= AF cos θ  

7 ⎛ 7⎞
= ⎜ ⎟ 
12 ⎜⎝ 7 ⎟⎠

1 1 3
= =  or  
12 2 3 6

OR
uuur
exact length of projection from AF to the plane Π 2
uuur n
= AF × 2  
n2

9                                                                                                                 
Suggested Solutions to 2010 NJC SH2 H2 Math Prelim Paper 1

9 (iv) ⎛3⎞
⎜4⎟
⎛1⎞
uuur ⎜⎜ 3 ⎟⎟ ⎜ ⎟
OP = + λ ⎜ 1 ⎟ , for some λ ∈  
⎜4⎟
⎜ ⎟ ⎜ −2 ⎟
0 ⎝ ⎠
⎜⎜ ⎟⎟
⎝ ⎠

Π 3 has equation px + qy = 1 .

⎛ p⎞
⎜ ⎟
∏3 : r • ⎜ q ⎟ = 1  
⎜0⎟
⎝ ⎠

For the three planes to intersect exactly a point, l is not parallel to Π 3 , then:

⎛ 1 ⎞ ⎛ p⎞
⎜ ⎟ ⎜ ⎟
⎜ 1 ⎟•⎜ q ⎟ ≠ 0
⎜ −2 ⎟ ⎜ 0 ⎟
⎝ ⎠ ⎝ ⎠

⇒ p+q ≠ 0 

∴ p ≠ − q (ans)

10(a) z 2 + (a − i) z * + 16 + bi = 0

( 2 + 3i ) + (a − i) ( 2 − 3i ) + 16 + bi = 0
2

⇒ −5 + 12i + 2a − 3ai − 2i − 3 + 16 + bi = 0
⇒ 8 + 2a + (10 − 3a + b ) i = 0

By comparing real and imaginary coefficient,


Real: 8 + 2a = 0 ⇒ a = −4 (ans)
Im :10 − 3a + b = 0 ⇒ b = −22 (ans)

10(b) z5 +1 = 0
z 5 = −1
z 5 = eiπ  
z 5 = ei( 2 k +1) π
i ( 2 k +1) π

z=e 5
, k = 0, ±1, ± 2

10                                                                                                                 
Suggested Solutions to 2010 NJC SH2 H2 Math Prelim Paper 1

iπ iπ i3π i3π
− −
z = −1, e 5 , e 5
,e 5 ,e 5

Im( z )  

1
× 

× 
×  Re( z )  
–1   O 1
× 

× 
 –1 

(i) iθ iθ
⎛ −iθ2 iθ ⎞
1+ e = e ⎜e + e 2 ⎟
2

⎝ ⎠
iθ⎛ ⎛ θ⎞ ⎛ θ⎞ ⎛θ ⎞ ⎛θ ⎞⎞
= e 2 ⎜ cos ⎜ − ⎟ + i sin ⎜ − ⎟ + cos ⎜ ⎟ + i sin ⎜ ⎟ ⎟
⎝ ⎝ 2⎠ ⎝ 2⎠ ⎝2⎠ ⎝ 2 ⎠⎠
θ
⎛θ ⎞ i
= 2 cos ⎜ ⎟ e 2 (shown)
⎝2⎠

(ii) Replace complex number z with w − 1 ,


i ( 2 k +1) π

z=e 5

i( 2 k +1) π

⇒ w −1 = e 5

i ( 2 k +1) π

∴ w = 1+ e 5

From (i),
i( 2 k +1) π
⎛ ( 2k + 1) π ⎞ i
( 2k +1) π
∴ w = 1+ e 5
= 2 cos ⎜ ⎟e
10
⎝ 10 ⎠
for k = 0, ±1, ± 2 .

11
                                                                                                                 
Suggested Solutions to 2010 NJC SH2 H2 Math Prelim Paper 1

11 ax 2 + bx + 1 1 − bc + ac 2
f(x) = = (ax + b − ac) +
x+c x+c
ax + b − ac = 2 x − 1
∴ a = 2 (ans) and b − ac = −1 ⇒ b = 2c − 1 (shown)

(i) 2
Given c = 1, f(x ) = 2 x − 1 +
x +1
2
Let y = 2 x − 1 +  
x +1
( x + 1) y = (2 x − 1)( x + 1) + 2
2 x 2 + (1 − y ) x + (1 − y ) = 0

For all real values of x,


D ≥ 0
⇒ (1 − y ) − 4(2)(1 − y ) ≥ 0
2

⇒ ( y + 7)( y − 1) ≥ 0
⇒∴ y ≤ −7 or y ≥ 1
Hence, y cannot lie between -7 and 1.

(ii)
y

y = 2x −1 

( 0,1)  

( −2, −7 )  

      
x = −1  

f is concaving downwards for x < −1 .

12
                                                                                                                 
Suggested Solutions to 2010 NJC SH2 H2 Math Prelim Paper 1

(iii) 2 x 2 + x + 1 = (kx + k − 3)( x + 1)


2x2 + x + 1
⇒ = kx + k − 3
x +1
The line kx + (k − 3) passes through the point (−1, −3) , which is the intersection of the
asymptotes. Since the oblique asymptote passes through the point (−1, −3) and using the
graph in (ii), the gradient of the line kx + k − 3 has to be more than 2 for the above equation to
have 2 real solutions.

Hence, k > 2 .

12

(18, 29 )

(0 , 2 )
             

x = 2 ( 0 ) = 0 ; y = ( 03 ) + 2 = 2
2
When λ = 0,

x = 2 ( 3) = 18 ; y = ( 3) + 2 = 29
2 3
When λ = 3,

dx dy
= 4λ , = 3λ 2
dλ dλ

dy 3λ 2 3λ
= =
dx 4λ 4

Equation of tangent:

13
                                                                                                                 
Suggested Solutions to 2010 NJC SH2 H2 Math Prelim Paper 1


y − (λ 3 + 2) = ( x − 2λ 2 )
4
4 y − 4 ( λ 3 + 2 ) = 3λ x − 6λ 3
4 y = 3λ x − 6λ 3 + 4λ 3 + 8
4 y = 3λ x + 8 − 2λ 3
4 y = 3λ x + 2 ( 4 − λ 3 ) (shown)

(a)

(i)

R ( 0, 2 ) P( 2λ2,λ3 +2)

⎛ 4 − λ3 ⎞
Q ⎜ 0, ⎟
⎝ 2 ⎠

4 − λ3
When x = 0, 4 y = 3λ ( 0 ) + 2 ( 4 − λ 3 ) = 2 ( 4 − λ 3 ) ⇒ y =
2

⎛ 4 − λ3 ⎞
Coordinates of Q is ⎜ 0 , ⎟.
⎝ 2 ⎠

Area of triangle PQR,

1⎛ 4 − λ3 ⎞ 2 ⎛ 4−4+λ ⎞
3
λ5
− ⎟( λ ) ⎜ 2 ⎟ 2 units2
= λ
2
A= ⎜ 2 2 =
2⎝ 2 ⎠ ⎝ ⎠

(ii) dx
Given: = 4 when λ = 2 .
dt

λ5 dA 5λ 4
A= ⇒ =
2 dλ 2

14                                                                                                                 
Suggested Solutions to 2010 NJC SH2 H2 Math Prelim Paper 1

dA dA dλ
By chain rule, = ⋅ .
dt dλ dt

dλ dx dx dλ dλ
(Find ) For λ = 2 ,  = ⋅ ⇒ 4 = 4 ( 2) ⋅
dt dt dλ dt dt

dλ 1
∴ =
dt 2

dA 5 ( 2 )
4

= ( 0.5) = 20 units2 /sec .


dt 2

(b)

R ( 0, 2 ) P ( 8 , 10 )

Q ( 0, −2 )

For λ = 2 ,equation of tangent at P is 2 y = 3x − 4.

Coordinates of P is ( 8,10 ) .

Coordinates of Q is ( 0, −2 ) .

Method 1

Area

8 8 1
= ∫ y dx − ∫ ( 3x − 4 ) dx
0 0 2
8
1 ⎡ 3x 2 ⎤
= ∫ ( λ + 2 ) ( 4λ ) dλ − ⎢
2
3
− 4x⎥
0 2⎣ 2 ⎦0
1
[96 − 32 − 0]
2
= ∫ 4λ 4 +8λ dλ −
0 2

15                                                                                                                 
Suggested Solutions to 2010 NJC SH2 H2 Math Prelim Paper 1

2
⎡ 4λ 5 ⎤
=⎢ + 4λ 2 ⎥ − 32
⎣ 5 ⎦0
⎡ 208 ⎤
=⎢ − 0 ⎥ − 32
⎣ 5 ⎦
48 3
= or 9 or 9.6 (ans)
5 5

OR Method 2

1
10 10
Area of region = ∫
−2 3
( 2 y + 4 ) dy − ∫ x d y
2

Area bounded by the tangent at P and y-axis

1
10
=∫
−2 3
( 2 y + 4 ) dy
1 10
= ⎡⎣ y 2 + 4 y ⎤⎦
3 −2

1
= (140 − ( −4 ) )
3
= 48

Area bounded by the curve and y-axis

10
= ∫ x dy
2
2
⎡6 ⎤
= ∫ 2λ 2 ( 3λ 2 ) dλ = ⎢ λ 5 ⎥
2

0
⎣ 5 ⎦0
2
= ∫ 6λ dλ
4
0

6
= ( 32 − 0 )
5
192
=
5

192 48 3
Area of region = 48 − = or 9 or 9.6 (ans)
5 5 5

16                                                                                                                 
Suggested Solutions to 2010 NJC SH2 H2 Math Prelim Paper 2

1(i) ⎛ 1 5⎤
Rg = ⎜ , ⎥ and Dh = [ 0, ∞ ) .
⎝ 2 2⎦

Since Rg ⊆ Dh , thus hg exists.

⎛ 1 5⎤ ⎡ 1 3⎤
Dh g = Dg = ( −∞, 0] → ⎜ , ⎥ → ⎢ − , ⎥ = Rhg
⎝ 2 2⎦ ⎣ 4 4⎦

OR any appropriate method such as graphical method

(ii) For inverse of function h to exist, b = 1.5

(ii) y y=x
y=h −1
( x)

y = h (x )

( –0.25,1.5)
x
0
(1.5, –0.25)
(iii) To solve for exact value of h −1 ( x) = x is same as solving h( x) = x ,

x 2 − 3x + 2 = x
x2 − 4x + 2 = 0

( −4 ) − 4 (1)( 2 )
2

∴x =
2
4± 8
=
2
= 2 − 2 (rejected Q x ≥ 1.5) or 2+ 2

From sketch above, for h −1 (x) ≤ x , x ≥ 2 + 2 .

2(a) As n → ∞ , xn → L , xn +1 → L .

xn +1 = 3 − ( ln ( xn + 1) )
2

 
Suggested Solutions to 2010 NJC SH2 H2 Math Prelim Paper 2

⇒ L = 3 − ( ln ( L + 1) )
2

⇒ 3 − ( ln ( L + 1) ) − L = 0
2

Using GC to solve 3 − ( ln ( L + 1) ) − L = 0 ,
2

L = −0.860 (rejected as xn > 0 )

or L = 1.8806225 ≈ 1.88 (3sf)

2 (b) r 2 + 7 r + 11 A B
= +
( r + 4 )! ( r + 2 )! ( r + 4 )!

r 2 + 7 r + 11 = A ( r + 3)( r + 4 ) + B

When r = −3, B = −1

When r = 0, A = 1

r 2 + 7r + 11 1 1
Hence, = − .
( r + 4 )! ( r + 2 )! ( r + 4 )!
n ⎛ r 2 + 7 r + 11 ⎞ n ⎛ 1 1 ⎞
∑ ⎜⎜ ( r + 4 )! ⎟⎟ = ∑ ⎜⎜ − ⎟⎟

r =1 ⎠ r =1 ⎝ ( r + 2 ) ! ( r + 4 ) ! ⎠
1 1
= −
3! 5!
1 1
+ −
4! 6!
1 1
+ −
5! 7!
1 1
+ −
6! 8!

+ M
1 1
+ −
( n − 1)! ( n + 1)!
1 1
                                    + −
n ! ( n + 2 )!
1 1
+ −
( n + 1)! ( n + 3)!
1 1
+ −
( n + 2 )! ( n + 4 )!
Page 2 of 16 

 
Suggested Solutions to 2010 NJC SH2 H2 Math Prelim Paper 2

1 1 1 1
                                   = + − −  
3! 4! ( n + 3) ! ( n + 4 ) !

4 +1 n + 4 +1
= −
4! ( n + 4 ) !
5 n+5
= −
4! ( n + 4 ) !

(ii) n ⎛ r 2 + 7 r + 11 ⎞ 5 n+5
Let Pn be the proposition that ∑ ⎜

r =1 ⎝
⎟⎟ = −
( r + 4 )! ⎠ 4! ( n + 4 )!
for all n ∈ +
.

Consider P1:

5 1+ 5 5 6 25 − 6 19
RHS of P1 = − = − = = .
4! (1 + 4 ) ! 4! 5! 5! 5!
1 ⎛ r 2 + 7 r + 11 ⎞ 12 + 7 + 11 19
LHS of P1 = ∑ ⎜⎜
r =1 ⎝
⎟= =
( r + 4 )! ⎟⎠ (1 + 4 )! 5!
= RHS of P1

Hence, P1 is true.
+
Assume Pk is true for some k ∈ .

k ⎛ r 2 + 7 r + 11 ⎞ 5 k +5
ie. ∑ ⎜

r =1 ⎝
⎟⎟ = −
( r + 4 )! ⎠ 4! ( k + 4 )!
Consider Pk+1:

5 k +1+ 5 5 k +6
RHS of Pk+1 = − = − .
4! ( k + 1 + 4 ) ! 4! ( k + 5 ) !
k +1 ⎛ r 2 + 7 r + 11 ⎞
LHS of Pk+1 = ∑ ⎜⎜
r =1 ⎝

( r + 4 )! ⎟⎠
⎛ r 2 + 7 r + 11 ⎞ ( k + 1) + 7 ( k + 1) + 11
2
k
= ∑ ⎜⎜ ⎟+
r =1 ⎝ ( r + 4 )! ⎟⎠ ( k + 1 + 4 )!
5 k +5 k 2 + 2k + 1 + 7 k + 7 + 11
= − +
4! ( k + 4 ) ! ( k + 5)!
5 k 2 + 10k + 25 − k 2 − 2k − 1 − 7 k − 7 − 11
= −
4! ( k + 5)!
5 k +6
= − = RHS of Pk+1
4! ( k + 5 ) !

Page 3 of 16 

 
Suggested Solutions to 2010 NJC SH2 H2 Math Prelim Paper 2

Hence, Pk is true implies Pk+1 is also true.

Since P1 is true and Pk is true implies Pk+1 is also true, by Mathematical Induction, Pn is
true for all n ∈ + .

(iii) Let r = j – 1. Hence, we have

∞ ⎛ r 2 + 9r + 19 ⎞ ∞ ⎛ ( j − 1)2 + 9 ( j − 1) + 19 ⎞
∑ ⎜⎜
r =1 ⎝
⎟=
( r + 5)! ⎟⎠ ∑ ⎜⎜ ( j − 1 + 5 ) !


j −1=1
⎝ ⎠
∞ ⎛ 2
j − 2 j + 1 + 9 j − 9 + 19 ⎞
= ∑ ⎜⎜ ⎟⎟
j =2 ⎝ ( j + 4 )! ⎠
∞ ⎛ 2
j + 7 j + 11 ⎞
= ∑ ⎜⎜ ⎟
j =2 ⎝ ( j + 4 )! ⎟⎠
∞ ⎛ 2
r + 7 r + 11 ⎞
= ∑ ⎜⎜ ⎟, since j is a dummy variable
r =2 ⎝ ( r + 4 )! ⎟⎠
∞ ⎛ 2
r + 7 r + 11 ⎞ 12 + 7 + 11
= ∑ ⎜⎜ ⎟−
r =1 ⎝ ( r + 4 )! ⎟⎠ (1 + 4 )!
⎛5 n + 5 ⎞ 19
= lim ⎜⎜ − ⎟−
n →∞ ⎝ 4! ( n + 4 )! ⎟⎠ 5!
5 19
= −
4! 5!
5 19
= −
4! 4!( 5 )
6 1
= or
5! 20

3 Let  y = 2 esin x  

y 2 = 4esin x
dy
  2y = 4esin x cos x = y 2 cos x           
dx
dy
2 = y cos x (shown)
dx

Page 4 of 16 

 
Suggested Solutions to 2010 NJC SH2 H2 Math Prelim Paper 2

d 2 y dy
⇒2 = cos x − y sin x
dx 2 dx
d3 y d 2 y dy dy
⇒ 2 3 = 2 cos x − sin x − sin x − y cos x  
dx dx dx dx
3 2
d y d y dy
∴ 2 3 = 2 cos x − 2 sin x − y cos x
dx dx dx
When x = 0, y = 2
dy
=1
dx
d2 y 1
=
dx 2 2
d3 y 3
3
=−   
dx 4
⎛1⎞ 2 ⎛ 3⎞ 3
⎜ ⎟x ⎜− ⎟x
2 4⎠
∴ y ≈ 2+ x+ ⎝ ⎠ + ⎝
2! 3!
x 2 x3
y = 2+ x+ − (ans)
4 8
Method 1

y2 1 x 2 x3
esin x = ≈ (2 + x + − ) 2
4 4 4 8

1 x 2 x3
e− sin x = esin( − x ) ≈ [2 − x + + ]2 (Replace x by − x)
4 4 8
1 x 2 x3
= [2 − ( x − − )]2
4 4 8
1⎡ x 2 x3 ⎞ ⎤
2
⎛ x2 ⎞ ⎛
= ⎢ 4 − 4 ⎜ x − + ... ⎟ + ⎜ x − − ⎟ ⎥
4⎢ ⎝ 4 ⎠ ⎝ 4 8 ⎠ ⎥
⎣ ⎦
1
≈ ⎡⎣ 4 − 4 x + x 2 + x 2 ⎤⎦
4
1
= ( 4 − 4x + 2x2 )
4
x2
= 1− x + (ans)
2

OR Method 2

y2 1 x 2 x3
esin x = ≈ (2 + x + − ) 2  
4 4 4 8

Page 5 of 16 

 
Suggested Solutions to 2010 NJC SH2 H2 Math Prelim Paper 2

e − sin x = ( esin x )
−1

−1
⎛ y2 ⎞
=⎜ ⎟
⎝ 4 ⎠
x 2 x3 −2
= 4(2 + x + − )
4 8
x x 2 x3
= (1 + + − ) −2  
2 8 16
⎛ x x 2 x3 ⎞ ( −2 ) ( −3 ) ⎛ x x 2 x3 ⎞
2

= 1− 2 ⎜ + − ⎟ + ⎜ + − ⎟ +L
⎝ 2 8 16 ⎠ 2! ⎝ 2 8 16 ⎠
x2 ⎛ x2 ⎞
= 1− x − + 3 ⎜ ⎟ +L
4 ⎝ 4 ⎠
2
x
= 1− x + (ans)
2

4(i) dx
x = 2 tan θ ⇒ = 2 sec 2 θ

4 − x2 4 − 4 tan 2 θ
∫ dx = ∫ ( 2sec θ ) dθ
2

(4 + x )
2 2
( 4 + 4 tan θ ) 2 2

4 − 4 tan 2 θ
=∫ ( 2sec θ ) dθ
2

( 4sec θ ) 2 2

4 − 4 tan 2 θ
=∫ dθ
8sec 2 θ
1 1 tan 2 θ
= ∫ − dθ
2 sec 2 θ sec 2 θ
1
= ∫ cos 2 θ − sin 2 θ dθ
2

1
2∫
= cos 2θ dθ

1⎛1 ⎞
= ⎜ sin 2θ ⎟ + C
2⎝2 ⎠
1
= ( sin θ cos θ ) + C
2
1⎛ x ⎞⎛ 2 ⎞
= ⎜ ⎟⎜ ⎟+C
2 ⎝ 4 + x ⎠ ⎝ 4 + x2 ⎠
2

x
= +C
                             4 + x
2

Page 6 of 16 

 
Suggested Solutions to 2010 NJC SH2 H2 Math Prelim Paper 2

(ii) Volume of region rotated about x-axis, V


(a) 2
⎛ ⎞ ⎡ 2
4− x ⎟ − 3 ( x + 2 ) ⎤ dx
2
=π∫ ⎜
1

⎟ ⎢⎣ 15 ⎥
−2 ⎜
⎝ (4 + x )2 2
⎠ ⎦
2
1 4 − x2 ⎛ 3⎞
= π∫ ⎟⎟ ( x + 2 ) dx
2
− ⎜⎜
−2
(4 + x )2 2
⎝ 15 ⎠
1
⎡ x ⎛ 3 ⎞ ( x + 2) ⎤
3

= π⎢ −⎜ ⎟ ⎥
⎢⎣ 4 + x ⎝ 225 ⎠ 3 ⎥⎦ −2
2

⎡ 1 ⎛ 3 ⎞ ⎛ 27 ⎞ ⎛ −2 ⎞ ⎤
= π⎢ −⎜ ⎟ ⎜ ⎟ − ⎜ ⎟⎥
⎣ 5 ⎝ 225 ⎠ ⎝ 3 ⎠ ⎝ 8 ⎠ ⎦
33
= π (ans)
100

33
∴b =
100

(ii) dy
(b)
y y=
dx

–2 O 2 x

Section B: Statistics [60 marks]

5 (i) It is the list of all the 700 graduating students.

(ii) Arrange the list of all 700 students in some order (can be by surname, class, other
reasonable category.)

Page 7 of 16 

 
Suggested Solutions to 2010 NJC SH2 H2 Math Prelim Paper 2

700
Calculate the interval to take samples from to obtain the 140 samples. ie. =5.
140

From the first group of 5 students, select the first student using random sampling. Then
select every 5th student after that.

Note:

Students may say they use random sampling to pick anyone in the list as the first student,
but they will then need to qualify that they will need to cycle back to the names at the front
of the list if it reaches the end before getting all the 140 students.

6(i)               s  

   

   

                                                          

(ii) Using GC,


α = −4.975 ≈ −4.98 , β = 2.111 ≈ 2.11
r = 0.951 (to 3 sf)

Since the value of r is close to 1, it suggests an almost linear relationship. Hence a linear
model is appropriate.

(iii) When d = 28 ,

s = −4.97519 + 2.110569 ( ln 28)

s = 2.05766 ≈ 2.06 or 2.058

Thus average sales is 2060 or 2058 bottles.

The answer is reliable since based on the new model, the value of r is close to 1 and that
suggests a linear relationship. Furthermore we are predicting s based on d which is within
the range.

Page 8 of 16 

 
Suggested Solutions to 2010 NJC SH2 H2 Math Prelim Paper 2

7(i) Number of ways = 9! = 362880 (ans)


(ii) Number of ways to arrange the couple among themselves = 2!
6!
Number of ways to arrange 4 couples and 2 children =
6
6!
Number of ways = (2!)4 = 1920 (ans)
6
⎛8⎞
Number of ways to select 2 adults to in the front row = ⎜ ⎟
⎝ 2⎠

Number of ways to arrange the bride and groom =2!

Number of ways to arrange the children and two adults in the front row =4!

Number of ways to arrange the 6 adults in the back row =6!

⎛8⎞
Number of arrangements = ⎜ ⎟ ( 4!)( 2!)( 6!) = 967680 (ans)
⎝ 2⎠

OR Alternative Solution
⎛ 4⎞
(iii) Number of ways to arrange the children front row = ⎜ ⎟ ( 2!)
⎝ 2⎠
Number of ways to arrange the bride and groom =2!

Number of ways to arrange the 8 adults = 8!


⎛ 4⎞
Number of arrangements = ⎜ ⎟ ( 2!)( 2!)( 8!) = 967680 (ans)
⎝ 2⎠

8(a)

(i) A B

2
 
15

P ( A) = P ( A ∪ B ) − P ( A ' ∩ B )
3 2
= −
4 15
37
=
60

Page 9 of 16 

 
Suggested Solutions to 2010 NJC SH2 H2 Math Prelim Paper 2

Since events A and B are independent,

2
P ( A '∩ B ) = P ( A ') P ( B ) =
15

(1 − P ( A) ) P ( B ) = 152
⎛ 37 ⎞ 2
⎜1 − ⎟ P ( B ) =
⎝ 60 ⎠ 15

8
P ( B) =
23

OR Alternative solution

P ( A ∪ B ) = P ( A) + P ( B ) − P ( A ∩ B )
3 37 37
⇒ = + P ( B) − P ( B )
4 60 60
23 2
⇒ P ( B) =
60 15
8
∴P ( B) = (ans)
23

(ii)  P ( A ∩ B A ∪ B)

P (( A ∩ B ) ∩ ( A ∪ B ))
=
P ( A ∪ B)

P ( A ∩ B)
=
P ( A ∪ B)

P ( A) × P ( B ) P ( B ) − P ( A' ∩ B )
= (OR: = )
P ( A ∪ B) P ( A ∪ B)

⎛ 37 ⎞⎛ 8 ⎞
⎜ ⎟⎜ ⎟
60 23
= ⎝ ⎠⎝ ⎠
3
4

296
=
1035

Page 10 of 16 

 
Suggested Solutions to 2010 NJC SH2 H2 Math Prelim Paper 2

8(b) P(all 3 boys in the same group)

(i) ⎛ 3 ⎞⎛ 5 ⎞⎛ 4 ⎞
⎜ ⎟⎜ ⎟⎜ ⎟
⎝ 3 ⎠⎝ 1 ⎠⎝ 4 ⎠
= 2!
⎛ 8 ⎞⎛ 4 ⎞
⎜ ⎟⎜ ⎟
⎝ 4 ⎠⎝ 4 ⎠
2!

5 1
= = (ans)
70 14

(ii) P(either Ivy or Tamie is in the same group as Cassy)


= P(Ivy together with Cassy but not Tamie)
+ P(Tammy together with Cassy but not Ivy)
+ P(Ivy and Tamie together with Cassy)
⎛ 5 ⎞⎛ 3 ⎞ ⎛ 5 ⎞⎛ 3 ⎞ ⎛ 5 ⎞ ⎛ 4 ⎞
⎜ ⎟⎜ ⎟ ⎜ ⎟⎜ ⎟ ⎜ ⎟ ⎜ ⎟
⎝ 2 ⎠⎝ 3 ⎠ ⎝ 2 ⎠⎝ 3 ⎠ ⎝ 1 ⎠ ⎝ 4 ⎠
= 2! + 2! + 2!
⎛ 8 ⎞⎛ 4 ⎞ ⎛ 8 ⎞⎛ 4 ⎞ ⎛ 8 ⎞⎛ 4 ⎞
⎜ ⎟⎜ ⎟ ⎜ ⎟⎜ ⎟ ⎜ ⎟⎜ ⎟
⎝ 4 ⎠⎝ 4 ⎠ ⎝ 4 ⎠⎝ 4 ⎠ ⎝ 4 ⎠⎝ 4 ⎠
2! 2! 2!
25 5
= = (ans)
70 14

OR
(either Ivy or Tamie is in the same group as Cassy)
= P(Ivy together with Cassy)
+ P(Tammy together with Cassy)
– P(Ivy and Tamie together with Cassy)
⎛ 6 ⎞⎛ 4 ⎞ ⎛ 6 ⎞⎛ 4 ⎞ ⎛ 5 ⎞⎛ 4 ⎞
⎜ ⎟⎜ ⎟ ⎜ ⎟⎜ ⎟ ⎜ ⎟⎜ ⎟
⎝ 2 ⎠⎝ 4 ⎠ ⎝ 2 ⎠⎝ 4 ⎠ ⎝ 1 ⎠⎝ 4 ⎠
= 2! + 2! − 2! = 25 = 5 (ans)
⎛ ⎞⎛ ⎞ ⎛ ⎞⎛ 4 ⎞ ⎛ 8 ⎞⎛ 4 ⎞ 70 14
8 4 8
⎜ ⎟⎜ ⎟ ⎜ ⎟⎜ ⎟ ⎜ ⎟⎜ ⎟
⎝ 4 ⎠⎝ 4 ⎠ ⎝ 4 ⎠⎝ 4 ⎠ ⎝ 4 ⎠⎝ 4 ⎠
2! 2! 2!

Page 11 of 16 

 
Suggested Solutions to 2010 NJC SH2 H2 Math Prelim Paper 2

9(i) Let X be the number of arrivals at the airport in a two-hour period.

X ~ Po ( 8 )

P ( X ≥ 13) = 1 − P ( X ≤ 12 ) = 0.063797 ≈ 0.0638

(ii) Let W be the number of arrivals in a one-hour period.

Let Y be the number of departures in a one-hour period.

W ~ Po ( 4 ) ; Y ~ Po ( 3) ; W + Y ~ Po ( 7 )

P (Y < 2 | W + Y = 9 )
P (Y < 2 ∩ W + Y = 9 )
=
P (W + Y = 9 )
P (Y = 0, W = 9 ) + P (Y = 1, W = 8 )
=
P (W + Y = 9 )
P (Y = 0 ) P (W = 9 ) + P (Y = 1) P (W = 8 )
=
P (W + Y = 9 )
0.0051052524
=
0.1014046695
= 0.0503453384
≈ 0.0503 (ans)

(iii) (1)There are two mutually exclusive outcomes – either there are at least 13 arrivals in each
two-hour period or there isn’t.

(2)The probability of having at least 13 arrivals for each two hour period remains constant
for each of the 60 two-hour periods.

(3)There is a fixed number of 60 two-hour periods independently selected under


consideration.

(iv) Notice that we are unable to define the random variable as the number of two-hour periods, out of 60, with
less than 13 arrivals each as it will not be possible to do any approximations.

np = 56.172 ( > 5) and nq = 3.828 ( < 5)

Let V be the number of two-hour periods, out of 60, with at least 13 arrivals each.

V ~ B ( 60, P ( X ≥ 13) )

Since n = 60 ( > 50, large) and p = P ( X ≥ 13) = 0.0638 ( < 0.1, small), such that np = 3.828

Page 12 of 16 

 
Suggested Solutions to 2010 NJC SH2 H2 Math Prelim Paper 2

( < 5), we have V ~ Po ( 3.828 ) approximately.

At most 50 two-hour periods with less than 13 arrivals each means the same as at least 10
two-hour periods with at least 13 arrivals each.

P (V ≥ 10 ) = 1 − P (V ≤ 9 ) = 0.0060899731 = 0.00609

Note:

Students may choose to use the more accurate value for

P ( X ≥ 13) = 0.0637971966 . If they do so, the following values will be obtained:

np = 3.827831796

V ~ Po ( 3.827831796 )

P (V ≥ 10 ) = 0.0060881936 = 0.00609.

This will still be considered as correct and will be awarded the necessary marks.

10 Let X denote the monthly electricity usage of a 3-room unit.

X N ( 290, σ 2 )

Let Y denote the monthly electricity usage of a 5-room unit.

Y N ( 450,105 )

(i) P ( X > 290 ) × P ( X > 290 ) × P (Y < 450 ) = 0.5 × 0.5 × 0.5
= 0.125 (ans)
(ii) X1 + X 2 + X 3 + X 4 N (1160, 4σ 2 )

P ( X 1 + X 2 + X 3 + X 4 > 3 ( 380 ) ) = 0.868

⇒ P ( X 1 + X 2 + X 3 + X 4 < 1140 ) = 0.132

⎛ 1140 − 1160 ⎞
⇒ P⎜Z < ⎟ = 0.132
⎝ 4σ 2 ⎠

Page 13 of 16 

 
Suggested Solutions to 2010 NJC SH2 H2 Math Prelim Paper 2

1140 − 1160
⇒ = −1.1169867  
4σ 2
20
⇒ 2σ =
1.1169867
⇒ σ = 8.9526579
∴σ 2 = 80.1500827 ≈ 80 (ans)

(iii) Let C denote the monthly electricity charge of a 5-room unit.

C = 0.24Y N ( 0.24(450), 0.242 (105) )

∴C N (108, 6.048 )

P ( C > a ) = 0.9
⇒ 1 − P ( C ≤ a ) = 0.9
⇒ P ( C ≤ a ) = 0.1

∴ a = 104.848321 ≈ 104.85  (ans)

(iv) Let W denote the number of 5-room units with monthly electricity bill exceeding $a, out of
eighty 5-room units.

W B ( 80, 0.9 )

Since n = 80 is large and np = 80(0.9) = 72( > 5), nq = 80(0.1) = 8( > 5) ,

∴W N ( 72, 7.2 ) approx

P (W ≥ 70 ) = P (W > 69.5 )
= 0.8242529
≈ 0.824 (ans)

11(a) 3791
x= = 75.82
50

1 ⎡ 37912 ⎤ 525.38 26269


s2 = ⎢ 287959 − = = = 10.72204
49 ⎣ 50 ⎥⎦ 49 2450

Let μ be the mean drying times of Noppin paint.


Page 14 of 16 

 
Suggested Solutions to 2010 NJC SH2 H2 Math Prelim Paper 2

Test H0: μ = 75 against H1: μ ≠ 75

Level of Significance: 5% (2-tailed Z-test)

X −μ
Test Statistic: Z = s
~ N ( 0 ,1) approximately.
n

Method 1: Using critical region and observed test statistic, zcalculated

Critical region: z > 1.95996


0.05

75.82 − 75
zcalculated = = 1.77076 −1.95996 0 1.95996
10.72204
50
1.77076

Since zcalculated = 1.77076 < 1.95996 , we do not reject H0.

Method 2: Using p-value

p-value = 0.07660

Since p-value = 0.07660 > 0.05, we do not reject H0.

There is insufficient evidence at 5% level of significance to claim that the mean drying
times of the Noppin paint has changed. Hence, the manufacturers will not discard the
entire affected batch of paint.

11(a) Since X is unknown and n = 60 is large,

(ii) by Central Limit Theorem,

X1 + X 2 + X 3 + L + X 60 N ( 60 × 75, 60 ×10.72204082) approx .

Page 15 of 16 

 
Suggested Solutions to 2010 NJC SH2 H2 Math Prelim Paper 2

P ( 60 ( 72 ) < X 1 + X 2 + X 3 + L + X 60 < 60 ( 76 ) )
P ( 4320 < X 1 + X 2 + X 3 + L + X 60 < 4560 )
= 0.990999
≈ 0.991(ans)

(b)(i) Test H0: μ = 75 against H1: μ < 75

Level of Significance: 5% (lower-tailed)

n 20 20
σ n 2 = ( 7.5 ) ⇒ s = 7.5
2
s2 = = 7.69484  
n −1 19 19

X −μ
Test Statistic: T = s
~ t (19 )
n

0.05

x − 75
tcalculated = 7.69484
20

−1.72913 0 

For the chemist to claim that the new additive had been effective, we need to reject H0 in
favour of H1.

Hence, we need tcalculated < −1.72913 .

x − 75
7.69484
< −1.72913
20

x < 72.02482

Largest value of x = 72.0 (to 3 sig. fig.)

(ii) It is necessary to assume that the drying time of Noppin paint, X, follows a normal
distribution.

Page 16 of 16 

 
NANYANG JUNIOR COLLEGE
JC2 PRELIMINARY EXAMINATION
Higher 2

MATHEMATICS 9740/01

Paper 1 16th September 2010

3 Hours

Additional Materials: Answer Paper


List of Formulae (MF15)

READ THESE INSTRUCTIONS FIRST

Write your name and class on all the work you hand in.
Write in dark blue or black pen on both sides of the paper.
You may use a soft pencil for any diagrams or graphs.
Do not use staples, paper clips, highlighters, glue or correction fluid.

Answer all the questions.


Give non-exact numerical answers correct to 3 significant figures, or 1 decimal place in the case of angles in
degrees, unless a different level of accuracy is specified in the question.
You are expected to use a graphic calculator.
Unsupported answers from a graphic calculator are allowed unless a question specifically states otherwise.
Where unsupported answers from a graphic calculator are not allowed in a question, you are required to
present the mathematical steps using mathematical notations and not calculator commands.
You are reminded of the need for clear presentation in your answers.

At the end of the examination, fasten all your work securely together.
The number of marks is given in brackets [ ] at the end of each question or part question.

This document consists of 6 printed pages.


NANYANG JUNIOR COLLEGE
Internal Examinations

© NYJC 2010 [Turn Over


2

1
1 Find the equation of the quadratic function which has a line of symmetry at x  and passes through
3
the points (1,9) and (1,5) . [4]

2 The complex numbers z and w are 1 + ai and b – 2i respectively where a and b are real and a is
negative. Given that zw* = 8i, find the exact values of a and b. [3]

Find the smallest positive integer value of n such that wn is purely imaginary. [2]

1
3 Find the expansion of in ascending powers of x, up to and including the term in x 2 . Hence
4x
2 1
find an approximate value of the integral  0
4  2x
dx , expressing your answer in the form

a  b 2 , where a and b are constants to be determined. [6]

d2 x dx
4 The differential equation 2  9 x  3t is such that x = 0, = 1 when t = 0. Find the Maclaurin’s
dt dt
series for x up to and including the t 3 term. [3]

It is known that this differential equation has a general solution of the form x = Asin3t +Bt, where A
and B are constants. Assuming that t is sufficiently small for terms with powers of 4 and above to be
neglected, find the values of A and B. [4]

NYJC 2010 JC2 Preliminary Examination 9740/01


3
5 With respect to the origin O, the points A, B and C have position vectors a, b and c respectively, and
are such that OACB is a parallelogram in an anti-clockwise sense.
(i) Express c in terms of a and b. [1]
(ii) Show that the area of parallelogram OACB is given by a  b . [2]

(iii) Show that the maximum area of parallelogram OACB is a b . [2]

 a1   b1 
   
(iv) By considering a·b where a   a2  and b   b2  , show that
 a   b 
 3   3 

(a1b1  a2b2  a3b3 ) 2   a12  a22  a32  b12  b22  b32  . [3]

6 The diagrams below show the graphs of y = |f(x)| and y = f ( x) for x . The point A, B and A’ has

the coordinates of (1, 2) , (1,1.5) and (1, 2) respectively.

y y  f ( x) y
y  f ( x)

A (1, 2)
B (1,1.5)
A ' (1, 2)
x x

On separate diagrams, sketch the graphs of,


(i) y  f ( x) , [2]
1
(ii) y , [3]
f ( x)
(iii) y  f '( x) , [3]
showing clearly any asymptotes and the coordinates of any stationary point(s).

NYJC 2010 JC2 Preliminary Examination 9740/01 [Turn Over


4
7 Functions f and g are respectively defined on the domain of real numbers by
f : x  x2  2x  2 , x > 1,
g: x  x3 , x  –3.
(i) By considering the derivative of f(x), prove that f is a one-one function. [2]
(ii) Solve the equation f(x) = f 1  x  where f 1 x  denotes the inverse function of f(x). [2]

(iii) Deduce the solution set to the inequality f  x   f 1  x  . [2]

(iv) Show that the composite function gf exists and define it in a similar form. State also its
range. [3]

A curve C is defined by the parametric equations x  2t , y  3et , where t  .


2
8
(i) Sketch the curve C. [1]
(ii) Given that the point P lies on the curve with coordinates (2, 3e), show that the equation of the
tangent to the curve at point P is y  3e( x  1) . [2]
(iii) The normal to the curve at point P cuts the x-axis at point Q. Given also that the tangent to the
curve at point P cuts the y-axis at point R, find the coordinates of Q and R.
Hence, show that the area of PQR is 3e(1  9e 2 ) units2. [5]
(iv) State the range of values of m for which the line y  mx  3e intersects the curve at 2 distinct
points. [1]

a 1
9 Given that f ( x)  ax  1  , x  , a 0.
4x 1 4

(i) Find the equation of the asymptote(s) of y  f ( x) . [2]

(ii) Find the coordinates of the turning point(s) for y  f ( x) . [4]

(iii) (a) Sketch the graph of y  f ( x) , for 0 < a < 1, indicating clearly the equations of the
asymptote(s), and the coordinates of any turning point(s) and axial intercept(s). [3]

(b) Determine the range of values of m such that there will be intersection between the
1   1
line y   a  1 = m  x   and y = f(x). [1]
4   4

NYJC 2010 JC2 Preliminary Examination 9740/01


5
10 An innovation is introduced into a community of 100 farmers at time t  0 . Let x denote the number
of farmers who have adopted the innovation at time t. Assume that x is a continuous function of time.
The rate at which the number of farmers in that community who adopted the innovation at a
particular instant is proportional to the product of the number of farmers who have already adopted
and the number of farmers who have not adopted the innovation.

Initially, one farmer adopted the innovation and the rate at which the number of farmers who adopted
the innovation is one farmer per unit time.

dx
(i) Show that  k(100x  x 2 ) , where k is to be determined. [1]
dt
(ii) Find the particular solution of x , in terms of t. [5]
(iii) Sketch the graph of x versus t, for t  0 . [2]
(iv) Using the graph in (iii) or otherwise, find the time taken for 75% of the population of farmers
to adopt the innovation, leaving your answer to 2 decimal places. [1]
(v) Give a reason why the model may not be suitable. [1]

11 Solve the equation (  2)5  32  0 , leaving your answers in the form rei , where r  0 and

     . [4]
(i) State the equation of the circle in the form | z  a | b that passes through all the points
represented by the roots.
Sketch this circle, showing clearly the relationship between the roots and the locus clearly. [3]
 
(ii) The roots 1 and 2 are such that   arg(1  2)  0  arg(2  2)  . On the same
2 2
3
Argand diagram, sketch the locus of the points representing z given that arg(z  1 )  . [2]
5
Find the complex number represented by the point of intersection between the two loci. [1]
3
(iii) Find the least value of z  2 if arg(z  1 )  . [2]
5

NYJC 2010 JC2 Preliminary Examination 9740/01 [Turn Over


6
12 (a) (i) Write down the derivative of (1  x 2 )n . [1]

(ii) Find  x3 (1  x 2 )n dx where n  1,  2 . [3]

3
(b) Region A is bounded by the curve y  sin x  cos x , the lines y  2 and x  .
4
(i) Find the exact area of region A. [4]
(ii) Deduce the exact area of the region in the first quadrant bounded by the curve

y  sin  x  4   cos  x  4  and the two axes. [2]

(iii) Find the volume generated when region A is rotated through four right angles about the
x-axis. [2]

-----END OF PAPER-----

NYJC 2010 JC2 Preliminary Examination 9740/01


NANYANG JUNIOR COLLEGE
JC2 PRELIMINARY EXAMINATION
Higher 2

MATHEMATICS 9740/02

Paper 2 17th September 2010

3 Hours

Additional Materials: Answer Paper


List of Formulae (MF15)

READ THESE INSTRUCTIONS FIRST

Write your name and class on all the work you hand in.
Write in dark blue or black pen on both sides of the paper.
You may use a soft pencil for any diagrams or graphs.
Do not use staples, paper clips, highlighters, glue or correction fluid.

Answer all the questions.


Give non-exact numerical answers correct to 3 significant figures, or 1 decimal place in the case of angles in
degrees, unless a different level of accuracy is specified in the question.
You are expected to use a graphic calculator.
Unsupported answers from a graphic calculator are allowed unless a question specifically states otherwise.
Where unsupported answers from a graphic calculator are not allowed in a question, you are required to
present the mathematical steps using mathematical notations and not calculator commands.
You are reminded of the need for clear presentation in your answers.

At the end of the examination, fasten all your work securely together.
The number of marks is given in brackets [ ] at the end of each question or part question.

This document consists of 6 printed pages.


NANYANG JUNIOR COLLEGE
Internal Examinations

© NYJC 2010 [Turn Over


2
Section A: (40 Marks)

1 Two scientists are studying the growth of a certain species on an island.


(a) The first scientist proposes the following model:
“Let xn be the population of the species in the nth year and d n  xn  xn1 , n  2 with

d1  x1  100 . It is believed that d n is a geometric sequence with common ratio e r .”

(i) Find an expression for xn in terms of r and n. [2]

(ii) Obtain an inequality for r in order for the population to stabilise at K. Find the value
of K in terms of r. [2]
(b) The second scientist proposes the following model:
“Let yn be the population of the species (in thousands) in the nth year. It can be hypothesised

that y1  0.1 and yn 1  2 yn (1  yn ) .”

(i) Find the limiting population of this model. [2]


(ii) Find the value of r in order for both models to have the same limiting population. [2]

2 A circular cylinder is inscribed in a right circular cone of base radius 2c and vertical height c. One of
the circular ends of the cylinder lies on the base of the cone and the other end is in contact with the
inner surface of the cone. If the cylinder has base radius x, find expressions for the volume V and the
total surface area S of the cylinder in terms of c and x. If c is fixed and x varies, find the maximum
value of V. [8]

Prove that S cannot exceed 8πc2. [2]

3 The sequence of numbers {u r } where r = 1, 2, 3, …, is such that it satisfies the recurrence relation
rur 1
 ur  r 2 and u1 = 1.
r 1
(i) By dividing the above recurrence relation by r and using the method of difference, show that
n 2
un 
2
 n  n  2 for n = 1, 2, 3, …. [5]

(ii) Prove the result in (i) using mathematical induction. [4]


un
(iii) Find the exact value of as n   . [1]
n3

NYJC 2010 JC2 Preliminary Examination 9740/02


3
4 (a) The line l passes through the point A with coordinates (1,-1,1) and is parallel to the vector
2i + j – 2k. The plane p has equation r  (i + j + k) = 3. Find, in exact form,

(i) the position vector of B, the point of intersection between l and p, [3]
(ii) the sine of the acute angle between l and p, [2]
(iii) the shortest distance from A to p, [1]
(iv) the length of the projection of AB onto p. [2]

(b) Given that the system of linear equations

x + y + 3z =  (,   )
3x + y + 4z = 9
x+y = 3

has infinite solutions, obtain the numerical values of  and . [4]

Section B: (60 Marks)

5 (a) Out of the 29 basketball teams, 14 teams have 12 players and 15 teams have 13 players. A
sample of 58 players is to be chosen as follows. Each team will be asked to place cards with
its players’ names (1 card for 1 name) into a hat and randomly draw out two names. The two
names from each team will be combined to make up the sample. Explain why this procedure
will not result in a simple random sample of the 363 basketball players. Describe a procedure
that will result in a simple random sample in this situation. [2]

(b) A cable company plans to survey potential customers in a small city currently served by
satellite dishes. It intends to select a sample of families from each of the five non-overlapping
neighbourhoods that make up the city. Suggest, with justification, a sampling technique that
could be used in this case. [2]

NYJC 2010 JC2 Preliminary Examination 9740/02 [Turn Over


4
6 A coffee production factory claims that the average amount of coffee in a packet is at least 10 grams.
A consumer suspects the factory has overestimated the mean. To check the claim, a random sample
of 8 packets is weighed and the values are given as follows.
9.5 10.1 9.3 10.4 9.6 9.1 9.9 9.0
Making necessary assumption(s), test the factory’s claim at 3% level of significance. [4]

A second sample of 64 packets is obtained and the data are summarised as follows:

 (x  10)  25.6,  (x  10) 2


 151.99

Test the factory’s claim at 5% level of significance. [2]


If an observation of mass 9.6 grams is added to the second sample, without conducting the test,
decide whether the conclusion of the second test will still remain the same? [1]

7 An experiment was conducted to investigate the relationship between the amount of unreacted
chemical, x, and the time that elapsed since the start of the experiment, t.

x 25.5 28 31 33.5 41 43.5 45.5 51 57.5 58 73


t 46 44 35.5 30 20 15.6 17 12.3 11 8.3 6

(i) Obtain the scatter diagram for x against t and comment on any relationship between x and t.
Calculate the linear product moment correlation coefficient, r, between x and t. [4]
(ii) State with a reason (without any calculations) which of the following models is more
appropriate for the data:
(a) x = atb, where a > 0 and b < 0
(b) x = a + bt2, where a > 0 and b < 0. [1]
(iii) If there was an error in recording the t values and all the t-values must be increased by 3, what
would be the effect on

(a) t , [1]
(b) standard deviation of t, [1]
(c) the correlation coefficient, r? [1]

NYJC 2010 JC2 Preliminary Examination 9740/02


5
8 Four soccer players, three tennis players, two badminton players and one swimmer sit at a round
table with 10 seats. Find the number of possible seating arrangements
(i) if the two badminton players sit together, [2]
(ii) if the two badminton players sit directly opposite each other, [2]
(iii) if the two badminton players sit together and none of the four soccer players sit next to either
of the two badminton players, [2]
(iv) if the two people sitting beside the swimmer are from different sports. [2]

9 A vehicle rental company has 7 cars and 4 vans available for rental per day. It is known that the
request for cars has a mean of 4 per day; and independently, the request for vans has a mean of 2 per
day.
(i) Find the probability that the number of request for a vehicle exceeds 11 on a particular day.[2]
(ii) Find the probability that some requests for a vehicle have to be refused on a particular day. [2]
(iii) Explain why the value found in (ii) is larger than the value found in (i). [1]
(iv) Using a suitable approximation, find the least number of days such that the average number
of requests for vehicles exceeding 7 is less than 0.001. [3]

10 Chris takes a bus to school every day in a 5-day week. The bus journey consists of two intermediate
4 2
stops. The probability of delay at stop A is while the probability of delay at stop B is . The
5 5
delays are independent of each other. If the bus is delayed at either stop, Chris will be late for
school.
(i) Given that Chris was late on Monday, find the probability he was late exactly three times in a
week. [3]
(ii) Given that Chris was late one day, find the probability that he was delayed at stop A. [2]
(iii) Given that Chris was delayed at exactly one stop, find the probability that he was delayed at
stop B. [3]

NYJC 2010 JC2 Preliminary Examination 9740/02 [Turn Over


6
11 On the tropical island of Stabletree, records show that the number of floods occurring each month
may be modelled using a Poisson distribution with mean 2. A “bad” month is a month where there
are at least 4 floods occurring, and a “bad” year is one where there are more than 2 bad months in the
year. Regarding a month as a twelfth part of a year, and assuming independence of flood
occurrences,
(i) show that the probability that there are at most 2 bad months in a year is 0.760. [3]
(ii) use a suitable approximation to find the probability that out of fifty years, there are less than 5
bad years. [4]
(iii) Comment on whether it is suitable to use the model to estimate the probability that the year
2060 (which is 50 years from now) will be a bad year. [1]

12 The mass of a Munchi pear is normally distributed with mean mass 120g and standard deviation 10g.

(i) If two Munchi pears are chosen at random, find the probability that one of the pears will have
a mass between 100g and 126g while the other will have a mass of less than 115g. [2]

(ii) If ten Munchi pears are chosen at random, find the probability that exactly three of the pears
will have a mass more than 122g each. [2]

The mass of a Fuchi apple is normally distributed with mean mass 115g and standard deviation 8g.

(iii) Find the probability that the total mass of three randomly chosen Fuchi apples will be more
than three times the mass of a randomly chosen Munchi pear. [2]

(iv) A random sample consisting of n Fuchi apples is chosen. Find the least value of n such that
there is a probability of not more than 0.3 that the sample mean differs from its mean mass by
more than 4g. [3]

-----END OF PAPER-----

NYJC 2010 JC2 Preliminary Examination 9740/02


2010 NYJC JC2 Prelim 9740/1 Solutions 2010 NYJC JC2 Prelim 9740/1 Solutions
Qn
Qn 3 1 −
1

1 = (4 − x) 2
Let the equation of parabola be y = ax 2 + bx + c 4− x
At points (-1, 9) and (1, 5) 1 −
1
2  x 2

9 = a− b+ c = (4) 1 − 
5= a + b+ c  4
dy
= 2ax + b 1

 1  x 
=  1 +  −  −  + 2( )( )
−1 −3 2 
2  − x  + ... 
 
dx 2   2  4  (1)(2)  4 
dy 1  
At line of symmetry, = 0, x =
dx 3 1 x 3 2
≈ 1 + + x 
2 2  8 128 
0 = a+b
3
using GC, a = 3, b = −2, c = 4
2 1
( 2 x )  dx
2 1 2x 3
Equation of parabola is y = 3x 2 − 2 x + 4 ∫ dx ≈ ∫ 1 + +
2
0
4 − 2x 0 2 8 128 
1 2  x 3 2
= ∫  1 + + x  dx
2 0  4 32 
2
1 x2 1 3 
=  x + 8 + 32 x 
2 (1+ai)(b+2i) = 8i 2 0
=> (b-2a)+(ab+2)i = 8i

( 2) 
2
Comparing real/imaginary parts, b-2a = 0 and ab+2 = 8 1 1
( 2 )  = 17322 + 81
3
i.e. b = 2a and a(2a) + 2 = 8 = 2+ +
2 8 32
i.e. a 2 = 3  
i.e. a = - 3 (since a < 0) and b = - 2 3
17 1
∴b = , a=
5π 32 8
w = - 2 3 - 2i => arg(w) = −
6
5nπ
Hence, arg(wn) = − 4 d2 x d 3x dx
6 Since 2
+ 9 x = 3t , 3 + 9 = 3 .
∴ Least n = 3 dt dt dt
−5π dx d2x d 3x
(then argument is , so that wn is of form – ki with k > 0). When t = 0, x=0, = 1, 2 = 0 , 3 = −6 . Thus the Maclaurin’s series is
2 dt dt dt
x = t − t 3 +⋯
x = Asin 3t + Bt
 (3t)3 
≈ A  3t − + Bt
 6 
9A 3
= (3A + B)t −
t
2
Equating coefficients:
 3A + B = 1
 2 1
 9A ⇒A= ,B=
 =1 9 3
 2

Page 1 of 9 Page 2 of 9
2010 NYJC JC2 Prelim 9740/1 Solutions 2010 NYJC JC2 Prelim 9740/1 Solutions
Qn
5 (i) c=a+b 6 i y
   
(ii) ˆ = OA OB sin AOB
Area = OA × OB × sin AOB ˆ = OA × OB = a × b
ˆ has maximum value of 1, thus maximum area of OACB is
(iii) Since sin AOB
a b.
 a   b 
 1   1 
(iv) Let a =  a2  and b =  b2  . We have aib = a b cos θ ≤ a b .
   
 a3   b3 
2 2 2 x
Thus aib ≤ a b . Since aib = a1b1 + a2b2 + a3b3 , we have
(a1b1 + a2b2 + a3b3 ) 2 ≤ ( a12 + a22 + a32 )( b12 + b22 + b32 ) .

ii

iii

7 7(i) f ′( x ) = 2 x − 2 = 2( x − 1) > 0 since x > 1


So f is strictly increasing on its domain and therefore one-one.

(ii) f(x) = f −1 ( x ) ⇒ f(x) = x since the graphs of y = f(x), y = f −1 ( x ) and y


= x all meet at the same point(s).
Thus x 2 − 2 x + 2 = x
(x − 1)(x − 2) = 0
x = 2 or x = 1 (reject since x > 1)
Hence x = 2.
Page 3 of 9 Page 4 of 9
2010 NYJC JC2 Prelim 9740/1 Solutions 2010 NYJC JC2 Prelim 9740/1 Solutions
Coordinates of point Q: ( 2 + 9e 2 , 0)
(iii) y
y = f-1(x)
( 2)
2
PR = + (6e) 2 = 4 + 36e 2 = 2 1 + 9e 2
y = f(x)
( 2 + 9e − 2 ) + (0 − 3e) 2
2

Soln set = (1, 2] PQ = 2

y
O 1 2 x = 81e4 + 9e 2
(iv) Since Rf = (1,∞) ⊂ [–3, ∞) = Dg, gf exists. = 3e 1 + 9e 2
gf(x) = g( x 2 − 2 x + 2 ) P
Area of ∆PQR
= (x 2
− 2x + 2 + 3 ) 3 Q
1 x
(x ) (x > 1) =   ( PR )( PQ )
O
= 2
− 2x + 5
2 R

( x − 2 x + 5) , x > 1
( )( )
2
Therefore, gf : x ֏ 1
=   2 1 + 9e2 3e 1 + 9e 2
Rgf = (2,∞). 2
= 3e (1 + 9e 2 ) (shown)
8 (i)
(iv) For the line y = mx − 3e to intersect the curve at 2 distinct points, the
absolute value of m must be greater than the tangent to the curve at
point P :
∴ m > 3e

3
x 9 (i) 1
O Asymptotes needed: x = and
4
(ii) x = 2t , y = 3et
2
y = ax – 1.
2 (ii) a 1
dy dy dt 6tet 2 y = ax − 1 + , x≠ , a>0
= × = = 3tet 4x −1 4
dx dt dx 2
dy 4a
=a −
( 4 x − 1)
2
At point P, by observation, t = 1 . dx
dy 4a
Equation of tangent at point P: = 0 when a − =0
( 4 x − 1)
2
dx
y − y1 = m ( x − x1 ) i.e. ( 4 x − 1) = 4
2

y − 3e = 3e( x − 2) 3 1
x= or −
∴ y = 3e( x − 1) (shown) 4 4
(iii) Coordinates of point R: (0, -3e) Coordinates of turning points:
Equation of normal at point P: 3 5   1 3 
1  , a − 1 or  − , − a − 1
y − y1 = − ( x − x1 ) 4 4   4 4 
m (iii) (a)
1
y − 3e = − ( x − 2)
3e
1  2 
∴ y = − x +  + 3e 
3e  3e 
Let y = 0: x = 2 + 9e2

Page 5 of 9 Page 6 of 9
2010 NYJC JC2 Prelim 9740/1 Solutions 2010 NYJC JC2 Prelim 9740/1 Solutions
100 100
t t
x = A100e 99
− xAe 99

100 100
t t
99
A100e 100e 99
x= 100
= 100
t t
1 + Ae 99
99 + e 99

(iii) x
100

1
0 t

1   1
(b) The line y −  a − 1 = m  x −  passes through the point of
4   4 (iv) Using GC, 5.64 years.
intersection between the 2 asymptotes. For the 2 graphs to intersect, m >
a. (v) The farmers may be influenced by adoption of innovation from other
sources, eg mass media, besides farmers.
10 (i) dx Or any other reasonable answer
= k ( x)(100 − x) 11 (ω − 2)5 − 32 = 0 ⇒ (ω − 2)5 = 25 e 2 kπ i
dt
dx
At t = 0 , x = 1, = 1 ω = 2 + 2e
2 kπ i
5
kπ i
= 2e 5 e 5 + e ( kπ i − k5π i
)
dt
 kπ  kπ5 i
1 = k (1)(99) ⇒ k =
1 = 4 cos   e , k = 0, ± 1, ± 2
99  5 
dx 1 (i) z−2 = 2
therefore = (100 x − x 2 ) Im
dt 99
(ii) dx 1
= (100 x − x 2 ) C W2
dt 99
B 4
∫ ∫
1 1
dx = dt
x(100 − x) 99

∫ ∫
1 1 1 1
+ dx = dt 2π
100 x 100 − x
5
99 A
O 2 4 4 Re
100
ln x − ln(100 − x) = t +C
99 3π
5
 x  100 4
ln  = t +C
 100 − x  99 W1
100 100
x t +C t 4
= e 99 = Ae 99
100 − x
3π i
(ii) c = 2 + 2e 5 .
1 π π 
When t = 0 , x = 1 , A = (iii) By symmetry, ∡BAW2 = . Thus BW2 = 2sin   .
99 5 5

Page 7 of 9 Page 8 of 9
2010 NYJC JC2 Prelim 9740/1 Solutions
12 d
(a)(i) (1 − x ) = −2xn(1 − x 2 ) n−1
2 n
dx
(ii)
3 2 n 1 2 
( ) 2 n
∫ x (1 − x ) dx = 2 ∫ − x  −2 x 1 − x  dx
 
( )

( )
dv n
Let u = - x 2 , = −2 x 1 − x 2
dx

(1 − x2 )
n +1
du
= −2 x , v =
dx n +1
x 2 (1 − x 2 )n +1
3
∴∫ x 1− x ( )
2 n
dx = −
2(n + 1)
− ∫ −2 x ⋅
1
2(n + 1)
(1 − x 2 )n +1 dx

2 2 n +1
x (1 − x ) 1 2 n +1
=− + ∫ x ⋅(1 − x ) dx
2(n + 1) n +1
x 2 (1 − x 2 )n+1 1 1
=− − (1 − x 2 ) n+ 2 + C
2(n + 1) 2 (n + 1)(n + 2)

(b)(i) Area of A =
 3π π  3π

−  − ∫π sin x + cos x dx
4
2
 4 4 4

π 2 3π
= − [sin x − cos x ]π4
2 4

π 2
= − 2
2

π 2
(ii) By translation, req’d area = − Area of A = 2
2

π 3π

(iii) Volume = π ( 2 )2 − π ∫π (sin x + cos x)2 dx


4

2 4

= 4.93

Page 9 of 9
2010 NYJC JC2 Prelim 9740/2 Solutions 2010 NYJC JC2 Prelim 9740/2 Solutions
Qn
Qn
1 n
d1 (1 − (er )n ) 100(1 − ern ) dV
(a)(i) Since xn = ∑ d k = = . At max/min V, = 0:
k =1 1 − er 1 − er dx
(a)(ii) In order for population to stabilise at K, the sum of the geometric dV π x 2
= ( −1) + π x ( 2c − x )
sequence must converge. Thus er < 1 . Hence r < 0 . dx 2
 x
K = lim xn =
100
. = π x  2c − x − 
n →∞ 1 − er  2
(b)(i) Let y = lim yn . Taking limits πx
n →∞ = ( 4c − 3 x )
2
lim yn +1 = lim 2 yn (1 − yn )
n →∞ n →∞

⇒ y = 2 y (1 − y ) πx
∴ ( 4c − 3 x ) = 0
1 2
⇒ y = 0, 4c
2 x= or x = 0 (rejected)
1 3
Using GC, yn is an increasing sequence, thus y = .
2
d 2V π x π
Thus, the limiting population is 500. = ( − 3 ) + ( 4c − 3 x )
(b)(ii) Need K = 500. Thus dx 2 2 2
100 4 π
= 500 ⇒ r = ln   . = ( − 3 x + 4c − 3 x )
1 − er 5 2
2 = π ( 2c − 3 x )

4c d 2V   4c  
x when x = , = π  2c − 3    < 0
3 dx 2   3 
c h 4c
Therefore, volume V is maximum when x = .
3
2c
2
 4c   1  4c  
max volume = π    c −   
Using similar triangle,  3   2  3 
c c−h 16π c3
= max V =
2c x 27
x
h=c−
2 Surface area, S = π x ( x + 2c )
 x S is a strictly increasing function for x > 0
Volume, V = π x 2  c − 
 2 ∴ S is maximum when x is maximum:
π x2 Since x < 2c
V= ( 2c − x )
2 S = π x ( x + 2c ) < π ( 2c )( 2c + 2c ) = 8π c 2
∴ S < 8π c 2
 x
Surface area, S = 2π x 2 + 2π x  c − 
 2
= 2π x 2 + 2π xc − π x 2
S = π x ( x + 2c )
Page 1 of 9 Page 2 of 9
2010 NYJC JC2 Prelim 9740/2 Solutions 2010 NYJC JC2 Prelim 9740/2 Solutions
Qn Qn
3 3(i) Dividing the reurrence relation by r gives
ur +1 ur
− =r. un 1  n2 − n + 2 
(iii) =  
r +1 r n 3
2 n2 
Summing both sides from r = 1 to r = n – 1, we have
n −1 n −1 1 1 2  1
= 1 − +  → as n → ∞ .
∑  r + 1 − r  = ∑ r
ur +1 ur
2  n n2  2
r =1 r =1 4 4(a)(i) Equation of l is r = i – j + k + λ(2i + j – 2k).
u2 u1  1 + 2λ 
−  
2 1 Substitute r =  − 1 + λ  into equation of p gives
u u n −1  1 − 2λ 
+ 3− 2 = (1 + n − 1)  
3 2 2
⋮  1 + 2 λ    1
   
u u
+ n − n−1  − 1 + λ  •  1 = 3
n n −1  1 − 2λ   1
   
[Note that LHS is a telescopic series and RHS is an arithmetic series] ⇒ 1 + 2λ – 1 + λ + 1 – 2λ = 3 ⇒ λ = 2.
n ( n − 1) 5 
Thus
un u1
− = which gives  
n 1 2 Therefore OB = 1  .
 − 3
 n ( n − 1)  n 2  
un = n 1 +
2  2
(
 = n − n + 2 as desired. )

(ii) A
φ
(ii) Let Pn denotes the proposition in (i)
θ
For n = 1, LHS = u1 = 1
B p (side view)
1
(
RHS = 12 − 1 + 2 = 1 = LHS.
2
) Let φ be the angle between l and the normal vector of p. Taking
So P1 is true. scalar product of the direction vector of l and the normal vector of
Assume Pk is true for some k = 1, 2, 3, …. p gives
That is, uk = ( k 2 − k + 2 ) ------ (IH)
k  2   1
2     1
1  • 1 = 3 3 cos φ ⇒ cos φ = .
k +1   − 2   1 3 3
( k + 1) − ( k + 1) + 2  .
2
To prove uk +1 =    
2 
For n = k + 1, [Note that φ is acute since cos φ > 0]
k +1 Let θ be the acute angle between l and p.
(
LHS = uk +1 = k 2 + uk
k
) by the recurrence reln
sin θ = sin(90° – φ) = cos φ =
1
.
  k +1 3 3

k
2
(
= k 2 + k 2 − k + 2 
 k
) by (IH)
(iii) shortest distance from A to p
k +1 2
=
2
(
k +k +2 ) 4 
 1 
= AB sin θ =  2  
6 2
k +1   = = .
( k + 1) − ( k + 1) + 2 = RHS.
2
=  −4   3 3  3 3 3
2   
So Pk+1 is true.
Since P1 is true and Pk true ⇒ Pk+1 is true, by the principle of MI, Pn is
true.

Page 3 of 9 Page 4 of 9
2010 NYJC JC2 Prelim 9740/2 Solutions 2010 NYJC JC2 Prelim 9740/2 Solutions
Qn Qn
4  X − 10
Under H0, T = ~ t (7)
AB =  2  = 42 + 22 + ( −4 ) = 6
2
(iv) s/ n
 −4  Reject H0 if p-value < 0.03
  Since x = 9.6125 , s = 0.49117 , p-value = 0.0304
length of the projection of AB onto p
2
 2  26 Since p-value > 0.03, there is insufficient evidence at 3% level of significance
= 62 −   = 2 3 using Pythagoras’ Theorem level to reject H0, and we conclude that the factory’s claim could be correct.
 3
To test H 0 : µ = 10 vs H1 : µ < 10 at 5% level of significance
X − 10
Under H0, using Central Limit Theorem, Z = ~ N (0,1) approx
(b) By GC, the equation of the line of intersection of the planes given by s/ n
the last 2 equations is Reject H0 if p-value < 0.05
3  −2  25.6 1  (614.4 − 10 × 64) 2 
    Since x = 10 − = 9.6 , s 2 = 151.99 −  = 2.25
r = 0 + λ 2  . 64 63  64 
0  
  1  Thus, p-value = 0.0164
For the system to have infinite solutions, the above line must lie on the Since p-value < 0.05, there is sufficient evidence at 5% level of significance
plane given by the first equation. So the line is perpendicular to the level to reject H0, and we conclude that the factory’s claim may not be correct.
normal vector of the plane. That is,
Since sample mean remains unchanged and s2 decreases, the p-value will
 1   −2 
    1 decrease. Thus we will still reject H0.
 α  •  2  = 0 ⇒ -2 + 2α + 3 = 0 ⇒ α = − 2
 3  1  7 1(i)
   
Furthermore, any point of the line is also a point of the plane. Take 80
(3,0,0) and substitute into the first equation gives 3 + α(0) + 3(0) = β 70
⇒ β=3 60
50
5 (a) - The first method does not give a simple random sample as each player 40 x
does not have the same chance of being selected. 30
- A simple random sample can be obtained by numbering all the 363 from 20
001 to 363 and then picking three digits at a time from a random number 10
table, ignoring numbers over 363 and ignoring repeats, until a group of 0
0 10 20 30 40 50
58 numbers is obtained. The players corresponding to these 58 numbers
will be a simple random sample.
There appears to be a curvilinear/non-linear relationship between x and t. r ≈ -
(b) Stratified sampling, where the population is divided into homogeneous 0.920.
groups called strata and random individuals from each stratum are (ii) Model (a) is more appropriate because the graph of x = atb, where a > 0
chosen. Stratified samples can give useful information about each stratum and b < 0 is concave upwards whereas the graph of x = a + bt2, where a > 0
(in this case, about each of the five neighbourhoods) in addition to and b < 0 is concave downwards. The points on the scatter diagram fall on a
information about the whole population (the city population) graph that is concave upwards.
(iii)(a) t will increase by 3 (b)standard deviation of t remains the same
Quota sampling as the sampling frame is not available. The population is (a) r remains unchanged
divided into non-overlapping groups and the sample size is determined
without any basis. 8 (i) 8! x 2! = 80640
(ii) 8C4 x 4! x 4! or 7! x 8 = 40320
6 To test H 0 : µ = 10 vs H1 : µ < 10 at 3% level of significance (iii) 4C2 x 2! x 2! x 6! = 17280
Assume that the distribution of the amount of coffee in a packet is normally (iv) [4C1. 3C1 +3C1. 2C1 + 4C1 .2C1] x 2! x 7! = 262080
distributed.
Page 5 of 9 Page 6 of 9
2010 NYJC JC2 Prelim 9740/2 Solutions 2010 NYJC JC2 Prelim 9740/2 Solutions
Qn Qn
9 Let X be the number of requests for cars on a particular day. X ~ Po(4) (iii) P(Chris was delayed at B | he was delayed at exactly one stop)
Let Y be the number of requests for vans on a particular day. Y ~ Po(2) = P(Chris was delayed at stop B only)/P(delayed at exactly one stop)
(i) Let T be the number of requests for vehicles on a particular day. Thus 1 2
( )( )
T ~ Po(6) 5 5 1
= =
prob.req′d = P (T > 11) 4 3 1 2
( )( ) + ( )( ) 7
5 5 5 5
= 1 − P (T ≤ 11)
= 0.0201 11 i) Let X be the no. of floods in a month.
(ii) Either demand for a car or a van is not met. Thus Then, X ~ Po(2), and P(bad month) = P(X ≥ 4) = 0.142877
prob.req′d = P ( X > 7 or Y > 4) Let Y be the no. of bad months in 12 months (note: 1 year = 12 months)
= 1 − P ( X ≤ 7 and Y ≤ 4) Then, Y ~ B(12, 0.142877)
= 1 − P ( X ≤ 7) P (Y ≤ 4) Hence, P(Y ≤ 2) = 0.76006
= 0.101 ii) Let W be the no. of bad years, out of fifty years.
(iii) The event in (i) is a subset of the event in (ii). Thus the value obtained Then, W ~ B(50, 1 – 0.76006) ~ B(50, 0.23994)
in (i) will be smaller.
(iv) Let n be the number of days needed. Assume that n is large. By Central Since n = 50 is large, np = 11.997 > 5 and nq = 38.003 > 5,
 6 W may be approximated using N(11.997, 9.11844).
Limit Theorem, T ~ N  6,  approx.
 n
P (T > 7) < 0.001 P(W < 5) → (c.c.) P(W < 4.5) = 0.00652 or 0.00650
 7−6
⇒ PZ >  < 0.001
 6 
 n 
iii) It is not suitable since the weather conditions may have changed over
 n such a long time period and the model may no longer be applicable.
⇒ P  Z ≤  > 0.999
 6 
n 12 (i) Let M be the mass of a randomly chosen Munchi pear.
Thus > 3.09023 ⇒ n > 57.3
6 M ∼ N (120,10 2 )
Thus least number of days required is 58.
Using GC,
10 4 2 4 2 22
(i) P(Chris is late in a day) = + − ( )( ) = P (100 < M < 126 ) ≈ 0.7030
5 5 5 5 25
22 P ( M < 115) ≈ 0.3085
Let X be the number of days Chris is late out of 4 days. X~B(4, ).
25 Required probability = ( 0.7030 )( 0.3085 ) × 2! ≈ 0.434 (3 s.f.)
P(Chris was late exactly thrice in a week | Chris was late on Mon)
= P(Chris was late twice in the remaining four days of the week)
(ii) Using GC,
= P(X = 2)
P ( M > 122 ) ≈ 0.4207
≈ 0.0669
(ii) P(Chris was delayed at A | Chris was late) Let S be the number of pears of mass that is more than 126g.
4 S ∼ B (10, 0.4207 )
= P(Chris was delayed at A)/P(Chris was late) 5 =
10 P ( S = 3 ) ≈ 0.196 (3 s.f.)
22 11
25 (iii) Let F be the mass of a randomly chosen Fuchi apple.
F ∼ N (115,82 )

Page 7 of 9 Page 8 of 9
2010 NYJC JC2 Prelim 9740/2 Solutions
Qn
(
F1 + F2 + F3 − 3M ∼ N ( 3 × 115 − 3 × 120 ) , ( 3 × 82 + 9 × 102 ) )
⇒ F1 + F2 + F3 − 3M ∼ N ( −15,1092 )

Using GC,

Required probability
= P ( F1 + F2 + F3 > 3M )
= P ( F1 + F2 + F3 − 3M > 0 ) ≈ 0.325 (3 s.f.)
(iv) F ∼ N (115,82 )
 82 
⇒ F ∼ N 115, 
 n
( )
P F − 115 > 4 ≤ 0.3

⇒ P( F − 115 < −4) + P( F − 115 > 4) ≤ 0.3


   
 4   4 
P Z < −  + P Z > 2  ≤ 0.3
2
 8   8 
 n   n 
 n n
∴P − <Z <  ≥ 0.7
 2 2 
 n
⇒ P Z <  ≥ 0.85
 2 
invNorm(0.85) = 1.036
n
≥ 1.036 ⇒ n ≥ 4.296
2
Hence, least n = 5.

Page 9 of 9
3

1 It is known that the number of diagonals that can be drawn in a polygon of n sides
can be expressed as a quadratic polynomial in n. By considering the number of
diagonals in a triangle, a quadrilateral and a pentagon, find the number of
diagonals that can be drawn in a polygon of 200 sides. [4]

2 Given that f ( x)  ln(cos x) , show that

f ''( x)   sec 2 x .

By further differentiation, find the first two non-zero terms in the Maclaurin’s
series of f ( x) .

Hence obtain an approximate value for

0.4
0 ln  cos x  dx . [6]

3 Solve the inequality

ln( x  1)  9  x 2 .
Hence, solve the inequality

ln( x  1)  9  x 2 . [5]

4 A

3m
C 10 ms 1

xm 2m
B
An athlete at point C is running towards the finishing line AB, at a constant speed
of 10 ms 1 in a direction perpendicular to AB, as shown in the diagram above.

(i) Given that ACB is  and x is the distance of the athlete from AB, show
that

3 2
  tan 1  tan 1 . [2]
x x

(ii) Find the exact rate of change of  when the athlete is 10 m from the
finishing line. [3]
[Turn over
4
1
5 (i) Sketch the graph of y  x  , stating the equations of any asymptotes and
x
the coordinates of any points of intersection with the axes. [2]

(ii) The curve C has equation

p 2 x 2  pqx  1
y ,
px  q

where p and q are positive constants.

State a sequence of transformations which transform the graph in (i)


to the graph of C. [3]

1
6 Find the first three terms in the expansion, in ascending powers of , of 3  x
x
expressing the coefficients in their simplest form. State the set of values of x for
which the expansion is valid.

Hence, by substituting x  25 , find an approximate value for 7 as a fraction.


[6]

7 Find the exact value of


6 2x
e sin x dx . [5]
0
5

y
8
C
(1, 4)
(3, 2)
D y  f ( x)
1
y
A B 2
x
1 0 5
2

x2

The diagram shows the graph of y  f ( x) . The curve crosses the x-axis at the

5 
points A (1, 0) and B  , 0  , and has a minimum point at C (1, 4) and a
2 
1
maximum point at D (3, 2) . The lines y  , x  0 and x  2 are asymptotes to
2
the curve. Sketch, on separate diagrams, the graphs of

(i) y   f ( x) , [3]

(ii) y  f '( x) , [3]

labeling each graph clearly and showing the asymptotes and coordinates of the
points corresponding to A, B, C and D.

[Turn over
6

9 Find the roots of the quartic equation z 4  2  i2 3  0 . Give your answers


exactly, in the form rei , where r  0 and      . [3]

The polynomial P(z) has degree eight and real coefficients. All the roots of the
equation z 4  2  i2 3  0 are also the roots of the equation P(z) = 0. By
considering P(z) as a product of two quartic factors, find P(z), expressing all
the coefficients in real and non-trigonometrical form. [3]

10 A curve is defined by the parametric equations

x  et , y  t2  t .

dy
Find an expression for in terms of t. [1]
dx

(a) Given that the tangent to the curve at the point with parameter p passes
through the origin, find the exact values of p. [3]

d 2 y d  dy  dx
(b) Given that    , deduce the exact range of t for which the
dx 2 dt  dx  dt
curve is concave upwards. [3]

11 A seed from a type of tree was planted and the change in the height of the tree
is noted at the end of each year. It is observed that the change in height of the tree
follows an arithmetic progression in the first 10 years of growth, and
subsequently, follows a geometric progression from the end of the 10th year.
Given that the tree’s change in height was 40 cm and 70 cm at the end of the 3rd
year and 5th year of its growth respectively, find

(i) the change in height of the tree at the end of the 4th year of its growth, [1]

(ii) the height of the tree at the end of the 10th year of its growth. [3]

Given that the tallest the tree can ever grow is 20 m, find the year at which
the tree is at least twice as tall as at the end of 10th year of its growth. [6]
7

12 By using substitution u  4  e x or otherwise, show that

ln p


1 1  5p 
x dx  4 ln  4  p  . [4]
0 4e  

Hence,

(i) find the exact volume of the solid of revolution formed when the region
1
bounded the curve y 2  , the y-axis and the line x  ln 6 is rotated
4  ex
through 2 right angles about the x-axis, [3]

1
(ii) by considering the area under the curve y  , find the exact value of
4  ex


5  1 4 y 
ln   dy . [3]
1  y 
8

13
x
jogging direction

O treadmill
A woman starts to jog at a rate proportional to the distance x from O, along a
treadmill which is running at a constant rate of a ms 1 , as shown in the diagram
above. At time t seconds, the distance she has moved is x m. When her distance
from O is 1 m, her position remains stationary. Show that

dx
 a  x  1 .
dt
[3]
1
Given that at t = 2s, she is moving at 2 ms1 , find the speed of the treadmill.
e
[7]

Hence find the exact distance she has further moved after another 2 seconds. [2]

[Turn over
8

 2 1
14 The planes p1 and p2 have equations r.  1   4 and r.  2    respectively,
 
1  
   
where  and  are negative constants. p1 and p2 are inclined at 60 to each
other. The point A with position vector i − 2j, where  is a constant, is in both
p1 and p2 .

(i) Find the position vector of A. [1]

(ii) The point B has position vector 3i − 3j + 2k. Find the shortest distance
from B to p1 . [3]

(iii) Find the equation of p2 in the form r.n  d . [4]

The plane p3 has equation  x  y  z   , where  and  are constants.

(iv) Find the values of  and  if p3 passes through the origin and p1 ,
p2 and p3 have only one common point, which is also the foot of
perpendicular from A to p3 . [3]

(v) What can be said about the values of  and  if p1 , p2 and p3 have no
common point and all points on p3 are equidistant from p1 ? [2]
3

Section A: Pure Mathematics [40 marks]

1 x cm
A B
5 cm
l cm
D

10 cm C
A student wants to draw a straight line of length l cm that begins at the side AB,
ends at the side BC and passes through the point D, as shown in the diagram
above. (AB and BC may be assumed to be infinitely long.)

(i) By using similar triangles, show that

25 x 2
l 2  x2  . [3]
 x  10 
2

(ii) Hence find the length of the shortest straight line he can possibly draw. [3]

2 The function f is defined by

b 1
f :x a 2 , x ,
x 2

where a and b are positive constants.

(i) Sketch the graph of f and state range of f. [2]

(ii) Find f 1 . [3]

(iii) Given that


b2 1
gf : x   2 , x ,
x 2

find g( x) in terms of a and b. [2]

[Turn over
4

3
D C

5m
6m 6m
A 10 m
B
H G
3m k j 3m
4m
O i
E 10 m F

The diagram above shows a partial design of the roof of the gallery of an
amphitheatre. ABCD is an inclined rectangular roof, where AB = 10 m, and
BC  5 m . EFGH is a rectangle on a horizontal ground, where EF = 10 m and
EH  4 m . The points A and B are 3 m directly above E and F respectively. The
points C and D are 6 m directly above G and H respectively.

Point O is the centre of EFGH andis  taken as


the
 origin. Perpendicular unit
vectors i, j, k are in the direction of EF , EH and EA respectively.

10 
  
(i) Show that AC   4  and find a vector equation of the line passing
3
 
through A and C. [3]

(ii) A spot light is fixed at the top of a vertical pillar erected at point Q
with coordinates 15, 6, 0  . Find the height of the pillar if the top of the
pillar is collinear with A and C. [3]

(iii) The main electrical supply to the gallery runs along the diagonal AC. Find
the position vector of point X on AC such that the length of the electric
cable that is required to provide electricity from the main electrical supply
to an amplifier mounted at D, will be the least. [3]
5

4 A sequence {xn } of negative numbers is defined by x0  2 and

n 1
 1
xn  xn1     for n  1 .
 2

n 1
4  1  
(i) Prove by mathematical induction that xn      1 for n  0 . [4]
3  2  

(ii) The sequence {xn } converges to l as n tends to infinity. State the exact
value of l. [1]

N
(iii) Determine, with a reason, whether  xn converges. [1]
n1

(iv) By considering xn  xn1 , deduce that xn  xn1 if n is odd and xn  xn1


if n is even. [2]

5 Sketch on an Argand diagram the set of points representing all complex numbers
z satisfying both the inequalities

iz  2i  2  2 and Re  z   1   3i . [4]

Find

(i) the range of arg  z  2  2i  , [2]

(ii) the complex number z where arg  z  2  2i  is a maximum. [2]

The locus of the complex number w is defined by w  5  2i  k , where k is a


real and positive constant. Find the range of values of k such that the loci of w
and z will intersect. [2]

[Turn over
6

Section B: Statistics [60 marks]

6 (a) The Royal Club has membership of 1600 people, of which 1000 are males
and 600 are females. The boss wants to find out what members think of
the new set meals menu offered by the Oiishi restaurant of the Club. A
sample of 80 members is to be chosen. State a suitable sampling method
and describe in detail how this can be carried out. [3]

(b) The amount of tips a member gives after a meal in Oiishi restaurant of
Royal Club has mean $5 and standard deviation $1. Estimate the
probability that the amount of tips collected from 80 randomly chosen
customers is between $350 and $410. [2]

7 In an experiment, a new computer game and a new mathematics quiz are given
to a group of teenagers. It may be assumed that the teenagers are playing the
computer game and attempting the mathematics quiz for the first time. The
computer game score, u, and the mathematics quiz score, v, of 8 teenagers are
given in the table below.

Teenager 1 2 3 4 5 6 7 8
Computer game score, u 52 90 64 74 80 82 76 74
Mathematics quiz score, v 60 90 68 74 74 90 82 70

(i) Find the linear product moment correlation between v and u and the
equation of the regression line of v on u. [2]

(ii) Comment on the suitability of using the regression line in (i) to predict u
given v. Use the appropriate regression line to predict u given that v is 85,
giving your answer to the nearest whole number. [3]

The scores are actually recorded for 9 teenagers. However, the scores for the
last teenager are lost.

Teenager 1 2 3 4 5 6 7 8 9
Computer game score, u 52 90 64 74 80 82 76 74 p
Mathematics quiz score, v 60 90 68 74 74 90 82 70 q

It is known that the inclusion of the scores of the last teenager does not alter
the mean computer game score and one of the regression lines is given by
u  45.556  0.395v .

(iii) Find p and q, giving your answers to the nearest whole number. [3]
7

8 A taxi company wants to track the number of morning, afternoon and night trips
made in a day. Of the total trips in a day, p% are morning trips, q% are
afternoon trips and the remaining are night trips. Past records indicated that the
probability that a randomly chosen morning, afternoon and night trip is paid by
NETS is 0.85, 0.65 and 0.25 respectively.

(a) If p  30 and q  50 , find the probability that a randomly chosen trip is


paid by NETS. [3]

(b) Given that the probability of a randomly chosen trip paid by NETS
is taken in the morning is 0.5, express q in terms of p. [4]

9 Defective spots are found randomly on a roll of ribbon. On average, 5 defective


spots are found per 10 m of ribbon. The roll of ribbon is cut into short ribbons,
each of length 1 m. A short ribbon is not discarded if it contains at most 1
defective spot.

(i) State a condition under which a Poisson distribution would be a suitable


probability model and find the percentage of short ribbons that do not need
to be discarded. [3]

(ii) Given 52 randomly chosen short ribbons, find an approximate probability


that there are more than forty-five short ribbons that do not need to be
discarded. [4]

(iii) Determine the maximum number of short ribbons required for which the
probability of at least 3 short ribbons discarded does not exceed 0.1. [2]

10 A six-digit number is to be formed from the digits 1, 2, 3, 4, 5, 6, 7, 8 and 9. For


each of the following cases, find how many different ways the six-digit number
can be formed.

(i) The even and odd digits of the number must alternate and any digit may
appear more than once. [2]

(ii) The number must be odd and is less than 600 000 and no digit may appear
more than once. [3]

(iii) The number is formed from four different digits, eg. 621313, 255567.
[4]

[Turn over
8

11 A star fruit plantation claims that it has developed a method of producing star
fruits that are larger in size with a mean weight of 200 grams. A wholesaler
suspects that the plantation is overstating the weight of the star fruits
produced and decides to test the plantation’s claim. The wholesaler obtains a
random sample of 10 star fruits. The weights (to the nearest gram) of the 10
star fruits are as follows:

189 192 200 205 206 198 188 190 200 192

Find an inequality satisfied by the significance level that will confirm the
wholesaler’s suspicion. State an assumption made in conducting the test. [5]

It is later known that the standard deviation of the weight of the star fruits
produced by the plantation is 6 grams. Another random sample of 10 star fruits
is obtained. A second test is conducted at 2% significance level and the
wholesaler is unable to confirm his suspicion. Find, to the nearest gram, the
range of values for the mean weight of these 10 star fruits in order to arrive at this
conclusion. [4]

Explain, in the context of the second test, the meaning of ‘at 2% significance level’?
[1]

12 The weight, Y, of a Yummy cereal bar is normally distributed with mean


(120 – k) g and standard deviation 10 g. The weight, F, of a Fullness cereal bar is
normally distributed with mean 180 g and standard deviation 20 g.

(i) Given that P(Y < 80) = P(Y > 150), show that the value of k is 5. [1]

(ii) 5 Yummy cereal bars are randomly chosen. Find the probability that
exactly one bar weighs lesser than the lower quartile weight and exactly
one bar weighs more than the median weight. [2]

(iii) Find the probability that the weight of 2 randomly chosen Yummy cereal
bars differs from one and a half times the weight of a randomly chosen
Fullness cereal bar by at most 5g. State the assumption needed for your
working. [4]

The Fullness cereal bars are sold at $2 per 100g.

(iv) Find the probability that a randomly chosen Fullness cereal bar costs more
than $3.50. [2]

(v) A random sample of 100 Fullness cereal bars is to be taken. Using a


suitable approximation, find the probability that there are less than 65
bars with each costing more than $3.50. [3]
1

RI 2010 9740/01/10 [Turn over


2

1 By writing z  x  iy, x, y  , solve the simultaneous equations

w
z 2  zw  2  0 and z *  , where z * is the conjugate of z . [3]
1 i

2 Sketch the graph of y  e x  x . By adding a suitable graph to the sketch, find the set of
2
values of x that satisfies 2 x  5  2e x  . [4]
x2

3 Four friends returned from a trip to Europe and converted their foreign currencies back to
Singapore Dollars. The amounts of foreign currencies converted and the total amounts
received in Singapore Dollars are shown in the following table.

Donald Leonard Michael Raphael


Sterling Pound 36 55 40 k
Euro Dollar 77 18 31 59
Swiss Franc 42 63 26 24
Total amount in 269.90 233.45 175.50 313.00
Singapore Dollars

Assuming that, for each foreign currency, the exchange rate quoted for each of the friends
is the same, calculate the value of k. [4]


1
4 Find (a) dx, [2]
1  2 x  x2


x2  x  3
(b) dx . [3]
( x 2  2)(1  x)

RI 2010 9740/01/10
3

5 A sequence u0 , u1 , u2 , is defined by

u0  1 and un 1  run  d for n  0

where r and d are non-zero constants.

(i) If r = d = 1, write u n in terms of n. [1]

 d  d
If r  1, prove by induction that un  r n 1   for all non-negative integral
 r 1  r 1
values of n. [4]

(ii) If r  1, state the limit of the sequence as n   . [1]

6 The diagram shows the graph of y  f ( x) . The curve has a maximum point at (0,2) and it
cuts the x-axis at the points (a,0) and (a,0) where a is a positive constant. The lines
x  2 , x  2 and y  2 are asymptotes to the curve.
y
y  f(x)

2
y2

a O a x

x  2 x2

Sketch, on separate diagrams, the graphs of

(i) y 2  f ( x), [3]


1
(ii) y . [3]
f (2 x)

Your sketches should show clearly the equations of asymptotes, stationary points and
intersection with the axes, where applicable.

RI 2010 9740/01/10 [Turn over


4

a
7 The curve C has equation y  f (x)  2 x  1  where a and b are positive constants.
bx  1

(i) Show that C has exactly two stationary points. [3]


(ii) Given that C passes through the point (0,3) , find the value of a. [1]
Hence sketch the graph of y  f '(x) , indicating clearly any intersection with the axes
and the equation(s) of asymptote(s) of the curve in terms of b. [3]

8 A cylindrical water tank has a horizontal base with a fixed area of A m 2 and is initially
empty. Water is poured into the tank at a constant rate of 5 m3s1 , and leaks out through a
small hole in the base at a rate which is proportional to the depth of water in the tank. The
depth of water in the tank is x metres at time t seconds. Show that

dx
A  5  kx , where k is a positive constant. [2]
dt
Solve the differential equation, expressing x in terms of A, k and t . [5]

9 (a) Solve the equation z 6  4 2(1  i) , expressing the solutions in the form rei , where
r  0 and       . [3]

(b) (i) The two complex numbers 2  3 + 2i and 2  3 + 2i satisfy the cubic
equation a3 z 3  a2 z 2  a1 z  a0  0 . Explain clearly whether it is possible for all
the coefficients a3 , a2 , a1 , a0 to be real numbers. [1]

(ii) In an Argand diagram, the point A represents the complex number 2  3  i .


If A, B, and C are the vertices of an equilateral triangle taken in clockwise order,
and these three points lie on a circle with centre at the origin, find the complex
number represented by B in the form p  iq where p, q  , giving the exact
values of p and q. [3]

RI 2010 9740/01/10
5

10 The curve C has equation y  xe  x for x  0 and P(a, ae  a ) is a point on C .

(i) Sketch the curve C . [1]

(ii) Find, in terms of a , the equation of the tangent to the curve at P . [2]

This tangent cuts the y  axis at the point Q(0, h) .

Using differentiation, find, as a varies, the exact maximum value of h . [6]

11 The equations of two planes p1 , p2 are

2x  5y + 3z = 3,

3x + y + 6z =  ,

respectively, where  and  are constants.

(i) Given that the two planes intersect in a line l, with a vector equation given by

 4   2 
r =  2   s  1  , s  ,
 5   3 
   

show that the value of  is 12 and find the value of  . [3]

A third plane p3 has equation given by

5x + 8y + tz = 12,

where t is a constant.

(ii) With the values of  and  found in (i), find the exact value of t if the three planes
have no point in common. [2]

(iii) The plane p4 contains the line l and the point (1, 1, 2). Find the cartesian equation of
p4 and the acute angle between p1 and p4. [5]

RI 2010 9740/01/10 [Turn over


6

12 Relative to the origin O, three points A, B and P have position vectors given by

a = 14i + 8j + 6k, b = 11i + tj + 12k and p = 12i  4j + 10k respectively, where t is a


constant.

(a) If P divides the line segment AB in the ratio  : 2, find the values of  and t.

[3]

(b) It is given that t = 2.

(i) Find the exact length of projection of PB onto AB .

Deduce the exact shortest distance of P from line AB. [4]

(ii) BAPQ forms a parallelogram. Find the position vector of the point Q and the
area of the parallelogram correct to 2 decimal places. [4]

4 1
13 Given that f ( x)  ,x  .
(1  3x) x 2  4 3

(i) Express f ( x) as a series expansion in ascending powers of x, up to and including


the term in x3 . [4]

(ii) Find the range of values of x for which the expansion in (i) is valid. [2]

(iii) A curve is given by the equation


2tan y  f (x).
By differentiation, find the series expansion for y in ascending powers of x, up to an
including the term in x 2 . [5]

RI 2010 9740/01/10
7

pn
14 (a) The sum of the first n terms of a series, Sn , is given by n1  5 , where p is a non-
5
zero constant and p  5 .
Obtain an expression for Tn , the nth term of the series and prove that this is a
geometric series.
Find the range of values of p for the sum to infinity to exist. [5]

(b) An arithmetic progression is grouped into sets of numbers as follows:


{2} , {6 , 10} , {14 , 18 , 22} , {26 , 30 , 34 , 38} , …

where the number of terms is 1 for the first set, 2 for the second set, 3 for the third set,
4 for the fourth set and so forth.

(i) Show that the first term in the nth set is 2n2  2n  2 .

(ii) Find, in terms of n, a simplified expression for the last term in the nth set.

(iii) Hence, find the sum of all the terms in the nth set. [5]

RI 2010 9740/01/10 [Turn over


1

RI 2010 9740/02/10 [Turn over


2

Section A : Pure Mathematics [40 marks]

1 1 na 1
1 (i) Given that  2 2 , show that a   . [1]
2(n  1) 2n
2
n (n  1) 2
2
N
2n  1
(ii) Given that S N   2n (n  1)
nM
2 2
, state the smallest possible value of M, where M  

and M  N , such that S N can be defined. [1]

(iii) If M = 3, find S N in terms of N. [3]

(iv) Deduce that the sum to infinity of the series

1 1 1
   ...
(2)(3) (3)(4) (4)(5)2
2 2

1
is less than . [3]
8

 
2 The point A represents a fixed complex number a where  arg(a)  . The complex
4 2
numbers ia and  a are represented by the points B and C respectively.

On a single clearly labelled Argand diagram, show the points A, B, C and the set of points
representing all complex numbers z satisfying both relations


z  ia and arg( z  a)  arg(a)  . [4]
4
Find

(i) the minimum value of | z | in terms of | a | , [2]

(ii) the range of values of arg( z ) in terms of arg(a) . [2]

RI 2010 9740/02/10
3

3 (a) Functions f and g are defined by

f:x ( x  2)( x  4) , for x  , x  4 ,


x
g:x , for x  \ 3 .
x3

Determine if f is one-one, justifying your answer. [1]

Only one of the composite functions fg and gf exists. Give a definition (including the
domain) of the composite that exists, and explain why the other composite does not
exist.

Find the range of the composite that exists. [6]

(b) The function h is defined by


h : x a(1  e  x ) , for x  
0
, where a is a positive constant.

Find h 1 (x) and state the domain of h 1 . [3]

Given that the graphs of y  h( x) and y  h 1 ( x) intersect at the point where x = b, and
b
that  0
h( x) dx  I , find the area bounded by the curves y  h( x), y  h 1 ( x) and the
axes, giving your answer in terms of I and b. [2]

4x
4 The curve C has equation y  f ( x) , where f ( x)   , x .
x 1
2

1
(i) Find the exact value of  1
f (| x |) dx . [2]

(ii) Using the substitution u  3x  e2  3 , find  ln(3x  e2  3) dx .


Hence find the exact area of region R bounded by C, the curve y  ln(3 x  e 2  3) and
the line y  0 . [7]

(iii) Find the volume of solid formed when R is rotated completely about the x-axis,
giving your answer correct to 2 decimal places. [3]

RI 2010 9740/02/10 [Turn over


4

Section B : Statistics [60 marks]

5 (a) Find the number of three-letter code-words that can be formed from the letters of the
word WHYOGEE. [3]

(b) A country is invited to send a delegation of six youths selected from six badminton
players, six tennis players and five football players to participate in the opening
ceremony of the Singapore 2010 Youth Olympic Games. No youth plays more than
one game. The delegation is to consist of at least one, and not more than three players
selected from each sport.

(i) Find the number of ways in which the delegation can be selected. [3]

During the ceremony, the youths from the delegation are to be seated in six out of ten
chairs which are arranged in a row.

(ii) Find the number of ways this can be done if no two empty chairs are adjacent.
[2]

1
6 A six-sided die is thrown. The probability of getting a ‘1’ is p , where 0  p  , and the
15
probabilities of getting ‘1’, ‘2’, ‘3’, ‘4’ and ‘5’ are consecutive terms of an arithmetic
progression with common difference p .
Show the probability of getting ‘6’ is 1 15p . [2]

The die is thrown twice and events A and B are defined as follows:

A : the sum of the scores of the two throws is at least 10;

B : the score of the first throw is 5.

Find

(i) P(A) , [3]


(ii) P(A|B). [2]

Hence, show that there is no value of p for which A and B are independent. [2]

RI 2010 9740/02/10
5

7 A researcher investigates the relationship between the stress level of a person and his job
performance. The table below shows the findings for 8 different workers for the same job.

Stress level, x 2 2.5 3 3.5 4 5 6 7

Performance, y 3.30 3.10 3.00 2.86 2.72 2.60 2.55 2.50

(i) Draw a scatter diagram for the data and calculate the product moment correlation
coefficient between x and y. [2]

(ii) Comment on whether a linear model is appropriate. [2]

(iii) The researcher proposes that a model of the form


b
y  a  where a and b are positive constants
x

is more appropriate.

Explain why he thinks this is so. [1]


(iv) Calculate the least square estimates of a and b for the model in (iii). [2]
(v) Estimate the performance when the stress level is 4.5. Comment on the reliability of
the estimate. [2]

8 The mass of a male student in Aishan Secondary School is denoted by X kilograms. The
masses of a random sample of 150 male students are summarized by

 x  8400  ( x56)  5555.


2
and

(i) Calculate unbiased estimates of the mean and variance of X. [2]

Aishan Secondary School claimed that the mean mass of a male student in the school is 55
kilograms.

(ii) Test, at the 3% significance level, whether the school is understating the mean mass
of a male student. Does the Central Limit Theorem apply in this context? [5]
(iii) State what you understand by the expression ‘at the 3% significance level’ in the
context of the question in (ii). [1]

In a separate study, the opinions of female students in Aishan Secondary School are to be
collected to determine if they are satisfied with their own weights. Describe how a quota
sample of size 100 might be obtained. [2]

RI 2010 9740/02/10 [Turn over


6

9 Every weekday, the last train from Bedok Station leaves the station at 11.29 pm. Its travel
time X (in minutes) from Bedok Station to Redbridge Station may be taken to follow a
normal distribution with mean 30 and variance 9.

(i) Find the travel time exceeded by 90% of the trips made by the last train. [2]

At 11.40 pm every weekday, Abel walks from his workplace to Redbridge Station to catch
the last train. The time Y (in minutes) he takes follows a normal distribution with mean 16
and variance 4.

(ii) Find the probability that the train arrives at Redbridge Station after 12 am and Abel
arrives before 12 am. [2]

(iii) Find the probability that Abel will not miss the train. [3]

(iv) Explain why the answer to part (iii) is greater than the answer to part (ii). [1]

The time W (in minutes) taken by Dina to walk from her workplace to Redbridge Station
has a mean of 4 and a variance of 6.

(v) Explain why W is unlikely to be normally distributed. [1]

(vi) Find the probability that the mean time taken by Dina to walk from her workplace to
Redbridge Station, in a random sample of 40 trips, is less than 3.5 minutes. [2]

10 Incoming telephone calls to the management office of a shopping mall are received
randomly and independently, at an average rate of 6.75 per hour. The mall (including its
management office) is open from 10 am to 10 pm.

(i) State, in the context of the question, a condition required for a Poisson distribution to
be a suitable model for the number of incoming calls from 10 am to 10 pm. [1]
Assume that the condition in (i) is satisfied.

(ii) Find the probability of receiving no fewer than 8 calls in a particular hour. [2]
(iii) In 4 non-overlapping one-hour periods, find the probability of receiving at most 6
calls in a one-hour period, exactly 7 calls in another one-hour period, and no fewer
than 8 calls in each of the 2 remaining one-hour periods. [3]
(iv) The number of incoming calls from 10 am to 10 pm, Y , has mean  and standard
deviation  . Use a suitable approximation to find P      Y      , giving your
answer correct to 4 decimal places. [4]
(v) A day is considered busy if there are more than 90 incoming calls received from
10 am to 10 pm. Find the probability that the 3rd busy day in a month occurs on the
15th day of the month. [3]

RI 2010 9740/02/10
RAFFLES INSTITUTIION
matics 974
H2 Mathem 40
2010 Year 6
__
_________
________
________
_____
20
010 H2 Math
hs Prelimin
nary Exam Paper
P 1 Solu
utions

Q1 [3] Let z = x + iy. Su ubstitute the second


s equaation into thee first.
z + z ( z * (1 + i)) − 2 = 0
2

( x + iy ) 2 + ( x 2 + y 2 )(1 + i) − 2 = 0
2 x 2 + 2ixy + ( x 2 + y 2 )i − 2 = 0
On com mparing reall and imaginnary parts,
2x − 2 = 0, 2xy + x 2 + y 2 = ( x + y ) 2 = 0
2

x = ±1,1 y = m1 .
When z = 1 − i , w = 2i
When z = −1 + i , w = − 2i

Q2 [4]
y
y = ex − x

5 1
y= −
2 x2
α βO x

2
2x + 5 − 2e x ≤
x2
2
⇒ 5− ≤ 2e x − 2 x
x2
5 1
⇒ − 2 ≤ ex − x
2 x

t diagram,, x ≤ α or β ≤ x < 0 or
From the o x>0
Using G.C., x ≤ −2.18 or − 0.920 ≤ x < 0 or x > 0 (3s.f.)
Thereffore solution n set is ( −∞, −2.18] ∪ [ −00.920, 0) ∪ (00, ∞ ) .
Q3 [4] Let x, y,
y z be the exxchange ratee quoted for Sterling Pouund, Euro Dollar and Sw
wiss Franc,
respectively (i.e. 1 Sterling Poound = x Sinngapore Dolllars, 1 Euro Dollar = y Singapore Dollars
D
and 1 Swiss
S Franc = z Singappore Dollars)).
36x + 77 y + 42 z = 269.9
55x + 18 y + 63z = 233.45
40x + 31y + 26 z = 175.5
Using the GC, x = 2.15, y = 1.7 78, z = 1.32
kx + 599 y + 24 z = 313
3
313 − 59(1.778) − 24(1.32)
∴k = = 82
2..15

____
____________
_____________
________
∫ ∫
Q4 [5] 1 1
dx = dx
(a)[2] 1 + 2 x − x2 2 − ( x − 1)2
⎛ x −1 ⎞
= sin −1 ⎜ ⎟+c
⎝ 2 ⎠

∫ ∫
(b)[3] x2 − x + 3 1 1
dx = dx+
2
( x + 2)(1 − x) x + 2 1− x
2

1 ⎛ x ⎞
= tan −1 ⎜ ⎟ − ln |1 − x | + c
2 ⎝ 2⎠

Q5 [6] un = n + 1
(i) [1]

(ii) [4] ⎛ d ⎞ d
Let Pn be the statement un = r n ⎜1 + ⎟− for n ∈ Z0+ .
⎝ r −1 ⎠ r −1
When n = 0,
LHS: u0 = 1 (given)
⎛ d ⎞ d
RHS: r 0 ⎜1 + ⎟− =1
⎝ r −1 ⎠ r −1
∴ P0 is true.
Assume Pk is true for some k ∈ Z + ,
⎛ d ⎞ d
i.e. uk = r k ⎜1 + ⎟−
⎝ r −1 ⎠ r −1
Prove that Pk +1 is true,
⎛ d ⎞ d
i.e. to prove uk +1 = r k +1 ⎜1 + ⎟−
⎝ r −1 ⎠ r −1
uk +1 = ruk + d
⎛ ⎛ d ⎞ d ⎞
=r ⎜ r k ⎜1 + ⎟− ⎟+d
⎝ ⎝ r −1 ⎠ r −1 ⎠
⎛ d ⎞ rd
=r k +1 ⎜1 + ⎟− +d
⎝ r −1 ⎠ r −1
⎛ d ⎞ −rd + rd − d
=r k +1 ⎜1 + ⎟+
⎝ r −1 ⎠ r −1
⎛ d ⎞ d
=r k +1 ⎜1 + ⎟−
⎝ r −1 ⎠ r −1
Since Pk is true implies Pk +1 is true, and P0 is true, by mathematical induction, Pn is true for all
n ∈Z0+ .

(iii) [1] d
As n → ∞, un →
1− r

__________________________________
2010 H2 Maths Preliminary Exam Paper 1
Page 2 of 8
Q6 [6] y
(i) [2] y 2 = f(x)

2
y= 2
−a O a x

− 2 y=− 2

x = −2 x=2
(ii)[2] y

1
y=
f ( 2x)
1
2 y= 1
2

| |
−1 O 1 x

x = − a2 x= a
2

Q7[7]
2
(i)[3] y = 2x + 1 +
bx + 1
dy 2b
= 2−
( bx + 1)
2
dx
dy 2b
When = 0, =2
( bx + 1)
2
dx

( bx + 1) = b
2

x=
1
b
(
−1 ± b )
Since b is a positive constant, there are 2 distinct real solutions for x.
Hence, C has exactly two stationary points. (shown)
y
(II)[3]
y = f ' (x)

2 y=2

| |
x
( ) ( )
1 1
− b +1 O b −1
b
2 − 2b
__ b

1
x=−
b
__________________________________
2010 H2 Maths Preliminary Exam Paper 1
Page 3 of 8
8 [7] dV dx
V = Ax ⇒ =A
dt dt
[2]
dV
= 5 − kx , where k is a positive constant.
dt
dx
⇒ A = 5 − kx (shown)
dt

dx
A = 5 − kx
dt
1
⇒ A∫ dx = ∫ dt
5 − kx
A
[5] ⇒ − ln 5 − kx = t + c1 , where c1 is an arbitrary constant
k
k
⇒ ln 5 − kx = − t + c, where c is an arbitrary constant
A
k
− t +c
⇒ 5 − kx = e A

k
− t
⇒ 5 − kx = Be A
, where B is an arbitrary constant
k
1 − t
⇒ x= (5 − Be A )
k
When t = 0, x = 0 ⇒ B = 5
k
5 − t
∴ x= (1 − e A )
k

Q9 [7] π ⎛π ⎞
i i ⎜ + 2 kπ ⎟
z = 4 2(1 + i) = 8e
6 4
= 8e ⎝4 ⎠

(a)[3] iπ (8 k +1)
z = 2e 24
, k = −3, −2, −1, 0,1, 2
(b)(i)[1] Since the two complex roots are non-real, and not conjugate, for all the coefficients a3 , a2 , a1 , a0
to be real, the degree of the polynomial has to be at least four. Since the degree is only three, it is
not possible.
(ii)[3] Since the points A, B and C are on a circle centered at the origin, and they form an equilateral
2π 2π
triangle, angle AOB is . OA rotated by clockwise about O will give OB. Let b be the
3 3
complex number represented by B.

− i
b=e 3
(2 − 3 + i)
⎛ 1 3⎞
b = ⎜⎜ − − i ⎟⎟ (2 − 3 + i)
⎝ 2 2 ⎠
b = 3 − 1 + i(1 − 3)
(i)[1]

__________________________________
2010 H2 Maths Preliminary Exam Paper 1
Page 4 of 8
(ii)[8] dy
y = xe − x ⇒ = x (−e − x ) + e − x = e − x (1 − x)
dx
[2]
dy
At P , x = a, y = ae − a , = e − a (1 − a )
dx
Equation of tangent to the curve at P is
y − ae − a = e − a (1 − a )( x − a )
⇒ y = e − a (1 − a )( x − a ) + ae − a
At Q , x = 0, y = h
⇒ h = e − a (1 − a )(0 − a ) + ae − a
= e − a (a − 1)(a ) + ae − a = a 2 e − a
[6] dh
⇒ = 2ae − a + a 2 (−e − a ) = ae − a (2 − a )
da
2− 2 2+
dh
For stationary values of h , =0 dh
da >0 0 <0
da
⇒ a = 0 (N.A. since a > 0 ) or a = 2
Tangent

Maximum value of h = 4e −2
Q11[10] Since the two planes intersect in a line l, with a vector equation given by
(i)[3] ⎛ 4 ⎞ ⎛ −2 ⎞
r = ⎜⎜ −2 ⎟⎟ + s ⎜⎜ 1 ⎟⎟ , s ∈ R,
⎜ −5 ⎟ ⎜ 3 ⎟
⎝ ⎠ ⎝ ⎠
The line l lies in p2.
⎛ −2 ⎞ ⎛ 3 ⎞
So ⎜⎜ 1 ⎟⎟ ⋅ ⎜⎜ λ ⎟⎟ = 0 ⇒ −6 + λ + 18 = 0 ⇒ λ = −12
⎜ 3 ⎟ ⎜6⎟
⎝ ⎠ ⎝ ⎠
⎛ 4⎞
⎜ ⎟
⎜ −2 ⎟ satisfies equation of p2.
⎜ −5 ⎟
⎝ ⎠
⎛ 4 ⎞ ⎛ 3⎞
So ⎜⎜ −2 ⎟⎟ ⋅ ⎜⎜ λ ⎟⎟ = μ ⇒ μ = 12 − 2λ − 30 = 12 + 24 − 30 = 6
⎜ −5 ⎟ ⎜ 6 ⎟
⎝ ⎠ ⎝ ⎠

(ii)[2] Since p1 and p2 intersect in line l, and the 3 planes have no common point of intersection, l does
not intersect p3.
Hence l is parallel to p3, but l does not lie in p3.
⎛ −2 ⎞ ⎛ 5 ⎞
⎜ ⎟ ⎜ ⎟ 2
⎜ 1 ⎟ ⋅ ⎜ 8 ⎟ = 0 ⇒ −10 + 8 + 3t = 0 ⇒ t = 3
⎜ 3 ⎟ ⎜t⎟
⎝ ⎠ ⎝ ⎠
⎛ ⎞
⎛ 4 ⎞ ⎜5⎟
⎜ ⎟ 2
Also ⎜⎜ −2 ⎟⎟ . ⎜ 8 ⎟ = ≠ 12 which means (4, −2, −5) is not in p3 and so l does not lie in p3.
⎜ −5 ⎟ ⎜ 2 ⎟ 3
⎝ ⎠⎜ ⎟
⎝3⎠

__________________________________
2010 H2 Maths Preliminary Exam Paper 1
Page 5 of 8
(iii)[5] ⎛ −2 ⎞
Since p4 contains l, ⎜⎜ 1 ⎟⎟ is parallel to p4.
⎜ 3⎟
⎝ ⎠
Since p4 contains the points (1, −1, 2) and (4, −2, −5).
⎛ 4⎞ ⎛1⎞ ⎛ 3⎞
Hence ⎜⎜ −2 ⎟⎟ − ⎜⎜ −1⎟⎟ = ⎜⎜ −1 ⎟⎟ is parallel to p4.
⎜ −5 ⎟ ⎜ 2 ⎟ ⎜ −7 ⎟
⎝ ⎠ ⎝ ⎠ ⎝ ⎠
⎛ 3 ⎞ ⎛ −2 ⎞ ⎛ 4 ⎞
A vector normal to p4 is ⎜⎜ −1 ⎟⎟ × ⎜⎜ 1 ⎟⎟ = ⎜⎜ 5 ⎟⎟ .
⎜ −7 ⎟ ⎜ 3 ⎟ ⎜ 1 ⎟
⎝ ⎠ ⎝ ⎠ ⎝ ⎠
⎛ 4⎞ ⎛ 1 ⎞ ⎛ 4⎞
Equation of p4 is given by : r⋅ ⎜⎜ 5 ⎟⎟ = ⎜⎜ −1⎟⎟ ⋅ ⎜⎜ 5 ⎟⎟ = 1 ⇒ 4 x + 5 y + z = 1
⎜1⎟ ⎜ 2 ⎟ ⎜1⎟
⎝ ⎠ ⎝ ⎠ ⎝ ⎠
Let θ be the angle between p1 and p4.
⎛ 2 ⎞ ⎛ 4⎞
⎜ ⎟ ⎜ ⎟
⎜ −5 ⎟ ⋅ ⎜ 5 ⎟
⎜ 3 ⎟ ⎜1⎟ −14
cos θ = ⎝ ⎠ ⎝ ⎠ = ⇒ θ = 110.514
2 + 5 + 3 4 + 5 +1
2 2 2 2 2 2
38 42

Acute angle between p1 and p4 = 180° − θ = 69.5° (1 d.p.)


Q12[11] By Ratio Theorem,
(a) [3] uur uuur
uuur 2OA + λ OB
OP =
λ+2
⎛ 12 ⎞ ⎛ 14 ⎞ ⎛ 11 ⎞
⎜ ⎟ 2 ⎜ ⎟ λ ⎜ ⎟
⇒ ⎜ −4 ⎟ = ⎜ 8 ⎟+ ⎜t ⎟
⎜ 10 ⎟ λ + 2 ⎜ 6 ⎟ λ + 2 ⎜ 12 ⎟
⎝ ⎠ ⎝ ⎠ ⎝ ⎠
28 11λ
So 12 = + ⇒ 12λ + 24 = 28 + 11λ ⇒ λ = 4
λ +2 λ + 2
16 λt 16 4t
Also, − 4 = + ⇒ −4 = + ⇒ t = −10
λ +2 λ + 2 6 6
(b) [8] ⎛ 11 ⎞ ⎛ 12 ⎞ ⎛ −1⎞
(i)[4] uur uuur uuur ⎜ ⎟ ⎜ ⎟ ⎜ ⎟
PB = OB − OP = ⎜ 2 ⎟ − ⎜ −4 ⎟ = ⎜ 6 ⎟
⎜12 ⎟ ⎜ 10 ⎟ ⎜ 2 ⎟
⎝ ⎠ ⎝ ⎠ ⎝ ⎠
⎛ 11 ⎞ ⎛ 14 ⎞ ⎛ −3 ⎞
uuur uuur uur ⎜ ⎟ ⎜ ⎟ ⎜ ⎟
AB = OB − OA = ⎜ 2 ⎟ − ⎜ 8 ⎟ = ⎜ −6 ⎟
⎜12 ⎟ ⎜ 6 ⎟ ⎜ 6 ⎟
⎝ ⎠ ⎝ ⎠ ⎝ ⎠
uur uuur
Length of projection of PB onto AB
⎛ −1⎞ ⎛ −3 ⎞
⎜ ⎟ ⎜ ⎟
⎜ 6 ⎟ • ⎜ −6 ⎟
⎜2⎟ ⎜ 6⎟ 3 − 36 + 12 21 7
⎝ ⎠ ⎝ ⎠
= = = =
32 + 62 + 62 81 9 3

Shortest distance of P from AB = BP 2 − ⎛⎜ ⎞⎟ = 41 −


7 49 320 8 5
= =
⎝3⎠ 9 3 3

__________________________________
2010 H2 Maths Preliminary Exam Paper 1
Page 6 of 8
(ii)[4] BAPQ forms a parallelogram.
uuur uuur
Hence AB = PQ
⎛ −3 ⎞ ⎛ 12 ⎞ ⎛ 9 ⎞
uuur uuur uuur ⎜ ⎟ ⎜ ⎟ ⎜ ⎟
⇒ OQ = AB + OP = ⎜ −6 ⎟ + ⎜ −4 ⎟ = ⎜ −10 ⎟
⎜ 6 ⎟ ⎜ 10 ⎟ ⎜ 16 ⎟
⎝ ⎠ ⎝ ⎠ ⎝ ⎠

Area of parallelogram BAPQ


uuur uuur
= AB × AP

⎛ −3 ⎞ ⎛ −2 ⎞
⎜ ⎟ ⎜ ⎟
= ⎜ −6 ⎟ × ⎜ −12 ⎟
⎜6⎟ ⎜ 4 ⎟
⎝ ⎠ ⎝ ⎠

⎛ 48 ⎞
⎜ ⎟
=⎜0⎟
⎜ 24 ⎟
⎝ ⎠

= 482 + 02 + 242
= 53.67 (to 2 dec places)

13[11] 4
f ( x) =
(i)[4] (1 − 3x) x 2 + 4
1

⎡ ⎛ x2 ⎞⎤ 2
= 4(1 − 3x) −1 ⎢ 4 ⎜1 + ⎟ ⎥
⎣ ⎝ 4 ⎠⎦
1

4 ⎛ x2 ⎞ 2
= (1 − 3x) −1 ⎜1 + ⎟
4 ⎝ 4⎠
⎡ 1 ⎛ x2 ⎞ ⎤
= 2 ⎡⎣1 + 3x + (3x) 2 + (3x)3 + ...⎤⎦ ⎢1 − ⎜ ⎟ + ...⎥
⎣ 2⎝ 4 ⎠ ⎦
⎡ ⎛ 1 ⎞ ⎛ 3 ⎞ ⎤
= 2 ⎢1 + 3x + ⎜ − + 9 ⎟ x 2 + ⎜ − + 27 ⎟ x3 + ...⎥
⎣ ⎝ 8 ⎠ ⎝ 8 ⎠ ⎦
71 213 3
= 2 + 6 x + x2 + x + ...
4 4

(ii)[2] For expansion in (i) to be valid,


x2
3 x < 1 and <1
4
1
⇒ x< and x 2 < 4
3
1
⇒ x< and x < 2
3
⎛ 1 1⎞
Range of values of x = ⎜ − , ⎟
⎝ 3 3⎠

__________________________________
2010 H2 Maths Preliminary Exam Paper 1
Page 7 of 8
(iii)[5] 2 tan y = f ( x) ----- (1)
Differentiate (1) with respect to x,
dy
2sec2 y = f '( x) ----- (2)
dx
Differentiate (2) with respect to x,
⎡ ⎛ dy ⎞
2
d2 y ⎤
2 ⎢ 2sec y tan y ⎜ ⎟ + sec 2 y 2 ⎥ = f "( x) ----- (3)
2

⎣⎢ ⎝ dx ⎠ dx ⎦⎥
When x = 0,
71
from (i), f (0) = 2, f '(0) = 6 and f ''(0) =
2
π
(1) gives 2 tan y = 2 ⇒ y =
4
dy dy 3
(2) gives 2(2) = 6 ⇒ =
dx dx 2
⎡ ⎛3⎞
2
d 2 y ⎤ 71 d 2 y 35
(3) gives 2 ⎢ 2(2)(1) ⎜ ⎟ + 2 2 ⎥ = ⇒ =
⎢⎣ ⎝2⎠ dx ⎥⎦ 2 dx 2 8
2
π 3 ⎛ 35 ⎞ x π 3 35
Hence y = + x + ⎜ ⎟ + ... = + x + x 2 + ...
4 2 ⎝ 8 ⎠ 2! 4 2 16

Q14 pn
[5] Given Sn = − 5.
5n −1
p n −1
⇒ S n−1 = n − 2 − 5.
5
Tn = S n − S n −1
∴ pn ⎛ p n −1 ⎞
= n −1
− 5 − ⎜ n−2 − 5 ⎟
5 ⎝5 ⎠
n−1
⎛ p⎞ ⎡ ⎛ 1 ⎞⎤
= ⎜⎜ ⎟⎟⎟ ⎢ p − ⎜⎜ −1 ⎟⎟⎟⎥
⎝⎜ 5 ⎠ ⎢⎣ ⎝⎜ 5 ⎠⎥⎦
n−1
⎛ p⎞
= ( p − 5)⎜⎜ ⎟⎟⎟
⎝⎜ 5 ⎠
n−2
⎛ p⎞
Now, Tn−1 = ( p − 5)⎜⎜ ⎟⎟⎟
⎜⎝ 5 ⎠
n−1
⎛ p⎞
( p − 5)⎜⎜⎜ ⎟⎟⎟
Tn ⎝5⎠ p
∴ = n−2
= (constant)
Tn−1 ⎛ p⎞ 5
( p − 5)⎜⎜⎜ ⎟⎟⎟
⎝5⎠

⇒ series is a geometric series. (shown)


p
For the sum to infinity to exist, <1
5
∴ −5 < p < 5 , p ≠ 0

__________________________________
2010 H2 Maths Preliminary Exam Paper 1
Page 8 of 8
RAFFLES INSTITUTION
H2 Mathematics 9740
2010 Year 6
____________________________
2010 H2 Maths Preliminary Exam Paper 2 Solutions

Q1 [8] 1 1
(i)[1] − 2 
2(n − 1) 2n
2

n 2 − n 2 + 2n − 1
=
2n 2 (n 2 − 1)
n − 12 1
= 2 ∴ a = − (shown)
n (n − 1) 2
2
(ii) [1] Smallest value of M = 2
(iii)[3] N
2n − 1
SN = ∑ 2
n =3 2n ( n − 1)
2

N
n − 12
=∑
n =3 n ( n − 1)
2 2

N
1 1
= ∑( − 2)
n =3 2( n − 1)
2
2n
1 1
= −
2(2) 2(3)2
2

1 1
+ −
2(3) 2(4) 2
2

M
1 1
+ −
2( N − 2) 2( N − 1) 2
2

1 1
+ −
2( N − 1) 2 N 2
2

1 1
= −
8 2N 2
1 1 1
+ + + ...
(2)(3) (3)(4) (4)(5) 2
2 2


1
=∑ 2
n = 3 n ( n − 1)

n −1
=∑
n = 3 n ( n − 1)
2 2


n − 12
<∑
n = 3 n ( n − 1)
2 2

N
n − 12
= lim
N →∞
∑ n (n − 1)
n =3
2 2

⎡1 1 ⎤
= lim ⎢ −

N →∞ 8 2 N 2 ⎥⎦

__________________________________
2010 H2 Maths Preliminary Exam Paper 2
Page 1 of 8
1
= (shown)
8
Q2 [8]
[4]

(i)[2] Minimum value will be the perpendicular distance from origin to the line segment CB. Let the
point of intersection of this perpendicular and CB be D. By observing the right angle triangle
π |a|
formed by O, C, and D, OD = | a | sin = .
4 2
(ii)[2] It is clear that the locus of z satisfying both relations is the line segment CB, not including C. The
π
argument of the point represented by B is arg(a) − , while the argument of the point represented
2
by C is arg(a ) − π .
Hence the range of values is
π
arg(a ) − π < arg( z ) ≤ arg(a) − .
2

Q3 [12] Since f (2.5) = f (3.5) = −0.75 , f is not one-one.


(a)[7] OR
y = f ( x)

O 2
y = −0.75
°4
(3, −1)
D f = (−∞, 4) R f = [ −1, ∞ )
Since the horizontal line y = −0.75 does not cut the graph of f at one and only one point, f is not
one-one.

y = g( x)
y =1

O
x = −3
D g = R \ {−3} R g = R \ {1}

fg does not exist because R g = R \ {1} ⊄ ( −∞, 4) = D f .


Since R f = [ −1, ∞ ) ⊆ R \ {−3} = D g , gf exists.
( x − 2)( x − 4) x2 − 6x + 8 3
gf ( x ) = = 2 = 1− D gf = D f = ( −∞, 4) .
( x − 2)( x − 4) + 3 x − 6 x + 11 ( x − 3) 2 + 2

__________________________________
2010 H2 Maths Preliminary Exam Paper 2
Page 2 of 8
R gf = R g whose domain is restricted to R f
= R g whose domain is restricted to [−1, ∞)
= [ − 12 ,1)

OR sketch y = gf(x)
y = gf ( x)
y =1

O 2 °4
(3, − ) 1
2
D gf = (−∞, 4) R gf = [− 12 ,1)

(b)[5] Let y = a (1 + e − x )
y
− 1 = e− x
a
x = − ln ( ay − 1)
h −1 (x) = − ln ( ax − 1)
D h −1 = R h = ( a , 2 a ]

y = h −1 (x)
A(0, 2a)
B(b, b) y = h(x)
y=a

y=x O C (2a,0) +
x=a Dh = 0 R h = ( a, 2a ]

Since y = h −1 ( x) can be obtained by reflecting y = h( x ) about the line y = x, therefore at the point
of intersection of y = h( x ) and y = h −1 ( x) , y = x = b.
b b
∫ 0
a (1 + e − x ) d x = ∫ a (1 + e − y ) d y = I .
0

∴ Area bounded by y = h( x), y = h −1 ( x) and the axes


b b
= ∫ a (1 + e − x ) dx + ∫ a (1 + e − y ) d y − (b × b)
0 0

= 2I − b2

OR
b
b b ⎡ x2 ⎤ b2
Area OAB = ∫ 0
h( x ) d x − ∫
0
x dx = I − ⎢ ⎥ = I −
⎣ 2 ⎦0 2
b2
Since y = h( x ) and y = h −1 ( x) are symmetrical about y = x , Area OAB = Area OBC = I − .
2
∴ Area bounded by y = h( x), y = h −1 ( x) and the axes
⎛ b2 ⎞
= 2 ⎜ I − ⎟ = 2I − b2
⎝ 2⎠

__________________________________
2010 H2 Maths Preliminary Exam Paper 2
Page 3 of 8
Q4 [12]

∫ ∫
1 1
−4 x
f (| x |)dx = 2 dx
(i)[2] −1 0 x2 + 1


1
2x
= 2(−2) dx
0 x +12

1
= −4 ⎡⎣ln( x 2 + 1) ⎤⎦
0
= −4 ln 2

(ii)[7]

∫ ln(3 x + e 2 + 3)dx


1
= ln(u ) du
3


1
= ln(u )du
3


1 1
= [u ln u ] − 1du
3 3
1
= [u ln u − u ] + c
3
1
= ⎡⎣ (3 x + e2 + 3) ln(3 x + e 2 + 3) − (3 x + e 2 + 3) ⎤⎦ + c
3
4 y

(−1, 2) y = ln(3 x + e 2 + 3)
2

R x

−4 x
–6 –4 –2 2 4 6

− ( e + 2)
2
y=
3 –1 x +1
2

–2

–3

–4

−1

∫ ∫
0
−4 x
ln(3 x + e 2 + 3)dx + dx
− ( e2 + 2)
−1 x2 + 1
3
1 −1
= ⎡⎣(3 x + e 2 + 3) ln(3 x + e 2 + 3) − (3 x + e 2 + 3) ⎤⎦ ( e2 + 2) + 2 ln 2
3 −
3
1
= [e2 ln e2 − e2 + 1] + 2 ln 2
3
1
= (e 2 + 1) + 2 ln 2
3
(iii)[3] −1

∫ ∫
0 2
⎛ −4 x ⎞
( )
2
π ln(3 x + e2 + 3) dx + π ⎜ 2 ⎟ dx
−1 ⎝ x + 1 ⎠
− ( e2 + 2)
3
= 20.55 (2d.p.)

__________________________________
2010 H2 Maths Preliminary Exam Paper 2
Page 4 of 8
Q5[8] W,Y,O,G – 1 letter each
(a)[3] E – 2 letters
Case 1: All 3 letters are different, i.e. W, Y, O, G, H, E.
Number of ways = 6C3 × 3! = 120 .
Case 2: 2 letters are identical, i.e. E, E and one of the remaining 5 letters, W, Y, O, G, H.
3!
Number of ways = 5C1 × = 15 .
2!
Total number of ways = 120 + 15 = 135
(b) [5] Ways to select the players
(i) [3] Badminton Tennis Football No. of ways
(6) (6) (5)
1 2 3 6
C1 × 6C2 × 5C3 =900
1 3 2 6
C1 × 6C3 × 5C2 =1200
2 1 3 C2 × 6C1 × 5C3 =900
6

2 3 1 C2 × 6C3 × 5C1 =1500


6

2 2 2 C2 × 6C2 × 5C2 =2250


6

3 1 2 C3 × 6C1 × 5C2 =1200


6

3 2 1 C3 × 6C2 × 5C1 =1500


6

Total number of ways = 9450.


(iii) [2] Arrange the 6 youths in 6! ways and slot the 4 empty chairs into the row in 7C4 ways.
Total number of ways = 6! × 7C4 = 25200 .
Q6 [7] Probability of getting ‘6’
[2] = 1 − (p + 2 p + 3 p + 4 p + 5 p )
= 1 − 15 p

(i)[3]
Second die
+ 1 2 3 4 5 6

1 2 3 4 5 6 7

2 3 4 5 6 7 8

3 4 5 6 7 8 9
First die

4 5 6 7 8 9 10

5 6 7 8 9 10 11

6 7 8 9 10 11 12

P(A) = P(sum = 10)+P(sum = 11)+P(sum = 12)


= 2(4p)(1 − 15 p)+(5 p )(5p )+2(5p)(1 − 15 p )+(1 − 15 p )2
= 1 − 12 p − 20 p 2

__________________________________
2010 H2 Maths Preliminary Exam Paper 2
Page 5 of 8
(ii)[2] P(A B ) = P(2nd die is 5 1st die is 5)+P(2nd die is 6 1st die is 5)
= 5 p + 1 − 15 p  
= 1 − 10 p

Alt :  

P(A B )
P(A ∩ B)
=
P(B )
P(1st die is 5, 2nd die is 5)+P(1st die is 5, 2nd die is 6)
=  
P(1st die is 5)
(5 p )(5 p )+(5 p )(1 − 15 p )
=
5p
= 1 − 10 p

For A and B to be independent, P(A) = P(A|B)

1 − 12 p − 20 p 2 = 1 − 10 p
20 p 2 + 2 p = 0  
1
∴ p = 0 or −
10

Since p > 0, there are no values of such that A and B are independent.

Q7[9]
(i)[2]

The product moment correlation coefficient between x and y,


r is -0.943(3 s.f.).
(ii)[2] Since r is close to 1, it suggests a linear model is appropriate. However the scatter diagram
shows the relationship is non-linear.
(iii)[1] The scatter diagram shows that as x increases, the rate of decrease in y becomes smaller and y appears to
approach a value. For a linear model, the rate of decrease is constant.
OR
The product moment correlation coefficient between 1/x and y, r2 is 0.993 where r2 is almost 1 as

compared to r =0.943.

b
Therefore the model y = a + where a > 0 ,b > 0 is better.
x
(iv) [2] a = 2.16 (3s.f.), b = 2.33 (3s.f.)

__________________________________
2010 H2 Maths Preliminary Exam Paper 2
Page 6 of 8
(v) [2] When x = 4.5, y = 2.68 (3 s.f..)
Since x = 4.5 is within the data range of the x values and value of r2 is close to 1, the estimate is
reliable.

8 [10] 8400
Unbiased estimate of the mean = = 56
150
(i)[2]
5555
Unbiased estimate of the variance = .
149
(ii)[5] (i) H0 : μ = 55 vs H1 : μ > 55

Perform a 1-tail test at 3% level of significance.

⎛ s2 ⎞
Under H0 , X ~ N ⎜ μ0 , ⎟ approximately, by Central Limit Theorem, where
⎝ n⎠
5555
μ0 = 55, s = , x = 56, n = 150
149

Using a z-test, p-value = 0.0224 (3 s.f.)

Since the p -value = 0.0224 < 0.03, we reject H0 and conclude that there is sufficient evidence, at
3% significance level, that the school is understating the mean mass of male students in the school.

Central Limit Theorem has to be applied here as X is not stated to be normally distributed and the
sample size of 150 is large.
(iii)[1] It means that the probability of concluding that the mean mass is more than 55 kg when it actually
is 55 kg is 0.03.
[2] Suppose that there are 4 different levels of female students in the school. Select 25 from each level
and carry out the data collection by interviewing the first 25 female students of each level who
enter the school gate in the morning.

Q9 [11] P ( X > t ) = 0.9


(i)[2] P ( X < t ) = 0.1
t = 26.2 (in minutes)

(ii)[2] P ( X > 31 and Y < 20 ) = P ( X > 31) × P (Y < 20 )


= 0.36944 × 0.97725
= 0.361 (3sf)

(iii)[3]
(
P ( X > Y + 11) = P ( X − Y > 11) where X − Y ~ N 14, 13
2
)
= 0.797 (3sf)
(iv) Part (iii) includes the case in (ii), as well as other cases in which Abel reaches the station before
the train, e.g. train arrives after 12.01am and Abel arrives before 12.01am.

(v) If W ~ N ( 4, 6 ) , then 99.7% of the values would lie within 4 ± 3 6 , which contains a significant
range of negative values.
__________________________________
2010 H2 Maths Preliminary Exam Paper 2
Page 7 of 8
(vi) ⎛ 6 ⎞
2
Since sample size 40 is large, by Central Limit Theorem, W ~ N ⎜ 4, ⎟ approximately.
⎜ 40 ⎟
⎝ ⎠
( )
P W < 3.5 = 0.0984 (3sf)

Q10[13] The average number of incoming calls received per hour is constant throughout the opening hours
(i)[1] of the mall.
OR
The probability of 2 or more incoming calls received in a very short interval of time is negligible.
(ii)[2] Let X be the number of incoming calls received in an hour. X ~ Po ( 6.75 ) .
P ( X ≥ 8) = 1 − P ( X ≤ 7 )
= 0.364 (3sf)
(iii)[3] Required probability is
4!
P ( X ≤ 6 ) × P ( X = 7 ) × ⎡⎣ P ( X ≥ 8 ) ⎤⎦ ×
2

2!
4!
≈ 0.48759 × 0.14832 × ( 0.36409 ) ×
2
= 0.115 (3sf)
2!
(iv)[4] Y ~ Po ( 81) . So μ = 81 , σ = 9 .

( )
Since μ = 81 > 10 , Y ~ N 81, 92 approximately.
P ( 81 − 9 < Y < 81 + 9 )
= P ( 72 < Y < 90 )
= P ( 72.5 < Y < 89.5 ) by continuity correction
= 0.6551 (4dp)
(v)[3] Let W be the number of busy days in 14 days.
W ~ B (14, P (Y > 90 ) ) ,that is, W ~ B (14, 0.14593) .
Required probability is
P (W = 2 ) × P (Y > 90 ) ≈ 0.29191× 0.14593
= 0.0426 (3sf)

__________________________________
2010 H2 Maths Preliminary Exam Paper 2
Page 8 of 8
Name ( ) Class

RIVER VALLEY HIGH SCHOOL


2010 Year 6 Preliminary Examinations
Higher 2

MATHEMATICS 9740/01
Paper 1 14 September 2010
3 hours
Additional Materials: Answer Paper
List of Formulae (MF15)
Cover Page

READ THESE INSTRUCTIONS FIRST

Do not open this booklet until you are told to do so.


Write your name, class and index number in the space at the top of this page.

Write your name and class on all the work you hand in.
Write in dark blue or black pen on both sides of the paper.
You may use a soft pencil for any diagrams or graphs.
Do not use staples, paper clips, highlighters, glue or correction fluid.

Answer all the questions.


Give non-exact numerical answers correct to 3 significant figures, or 1 decimal place in the case
of angles in degrees, unless a different level of accuracy is specified in the question.
You are expected to use a graphic calculator.
Where unsupported answers from a graphic calculator are not allowed in a question, you are
required to present the mathematical steps using mathematical notations and not calculator
commands.
You are reminded of the need for clear presentation in your answers.

At the end of the examination, place the cover page on top of your answer paper and fasten all
your work securely together.

The number of marks is given in brackets [ ] at the end of each question or part question.

This document consists of 6 printed pages.


2

1 A shampoo manufacturing company produces shampoo that comes in three different sizes
of bottles (small, medium and large). The amount of plastic required to manufacture the
bottles, and the profit made from the sale of each bottle of shampoo are shown in the
table below:

Small Medium Large


Volume of shampoo (cm3) 200 450 600
Amount of plastic (cm2) 150 335 475
Profit ($) per unit sold 2.50 3.80 4.20

On a particular round of production, the volume of shampoo and the amount of plastic
used were 370700cm3 and 280400cm2 respectively. When all the bottles of shampoo
from this round of production were sold, the profit made from the sale of the medium
bottles was twice the total profit made from the sale of the small and large bottles.
Determine the number of each size of shampoo produced and state an assumption made
about the bottles manufactured. [6]

2 The complex number z satisfies the relations iz + 1 ≤ i and z − z* ≤ 2i .

(i) Illustrate both of these relations on a single Argand diagram. [3]

(ii) Find the greatest and least possible values of arg ( z + 1 + i) , giving your answers in
radians correct to 3 decimal places. [3]

3 Given that θ is sufficiently small for θ 3 and higher powers of θ to be neglected, express
4 − cos θ
as a quadratic expression in θ . [4]
2 + tan θ

π π 55π 2
9 11
By letting θ = , show that 3≈ + − . [2]
6 7 56 2016

RIVER VALLEY HIGH SCHOOL 9740/01/2010


3

4 (i) Find
d
dx
( sin −1 x 2 ) . [1]

  x 

5
(ii) Evaluate the integral ln  tan −1    dx numerically. [1]
1   10  

(iii) Given that 0 < k ≤ 1 and

  x 
∫ ∫
k 5
x 2 sin −1 x 2 dx = ln  tan −1    dx ,
0 1   10  


k
x4
show that = dx ak 3 sin −1 k 2 + b , where a and b are constants to be
0 1− x 4

determined. [4]

n+2
5 The sequence of numbers u1 , u2 , u3 , is given by u1 = 2 and un +1 = for all positive
nun
integers.

(i) By writing down the terms u2 and u3 , make a conjecture for un in terms of n . [2]

(ii) Prove your conjecture by mathematical induction. [4]

(iii) Write down the limit of unun −1 as n tends to infinity. [1]

6 (i) Verify that r 4 + r 2 + 1= (r 2 + r + 1)(r 2 − r + 1). [1]

1 1
(ii) By considering − 2 , show that
r + r +1
2
r − r +1

∑r
N
r 1 1 
= 1 − 2  . [4]
r =1
4
+ r2 +1 2  N + N +1


r
(iii) Evaluate ∑
r =2 r + r2 +1
4
. [2]

RIVER VALLEY HIGH SCHOOL 9740/01/2010


4

ax 2 + bx + c
7 The diagram below shows a sketch of the graph of y = where a , b , c and
x+d
d are constants. The asymptotes, also shown in the diagram, are x = −2 and y= x − 4 .

4
−2 O x
−4

(i) Write down the value of d and find the values of a and b . [4]

(ii) Given that the curve passes through the point ( 0 , −4.5 ) , find the value of c . [1]

By sketching the graph of ( x − 4 ) + y 2 =,


2
(iii) k 2 determine the range of values of k
2
 ax 2 + bx + c 
such that the equation ( x − 4 ) +   =
2
k 2 , where a , b , c and d are
 x+d 
the values found above, has at least one negative root. [3]

8 (i) Solve the equation

( z + 2)3 =
−8 ,

giving the roots in the form reiθ − 2 , where r > 0 and − π < θ ≤ π . [4]

(ii) Express ( z + 2)3 + 8 as a product of three linear factors. [2]

(iii) Hence, find the values of the real numbers a and b in the equation z 2 + az + b = 0
where the roots are the complex numbers found in (i). [2]

(iv) Explain why the roots in (i) lie on a circle centre (−2,0) and radius 2. [1]

RIVER VALLEY HIGH SCHOOL 9740/01/2010


5

3π 3π
9 It is given that f (=
x ) ln (1 + sin x ) , where − ≤x≤ .
8 8

(i) Show that (1 + sin x)f ′ ( x ) =


cos x . [1]

(ii) By further differentiation of this result, find the Maclaurin series for f ( x ) , up to
and including the term in x3 . Hence, write down the equation of the tangent of
y = f ( x ) at x = 0 . [5]

Denote the Maclaurin series of f ( x ) in (ii) by g ( x ) .

(iii) On the same diagram, sketch the graphs of y = f ( x ) and y = g ( x ) for


3π 3π
− ≤x≤ . [2]
8 8

3π 3π
(iv) Find, for − ≤x≤ , the set of values of x for which the value of g ( x ) is
8 8
within ±0.05 of the value of f ( x ) . [2]

10 (a) Given that the first, third and fourth terms of an arithmetic progression are three
consecutive terms of a geometric progression, and that the sum of the first twenty
even-numbered terms of the arithmetic progression is 960, find the common
difference of the arithmetic progression. [4]

(b) Annie puts $x on 1 January 2010 into a bank account which pays compound
interest at a rate of 3% per month on the last day of each month. On the first day
of each subsequent month, she puts in an amount which is $x more than the
amount she puts in the previous month. For example, she puts in $x on 1 January,
$2x on 1 February and so on.

(i) Show that the amount of money in the bank account on the last day of
103 x
March is $ 103(1.03)3 − 112  . [4]
9

(ii) Find the least integer value of x so that the amount of interest earned for
the first three months of the year 2010 exceeds $100. [2]

RIVER VALLEY HIGH SCHOOL 9740/01/2010


6

11 The lines l1 and l2 meet at P. The line l3 lies on the same plane containing l1 and l2 , and
is perpendicular to l1 . Given that l1 and l2 are parallel to the vectors a and b respectively,
 a ⋅b 
show that l3 is parallel to the vector b −  2  a . [3]
 a 
 

The lines l1 and l2 have equations r = 3i − 5 j + 2k + t ( 4i − j + 2k ) and


r = 3i − 5 j + 2k + s ( 8i − 4 j + 3k ) respectively. The line l3 lies on the plane containing l1
and l2 and is perpendicular to l1 . Given that the three lines intersect, find the equation of
l3 . Find also the angle between l2 and l3 . [3]

The plane Π1 contains the lines l1 , l2 and l3 , and the plane Π 2 has equation
5
9 . Given that Π1 is parallel to Π 2 , show that k = . Hence, find the exact
kx + 2 y − 4 z =
2
distance between the two planes. [6]

2
12 (i) Sketch the graph of y = 3
. [2]
(4 − x )2 2

(ii) The region R is bounded by the curve, the axes and the line x = 3 . Using the
substitution x = 2 cos θ , find the exact area of R. [5]

(iii) Find the exact volume of revolution when R is rotated completely about the
y-axis. [6]

− End of Paper −

RIVER VALLEY HIGH SCHOOL 9740/01/2010


1

Name ( ) Class

Tutor Calculator Model

RIVER VALLEY HIGH SCHOOL


2010 Year 6 Preliminary Examinations
Higher 2

MATHEMATICS 9740/01
Paper 1 14 September 2010

Cover Page

INSTRUCTIONS TO CANDIDATES

Attach this cover page on top of your answer paper.


Circle the questions you have attempted and arrange your answers in NUMERICAL ORDER.

Question Mark Max. Mark Question Mark Max. Mark

1 6 7 8

2 6 8 9

3 6 9 10

4 6 10 10

5 7 11 12

6 7 12 13

Total 100 Grade

RIVER VALLEY HIGH SCHOOL 9740/01/2010


Name ( ) Class

RIVER VALLEY HIGH SCHOOL


2010 Year 6 Preliminary Examinations
Higher 2

MATHEMATICS 9740/02
Paper 2 15 September 2010
3 hours
Additional Materials: Answer Paper
List of Formulae (MF15)
Cover Page

READ THESE INSTRUCTIONS FIRST

Do not open this booklet until you are told to do so.


Write your name, class and index number in the space at the top of this page.

Write your name and class on all the work you hand in.
Write in dark blue or black pen on both sides of the paper.
You may use a soft pencil for any diagrams or graphs.
Do not use staples, paper clips, highlighters, glue or correction fluid.

Answer all the questions.


Give non-exact numerical answers correct to 3 significant figures, or 1 decimal place in the case
of angles in degrees, unless a different level of accuracy is specified in the question.
You are expected to use a graphic calculator.
Where unsupported answers from a graphic calculator are not allowed in a question, you are
required to present the mathematical steps using mathematical notations and not calculator
commands.
You are reminded of the need for clear presentation in your answers.

At the end of the examination, place the cover page on top of your answer paper and fasten all
your work securely together.

The number of marks is given in brackets [ ] at the end of each question or part question.

This document consists of 6 printed pages.


2

Section A: Pure Mathematics [40 marks]

1 The diagram shows the graph of y = f(x). The curve passes through the origin O, the
points A(–3a, 2) and B(–a, –2), where 0 < a < 1 .
y

x=a
A(−3a, 2)

x
0

B (−a, −2)

Sketch on separate clearly labelled diagrams, the graphs of

1 
(a) y=
−f  x + 1  ; [4]
2 

(b) y = f '( x) . [2]

 
2 Points O, A and B are such that OA =i + 2 j − 3k and OB= 4i − j .

(i) Use vector product to find the exact area of the triangle OAB. [2]

(ii) A point P on line AB is such that AP : AB = 2 : 3. Given that point Q lies on OP



= c(3i − k ) , for some constant c.
produced, show that OQ [2]

(iii) Given also that triangle AQB has a right angle at Q, find the value of c. [3]

RIVER VALLEY HIGH SCHOOL 9740/02/2010


3

3 The curve C has parametric equations

x= 2t + t 2 , y= (1 − t ) 2 ,

where t ∈  .

(a) Find the range of values of t for which C is decreasing. [3]

(b) Find the equation of the normal to the curve at t = 2 . Determine if the normal
meets C again. [4]

4 A long cylindrical metal bar is submerged into iced water. A researcher claims that the
rate at which the length, l cm, of the bar is shrinking at any time t seconds is proportional
to the volume of the bar at that instant, assuming that the cross-sectional area of the bar
remains constant during the shrinking process.

Formulate and integrate a differential equation to show that l = Ae kt , where A and k are
constants. State the range of values of A and of k. [5]

Given that the initial length of the bar is L, sketch a graph to show the relation between l
and t. Comment on the suitability of the claim by the researcher. [3]

It is later found that l and t are related by the equation l= B + Le kt , where B is some
constant. Given that l = 0.5L when t = T, show that the length of the bar when t = 3T, is
( 0.5L − B )
3

given by B+ . [2]
L2

5 The functions f and g are defined by

f : x  2 x 3 + 1, x > 0,
g : x  ln( x − a ), x>a.

(i) Give a reason why f -1 exists. Hence find f -1 ( x) and state the domain of f -1 . [3]

(ii) Only one of the composite functions fg and gf exists. State the greatest possible
value of a such that this composite function exists and using this value of a, give a
definition of this composite function. Write down the range of the composite
function. Explain why the other composite function does not exist. [5]

(iii) By sketching suitable graphs or otherwise, find the set of values of x such that
f -1f ( x) = ff -1 ( x) . [2]

RIVER VALLEY HIGH SCHOOL 9740/02/2010


4

Section B: Statistics [60 marks]

6 Research has shown that 5 out of 8 students of a particular school make use of the
school’s e-learning portal. Three random samples of 40 students are chosen. Using a
suitable approximation, find the probability that more than 80 students make use of the
school’s e-learning portal. [3]

7 The sales of movie tickets sales are known to be high during the weekend, low during the
beginning of the week (i.e. Monday and Tuesday) and somewhat moderate at midweek. A
sample of 50 is to be chosen from 350 days of a year of movie ticket sales to estimate the
ticket sales per day.

Describe how the sample could be chosen using systematic random sampling. [2]

Describe any disadvantage of using systematic sampling and explain briefly whether it
would be more appropriate to use stratified sampling instead. [2]

8 A group of 10 students consisting of 6 males and 4 females are seated at a round table
with chairs of different colours for lunch. Find the number of arrangements such that all
the female students are seated together. [3]

After lunch, the same group of 10 students bought tickets to attend a concert. If the tickets
are for a particular row of 10 adjacent seats, find the number of possible seating
arrangement when

(i) the first and last seats are occupied by the same gender, [3]

(ii) four particular students did not turn up for the concert. [1]

9 The average daily sales at a small food store is estimated to be $452.80. The daily sales,
$x, over a period of 12 days is summarized by

∑ ( x − 450) =
622.8 and ∑ ( x − 450)2 =
78798 .

(i) Test, at 1% significance level, whether the food store has underestimated its
average daily sales. State an assumption you made about the data. [6]

(ii) Suppose in a test at 5% significance level, it is found that there is significant


evidence of an increase in average daily sales. Using only this information, and
giving a reason, discuss whether there is significant evidence at 5% significance
level that the average daily sales has changed. [2]

RIVER VALLEY HIGH SCHOOL 9740/02/2010


5

10 (a) A student threw a fair die four times and recorded the number obtained for each
throw. Find the probability that

(i) at least two of the four numbers obtained are the same, [2]

(ii) exactly two of the four numbers obtained are the same given that the
number 6 appeared at least once. [4]

1 7 4
(b) Events A and B are such that P( B′) = , P( A′ ∩ B′) = and P( B A) = . Find
2 20 7
the probability P( A) . [3]

11 An experiment was conducted to verify how the radiation intensity x from a radioactive
source varies with time t. The following data were obtained from a particular source.

x 1 4 6 8 9 10 12 15
t 15.3 6.9 4.4 2.7 2.1 1.7 1.0 0.49

(i) Plot a scatter diagram for the above data. [1]

(ii) Calculate the product moment correlation coefficient for the above data and
comment on it, in relation to your scatter diagram in (i). [2]

(iii) Calculate the equation of the least square regression line of x on t. Give a practical
interpretation of the coefficient of t. [2]

(iv) The variable y is defined by y = ln x. For the variables y and t, calculate the
product moment correlation coefficient and hence compare the suitability of this
logarithmic model and the linear model in (iii). [2]

(v) Use an appropriate regression line to give the best estimate that you can of the
time when x = 10. [2]

RIVER VALLEY HIGH SCHOOL 9740/02/2010


6

12 The two most common types of disciplinary offences in a particular boy school is keeping
long hair and failure to wear the school badge. The mean number of disciplinary offences
recorded per day involving long hair is 1.12. Assuming that each school week consists of
five school days, the mean number of disciplinary offences recorded per school week
involving failure to wear the school badge is 4.2. The number of cases for each
disciplinary offence is assumed to have an independent Poisson distribution.

(i) Find the probability that at most 9 cases of disciplinary offence are recorded in a
given school week. [3]

(ii) In a school week in which there are more than 7 cases of disciplinary offence
involving long hair, find the probability that at most 9 cases of disciplinary
offence are recorded. [3]

(iii) Calculate the probability that on a Thursday in a particular school week, it is the
third day in the school week in which the discipline master caught at least 4
students having long hair in a day. (You may assume that Monday is the first day
of school for a school week.) [3]

(iv) Explain why the Poisson distribution may not be a good model for the number of
disciplinary cases involving long hair, in a school year. [1]

13 A farm in Kranji rears chickens and ducks for sale to the local market.

(a) The mass of a randomly chosen chicken has mean 2.5 kg and standard deviation
0.4 kg. If the probability that the mean mass of a large sample of n chickens is
greater than 2.45 kg exceeds 0.95, find the least value of n. [3]

(b) The variable X denotes the mass, in kg, of a randomly chosen duck which is
normally distributed with mean 2.7 and standard deviation σ . The ducks are sold
at $4 per kg.

50
(i) Given that P( X < 2.5)
= P( X < 2.9) , show that the value of σ is
107
0.424, correct to 3 significant figures. [2]

(ii) A housewife brings $50 to the farm. Calculate the probability that she has
less than $27.50 after buying 2 ducks. [3]

(iii) A sample of 4 ducks is randomly chosen. Calculate the probability that the
most expensive duck costs less than $11. [2]

− End of Paper −

RIVER VALLEY HIGH SCHOOL 9740/02/2010


1

Name ( ) Class

Tutor Calculator Model

RIVER VALLEY HIGH SCHOOL


2010 Year 6 Preliminary Examinations
Higher 2

MATHEMATICS 9740/02
Paper 2 15 September 2010

Cover Page

INSTRUCTIONS TO CANDIDATES

Attach this cover page on top of your answer paper.


Circle the questions you have attempted and arrange your answers in NUMERICAL ORDER.

Question Mark Max. Mark Question Mark Max. Mark

1 6 6 3

2 7 7 4

3 7 8 7

4 10 9 8

5 10 10 9

11 9

12 10

13 10

Total 100 Grade

RIVER VALLEY HIGH SCHOOL 9740/02/2010


RVHS 2010 Preliminary Exam: 9740 H2 Mathematics Paper 1 Solutions

Question 1 [6 marks]
Let the no. of small, medium and large bottles manufactured be denoted by s, m and l
respectively.
So, 150 s + 335m + 475l = 280400 ,
200 s + 450m + 600l = 370700 and
3.8m = 2(2.5s + 4.2l )

Using GC, solving the augmented matrix:


⎛ 150 335 475 280400 ⎞
⎜ ⎟
⎜ 200 450 600 370700 ⎟
⎜ 5 −3.8 8.4 0 ⎟⎠

Ans: s = 166, m = 550 and l = 150

Assumption:
The plastic bottles are of negligible thickness. OR
The plastic bottles are of the same thickness.

Question 2 [6 marks]
(i) iz + 1 ≤ i
1
z+ ≤1
i
z −i ≤1

z − z* ≤ 2
(ii)
x + iy − ( x − iy ) ≤ 2
2iy ≤ 2
−1 ≤ y ≤ 1
Im(z)

1
1

Re(z)
0

RVHS 2010 H2 Math Prelim P1 1


arg( z + 1 + i) = arg( z − (−1 − i))
Im(z)

1
1

Re(z)
0
α
−1 − i

π
Max. arg( z + 1 + i) = .
2
Min. arg( z + 1 + i) =α.

1
α = β − θ = tan −1 2 − sin −1 = 0.644 (3 dec. pl.)
5

Question 3 [6 marks]
⎛ θ2 ⎞
4 − ⎜1 − ⎟
4 − cos θ ⎝ 2 ⎠
=
2 + tan θ 2 +θ
θ2
3+
= 2
2 +θ
−1
⎛ θ 2 ⎞ ⎡ ⎛ θ ⎞⎤
= ⎜ 3 + ⎟ ⋅ ⎢ 2 ⎜1 + ⎟ ⎥
⎝ 2 ⎠ ⎣ ⎝ 2 ⎠⎦
⎛ θ2 ⎞ 1⎛ θ θ2 ⎞
= ⎜ 3 + ⎟ ⋅ ⎜1 − + + ... ⎟
⎝ 2 ⎠ 2⎝ 2 4 ⎠
3 3 5
≈ − θ + θ2
2 4 8

3
4− 2
π 2 ≈ 3 − π + 5π
Let θ = :
6 2+ 1 2 8 288
3
(8 − 3) 3 3 π 5π 2
≈ − +
2(2 3 + 1) 2 8 288

RVHS 2010 H2 Math Prelim P1 2


(8 3 − 3)(2 3 − 1) 3 π 5π 2
≈ − +
2(4(3) − 12 ) 2 8 288
51 − 14 3 3 π 5π 2
≈ − +
22 2 8 288
9 11π 55π 2
∴ 3≈ + − (shown)
7 56 2016

Question 4 [6 marks]
(i) d 2x
dx
( sin −1 x 2 ) =
1 − x4
(ii) 5 ⎛ −1 ⎛ x ⎞ ⎞
∫ 1 ln ⎜⎝ tan ⎜⎝ 10 ⎟⎠ ⎟⎠ dx = −5.292774105 = −5.29 (3 s.f.)
(iii) k dv
∫0 x sin x dx
−1 2
2
(letting u = sin −1 x 2 and = x2 )
dx
k
⎡ x3 ⎤ k 2 x4
= ⎢ sin −1 x 2 ⎥ − ∫ ⋅ dx
⎣3 ⎦0 0 3 1 − x4
k3 k 2 x4
= sin −1 k 2 − ∫ ⋅ dx
3 0 3
1 − x4
k3 k 2 x4
∴ sin −1 k 2 − ∫ ⋅ dx = − 5.292774105
3 0 3
1 − x4
Hence
k x4 3 ⎡ k3 ⎤
∫0 1 − x4
−1 2
d x = ⎢ sin k + 5.292774105⎥
2⎣ 3 ⎦
1
= k 3 sin −1 k 2 + 7.94 ,
2
where a = 0.5 and b = 7.94

Question 5 [7 marks]
(i) 3 4
u2 = , u3 =
2 3
n +1
Conjecture: un =
n
(ii) n +1
Let P(n) be the statement “ un = ”, for all positive integers n.
n
When n = 1: LHS = u1 = 2 = RHS (given)

∴ P(1) is true.

RVHS 2010 H2 Math Prelim P1 3


k +1
Assume P(k) is true for some integers k ∈ Z + , i.e uk =
k
k+2
To show: P(k+1) is true, i.e. uk +1 = .
k +1
Show: LHS = uk +1

k +2
=
kuk
k +2 k
= .
k k +1
k +2
= = RHS
k +1
Since P(1) is true and P(k) is true ⇒ P(k+1) is true,
∴ By Mathematical Induction, P(n) is true for all positive integers n.
(iii) n +1
un =
(n − 1)un −1
n +1 2
⇒ unun −1 = = 1+ → 1 as n → ∞
(n − 1) n −1

Question 6 [7 marks]
(i) RHS = (r 2 + r + 1)( r 2 − r + 1)
= r4 − r3 + r 2 + r3 − r 2 + r + r 2 − r +1
= r 4 + r 2 + 1 =LHS

(ii) 1 1
− 2
r + r +1 r − r +1
2

r 2 − r + 1 − (r 2 + r + 1)
= 2
(r + r + 1)(r 2 − r + 1)
−2r
= 2
(r + r + 1)(r 2 − r + 1)
−2r
= 4 2
r + r +1

RVHS 2010 H2 Math Prelim P1 4


∑ ∑r
N N
r 1 2r
=−
r =1
r + r +1
4 2
2 r =1
4
+ r2 +1

∑ ⎛⎜⎝ r
N
1 1 1 ⎞
=− − 2 ⎟
2 r =1
2
+ r +1 r − r +1⎠
1⎛1
= − ⎜ −1
2⎝3
1 1
+ −
7 3
1 1
+ −
13 7
+...
1 1 ⎞
+ − 2 ⎟
+2
+ −
N N 1 N N 1⎠ +
1⎛ 1 ⎞
= − ⎜ −1 + 2 ⎟
2⎝ N + N +1 ⎠
1⎛ 1 ⎞
= ⎜1 − 2 ⎟
2 ⎝ N + N +1 ⎠
(iii) ∞ ∞

∑ ∑r
r r 1 1⎛ 1 ⎞ 1
= − 4 2 = lim ⎜1 − 2 ⎟−
r =2
r + r2 +1
4
r =1
4
+ r +1 1 +1 +1
2 N →∞ 2 ⎝ N + N +1⎠ 3
1
Since lim 2 → 0,
N →∞ N + N + 1

1⎛ 1 ⎞ 1 1 1 1
∴ lim ⎜1 − 2 ⎟− = − = .
N →∞ 2
⎝ N + N +1⎠ 3 2 3 6

Question 7 [8 marks]
(i) d =2

Method 1:
Using long division:
ax 2 + bx + c c − 2 ( b − 2a )
y= = ax + ( b − 2a ) +
x+d x+2
∴ a = 1 , b = −2

Method 2:
y = x−4+
h
=
( x − 4 )( x + 2 ) + h = x 2 − 2 x − 8 + h
x+2 x+2 x+2
∴ a = 1 , b = −2

(ii) x2 − 2x + c
y=
x+2
c
At ( 0, −4.5 ) : −4.5 =
2
∴ c = −9

RVHS 2010 H2 Math Prelim P1 5


(iii)
y
(4, k)

4
−2 O 4+k
x
−4

( x − 4)
2
+ y 2 = k 2 is a circle centre (4, 0), radius k
k > 4.52 + 42
145 145
⇒k<− or k >
2 2

Question 8 [9 marks]
(i) ( z + 2)3 = −8
( z + 2)3 = 23 e(π + 2 kπ )i
π 2 kπ
( + )i
z + 2 = 2e 3 3
, k = 0, ±1
π 2 kπ
( + )i
z = 2e 3 3 − 2, k = 0, ±1
(ii) When k = 1 , z = 2eπ i − 2 = 2(cos π + i sin π ) − 2 = −2 − 2 = −4
π
i ⎛ π π⎞
k = 0, z = 2e 3 − 2 = 2 ⎜ cos + i sin ⎟ − 2
⎝ 3 3⎠
⎛1 3 ⎞
= 2 ⎜⎜ − i ⎟⎟ − 2 = −1 − 3i
⎝2 2 ⎠
π
− i ⎛ ⎛ π⎞ ⎛ π ⎞⎞
k = −1, z = 2e 3 − 2 = 2 ⎜ cos ⎜ − ⎟ + i sin ⎜ − ⎟ ⎟ − 2
⎝ ⎝ 3⎠ ⎝ 3 ⎠⎠
⎛1 3 ⎞
= 2 ⎜⎜ + i ⎟⎟ − 2 = −1 + 3i
⎝2 2 ⎠
So the 3 roots are z = −1 ± 3i and −4 .

( z + 2)3 + 8
= [ z − (−1 + 3i ) ][ z − (−1 − 3i ) ][ z + 4 ]
= [ z + 1 − 3i ] [ z + 1 + 3i ] [ z + 4 ]

RVHS 2010 H2 Math Prelim P1 6


(iii) [ z + 1 − 3i ][ z + 1 + 3i ]
( ) (
= z 2 + 1 − 3i z + 1 + 3i z + 4 )
= z2 + 2z + 4

Hence a = 2 and b = 4 .
(iv) ( z + 2)3 = −8
3
z+2 =8
z+2 =2
Thus, the roots in (i) lie on a circle centre (-2,0) and radius 2.
(or deduce answer from the form in answers of (i))

Question 9 [10 marks]


(i) f ( x ) = ln (1 + sin x )
cos x
∴f ′( x) =
1 + sin x
⇒ (1 + sin x ) f ′ ( x ) = cos x (shown)
(ii) Differentiate with respect to x:
(1 + sin x ) f ′′ ( x ) + ( cos x ) f ′ ( x ) = − sin x
Differentiate with respect to x: (1 + sin x ) f ′′′ ( x ) + ( cos x ) f ′′ ( x ) − ( sin x ) f ′ ( x ) + ( cos x ) f ′′ ( x ) = − cos x
At x = 0 : f ( x ) = 0 , f ′ ( x ) = 1 , f ′′ ( x ) = −1 , f ′′′ ( x ) = 1
x 2 x3
∴f ( x) ≈ x − +
2 6

Equation of tangent of y = f ( x ) at x = 0 is y = x .
(iii) y

⎛ 3π ⎞
⎜ , 0.757 ⎟
⎝ 8 ⎠

⎛ 3π ⎞
⎜ , 0.654 ⎟
⎝ 8 ⎠

O x

1 2 1 3
g ( x) = x − x + x
2 6

⎛ 3π ⎞ f ( x ) = ln (1 + sin x )
⎜ − , − 2.14 ⎟
⎝ 8 ⎠

⎛ 3π ⎞
⎜ − , − 2.58 ⎟
⎝ 8 ⎠

RVHS 2010 H2 Math Prelim P1 7


(iv) To find x such that f ( x ) − g ( x ) < 0.05
Using GC, draw the graph of y = f ( x ) − g ( x ) to find the intersection with y = 0.05 .

−0.7745598 < x < 0.96923979


Range is −0.775 < x < 0.969.

Question 10 [10 marks]


(a) Let the first term and common difference of the AP be a and d respectively.
a + 2d a + 3d
=
a a + 2d
( a + 2d ) = a ( a + 3d )
2

a 2 + 4ad + 4d 2 = a 2 + 3ad
a = −4d ------ (1)

20
⎡ 2 ( a + d ) + (19 ) 2d ⎤⎦ = 960
2 ⎣
2a + 40d = 96 ------ (2)

Subs. (1) into (2):


−8d + 40d = 96
⇒d = 3

(b) Mth Amt at beginning of Amt at end of mth


(i) mth
Jan x 1.03x
Feb 1.03x + 2x 1.032x +2(1.03)x
Mar 1.032x +2(1.03)x + 1.033x + 2(1.03)2x +
3x 3(1.03)x

∴ Amt on the last day of March


= $[1.033x + 2(1.03)2x + 3(1.03)x]
= $x[1.033 + 1.032 + 1.03
+ 1.032 + 1.03
+ 1.03]
⎡1.03 (1.033 − 1) 1.03 (1.032 − 1) 1.03 (1.031 − 1) ⎤
=$x ⎢ + + ⎥
⎢⎣ 1.03 − 1 1.03 − 1 1.03 − 1 ⎥

1.03 (1.033 + 1.032 + 1.03 − 3)
= $x
0.03

RVHS 2010 H2 Math Prelim P1 8


103 x ⎛ 1.03 (1.03 − 1) ⎞
3

=$ ⎜ − 3⎟
3 ⎜ 0.03 ⎟
⎝ ⎠
103 x 103 (1.03 − 1) − 9
⎛ ⎞
3

=$ ⎜ ⎟
3 ⎜ 3 ⎟
⎝ ⎠
103x
=$
9
(103(1.03)12 − 112 ) (shown)
(b) 103x
(ii) 9
(103(1.03)3 − 112 ) − ( x + 2 x + 3 x ) > 100 ⇒ x > 328.37
∴ least value of x is 329

Question 11 [12 marks]


uuur
Let PB be b and F be the foot of perpendicular from B to l1.

l1
l3 a F
P
l2
B
b

⎛ a ⋅b ⎞ uuur
( )
Then ⎜ 2 ⎟ a = b ⋅ a$ a$ = PF
⎜ a ⎟
⎝ ⎠
⎛ a ⋅b ⎞ uuur uuur uuur
∴ b − ⎜ 2 ⎟ a = PB − PF = FB ,
⎜ a ⎟
⎝ ⎠
which is parallel to l3.

Alternative: Use cross product.

⎛4⎞ ⎛8⎞
⎜ ⎟ ⎜ ⎟
−1 ⋅ −4
⎛ 8 ⎞ ⎜⎜ ⎟⎟ ⎜⎜ ⎟⎟ ⎛ 4 ⎞
⎛ a ⋅b ⎞ ⎜ ⎟ 2 3 ⎜ ⎟
b − ⎜ 2 ⎟ a = ⎜ −4 ⎟ − ⎝ 2 ⎠ 2⎝ 2⎠ ⎜ −1⎟
⎜ a ⎟
⎝ ⎠ ⎜ 3 ⎟ (4 +1 + 2 ) ⎜ 2 ⎟
⎝ ⎠ ⎝ ⎠
⎛8⎞ ⎛4⎞ ⎛ 0⎞
= ⎜⎜ −4 ⎟⎟ − 2 ⎜⎜ −1⎟⎟ = ⎜⎜ −2 ⎟⎟
⎜ 3 ⎟ ⎜ 2 ⎟ ⎜ −1 ⎟
⎝ ⎠ ⎝ ⎠ ⎝ ⎠
Since P(3, –5, 2) is a common point,
⎛ 3⎞ ⎛ 0⎞
∴ l3 : r = ⎜ −5 ⎟ + μ ⎜⎜ −2 ⎟⎟ , μ ∈ .
⎜ ⎟
⎜ 2⎟ ⎜ −1 ⎟
⎝ ⎠ ⎝ ⎠

RVHS 2010 H2 Math Prelim P1 9


⎛ ⎛8⎞ ⎛ 0⎞ ⎞
⎜ ⎜ ⎟ ⎜ ⎟ ⎟
⎜ ⎜ −4 ⎟ ⋅ ⎜ −2 ⎟ ⎟
⎜ ⎜ 3 ⎟ ⎜ −1 ⎟ ⎟
⎝ ⎠ ⎝ ⎠
Angle between l2 and l3 = cos −1 ⎜ ⎟
⎜ 64 + 16 + 9 0 + 4 + 1 ⎟
⎜ ⎟
⎜ ⎟
⎜ ⎟
⎝ ⎠
o
= 76.3

⎛4⎞ ⎛ 0⎞ ⎛5⎞
⎜ ⎟ ⎜ ⎟ ⎜ ⎟
n1 = ⎜ −1⎟ × ⎜ −2 ⎟ = ⎜ 4 ⎟
⎜ ⎟ ⎜ ⎟ ⎜ ⎟
⎝ 2 ⎠ ⎝ −1 ⎠ ⎝ −8 ⎠
⎛k ⎞ ⎛5⎞
So, ⎜ 2 ⎟ = c ⎜⎜ 4 ⎟⎟
⎜ ⎟
⎜ −4 ⎟ ⎜ −8 ⎟
⎝ ⎠ ⎝ ⎠
⎧ 1
⎪⎪ c = 2
⇒⎨
⎪k = 5
⎪⎩ 2

⎛5⎞
⎜ ⎟
Π 2 : r ⎜ 4 ⎟ = 18
⎜ −8 ⎟
⎝ ⎠
⎛2⎞⎛5⎞
Since ⎜⎜ 0 ⎟⎟ ⎜⎜ 4 ⎟⎟ = 18 , a point on Π 2 is Q ( 2, 0, −1) .
⎜ −1⎟ ⎜ −8 ⎟
⎝ ⎠⎝ ⎠
⎛ 2 ⎞ ⎛ 3 ⎞ ⎛ −1 ⎞
uuur ⎜ ⎟ ⎜ ⎟ ⎜ ⎟
PQ = ⎜ 0 ⎟ − ⎜ −5 ⎟ = ⎜ 5 ⎟
⎜ −1 ⎟ ⎜ 2 ⎟ ⎜ − 3 ⎟
⎝ ⎠ ⎝ ⎠ ⎝ ⎠
uuur
Required distance = PQ ⋅n1

⎛ −1 ⎞ ⎛5⎞
⎜ ⎟ 1 ⎜ ⎟
= ⎜ 5 ⎟⋅ 4
⎜ −3 ⎟ 25 + 16 + 64 ⎜⎜ ⎟⎟
⎝ ⎠ ⎝ −8 ⎠
13 105
=
35

Alternative Method 1 to calculate distance:

RVHS 2010 H2 Math Prelim P1 10


⎛5⎞
⎜ ⎟
⎜4⎟
⎜ −8 ⎟
∏1 : r. ⎝ ⎠
105
= −21
105
⎛5⎞
⎜ ⎟
⎜4⎟
⎜ −8 ⎟
∏ 2 : r. ⎝ ⎠
105
= 18
105

Thus, O is between the planes.


So, distance = 105
21
+ 18
105
= 39
105

Alternative Method 2 to calculate distance:


Equation of line through P and perpendicular to Π1 and Π 2 :
⎛ 3⎞ ⎛5 ⎞
⎜ ⎟ ⎜ ⎟
r = ⎜ −5 ⎟ + μ ⎜ 4 ⎟
⎜ 2⎟ ⎜ −8 ⎟
⎝ ⎠ ⎝ ⎠
Let N be the foot of perpendicular from P to Π 2 .
⎡⎛ 3 ⎞ ⎛ 5 ⎞⎤ ⎛ 5 ⎞
⎢⎜ ⎟ ⎜ ⎟⎥ ⎜ ⎟ 13
⎢⎜ −5 ⎟ + μ ⎜ 4 ⎟ ⎥ ⎜ 4 ⎟ = 18 ⇒ μ = 35
⎢⎣⎜⎝ 2 ⎟⎠ ⎜ −8 ⎟ ⎥ ⎜ −8 ⎟
⎝ ⎠⎦ ⎝ ⎠
⎛ 3⎞ ⎛5 ⎞
uuur ⎜ ⎟ 13 ⎜ ⎟
∴ ON = ⎜ −5 ⎟ + ⎜ 4 ⎟
⎜ 2 ⎟ 35 ⎜ −8 ⎟
⎝ ⎠ ⎝ ⎠
⎛5 ⎞
uuur 13 ⎜ ⎟ 13 105
Required distance = PN = 4 =
35 ⎜⎜ ⎟⎟ 35
⎝ −8 ⎠

Question 12 [13 marks]


(i) 2
y= 3
y
(4 − x ) 2 2

0.25
–2 0 32 x

(ii) 3 2
Required area = ∫ 3
dx
(4 − x )
0
2 2

RVHS 2010 H2 Math Prelim P1 11


π
2
= ∫ 6
π 3
⋅ −2sin θ dθ
2
( 4 − 4 cos θ ) 2 2

π
2
= ∫ 2
π 3
⋅ 2sin θ dθ
6
8 ( sin θ )
2 2

π
1
=
2 ∫ 2
π
6
cos ec 2θ dθ
π
1
= [ − cot θ ] π2
2 6
1⎡
= ⎣0 + 3 ⎤⎦
2
3
= units2
2
(iii) 2
2 3
2 ⎛ 2 ⎞3
y= 3
⇒ (4 − x ) 2 2
= ⇒ x =4 −⎜ ⎟ 2

(4 − x )2 2 y ⎝ y⎠

⎛ 2 2

( 3 ) ( 2) − π ∫ ⎜⎝ 4 − 2
2 2 −
Required Vol = π 1
3
⋅y 3
⎟ dy
4 ⎠
2 2
(or π ∫ 3 ⋅ (2) dy )
0
2
⎡ 2 1

= 6π − π ⎢ 4 y − 2 ⋅ 3 y 3 ⎥
3

⎣ ⎦1
4

= 6π − π [ (8 − 6) − (1 − 3)]
= 2π units3

RVHS 2010 H2 Math Prelim P1 12


RVHS 2010 Preliminary Exam: 9740 H2 Mathematics Paper 2 Solutions

Question 1 [6 Marks]
(a)
y
x = 2(a − 1)

B '(−2a − 2, 2)

x
−2 0

A '(−6a − 2, −2)

dy
dx
(b)

−3a −a x
0 a

Question 2 [7 Marks]
(i) 1 uuur uuur
Area of triangle OAB = OA × OB
2
⎛1⎞ ⎛4⎞
1⎜ ⎟ ⎜ ⎟
= ⎜ 2 ⎟ × ⎜ −1⎟
2⎜ ⎟ ⎜ ⎟
⎝ −3 ⎠ ⎝ 0 ⎠
⎛ −3 ⎞
1 ⎜ ⎟
=
2 ⎜ −12 ⎟
⎜ −9 ⎟
⎝ ⎠
3 26
= units2
2

RVHS 2010 H2 Math Prelim P2 Solutions 1


uuur 1 uuur uuur
(ii)
( )
OP = OA + 2OB by ratio thm
3
⎡⎛ 1 ⎞ ⎛ 4 ⎞ ⎤ ⎛ 3 ⎞
1 ⎢⎜ ⎟ ⎜ ⎟ ⎥ ⎜ ⎟
= ⎢⎜ 2 ⎟ + 2 ⎜ −1⎟ ⎥ = ⎜ 0 ⎟
3 ⎜ ⎟ ⎜ ⎟
⎢⎣⎝ −3 ⎠ ⎝ 0 ⎠ ⎥⎦ ⎝⎜ −1⎠⎟

⎛3⎞
uuur uuur uuur ⎜ ⎟
Since OQ // OP , OQ = c ⎜ 0 ⎟ for some constant c.
⎜ ⎟
⎝ −1⎠
uuur uuur
(iii) AQ BQ = 0
⎛ 3c − 1⎞ ⎛ 3c − 4 ⎞
⎜ ⎟ ⎜ ⎟
⎜ −2 ⎟ ⎜ 1 ⎟ = 0
⎜ 3 − c ⎟ ⎜ −c ⎟
⎝ ⎠ ⎝ ⎠
9c 2 − 12c − 3c + 4 − 2 − 3c + c 2 = 0
5c 2 − 9c + 1 = 0
Using GC, c = 1.68 or c = 0.119 (rej. since OQ > OP)

Question 3 [7 Marks]
(a) x = 2t + t 2 , y = (1 − t ) 2
dx dy
= 2 + 2t , = 2(1 − t ) 2 (−1) = −2(1 − t )
dt dt
dy −2(1 − t ) −2(1 − t ) (t − 1)
= = =
dx 2 + 2t 2(1 + t ) t +1
dy
For decreasing C, < 0 .
dx
(t − 1)
<0
t +1

Method 1:
+ − +

−1 1

So −1 < t < 1 .

Method 2:
(t − 1)
Plot the graph of y = and find the range of values of t such that y < 0 .
t +1

RVHS 2010 H2 Math Prelim P2 Solutions 2


So −1 < t < 1 .
(b) At t = 2 ,
x = 2(2) 2 + 22 = 8, y = (1 − 2) 2 = 1 .
At ( 8, 1) , the equation of normal is
t +1 3
gradient = − = − = −3
t − 1 t =2 1
y − 1 = −3 ( x − 8 )
y = −3 x + 25
Substitute the parametric equations into the equation of the normal.
(1 − t ) 2 = −3(2t + t 2 ) + 25
1 − 2t + t 2 = −6t − 3t 2 + 25
4t 2 + 4t − 24 = 0
t2 + t − 6 = 0
1 − 4(1)(−6) = 25 > 0
Thus, the normal will meet the C again.
[Alternative: Factorise and solve the equation.]

Question 4 [10 Marks]


dl
= kl
dt
1
∫ l dl = ∫ k dt
ln l = kt + C (note: l > 0)
l = e kt + C
l = Ae kt (shown)
A > 0 and k < 0

When t = 0, l = L: A = L
∴ l = Le kt
l
L

0 t

RVHS 2010 H2 Math Prelim P2 Solutions 3


The model suggested by the researcher is not suitable as l → 0 when t → ∞ (i.e. the bar
vanished).

t = T , l = 0.5L : 0.5 L = B + Le kT
0.5 L − B
⇒ = e kT
L
( 0.5L − B )
3

When t = 3T, l = B + Le 3 kT
= B+
L2

Question 5 [10 Marks]


(i)

Any horizontal line cuts the graph of f at most 1 point.


Therefore, f is a one to one function and f -1 exists.

f : x a 2 x3 + 1, x > 0.
Let y = 2 x + 1 3

3
y −1
x=
2
3
x −1
f -1 : x → , x >1
2

(ii) For gf to exist,


(1, ∞) = R f ⊆ Dg = (a, ∞) .
Hence largest possible a = 1 .

For a = 1 ,
gf ( x)
= g (2 x3 + 1)
= ln(2 x3 + 1 − 1)
= ln(2 x3 ), x > 0

Rgf = (−∞, ∞) .

Since = Rg ⊄ D f = (0, ∞) , so fg does not exist.

RVHS 2010 H2 Math Prelim P2 Solutions 4


(iii) f -1f ( x) = x, x ∈ D f = (0, ∞)
ff -1 ( x) = x, x ∈ D = (1, ∞)
f −1
y y
y = f -1f ( x) y = ff -1 ( x)

(1, 1)
x x
0 0

The set of values of x such that f -1f ( x) = ff -1 ( x) is (1, ∞) .

Question 6 [3 Marks]
Let A be the r.v. denoting number of students out of 120 students who make use of the
school’s e-learning portal.
A ~ B(120, 85 )
Since n = 120 > 50 is large, np = 120(5 / 8) = 75 > 5 and nq = 120(3 / 8) = 45 > 5 .
A ~ N (75, 225
8
) approximately.
P( A > 80)
= P( A ≥ 81)
= P( A ≥ 80.5) (by cc)
= 0.150 (3.s.f)

Question 7 [4 Marks]
The 350 days of ticket sales volume are listed in order (numbered from 1 to 350) with
50 = 7 . So every 7 day is chosen. The first sample is selected
th
an interval of selection, 350
randomly from the first 7 days, say the 2nd night is selected. Then the day 9th, 16th,
23rd,… up to a total of 50 days are chosen.

To choose every 7th day means that a particular day sales will be chosen. E.g. if
Monday is chosen, then the next 7th day will also be a Monday. But since the movie
ticket sales are periodic in nature, the sample will be biased and not be representative
of the actual ticket sales.

Stratified sampling seems more appropriate here as we should choose samples from
Weekends, Beginning of the week and Midweek as our strata so as to get a more
representative sample.

RVHS 2010 H2 Math Prelim P2 Solutions 5


Question 8 [8 Marks]
(7 − 1)!× 4!×10 = 172800
(i) Case 1: First and last seats occupied by females
4 × 3 × 8! ways
Case 2: First and last seats occupied by males
6 × 5 × 8! ways
Total number of ways = 4 × 3 × 8!+ 6 × 5 × 8! = 1693440

(ii) 10!
= 151200
4!

Question 9 [8 Marks]
(i) Let X be the random variable for the sales per day.
n = 12

x=
∑ ( x − 450) + 450 = 622.8 + 450 = 501.9
12 12
1 ⎡⎢ ( ∑ ( x − 450 ) ) ⎤⎥
2

s = Σ ( x − 450 ) −
2 2

12 − 1 ⎢ 12 ⎥
⎣ ⎦
1⎡ ( 622.8) ⎤ = 4224.970909
2

= ⎢ 78798 − ⎥
11 ⎣⎢ 12 ⎦⎥
s = 64.99977622

Test H0 : μ = 452.80
against H1 : μ > 452.80

at 1% level of significance.

Test statistic:
X − 452.80
Under H0, T = ~ t (11) since n = 12 is small.
s
12

p-value = P(T > tcalculated ) = 0.011983849 > 0.01

We do not reject H0.

We conclude that there is insufficient evidence at 1% level of significance that the


food store has underestimated its average daily sales.

Assume that the daily sales follow a normal distribution.

RVHS 2010 H2 Math Prelim P2 Solutions 6


Since H0 is rejected at 5% level of significance, p-value < 0.05 for the one-tailed test.
For the two-tailed test to reject H0 at 5% level of significance,
2 x p-value* < 0.05, which may or may not be true. So there may or may not be
significant evidence.
* the original p-value for the one-tailed test

Question 10 [9 Marks]
(a) P(at least 2 of the 4 numbers are the same)
(i) = 1 − P(all the 4 numbers are different)
= 1 − ( 66 )( 56 )( 64 )( 63 )
= 18
13

Alternatively,
P(at least 2 of the 4 numbers are the same)
=P(exactly 2 same, and another 2 same) + P(exactly 2 same and 2 different) +
P(exactly 3 same)
+ P(all 4 same)
⎛6⎞ 4 ⎛ 6⎞⎛ 6⎞ 4 ⎛ 6⎞⎛ 2⎞ 4 ⎛ 6⎞
( 6 ) ⎜ 2 ⎟ + 2!4! ( 16 ) ⎜ 3 ⎟ ⎜1 ⎟ + 3!4! ( 16 ) ⎜ 2 ⎟ ⎜1 ⎟ + ( 16 ) ⎜1 ⎟
4! 1 4
= 2!2!
⎝ ⎠ ⎝ ⎠⎝ ⎠ ⎝ ⎠⎝ ⎠ ⎝ ⎠
= 18
13

(a) P(exactly 2 of the 4 numbers are same at least one 6)


(ii)
P(2 out of the 4 numbers are same ∩ at least 1 six)
=
P(at least one six)
P(six appear twice) + P(six appear once)
=
P(at least one six)
P(6, 6, B, C ) + P( A, A, 6, B)
=
1 − P(no six)
⎛ 5 ⎞ 4! 1 4 ⎛ 5 ⎞⎛ 2 ⎞
2! ( 6 ) ⎜ ⎟ + 2! ( 6 ) ⎜ ⎟⎜ ⎟
4! 1 4

= ⎝ 2⎠ ⎝ 2 ⎠⎝1 ⎠
1 − ( 56 )
4

360
= or 0.537
671
(b) 4 P( A I B) 4 4
P ( B A) = ⇒ = ⇒ P( A I B) = P ( A)
7 P( A) 7 7
P ( A U B ) = P( A) + P( B) − P( A I B )
4
1 − P( A′ I B′) = P ( A) + [1 − P( B′)] − P( A)
7
4
0.65 = P( A) + 0.5 − P( A)
7
3
0.15 = P ( A)
7
P( A) = 0.35

RVHS 2010 H2 Math Prelim P2 Solutions 7


Question 11 [9 Marks]
(i)

(ii) r = −0.885

This illustrates a moderate linear correlation between the radiation intensity and time
which agrees with the scatter diagram, where the points do not seems to lie close to a
straight line.

(iii) Equation of the regression line of x on t : x = 11.610575 − 0.806146t


x = 11.6 − 0.806t (correct to 3 s.f.)

There is a decrease of 0.806 unit in the radiation intensity with every unit increase in
time.

(iv) r = −0.995 (3 s.f)

Since the value of the product moment correlation coefficient for ln x and t is closer to −1,
the logarithmic model is more suitable than the linear model.

(v) Equation of the regression line of y on t : y = 2.624253 − 0.174726t

When x = 10, y = ln(10) = 2.624253 − 0.174726t ,


⇒ t = 1.84098 ≈ 1.84 (correct to 3 s.f.)

Question 12 [10 Marks]


(i) Let X and Y be the r.v. denoting the number of disciplinary cases involving long hair
and faiure to wear school badge respectively in a given week.
X ~ Po(1.12 × 5) and Y ~ Po(4.2)
X + Y ~ Po(9.8)
P( X + Y ≤ 9) = 0.483188233 ≈ 0.483 (3.s.f)

(ii) P( X + Y ≤ 9 X ≥ 8 )
P( X + Y ≤ 9 ∩ X ≥ 8)
=
P( X ≥ 8)
P( X = 8)P(Y ≤ 1) + P( X = 9)P(Y = 0)
=
P( X ≥ 8)
= 0.038145 ≈ 0.0381 (3.s.f)

RVHS 2010 H2 Math Prelim P2 Solutions 8


(iii) Let W be the r.v. denoting the number of disciplinary cases involving long hair
recorded in a given day.
W ~ Po(1.12)
P (W ≥ 4) = 1 − P(W ≤ 3)
= 0.0272442116 ≈ 0.0272

Require Prob
⎛ 3⎞
( )
= ⎜ ⎟ [ P(W ≤ 3)][ P(W ≥ 4)] P(W ≥ 4)
⎝ 2⎠
2

= 5.87 × 10−5 (3 s.f )

(iv) The mean number of disciplinary cases involving long hair fluctuates throughout the
year, usually having more such cases after the school vacation.

Question 13 [10 Marks]


(a) Let C denotes the mass of a randomly chosen chicken.
Since n is large, C ~ N (2.5, 0.42 ) approximately by Central limit Theorem.
n
P(C > 2.45) > 0.95

Method 1
1 − P(C ≤ 2.45) > 0.95
P(C ≤ 2.45) < 0.05
C − 2.5
Let Z = ~ N (0, 1)
0.42
n
2.45 − 2.5
P( Z ≤ 0.4
) < 0.05
n
− 18 n < −1.64485
n > 13.15882901
n > 173.15
The least value of n is 174.

Method 2

The least value of n is 174.

RVHS 2010 H2 Math Prelim P2 Solutions 9


(b) X ~ N (2.7, σ 2 )
(i)
P( X < 2.5) = 107
50 P( X < 2.9)

P( X < 2.5) = 107 [


50 1 − P( X ≥ 2.9)
]
P( X < 2.5) = 107 [
50 1 − P( X < 2.5)
]
157 P( X < 2.5) = 50 ⇒ P( X < 2.5) = 157
50
107 107
2.5−
P( Z < σ ) = 157
2.7 50

P( Z < − 51σ ) = 157


50

− 51σ = −0.4719777448
σ = 0.424 (3.s.f) (Shown)

Alternative method:
Plot using GC:
Y1=normalcdf(-E99,2.5,2.7,X)
Y2=(50/107)normalcdf(-E99,2.9,2.7,X)
Sketch the graphs and find point of intersection.

(b) X1 + X 2 ~ N (2(2.7), 2(0.424) 2 )


(ii)
4( X1 + X 2 ) ~ N (4 × 2(2.7), 42 × 2(0.424)2 )
P(50 − 4( X1 + X 2 ) < 27.5)
= P(4( X1 + X 2 ) > 22.5)
= 0.354

(b) P(most expensive duck cost less than $11)


(iii) = P(all ducks cost less than $11 each)
4
= [ P(4 X < 11)] = ⎡⎣ P( X < 11 )⎤
4
4 ⎦
= 0.0895

RVHS 2010 H2 Math Prelim P2 Solutions 10


SAINT ANDREW’S JUNIOR COLLEGE

Preliminary Examination

MATHEMATICS
Higher 2 9740/01

Paper 1

Wednesday 15 September 2010 3 hours

Additional materials : Answer paper


List of Formulae(MF15)
Cover Sheet

READ THESE INSTRUCTIONS FIRST

Write your name, civics group and index number on all the work you hand in.
Write in dark blue or black pen on both sides of the paper.
You may use a soft pencil for any diagrams or graphs.

Answer all the questions. Total marks : 100

Give non-exact numerical answers correct to 3 significant figures, or 1 decimal place in


the case of angles in degrees, unless a different level of accuracy is specified in the
question.

You are expected to use a graphic calculator.


Unsupported answers from a graphic calculator are allowed unless a question specifically
state otherwise.
Where unsupported answers from a graphic calculator are not allowed in a question, you are
required to present the mathematic steps using mathematical notations and not calculator
commands.
You are reminded of the need for clear presentation in your answers.

The number of marks is given in brackets [ ] at the end of each question or part question.
At the end of the examination, fasten all your work securely together.

This document consists of 6 printed pages including this page.


[Turn over
2

1. The first 4 terms of a sequence are given by u1  63 , u2  116 , u3  171 , u4  234 . Given
that un is a cubic polynomial in n , find un in terms of n . [3]
Hence write down the value of u50 . [1]

2. Without using a graphic calculator, solve 1  x  x  1 .


2  3x
Hence, solve 1  x 2  x 2  1 exactly.
2
[5]
2  3x

3. Iron Will, the magician, is constructing a prism with an equilateral triangle base for his
latest escape act.
The edges of the prism are made from iron rods, the
rectangular faces of the prism are made of glass panels, and
the triangular faces of the prism are made of wooden boards.
The volume of the prism is fixed at exactly 2 3 cubic
metres.
8
Show that h  2 , where x is the length of the sides of the
x
[2]
triangular base and h is the height of the prism.

The cost of the iron rods is $1 per metre, the cost of the
wooden boards is $ 2 3 per sq. metre and the cost of the
glass panels is $2 per sq. metre.

Show that the expression of the total cost C of constructing


the prism in terms of x is as follows:
C  3 x 2  6 x  48 x 1  24 x 2 [2]

Using an analytical method, find the minimum cost of constructing this prism. [4]

4. (a) Solve the equation w2  3  4i , expressing your answer(s) in the form x  iy . [3]

(b) Find the fourth roots of –16, expressing your answers in the form rei , where
r  0 and      . On an Argand diagram, mark the points that represent the
roots clearly. [5]

[Turn over
3

5. (a) Functions f and g are defined by

f : x  x 2  3 for x  0
g : x  2 x  4 for all x  R

(i) Find f 1 in a similar form.


[2]
(ii) Sketch the graphs of f, f 1 and f 1f on the same diagram. [1]
(iii) Given that gf 1 exists, find gf 1 ( x) . [1]

(b) The function h is defined by

h(x) = 12  x 2 for 0  x  3
3x  6 for 3  x  6

and h(x) = h( x  6 ) for all x  R

(i) Find h(16) + h(25). [2]


(ii) Sketch y = h(x) for 6  x  12 . [3]

6. A curve C is defined parametrically by the equations


 
x  1  cos t , y  t  sin t for t 
2 2

(i) State the exact range of values that x and y can take. Hence sketch the curve C .
Label your graph, indicating the axes intercept(s) clearly if any. [3]


(ii) The tangent to the curve, C at the point P where t  is denoted by l .
2
Find the Cartesian equation of l . [3]

(iii) Find the range of values of m such that the line y  mx intersects C at two points. [2]

7. The equations of two planes 1 , 2 are given by


1 : 2 x  4 y  z  8
2 : x  2 z  6

(i) Find the vector equation of the line of intersection l between the planes 1 and 2 . [2]

(ii) Find the foot of perpendicular, F1 from the point  6,9, 2  to the plane 1 . [3]

[Turn over
4

Another plane 3 contains the points F1 and F2 and is parallel to l .

 26 
 
  5 
(iii) Given that OF2   9  , show that the Cartesian equation of the plane 3 is given
2 
 
5 
by 15 x  8 y  40 z  22 . [3]

(iv) By moving plane 3 by m units in the direction of vector v, all three planes will
meet at l . Find m and v. [3]

8. In a “man vs machine” contest, an athlete competes against a robot to see who can pull
himself up a 16 m rope faster. The athlete climbs up 0.8 m in his first pull; the height
attained by each subsequent pull decreases by a factor of 1/20 as he grows more and
more tired. The robot is able to pull itself up by 0.4m each time as it never grows tired.

16m

(i) The athlete and robot both start climbing at the same time from the base of the
rope. Assuming they pull at the same rate, after how many pulls will the robot [4]
overtake the athlete?

(ii) Will the athlete be able to reach the top of his rope? Justify your answer. [2]

(iii) The contestants again start climbing from the base but the athlete changes his
strategy. He continues pulling at the same rate, but now he climbs up x m in each
of his first two pulls; (x – 0.02) m in each of his next two pulls; and every
subsequent pair of pulls shows a similar decrease of 2cm from the previous pair.

Show that the distance he travels after 2n pulls is 2nx  0.02n(n  1) .

Given that he won the contest, find, correct to 2 decimal places, the minimum
value of x. [4]

[Turn over
5

1
dy e tan x
9. It is given that  , where tan 1 x denotes the principal value.
dx 1  x 2

(i) Find an expression for y in terms of x given that y = 1 when x = 0. [2]

  ddx y  (2 x  1) ddyx  0 .
2
(ii) Show that 1  x 2 2
[2]

(iii) By further differentiation of the result in (ii), find the Maclaurin’s series for y up to
and including the term in x3. [4]

(iv) Hence, deduce the series expansion for


1
e tan x 1 x
 (a) (b) e2 x  tan up to and including the term in x2. [4]
(1  x) 2


10. (a) Prove by induction for all n  that
n

  r ! r    n  1!  1
r 1
[5]

(b) The diagram shows part of the graph of y  x3  2 x 2  5 x  6 . The 3 real roots of
the equation x3  2 x2  5x  6  0 are denoted by  ,  and  where    <  .

 0  

(i) Find the values of  ,  and  . [1]

A sequence of real numbers x1 , x2 , x3 , x4 ,... satisfies the recurrence relation



xn1  1  3 5xn 2  8 xn  5 for n  .

[Turn over
6

(ii) Prove algebraically that if the sequence converges to L, then L is equal to either [3]
 ,  or  .

(iii) Show that if xn 1  xn , then   xn   or xn   . [3]

11. (a) Given that


A(sin  + cos ) + B(cos  – sin )  4 sin ,
Find the values of the constants A and B. [2]

1
 4sin 
Hence find the exact value of  0
4
sin   cos 
d . [3]

(b) 2t
(i) Express in partial fractions.
(t  1) 2 [2]

5 1
(ii) Hence, find the exact value of  1
x  2x 1
dx , using the substitution

t= 2x 1 . [6]

End of Paper

[Turn over
SAINT ANDREW’S JUNIOR COLLEGE

Preliminary Examination

MATHEMATICS
Higher 2 9740/02

Paper 2

Monday 20 September 2010 3 hours

Additional materials : Answer paper


List of Formulae(MF15)
Cover Sheet

READ THESE INSTRUCTIONS FIRST

Write your name, civics group and index number on all the work you hand in.
Write in dark blue or black pen on both sides of the paper.
You may use a soft pencil for any diagrams or graphs.

Answer all the questions. Total marks : 100

Give non-exact numerical answers correct to 3 significant figures, or 1 decimal place in


the case of angles in degrees, unless a different level of accuracy is specified in the
question.

You are expected to use a graphic calculator.


Unsupported answers from a graphic calculator are allowed unless a question specifically
state otherwise.
Where unsupported answers from a graphic calculator are not allowed in a question, you are
required to present the mathematic steps using mathematical notations and not calculator
commands.
You are reminded of the need for clear presentation in your answers.

The number of marks is given in brackets [ ] at the end of each question or part question.
At the end of the examination, fasten all your work securely together.

This document consists of 6 printed pages including this page.


[Turn over
2

Section A: Pure Mathematics [40 marks]

1 
   
1 The diagram below shows a rectangle tank where AG   2  and AC is parallel to the
 4
 
 2
vector 1  .

 2
 
H G

F
E

D
C

A
B


(i) Find the length of projection of AG onto the line passing through A and C. [2]

(ii) Given that GI : IC  3: 2 , find AI . [2]
 
(iii) Find the acute angle between AG and GC . [2]

2 In a chemical reaction a compound X is formed from a compound Y. The mass in grams


of X and Y present at time t seconds after the start of the reaction are x and y
respectively. The sum of the two masses is equal to 100 grams throughout the reaction.
At any time t, the rate of formation of X is proportional to the mass of Y at that time.
dx
When t = 0, x = 5 and  1.9 .
dt

dx
(i) Show that x satisfies the differential equation  0.02(100  x) . [2]
dt

(ii) Solve this differential equation, obtaining an expression for x in terms of t. [4]

(iii) Calculate the time taken for the mass of compound Y to decrease to half its initial
[2]
value.

(iv) Sketch the solution curve obtained in part (ii) and state what happens to
[3]
compounds X and Y as t becomes very large.

[Turn over
3 (a) Indicate clearly on a single Argand diagram, the locus of z that satisfy

0  arg( z  1  i)  and z  i  2 .
2

Hence, find the complex number z , in the form x + iy, for which arg( z  1)  ,
4
leaving your answer in exact form. [6]

3
 3 3   
(b) Consider the complex number w  2  cos  i sin  cos  i sin  .
 4 4  6 6

(i) Find the exact values of the modulus and argument of w. Hence, express w
[3]
in modulus-argument form.

(ii) Find the possible value(s) of wn , when n is a multiple of 4. [2]

8x
4. The curve C has equation y  f  x  , where f  x   .
x 1
2

(i) Using a non-calculator method, find, in simplest form, the exact coordinates of the
turning points of C . [3]

(ii) Sketch the curve C , indicating clearly any axis intercept(s) and asymptote(s). [2]

 f  x  dx  2 ln  n 2  1 . Hence deduce the exact


n
(iii) Given that n  0 , show that
0

f  x  dx .
2
value of  2 [3]

(iv) Sketch on a separate diagram, the graph of g  x   f  x  , labeling clearly any


stationary point(s) and asymptote(s).

The region R is bounded by the curve y = g(x), the lines x  0 , x  1 and the
x-axis. Find, in exact form, the volume generated when R is rotated completely
about the x-axis. [4]

Section B: Statistics [60 marks]

5 ABC College has a student population of 1800, of whom 60% are female students and
40% are male students.

The College intends to get a sample of 100 student volunteers to take part in a survey on
the College’s National Education Programme.

(i) Comment whether such a sample consisting of all the volunteers is likely to give a
true picture of the opinions of all the students about the College’s National
Education Programme. [1]

(ii) Suggest a method of obtaining a more representative sample and describe how it
[3]
may be carried out.
4

6 James has 9 marbles, 4 of them are red, 3 of them are blue, and 2 of them are yellow.
He arranged these marbles on a straight line. All marbles are identical except for their
colour.

Find the number of ways that the marbles can be arranged if

(a) there are no restrictions, [1]

(b) the arrangement is symmetrical about the centre marble,


(e.g. R-R-Y-B-B-B-Y-R-R) [2]

(c) each blue marble is between two marbles of the same colour.
(e.g. Y-B-Y or R-B-R) [4]

7 (a) A & B are two events with non-zero probability. Explain if each of the following
statements is necessarily true, necessarily false, or neither necessarily true nor
necessarily false.

(i) If A & B are mutually exclusive, then they are independent. [1]

(ii) If A & B are independent, then they are mutually exclusive. [1]

(b) A teacher brings 4 black, 3 blue, 2 red and 1 green markers to the classroom for
each of his lessons. Unknown to him, the probabilities that a black, blue, red or
1 1 1
green marker is out of ink are p , p, p and p respectively, where
2 4 8
0  p 1.

(i) Find, in terms of p, the probability that a randomly chosen marker from his
set of ten markers is out of ink. [2]

(ii) In the classroom, the first marker he tries out is out of ink. Find, in terms of
p, the probability that the next marker he tries out is a red marker that is also
out of ink. [3]

(iii) After numerous lessons, the teacher realised that, in general, the first marker
he tries out works at least 7 out of 10 times. Find the range of possible values
of p. [2]

[Turn over
5

8 (a) A secret source claims that 0.3% of the residents in the suburbs of Zozoland are
spies from Buzzland. If the claim were true, what is the probability, using a
suitable approximation, that there are at least five spies living in a Zozoland suburb
of 1200 residents? [3]

(b) Another independent source (assumed to be reliable) claims that there is an average
of 1.3 Buzzland spies and 0.4 Dodoland spies in a typical Zozoland city.

(i) The investigation bureau will only do extensive combing of a city where
there are more than five spies present. Given that there are 10,000 Zozoland
cities, estimate the number of cities that will be under investigation. [3]

(ii) The bureau investigated a certain number of cities and caught 23 Buzzland
spies and 11 Dodoland spies. Find the most likely number of cities that had
been investigated, stating an assumption needed for your calculation in the
context of the question. [4]

9 Marine biologists discovered that in a remote island off Philippines, a type of algae is
growing at an exponential rate and is threatening the marine life in the region. The
growth of the algae was collected over a period of 10 years and was recorded as follows
(measured as cell density in millions).

Year (x) 1 2 3 4 5 6 7 8 9 10
Cell density(y) 1.21 1.66 2.83 4.35 4.91 6.55 8.01 9.66 12.72 18.01

(i) The relationship between the year, x, and the cell density recorded as y, are related

by y  ae , where a and b are unknown constants. By plotting a scatter diagram,


bx

[2]
comment on the relationship between x and ln y.

(ii) Find the estimated regression line of ln y on x. Hence calculate estimates of a and
b. [3]

(iii) Estimate the cell density at Year 12. Comment on the reliability of your answer. [2]

(iv) Estimate the year at which the cell density is at 7 million. Comment on the choice
of your regression line. [2]

10 (a) In a manufacturing company, the mean salary of its employees is S$30,000.


The salary structure of the company is such that only 20% of the employees earn
higher than the mean salary. Explain whether the use of Normal distribution to
model the salary distribution of the employees in this company is appropriate. [2]

[Turn over
6
(b) Anne travels to work each day by bus. The total time, T A in minutes for Anne’s
journey to her office, is a normal random variable with a mean of 55 and a variance
of 25.

Ben drives to work every day. The total time, T B in minutes, that Ben spends
driving to work is a normal random variable with a mean of 53 and a variance of
16.

(i) Find the probability that the average of the individual times taken by Anne
and Ben to travel to work in a day is between 50 to 60 minutes. [2]

(ii) Find the greatest value of a, if the probability that Ben’s travel time differs
[2]
from 53 minutes by at most a minutes is not more than 0.6.

(iii) Using a suitable approximation, calculate the probability, that for a randomly
chosen period of 60 days, there are at least 43 days on which Anne will take
[4]
longer than Ben to travel to work.

11 (a) A water treatment plant monitors their drinking water from their storage tanks on
an hourly basis. The water must maintain a pH level of 8.5 (they try to maintain an
alkaline level) for it to be ‘drinkable’. At 0800 hours, they recorded the following
pH level from 11 of their tanks:
Tank 1 2 3 4 5 6 7 8 9 10 11
pH level 8.31 8.41 8.51 8.46 8.52 8.48 8.33 8.1 8.39 8.42 8.52

(i) Using an appropriate test, determine at 1% level of significance, if this


sample provides sufficient evidence that the mean pH level of the water
[3]
differs from 8.5?

(ii) State an assumption necessary for the test in a(i) to be valid. [1]

(b) At 1500 hours, a random sample was recorded from 80 tanks. Denoting the pH
level readings by x, the results are summarized as follows:

 ( x  8.5)  15.2 ,  ( x  8.5) 2


 232.2

(i) Find the unbiased estimate of the population mean and variance. [2]

(ii) Another sample of n (assume n is large) readings was recorded. Using the
unbiased estimate of the population mean and variance found in (b)(i), find
the least value of n so that the probability that this sample mean has a pH
reading of less than 8.2 is less than 0.3. [5]

End of Paper

[Turn over
1

SERANGOON JUNIOR COLLEGE

2010 JC2 PRELIMINARY EXAMINATION

MATHEMATICS

Higher 2 9740/1

Wednesday 18 August 2010

Additional materials: Writing paper

List of Formulae (MF15)

TIME : 3 hours

READ THESE INSTRUCTIONS FIRST


Write your name and class on the cover page and on all the work you hand in.
Write in dark or black pen on both sides of the paper.
You may use a soft pencil for any diagrams or graphs.
Do not use staples, paper clips, highlighters, glue or correction fluid.

Answer all the questions.


Give non-exact numerical answers correct to 3 significant figures, or 1 decimal place in the case of
angles in degrees, unless a different level of accuracy is specified in the question.
You are expected to use a graphic calculator.
Unsupported answers from a graphic calculator are allowed unless a question specifically states
otherwise.
Where unsupported answers from a graphic calculator are not allowed in a question, you are required
to present the mathematical steps using mathematical notations and not calculator commands.
You are reminded of the need for clear presentation in your answers.

The number of marks is given in brackets [ ] at the end of each question or part question.
At the end of the examination, fasten all your work securely together.

Total marks for this paper is 100 marks.

This question paper consists of 6 printed pages and no blank pages.

[Turn Over
2

Answer all questions (100 marks).

1 Without using the calculator solve the inequality

2 x2 + x < 3 . [3]
Hence solve the inequality
1x
2e x + e 2 < 3 . [2]

d 1
2 (i) Show that sin −1 x = for x < 1 . [2]
dx 1− x 2

1 π
1
(ii) Given that ∫0 2
1 − x2
dx = ∫ 4
a ( 2 cos2 x ) dx , show that
sin 2a + 2a − 1 = 0 . [3]

3 A sequence of numbers { xn } is defined by the relation

( )
xn = 9 2n −1 − xn −1 for n = 2,3, 4,... and x1 = 7 .

( ) n
Write down the values of x2 , x3 , x4 and x5 in the form of a bn − ( −1) , where a and b are
real numbers. [2]

Hence make a conjecture for xn and prove the conjecture using Mathematical Induction.[4]

4 Given that e y = 3 e + x + sin x . Show that

2
d2 y  dy 
3e3 y + 9e3 y   + sin x = 0. [3]
dx 2  dx 

Hence, find in terms of e, the Maclaurin’s series for y, up to and including the term in x2.
[4]
3

5 The function f is defined by f : x  3 + x − 2 x 2 for x ∈  , x ≥ k .

(i) Find the least value of k such that f has an inverse. [2]
–1 –1
(ii) Using the value of k in part (i), find f (x) and state the domain of f . [4]
–1
(iii) Hence, find the exact solution of the equation f (x) = f (x). [2]

6 (a) The diagram, not to scale, shows the graph of y = f(x). The vertical asymptotes are
x = 0 and x = 2. Sketch the graph of y = f '( x) , showing all corresponding
coordinates and asymptotes where possible. [3]

A(-1, 2)
B(1, 3)

x
-1/2 2

y= f(x)

(b) A cylindrical silo with a hemispherical rooftop is constructed to store


rubbish. r

The cost of each unit area of the rooftop is fixed at $21 and the cost of
each unit area of the curved surface of the cylinder is fixed at $7. The
total cost of the rooftop and the curved surface is fixed at $2100.

Given that the radius of the hemisphere is r, show that the volume V of
the structure can be expressed as
7π 3
V = 150r − r . [3]
3

Find the exact cost of the hemispherical top when V is at its maximum. [4]
4
[Curved surface area of sphere = 4π r 2 ; volume of sphere = π r 3 ]
3

[Turn Over
4

7 (a) A complex number w is such that ww∗ + 64 3 i + 16iw = 0 and Im( w) < 5 , where w∗
is the conjugate of w .

(i) Find w in the form x + yi, where x, y ∈ . [3]


(ii) Find the integer values of n such that w n is real . [3]

(b) Find the Cartesian equation of the locus of the point P representing the complex
number z where z − 1 − i = 3 1 + iz . Hence sketch the locus of point P. [5]

8 Lovewell the Frog is stuck at the bottom of a well. It tries to make a series of vertical upward
jumps on the well wall to get out. Its first jump is 1 m, but because of fatigue from over-
9
exertion, each subsequent jump is of the distance of the previous jump.
10

However, as the wall is wet and slippery, Lovewell slips down some distance after every jump
is made. It slips down 0.3 m after its first jump, but because it learns to grip better, each
subsequent slip is 0.02 m less than that of the previous slip.

For example, its first jump gets it 1m up the well, and then it slides down 0.3 m.
From its new position it makes a second jump up 0.9 m, and then it slides down 0.28 m.
Its third jump sees it go up a further 0.81 m, and it slides down 0.26 m; so on and so forth.

(i) How many jumps are needed for Lovewell to clear the 4m mark in the well? [5]
(ii) How many jumps are needed for Lovewell to clear the 6.5m mark in the well? [4]
(iii) Determine the highest mark in the well that Lovewell is able to reach? [2]

9 The position vectors of the points A, B, C and D are given as i + 3j , 2 j + 4k , i + j + k and


4i + 5k respectively.

(i) Find the vector equation of plane π in the form r ⋅ n = p , that contains the points A, B
and C. [3]

(ii) Find the foot of the perpendicular of the point D to the plane π . Hence find the
shortest distance from the point D to the plane π . [4]

A line l parallel to the vector j + k passes through point D and it meets the plane π at the
point N.
(iii) Find the position vector of the point N and hence find the vector equation of the
reflection of line l about the plane π . [5]
5

x 2 + ax + b
10 (a) The curve C has equation y = f ( x ) = where a, b are real constants. If the
x+9
two asymptotes of the curve intersect at the point ( −9, −12 ) and the x-axis is a tangent
to the curve, find the values of a and b. [4]

Sketch the curve y = f ( x ) , stating clearly the equations of the asymptotes and the
coordinates of any points of intersection with the axes. Hence deduce the values of p
and q such that the equation f ( x + p ) − q = kx has no real roots for k ≤ 1 . [3]

(b) The diagram below shows the sketch of the graph y = f ( x ) for a > 0 .
y

( 5a,3a )

y = ax

( −3a, 2a )

−a x
y=0 0 2a

x=0 x = 4a

Sketch on separate diagrams the graphs of


1
(i) y= , [3]
f ( x)
(ii) y2 = f ( x ) , [2]

showing clearly the equations of any asymptote(s), the coordinates of any turning
point(s) and axial intercept(s).

[Turn Over
6

11 (i) Solve the equation z5 = −32 , giving your answers in the form reiθ ,where r > 0 and
−π < θ ≤ π . [4]

(ii) Hence, show the roots of the equation w5 = 32i in an Argand diagram.

π π
The roots represented by W1 and W2 are such that − < arg (w1) < arg (w2) < .
2 2
Find the area of triangle OW1W2 where O is the origin. [5]

(iii) Express z 5 + 32 in the form

( z 2 − p cosα z + 4)( z 2 − p cos β z + 4) ( z + k )


where p, α , β and k are constants to be found. [4]

End of Paper
1

SERANGOON JUNIOR COLLEGE

2010 JC2 PRELIMINARY EXAMINATION

MATHEMATICS

Higher 2 9740/2

Wednesday 25 August 2010

Additional materials: Writing paper

List of Formulae (MF15)

TIME : 3 hours

READ THESE INSTRUCTIONS FIRST


Write your name and class on the cover page and on all the work you hand in.
Write in dark or black pen on both sides of the paper.
You may use a soft pencil for any diagrams or graphs.
Do not use staples, paper clips, highlighters, glue or correction fluid.

Answer all the questions.


Give non-exact numerical answers correct to 3 significant figures, or 1 decimal place in the case
of angles in degrees, unless a different level of accuracy is specified in the question.
You are expected to use a graphic calculator.
Unsupported answers from a graphic calculator are allowed unless a question specifically states
otherwise.
Where unsupported answers from a graphic calculator are not allowed in a question, you are
required to present the mathematical steps using mathematical notations and not calculator
commands.
You are reminded of the need for clear presentation in your answers.

The number of marks is given in brackets [ ] at the end of each question or part question.
At the end of the examination, fasten all your work securely together.

Total marks for this paper is 100 marks.

This question paper consists of 9 printed pages and 1 blank page.

[Turn Over
2

Section A: Pure Mathematics [40 marks]

1 The computer company call “Orange” manufactures the latest electronic gadget in town
called the iBoard with 3 different storage capacities namely 16GB, 32GB and 64GB. The
profit earned from each unit sold is as shown in the table below.

Storage Capacity 16GB 32GB 64GB


Profit $x $y $z

Within the first week after it was officially launched, the sales from 3 of its outlets for the
three different storage capacities is as shown below.

Storage Capacity 16GB 32GB 64GB


Outlet A 75 120 20
Outlet B 180 230 70
Outlet C 45 50 10

The total profit collected from outlets B and C are $38 750 and $8750 respectively. If the
total profit earned due to both the sales of 16GB and 32GB iBoard is equal to 12 times
the total profit earned from the sales of the 64GB iBoard, find the value of x, y and z.
[4]
Find the total profit collected from outlet A. [1]

2 A curve C has parametric equations

π
x = a sin2 t, y = a cos t, 0 ≤ t ≤ where a > 0.
2

(i) Sketch the curve. [2]

π
(ii) Find the equation of the normal at the point P where t = . [3]
3

(iii) Using a non-calculator method, determine whether the normal at P will meet C
again. [3]

[Turn Over
3

3
y
• A(4, 3)

x
0 2 C(3,0)

B(1, – 1)

The diagram above shows the graph of y = f ( x ) . On separate diagrams, sketch the graphs
of

(i) y = f (1 − 2 x ) , [3]
(ii) y = f ( x ), [2]
(iii) y = 2f ( x ) + 1 . [3]

showing in each case, the coordinates of the points corresponding to A, B, C and the
equations of the asymptotes.

[Turn Over
4

2
 t +1
Show that ∫ 
( ln t ) + C.
4 (a) (i)  ln t dt = t ln t − t + [2]
 t  2

(ii) A curve C is defined by the parametric equations

x = t + ln t
y = t − ln t , t > 0.

The region R, which is bounded by the curve C, the line x = 1 ,


x = 2 + ln 2 and the x – axis is as shown below.
y

R
x
0 1 2 + ln 2

Find the exact area of R. [4]

2
(b) The region S is enclosed by the curve D with equation ( y + 2 ) = 4 − x and the
line y = x .

Find the volume generated when S is rotated through 2π radian about the x – axis,
giving your answer correct to 2 decimal places. [3]

[Turn Over
5

5 A disease is found to be present in a protected reserve containing 35 orangutans. The


rate at which the number of infected orangutans, x, is increasing at any time t is
proportional to the product of the number of infected orangutans and the number that
have yet to be infected at that instant. Initially there were 5 animals infected.

Form a differential equation that describes this model and show that

35 Ae35kt
x= ,
1 + Ae35kt

where A>0 is to be found. [6]

Deduce the total number of infected orangutans after a long period of time and represent
the solution to this model on an appropriate graph. [4]

[Turn Over
6

Section B: Statistics [60 marks]

6 A survey on dining experience was undertaken in a small town with 10 three-star


restaurants, 60 two-star restaurants and 30 one-star restaurants.

The 100 restaurants in the small town are numbered from 1 to 100. A sample of 10
restaurants is selected by randomly choosing 10 numbers that are assigned to the
restaurants.

(i) Suggest one disadvantage of this sampling method. [1]


(ii) Suggest a better method of sampling and explain briefly how this could be done.
[3]

1
7 In a certain country, the probability that it rains on a given Tuesday is . For each of the
5
next 2 days, Wednesday and Thursday, the conditional probability that it rains, given that
it rained the previous day is α and the conditional probability that it rains, given that it
did not rain the previous day is β . The situation is illustrated in the uncompleted tree
diagram below.
R
α
1 R
5 R’
R
β
R’

Tuesday R’
Wednesday
(i) Complete the tree diagram to represent all the possible outcomes up to Thursday.
[2]
1 2
For α = and β = , find
3 3
(ii) the probability that it rains on a Thursday, [2]

(iii) the probability that it rains on at least two days given that it rains on a Thursday.
[3]

[Turn Over
7

8 Paul ordered 9 plates of sushi, namely 3 plates of unagi sushi, 2 plates of uni sushi and 4
plates of ebi sushi.

(i) Find the number of ways Paul can eat the 9 plates of sushi. [2]
(ii) Find the number of ways Paul can eat the 9 plates of sushi such that the first plate
and last plate are different types of sushi. [4]
(iii) Find the number of ways of arranging the nine plates of sushi on a round table
such that no two plates of ebi sushi are placed together. [2]

9 Miss Curious wants to determine if there is any correlation between the amount of
preparation and the results obtained in a recently concluded exam.

She asked her friends how much time they spent preparing for the exam (x), with their
exam scores (y), and recorded her findings in the table below.

x (hour) 10 15 22 27 38 46 53 64
y (score) 11 40 51 56 61 62 64 66

(i) Give a sketch of the scatter diagram for the data and find the equation
of the least squares regression line of y on x. [2]

(ii) State, with a reason, which of the following would be an appropriate model
to represent the above data (where a and b are constants and b > 0).
b
A: y =a+ B : y = a + be− x C : y = a + b ln x [2]
x

(iii) For the appropriate model chosen, find the values of a and b. [1]
Explain how this model is a better one than the equation found in part (i). [1]

(iv) Obtain a good estimate of the score of a student who spent 8 hours
studying for the exam and comment on the reliability of your answer. [2]

[Turn Over
8

10 You are an Intelligence Quotient (IQ) expert. While reading the newspaper, you become
interested in a newspaper advertisement that reads as follows:

Increase the IQ of your children by 10 points in just 16 weeks!



Subscribe now to Dr. Dune’s Drill (DDD) program
and astound your children’s friends, teachers and grandparents!
Assure a university education for your children
(and security for you in your old age).

A scientific study of 15 children from all over Singapore
showed an average IQ score of 108*
after only six weeks of the fantastic DDD program .

*standard deviation is 15

As a concerned IQ expert, you would like to investigate the validity of this


advertisement. You know that for the general population of children, the mean IQ is 100.
Through close contacts in the industry, you confirmed that the scientific study as stated in
the advertisement is valid.

(i) Test at 5% significance level, whether the mean IQ points of children who
participated in the DDD program has increased. State any assumptions that you
have to make in carrying out the test. [6]

(ii) What do you understand by 5% significance level in this context? [1]

Suppose that the population standard deviation σ is now known and a larger sample size
of 50 children is taken.

(iii) Find the range of values of the sample mean in terms of σ if the conclusion at
5% significance level is now different from that concluded in part (i). [4]

[Turn Over
9

11 In a certain country, it is found that on average the number of pairs of twins born weekly
are 2.

(i) Find the most probable number(s) of pairs of twins that are to be born weekly.
[1]

(ii) Find the probability of having at most 7 pairs of twins to be born in a two-week
period. [2]

(iii) Assuming there are 26 two-week periods in a year, estimate the probability that
there are less than 73 two-week periods with at most 7 pairs of twins born in 3
years. [3]

(iv) Using a suitable approximation, find the least number of consecutive weeks such
that the probability of having at most 20 pairs of twins born falls below half. You
may assume that the number of weeks is more than 5. [3]

(v) Find the probability that the mean number of pairs of twins born in 50 weeks is
less than 1.8. [2]

12 An ornithologist, who studies the behavior of birds, captures one male and one female
hornbill from a forest in Osaka, Japan. The masses of hornbills in that forest are assumed
to follow normal distributions with male hornbills having mean 3500g and standard
deviation 150g while female hornbills having mean 3000g and standard deviation σ .

(i) It is found from research that 5% of the female hornbills from the forest have
masses exceeding 3.2kg. Show that σ = 122 . [2]

(ii) Find the probability that the difference in mass between two randomly chosen
male hornbills is at least 0.1kg. [3]

(iii) Find the probability that the mass of 5 randomly chosen female hornbills exceeds
twice the mass of 2 randomly chosen male hornbills. [3]

(iv) Five male hornbills are randomly chosen. Find the probability that the fifth male
hornbill is the third hornbill with mass exceeding 3.6kg. [3]

End of Paper

[Turn Over
10
BLANK PAGE

[Turn Over
1

SERANGOON JUNIOR COLLEGE

2010 JC2 PRELIMINARY EXAMINATION

MATHEMATICS

Higher 2 9740/1

Solutions

1 2x2 + x < 3

−3 < 2 x 2 + x < 3
−3 < 2x 2 + x and 2 x2 + x < 3
2 x2 + x + 3 > 0 and 2 x2 + x − 3 < 0
2
⎛ 1 ⎞ 23
⎜x+ ⎟ + >0 and ( 2 x + 3)( x − 1) < 0
⎝ 4 ⎠ 16
3
x∈ and − < x <1
2
3
∴− < x <1
2

2
1
x ⎛ 1x ⎞ 1
x
2e x
+ e2 < 3 ⇒ 2⎜ e2 ⎟ + e2 < 3
⎜ ⎟
⎝ ⎠
1
x
∴ replace x with e2
1 1
3 x x
− < e2 < 1 ⇒ 0 < e2 < 1
2
1
x < ln1
2
∴x < 0

[Turn Over
2

2(i) Now, − π ≤ sin−1 x ≤ π where x ≤ 1


2 2
dy
Let y = sin−1 x ⇒ sin y = x ⇒ cos y =1
dx

dy 1
⇒ =
dx cos y

dy 1
⇒ =
dx ± 1 − sin 2 y

dy 1
⇒ =
dx ± 1 − x 2

dy 1
⇒ =
dx + 1 − x 2

dy
(since from the graph ≥ 0 for all x ≤ 1 )
dx

1 π
⎡ sin −1 x ⎤ 2 = 4 (cos 2x + 1)dx
(ii)
⎣ ⎦0 ∫a
π
π ⎡1 ⎤
− 0 = ⎢ sin 2x + x ⎥ 4
4 ⎣2 ⎦a

π ⎛1 π⎞ ⎛1 ⎞
= ⎜ + ⎟ − ⎜ sin 2a + a⎟ ⇒ sin 2a + 2a − 1 = 0
4 ⎝ 2 4⎠ ⎝ 2 ⎠

x2 = 9 ( 2 2 −1 ) − x1 = 18 − 7 = 11 = 3 ( 2 2 ) − ( −1)
2

3
x3 = 9 ( 23−1 ) − x2 = 36 − 11 = 25 = 3 ( 23 ) − ( −1)
3

x4 = 9 ( 2 4 −1 ) − x3 = 72 − 25 = 47 = 3 ( 2 4 ) − ( −1)
4

x5 = 9 ( 25 −1 ) − x4 = 144 − 47 = 97 = 3 ( 25 ) − ( −1)
5

OR Using G.C,
x2 = 11 = 3 ( 2 2 ) − ( −1)
2

x3 = 25 = 3 ( 23 ) − ( −1)
3

[Turn Over
3

x4 = 47 = 3 ( 2 4 ) − ( −1)
4

x5 = 97 = 3 ( 25 ) − ( −1)
5

From above,
Conjecture: xn = 3 ( 2 n ) − ( − 1 )
n

Let Pn be the statement that xn = 3 ( 2 n ) − ( −1) for n ∈ +


n

When n = 1,
= x1 = 3 ( 21 ) − ( −1) = 7 = R.H.S
1

L.H.S
Hence P1 is true.

Assume Pk is true for some k ∈ + i.e xk = 3 ( 2 k ) − ( −1)


k

To show that Pk+1 is true, i.e xk +1 = 3 ( 2 k +1 ) − ( − 1)


k +1

So L.H.S = xk +1 = 9 2( ( k +1) −1
) − x( k +1) −1

= 9 ( 2k ) − ⎡3 ( 2k ) − ( −1) ⎤
k
⎣ ⎦

= 9 ( 2 k ) − 3 ( 2 k ) − ( −1)
k +1

= 9 ( 2 k ) − 3 ( 2 k ) − ( −1)
k +1

= 6 ( 2 k ) − ( −1)
k +1

= 3 ( 2 ) ( 2 k ) − ( −1)
k +1

= 3 ( 2 k +1 ) − ( −1)
k +1
= R.H.S
Hence Pk true => Pk+1 is true.

Since P1 is true and Pk=> Pk+1 is also true, by Mathematical Induction, Pn is true for all
positive integer n.

[Turn Over
4

4 e y = 3 e + x + sin x
e3 y = e + x + sin x

Differentiating w.r.t. x,
dy
3e3 y = 1 + cos x
dx

2
d2 y ⎛ dy ⎞
3e 3y
+ 9e3y ⎜ ⎟ + sin x = 0 (shown)
dx 2
⎝ dx ⎠

When x = 0,
1
e3 y = e + x + sin x ⇒ y =
3
dy dy 2
3e3 y = 1 + cos x ⇒ =
dx dx 3e
2 2
d2 y 3 y ⎛ dy ⎞ d2 y ⎛ 2⎞ d2 y 4
3e 3y
+ 9e ⎜ ⎟ + sin x = 0 ⇒ 3e + 9e ⎜ ⎟ = 0 ⇒ =− 2
dx 2
⎝ dx ⎠ dx 2
⎝ 3e ⎠ dx 2
3e

Hence, the Maclaurin series of y is


⎛ 4 ⎞
1 2 ⎜⎝ − 3e2 ⎟⎠
y= + x+ x 2 + ...
3 3e 2!
1 2 2
≈ + x − 2 x2
3 3e 3e

5
(i) Least value of k = 1.5
(ii) let y = 2 x 2 − x − 3

1 25 1 8 y + 25
⇒ y = 2( x − ) 2 − ⇒ x= ±
4 8 4 16

1 8 x + 25
∴ f −1 ( x ) = + , x≥0
4 16

1+ 7
(iii) 2 x2 − x − 3 = x ⇒ x=
2

[Turn Over
5

6(a)
y

A(-1, 0) B(1, 0) x

y= f’(x)

x =0 x =2

2
(b) Area = 2π r + 2π rh (where h is the height of the cylinder)

2100 = 2π r 2 (21) + 2π rh(7)


2100 = 14π r (3r + h)
150
h= − 3r
πr
2 3 2 ⎛ 150 ⎞ 2
V= π r + π r 2h = π r3 + π r 2 ⎜ − 3r ⎟ = π r 3 + 150r − 3π r 3
3 3 ⎝ πr ⎠ 3
7π 3
= 150r − r
3
dV
= 150 − 7π r 2 = 0
dr
150
⇒r= (reject r <0)

d 2V
= − 14π r < 0 ⇒ Volume is max.
dr 2

150
Thus, cost of hemispherical top = 2π r 2 (21) = 2π (21)= $900

[Turn Over
6

7(a) (i) ww∗ + 64 3 i + 16iw = 0

( x + yi )( x − yi ) + 64 3 i + 16i ( x + yi ) = 0

x 2 + y 2 + 64 3 i + 16ix − 16 y = 0

(x 2
) ( )
+ y 2 − 16 y + 16 x + 64 3 i = 0

Comparing coefficients,

(16 x + 64 3 ) = 0 and (x 2
)
+ y 2 − 16 y = 0

x = −4 3 y 2 − 16 y + 48 = 0

( y − 4 )( y − 12 ) = 0
y = 4 since y < 5

∴ w = −4 3 + 4i

5π ⎛ 5nπ 5nπ ⎞
(ii) w = 8 , arg ( w ) = , thus wn = 8n ⎜ cos + i sin ⎟
6 ⎝ 6 6 ⎠

5nπ
( )
Since wn is real, Im wn = 0 ,
6
= kπ , k ∈

6k
⇒n= ,k ∈
5

For n to be an integer, n = 6k , k ∈ .

(b) z − 1 − i = 3 1 + iz

Let z = x + iy , then x + iy − 1 − i = 3 1 + i ( x + iy )

x − 1 + i ( y − 1) = 3 1 + ix − y

( x − 1) + i ( y − 1) = 3 (1 − y ) + ix

( x − 1)2 + ( y − 1)2 = 3 x 2 + (1 − y )
2

(
x2 − 2 x + 1 + y 2 − 2 y + 1 = 9 x2 + 1 − 2 y + y 2 )

[Turn Over
7

8 x 2 + 2 x + 8 y 2 − 16 y = −7

2
⎛ 1⎞ 2 9
⎜ x + ⎟ + ( y − 1) =
⎝ 8⎠ 64

⎛ 1 ⎞ 3
The locus is a circle of centre ⎜ − ,1⎟ of radius units.
⎝ 8 ⎠ y 8
3
1
8

× 1

5
8
x
4 1 0 2
− −
8 8 8

1(1 − 0.9 n )
8 Total upward distance covered by n jumps = =10(1 − 0.9 n )
1 − 0.9
Total sliding distance after n jumps
n
= ⎡⎣ 2(0.3) + (n − 1)(−0.02) ⎤⎦ = 0.3n − 0.01n(n − 1) = −0.01n2 + 0.31n
2

Checking after which jump, the frog will not slide back:
0.3,0.28,0.26,...,0.02,0
U n = 0 ⇒ 0.3 + ( n − 1)( −0.02) = 0 ⇒ n = 16

(i) To clear the 4 m mark,


⎡10(1 − 0.9 n ) ⎤ − ⎡ −0.01n2 + 0.31n ⎤ ≥ 4
⎣ ⎦ ⎣ ⎦
From GC, n ≥ 8.8715
Therefore, 9 jumps are needed.

(ii) After 15 jumps/slides, there will be no more sliding.

[Turn Over
8

Hence, after 15 jumps/slides, distance climbed


= ⎡10(1 − 0.915 ) ⎤ − ⎡ −0.01(15)2 + 0.31(15) ⎤
⎣ ⎦ ⎣ ⎦
= 5.5411m
15
Subsequent jumps will follow GP with first term 0.9 and ratio 0.9.
0.915 (1 − 0.9 n )
Hence ≥ 6.5 − 5.5411
1 − 0.9
From GC, n ≥ 5.95
Therefore, 6+15=21 jumps are needed.

15
(iii) Subsequent jumps will follow GP with first term 0.9 and ratio 0.9. Hence

Highest mark that the frog can reach

0.915
= + 5.5411
1 − 0.9
= 7.60 m

−1
→ ⎛⎜ ⎞⎟ → ⎛⎜ ⎞⎟ → → ⎛⎜ ⎞⎟
1 7
9(i) AB = ⎜ −1⎟ , BC = ⎜ −1 ⎟ ⇒ AB× BC = ⎜ 1 ⎟
⎜4⎟ ⎜ −3 ⎟ ⎜ 2⎟
⎝ ⎠ ⎝ ⎠ ⎝ ⎠

⎛7⎞ ⎛1⎞ ⎛7⎞ ⎛7⎞


Equation of plane π : r ⋅ ⎜ 1 ⎟ = ⎜ 3 ⎟ ⋅ ⎜ 1 ⎟ = 10 ⇒ r ⋅ ⎜⎜ 1 ⎟⎟ = 10
⎜ ⎟ ⎜ ⎟ ⎜ ⎟
⎜ 2⎟ ⎜0⎟ ⎜ 2⎟ ⎜ 2⎟
⎝ ⎠ ⎝ ⎠ ⎝ ⎠ ⎝ ⎠

(ii) Let foot of perpendicular from D be F.

4 + 7μ ⎞
→ ⎛⎜
Now, OF = ⎜ μ ⎟⎟
⎜ 5 + 2μ ⎟
⎝ ⎠

⎛ 4 + 7μ ⎞ ⎛ 7 ⎞
14
Since F is on π , ⎜⎜ μ ⎟⎟ ⋅ ⎜⎜ 1 ⎟⎟ = 10 ⇒ μ = −
⎜ 5 + 2μ ⎟ ⎜ 2 ⎟ 27
⎝ ⎠ ⎝ ⎠

[Turn Over
9

⎛ 10 ⎞
→ ⎜ 27 ⎟
OF = ⎜ − 14
27 ⎟
⎜⎜ 107 ⎟⎟
⎝ 27 ⎠

⎛ 10 ⎞ ⎛ 4 ⎞ ⎛ − 98 ⎞
→ ⎜ 27 ⎟ ⎜ ⎟ ⎜ 27 ⎟ →
DF = ⎜ − 14 − = − 14 ⇒ DF = 392 = 14 6
27 ⎟ 0
⎜ ⎟ ⎜ 27 ⎟
⎜⎜ 107 ⎟⎟ ⎜ 5 ⎟ ⎜⎜ 28 ⎟⎟ 27 9
⎝ 27 ⎠ ⎝ ⎠ −
⎝ 27 ⎠

⎛ 4⎞ ⎛ 0⎞
(iii) Equation of line l : r = ⎜ 0 ⎟ + λ ⎜⎜ 1 ⎟⎟
⎜ ⎟
⎜5⎟ ⎜1⎟
⎝ ⎠ ⎝ ⎠

→ ⎛⎜
4 ⎞
Thus since ON = ⎜ λ ⎟⎟ lies on plane π ,
⎜5 + λ ⎟
⎝ ⎠

⎛ 4 ⎞ ⎛7⎞
⎜ ⎟ ⎜ ⎟ 28
⎜ λ ⋅ 1
⎟ ⎜ ⎟ = 10 ⇒ λ = −
⎜5 + λ ⎟ ⎜ 2⎟ 3
⎝ ⎠ ⎝ ⎠

⎛ 4 ⎞
→ ⎜ ⎟
Therefore ON = ⎜ − 28
3 ⎟
⎜⎜ 13 ⎟⎟
⎝− 3 ⎠

⎛ − 88 ⎞
→ 1⎛ → → ⎞ → ⎜ 27 ⎟
28
Now, OF = ⎜ OD + OD ' ⎟ ⇒ OD ' = ⎜ − 27 ⎟
2⎝ ⎠ ⎜⎜ 79 ⎟⎟
⎝ 27 ⎠

⎛ 4 ⎞ ⎛ −7 ⎞
⎛ −7 ⎞ ⎜ ⎟
→ ⎜ ⎟ = − 28 + β ⎜ 8 ⎟
28
Thus ⇒ ND ' = 27 ⎜ 8 ⎟ ⇒ l ' : r ⎜ 3 ⎟ ⎜ ⎟
⎜ 7 ⎟ ⎜
⎜− ⎟13 ⎟ ⎜7⎟
⎝ ⎠ ⎝ 3⎠ ⎝ ⎠

[Turn Over
10

x 2 + ax + b 81 − 9 a + b
10(a) y=
x+9
(
= x+ a−9 + ) x+9
Equation of asymptotes are: y = x + ( a − 9 ) and x = −9

When x = −9 , y = −12
So a = 6

When y = 0 , x2 + 6 x + b = 0
Since curve cuts the x-axis only once,
Discriminant = 62 − 4b = 0
∴b = 9

y = f ( x)

y=x−3

x
( −3, 0 )
( −9, −12 )

x=−9

For the line y = kx to have no intersection with the transformed curve f ( x + p ) − q , the point of
intersection has to be shifted to the origin. Hence p = −9 and q = −12

[Turn Over
11

(b) y

1
y=
f ( x)

⎛ 1 ⎞ ⎛ 1 ⎞
⎜ −3a, ⎟ ⎜ 5a, ⎟
⎝ 2a ⎠ ⎝ 3a ⎠
x
y=0 0 4a

x = −a x = 2a

(5a, )
3ay 2 = f ( x )

( −3a, 2a )
x
−a 0
y=0 2a
( −3a, − 2a )

(5a, − 3a )
x = 4a

[Turn Over
12

z 5 = 32e(
π + 2 k π )i
11(i) , k = 0, ±1, ±2

⎛ π + 2 kπ ⎞
⎜ ⎟i
z = 2e ⎝ 5 ⎠
, k = 0, ±1, ±2
π 3π π −3π
i i − i i
πi
z = 2e 5 , 2e 5 , 2 e 5
, 2e , 2e
5

w5 = 32 i ⇒ iw = −32 ⇒ ( iw) = −32


5 5
(ii)

Let z = iw ⇒ w = −iz
I

R
2

Area of triangle OW1W2 = 1 × 2 × 2 × sin 2π = 1.90 units2.


2 5

π π 3π 3π
⎛ i ⎞⎛ − i ⎞⎛ i ⎞⎛ − i⎞
⎟ ( z − 2e )
πi
(iii) ⎜ z − 2e ⎟⎜ z − 2e ⎟⎜ z − 2e ⎟⎜ z − 2e
5 5 5 5

⎝ ⎠⎝ ⎠⎝ ⎠⎝ ⎠

⎛ 2 ⎛ π5i πi
− ⎞ ⎞⎛ 2 ⎛ 3π5 i −
3π i
⎞ ⎞
⎜⎜ z − 2 z ⎜ e + e ⎟ + 4 ⎟⎜
5
z − 2z ⎜ e + e 5 ⎟ + 4 ⎟ ( z + 2)
⎟⎜ ⎟
⎝ ⎝ ⎠ ⎠⎝ ⎝ ⎠ ⎠
⎛ 2 π ⎞⎛ 2 3π ⎞
⎜ z − 4 cos z + 4 ⎟ ⎜ z − 4 cos z + 4 ⎟ ( z + 2)
⎝ 5 ⎠⎝ 5 ⎠

π 3π
p = 4,α = ,β = ,k = 2
5 5

End of Paper

[Turn Over
1

SERANGOON JUNIOR COLLEGE

2010 JC2 PRELIMINARY EXAMINATION

MATHEMATICS

Higher 2 9740/2

Solutions

Section A: Pure Mathematics [40 marks]

1
180 x + 230 y + 70 z = 38750
45 x + 50 y + 10 z = 8750
300 x + 400 y = 12 (100 z ) ⇒ 3 x + 4 y − 12 z = 0
Using G.C, x = 100, y = 75 and z = 50
Profit collected by outlet A = $100 ( 75) + $75 (120 ) + $50 ( 20 ) = $17500

2 y
(i)

x
a

(ii) x = a sin2 t, y = a cos t


dx dy
= 2a sin t cos t , = −a sin t
dt dt

dy − a sin t 1
= = − sec t
dx 2a sin t cos t 2

 
π dy
At the point P where t = , = −1
3 dx

[Turn Over
2

3 1
Equation of normal at P ( a, a ):
4 2

y – y1 = m (x – x1)

1 −1 3
y − a = [ x − a]
2 −1 4

1
Equation of normal: y = x− a
  4

(iii) Equation of Curve: x = a sin2 t, y = a cos t

1
Equation of normal: y = x− a
4

Solving, 4a cos t = 4a sin 2 t − a


4cos2 t + 4cos t − 3 = 0
(2cos t + 3)(2cos t − 1) = 0
1 3
cos t = , cos t = − (rejected)
2 2
π
t = (point P)
3

Hence, the normal at P does not meet the curve again.

3 (i)

y
A’(‐3/2, 3)
• 

1  

•  x
C’(-1/2,0) -1/2 1/2
• B’(0, -1)

[Turn Over
3

(ii)
y
A’’(‐4,3) A’(4, 3) 
•  • 

1  

0 •  x
• 
C’’(-3,0) -2 2 C’(3,0)
•  • 
B’’(-1, -1) B’(1, -1)

y
(iii)
A’(4, 7)
• 

3  

1 •  C’(3,1) x
2
B’(1, -1) • 

⎛ t +1 ⎞ ⎛ 1⎞
4(a)(i) ∫ ⎜⎝ t ⎠
⎟ ln t dt = ∫ ⎜1 + ⎟ ln t dt
⎝ t⎠
1
= ∫ ln t dt + ∫ ln t dt
t
( ln t )
2

= t ln t − ∫ 1 dt + +c
2
( ln t )
2

= t ln t − t + +c
2

[Turn Over
4

2 + ln 2
(a)(ii) Area R = ∫ y dx
1

⎛ 1⎞
= ∫ ( t − ln t ) ⎜1 + ⎟ dt
2

1
⎝ t⎠
2⎡ ⎛ t +1 ⎞ ⎤
= ∫ ⎢t + 1 − ⎜ ⎟ ln t ⎥ dt
1
⎣ ⎝ t ⎠ ⎦
2
⎡t2 ⎤ ⎡ ( ln t ) ⎤
2 2

= ⎢ + t ⎥ − ⎢t ln t − t + ⎥
⎣ 2 ⎦1 ⎢⎣ 2 ⎦⎥
1

1 ⎞ ⎡ ( ln 2 ) ⎤
2

= ⎜ 2 + 2 − − 1⎟ − ⎢ 2 ln 2 − 2 + − ( −1) ⎥
⎝ 2 ⎠ ⎢⎣ 2 ⎥⎦
1 ⎡ ( ln 2 ) ⎤
2

= 3 − − ⎢ 2 ln 2 − 2 + − ( −1) ⎥
2 ⎢⎣ 2 ⎥⎦
( ln 2 )
2
7
= − 2 ln 2 − units2
2 2

(b)
y

x
−5 4

(
V = π ∫ ⎡ −2 − 4 − x ) (
− x 2 ⎤ dx + π ∫ ⎡ −2 − 4 − x ) − ( −2 + )
4 − x ⎤ dx
0 2 4 2 2

−5 ⎢
⎣ ⎥⎦ 0 ⎢⎣ ⎥⎦
= 327.25 units3

[Turn Over
5

5 x is the number of infected orangutans at any time t.

dx dx
∝ x (35 − x ) ⇒ = kx (35 − x ), k > 0
dt dt
1
⇒∫ dx = ∫ k dt
x (35 − x )
1 ⎛1 1 ⎞
⇒ ∫ ⎜⎝ +
35 x 35 − x ⎠
⎟ dx = k ∫ dt
⇒ ln x − ln (35 − x ) = 35kt + c
⎛ x ⎞
⇒ ln ⎜ = 35kt + C
⎝ 35 − x ⎟⎠

= e35kt + c = Ae35kt , A > 0


x

35 − x

35 Ae35kt x
⇒x=
1 + Ae35kt 35
1
When t=0, x=5, ⇒ A =
6 5

⎡ 6 ⎤
x = 35 ⎢1 − ⎥
⎣ 6 + e35kt ⎦
0 t
As t → ∞, x → 35

Section B: Statistics [60 marks]

6 (i) As there are only 10 three-star restaurants, there is a high chance that they might not
get selected at all during random sampling.
OR
The numbers of the respective types of restaurants selected might not be representative of
the restaurants in the town.

(ii) Stratified sampling. Set up the 3 respective sub-groups.


- Use population proportions to determine the numbers required for each sub-group:
Three-star(1), two-star(6), one-star(3).
- Randomly select the restaurants based on the respective numbers determined
[Turn Over
6

7(i) α R
α R
R 1−α R’
1 1−α R’ β R
5
1− β R’

4 α R
5 β R
R’ 1−α R’
Tuesday
1− β R’ β R

Wednesday 1 − β R’
Thursday
(ii) P(rains on a Thursday)

1 1 1 1 2 2 4 2 1 4 1 2
= ⋅ ⋅ + ⋅ ⋅ + ⋅ ⋅ + ⋅ ⋅
5 3 3 5 3 3 5 3 3 5 3 3
21 7
= =
45 15

(iii) P(rains on at least 2 days given it rains on a Thursday)

P(A ∩ B)
= P(A\B) =
P(B)

1 1 1 1 2 2 4 2 1 13
⋅ ⋅ + ⋅ ⋅ + ⋅ ⋅
13
= 5 3 3 5 3 3 5 3 3 = 45 =
21 21 21
45 45

9!
8(i) No. of ways = = 1260
2!3!4!
(ii) Case 1:uni and unagi
7!
No. of ways = ( 2 ) = 210
2!4!
Case 2:uni and ebi
7!
No. of ways = ( 2 ) = 280
3!3!

[Turn Over
7

Case 3:ebi and unagi


7!
No. of ways = ( 2 ) = 420
2!2!3!
Total no. of ways = 910

(iii)

- Unagi sushi
- Uni sushi

Since there are only 2 ways of arranging the uni and unagi sushi plates on the round
table,
No. of ways = 5C4 ( 2 ) = 10

9 (i)

Least squares regression line: y = 0.785x + 24.4

(ii) Model C.
Possible reasons:
- Shape of the points follow the shape of an logarithmic graph
- y increases as x increases (other 2 choices have p decreasing as x increases)

(iii) a = - 37.4, b = 26.3


The value of r increases after transformation which indicates there is a better
linear correlation between ln x and y instead of just x and y

[Turn Over
8

(iv) y = a + b ln x
∴ when x = 8, y = 17.31
Extrapolation done in calculating y when x = 8. It might not be reliable as there
may not be a linear relationship between ln x and y outside the range of data

10(i) Assuming that the IQ points of children is normally distributed.

Ho: μ = 100
H1: μ > 100 (test that the average IQ points of students who have participated in the
DDD program has increased)
1-tailed test at 5% significance level.

X−μ
Under Ho , ~ t(n − 1) .
S
n

( sample variance )= (15 )2 =


n 15 3375
Where s 2 = = 241.07143
n −1 14 14

By using GC, ttest = 1.996 and p-value = 0.0230.

Since p-value=0.0230<0.05, we reject Ho and conclude that here, we have sufficient


evidence at 5% significance level that the IQ points of children in the DDD program has
increased.

(ii) 5% significance level means there is a probability of 0.05 that we conclude that the mean
IQ points have increased when in fact it did not (reject Ho when Ho is true).

(iii) Population standard deviation σ is now known and a larger sample size of 50 children is
taken.

⎛ σ2 ⎞
Under Ho , X ~ N ⎜100, ⎟ by Central Limit Theorem (since n=50 is large)
⎜ 50 ⎟⎠

At 5% significance level, Ho is not rejected when ztest < 1.64485 .

x − 100
Thus < 1.64485 ⇒ x < 100 + 0.233σ
σ
50

[Turn Over
9

11(i) Let X be the r.v “number of pairs of twins born weekly”, X Po ( 2 )


Using G.C,when x = 1 and 2, P ( X = 1) = P ( X = 2 ) = 0.27067 give the highest
probability.
Hence the most probable numbers are 1 pair or 2 pairs of twins.

(ii) In a two-week period, X1 + X 2 ~ Po(4)


P ( X1 + X 2 ≤ 7) = 0.948866 ≈ 0.949

(iii) Let Y be the r.v “number of two-week period with more than 7 pairs of twins born in 78
two-week period”
Y B ( 78, 0.051134 )

Since n = 78 is large, np = 78 ( 0.051134 ) = 3.9884 ( < 5 )


So Y Po ( 3.9884 ) approximately
P ( less than 73 two-week periods with at most 7 pairs of twins born )
= P (Y ≥ 6 )
= 1 − P (Y ≤ 5 )
= 0.21306
≈ 0.213

(iv) Let n be the number of consecutive weeks


X 1 + X 2 + ... + X n Po ( 2n )

Since 2n > 10 (Q n > 5 ) , so X 1 + X 2 + ... + X n N ( 2n, 2n ) approximately


P ( X 1 + X 2 + ... + X n ≤ 20 ) < 0.5 ⎯⎯→
C .C
P ( X 1 + X 2 + ... + X n < 20.5 ) < 0.5

Using G.C,when n =10, P ( X 1 + ... + X 10 < 20.5 ) = 0.50997


when n =11, P ( X 1 + ... + X 11 < 20.5 ) = 0.47282

Thus least n is 11

50
∑ Xi ⎛ 2 ⎞
i =1
(v) X= ~ N ⎜ 2, ⎟ approximately, by CLT since n is large.
50 ⎝ 50 ⎠

Thus P ( X < 1.8 ) = 0.15865 = 0.159(3 s.f)

[Turn Over
10

12

(i) Let M denote the r.v of the mass of a male hornbill.


Let F denote the r.v of the mass of a female hornbill.

M (
N 3500,150 2 ) F (
N 3000, σ 2 )
P ( F > 3200 ) = 0.05
P ( F ≤ 3200 ) = 0.95
⎛ 3200 − 3000 ⎞
P⎜ Z ≤ ⎟ = 0.95
⎝ σ ⎠
200
= 1.64485 ⇒ σ = 121.59 = 122 (3s.f.)
σ

(ii) M1 − M 2 ~ N (0, 2 × 150 2 )


P ( M1 − M 2 ≥ 100 )= P (M1 − M 2 ≥ 100 ) + P (M1 − M 2 ≤ −100 )
= 2P ( M1 − M 2 ≤ −100 )
=0.36265=0.363 (3 s.f.)

(iii) Let T = F1 + F2 + F3 + F4 + F5 − 2 ( M 1 + M 2 )
T (
N 5 × 3000 − 4 × 3500, 5 × 121.49 2 + 23 × 150 2 )
P (T > 0 ) ≈ 0.976

(iv) Probability required

4!
= [ P(M > 3600)]3 [ P( M ≤ 3600)]2
2!2!

=0.053967

=0.0540 (3 s.f.)

End of Paper

[Turn Over
TEMASEK JUNIOR COLLEGE, SINGAPORE
Preliminary Examination
Higher 2

MATHEMATICS 9740/01
Paper 1 15 September 2010

Additional Materials: Answer paper 3 hours


List of Formula (MF15)

READ THESE INSTRUCTIONS FIRST

Write your Name and Civics Group on all the work you hand in.
Write in dark blue or black pen on both sides of the paper.
You may use a soft pencil for any diagrams or graphs.
Do not use staples, paper clips, highlighters, glue or correction fluid.

Answer all questions.


Give non-exact numerical answers correct to 3 significant figures, or 1 decimal place in
the case of angles in degrees, unless a different level of accuracy is specified in the
question.
You are expected to use a graphic calculator.
Unsupported answers from a graphic calculator are allowed unless a question
specifically states otherwise.
Where unsupported answers from a graphic calculator are not allowed in a question, you
are required to present the mathematical steps using mathematical notations and not
calculator commands.
You are reminded of the need for clear presentation in your answers.

At the end of the examination, fasten all your work securely together.
The number of marks is given in brackets [ ] at the end of each question or part question.

This document consists of 5 printed pages and 1 blank page.

© TJC 2010 [ Turn over

1
1 The points A, B and C have position vectors given respectively by 2i +3k , 2j − k and
αi − (α – 2)j + (2α – 1)k where α ∈  .

Show that A, B and C are collinear. [3]

Hence or otherwise, find α such that the C divides the line segment AB in the ratio 2:1. [3]

2 In a chemical reaction, a compound A is converted to a compound B at a rate


proportional to square of the amount of A that has yet to be converted. Initially, 120 g of
A is present. After 1 hour, 20 g of A remains. If x denotes the amount of A that is
pt
converted after t hours, show that x = , where p is an integer to be determined. [6]
5t + 1

3 The complex number z satisfies the equation

z 3 + az 2 − az − 1 =0 , where a ∈  .

(i) Verify that z = 1 is a root of the equation. [1]

(ii) Given that one of the complex roots is 2i, find a. [2]

(iii) Find the third root of the equation. [3]

4 A rectangular cuboid ABCDEFGH has a constant height of 2 cm and a base length of x


cm which increases with time t (measured in seconds). The cuboid, which is initially a
cube, is expanding at a constant rate of 104 cm3s−1.
E F

H G

D C
x
A x B

(i) Show that =


AF 2x2 + 4 . [1]

(ii) Find the rate of increase of the distance AF when t = 3. [6]

2
5 The functions f, g and h are defined by

f : x  x2 + 4 x ≥ −2 , x ∈  ,
g : x  ln ( − x ) x < 0, x∈ ,
h : x  x − 4 +λ x > 4, x∈ .

(i) Given that the composite function fh ( x ) exists, find the least value of λ . [3]

(ii) Find g −1 ( x ) and state the domain of g −1 . [2]

(iii) On a clearly labeled diagram, sketch the graphs of y = g ( x ) , y = g −1 ( x ) and


y = g −1g ( x ) . [2]

6 It is given that for all x ∈  ,

f ( x ) = x2 + 4x + 6 ,
g ( x ) = x 2 − x − 2 and
h ( x ) = ax3 + bx 2 + c .

f ( x)
(a) Solve algebraically the inequality ≤ 1. [3]
g ( x)
f ( e− x )
Hence solve the inequality ≤ 1. [2]
g ( e− x )
(b) =
If y h ( x ) + f ( x ) passes through the points ( −1, −18 ) , (1, −14 ) and ( 3,30 ) ,
evaluate h (101) . [4]

n
2r − 1
7 It is given that S n = ∑ r
.
r =1 2

 a
(i) Express S1, S2, S3 and S4 in the form  3 −  , where a, b ∈  . Hence obtain a
 b
conjecture for Sn in terms of n. [3]

(ii) Prove your conjecture using mathematical induction. [4]

N
2r + 1
(iii) Hence find ∑
r= 0 2 r +1
in terms of N. [2]

3
5

8 (i) Expand (1 − 2x ) 2 as a series of ascending powers of x up to and including the
term in x 2 . State the range of x for which the expansion is valid. [3]

3
− dy
(ii) Given that y= (1 − 2 x ) 2 , show that (1 − 2 x )
=3y .
dx [1]
By repeated differentiation of this result, show that the Maclaurin’s series
expansion of y in ascending powers of x, up to and including the term in x3 , is

1 + 3x + px 2 + qx 3 ,

where the numerical values of p and q are to be determined.


[4]
(iii) Explain briefly how the result in (i) can be used as a check on the correctness of
your answer in (ii).
[2]
9 (a) A geometric sequence { xn } has first term a and common ratio r. The sequence of
numbers { yn } satisfies the relation yn = log 3 ( xn ) for n ∈  + .
20
(i) If the product of x5 and x16 is 81, find the value of ∑ log3 xk . [4]
k =1
(ii) Show that { yn } is an arithmetic sequence. [2]

(b) The output of a coal mine in any year is 10% less than in the preceding year. Prove
that the output of the coal mined cannot exceed ten times the output in the first year. [2]

It is decided to close the mine when the total output exceeds nine times the output in
the first year. Determine the maximum number of years the mine will be in
operation, and give your answers correct to the nearest year. [3]

10 The plane p1 has equation x + y − 3 z = 6 and the point A has position vector i + 2k.
Given that the point B is the foot of perpendicular from A to p1, find the position vector
of B. [5]
1 0
   
Another plane p2 has equation r= λ  0  + µ  −1 , λ , µ ∈  . Given that p2 intersects
1 1
   
p1 at the line l, find the vector equation of l and show that the shortest distance from A to
77
l is . [6]
6

A plane p3 passes through the points A and B. Given that p1, p2 and p3 do not have a
common point, find the equation of p3. [3]

4
11 (a) Find ∫ sec x cosec x dx . [2]

1
(b) By using the substitution u = ln x , find ∫ ln ( ln x ) dx . [4]
x

(c) The diagram shows the region R bounded by the circle x 2 + y 2 =


4 , the curve
1
y = and the line x = −1 .
x

1
(i) The curve y = intersects the circle at a point A, and the line x = −1 cuts the
x
circle at the point B. Find the coordinates of the points A and B. [2]

(ii) Calculate the area of the region R. [3]

(iii) Find the volume of the solid formed when R is rotated through 4 right angles
about the y –axis. [4]

END OF PAPER

5
TEMASEK JUNIOR COLLEGE, SINGAPORE
Preliminary Examination
Higher 2

MATHEMATICS 9740/02
Paper 2 20 September 2010

Additional Materials: Answer paper 3 hours


Graph paper
List of Formula (MF15)

READ THESE INSTRUCTIONS FIRST

Write your Name and Civics Group on all the work you hand in.
Write in dark blue or black pen on both sides of the paper.
You may use a soft pencil for any diagrams or graphs.
Do not use staples, paper clips, highlighters, glue or correction fluid.

Answer all questions.


Give non-exact numerical answers correct to 3 significant figures, or 1 decimal place in
the case of angles in degrees, unless a different level of accuracy is specified in the
question.
You are expected to use a graphic calculator.
Unsupported answers from a graphic calculator are allowed unless a question
specifically states otherwise.
Where unsupported answers from a graphic calculator are not allowed in a question, you
are required to present the mathematical steps using mathematical notations and not
calculator commands.
You are reminded of the need for clear presentation in your answers.

At the end of the examination, fasten all your work securely together.
The number of marks is given in brackets [ ] at the end of each question or part question.

This document consists of 6 printed pages.

© TJC 2010 [ Turn over


1
Section A: Pure Mathematics [40 marks]

1 A curve C is represented by the parametric equations


x= t 2 − 2t , y= t 3 − 9t for t ≤ 2 .
Find the equation of the tangent to the curve C which is parallel to the y-axis. [4]

The figure below shows the region R bounded by the curve C and the x-axis.

Show that the curve C intersects the x-axis at the point (15, 0). [1]

Hence find the exact value of the area of R. [4]

2 By sketching the graphs of y = e x and =


y 2 x + 2 on the same diagram, find the number
of roots of the equation e − 2 x − 2 =
x
0. [2]

(i) Given that the root β lies in the interval (a, 0), state the largest integer value of a. [1]

A sequence of negative numbers, xn , satisfy the relation


1 xn
=
xn + 1 (e − 2) , for n ∈  + .
2
(ii) If the sequence converges, show that it converges to β . [2]

It is given that β < xn < 0 .

(iii) By considering xn +1 − xn , and with the aid of the sketch in (i), show that
xn +1 < xn . [2]

(iv) Show that xn +1 > β . [2]

(v) Describe the behaviour of the sequence for the case when β < x1 < 0 . [1]
2
3
 z  3+i
3 (a) Find the roots of the equation   + =
0. [4]
 3+i z

(b) A complex number z satisfies iz + 3 =


5.
(i) Sketch the locus of the points which represents z in an Argand diagram. [2]

(ii) Find z when z + 3 is a minimum. [4]

4 (a)
y =
y 6x + 4

y = f ( x)

−2 1 x

x = −1

The diagram shows the graph of y = f ( x ) with asymptotes x = −1 and =


y 6x + 4 .
On separate diagrams sketch the graphs of

(i) y = f ′( x) [2]

(ii) y2 = f ( x) . [2]

− x2 − 2x + 5
(b) The curve C has equation y = .
x −1
2
(i) Describe how the curve with equation y = x − 4 − can be transformed to C. [3]
x
(ii) State the equations of all the asymptotes of C and hence sketch C. [2]
(You are not required to find the x and y intercepts.)

(iii) By adding a suitable graph to the sketch in (ii), deduce the number of roots of
( x − 1)
2 2
 − x2 − 2 x + 5 
the equation + + 4 =
1. [2]
22  x −1 

3
Section B: Statistics [60 marks]

5 Through-Train Junior College (TTJC) has four levels of students: IP1, IP2, JC1 and JC2.
The population breakdown for the year 2010 is as follows:
Level IP1 IP2 JC1 JC2
Number of students 78 70 692 760
Mr Koh wants to find out how students spend their time after lessons. He plans to get
10% of the student population to do a survey. His method of conducting the survey is to
spend one hour each day at the canteen and ask if the students entering the canteen are
willing to do a survey for him. He continues to do that until he obtains the number of
surveys required.

(i) State a flaw in his sampling method. [1]

Mr Koh’s friend, Mr Toh, suggests he modify his sampling method to collect 40 surveys
from each level instead.

(ii) State the name of this sampling method. [1]

Mr Koh finally decided to obtain a stratified sample across different levels of students.

(iii) Describe how a sample of size 160 might be obtained. [2]

6 The time taken, in minutes, for John, a typist, to type 5000 words is normally distributed with
mean 34 and standard deviation 2.1.
John attends a training course to improve his typing speed. After the course, John records the
times he take to type 5000 words on 8 separate occasions, and obtains the following times (in
minutes):

36.5 33.4 27.7 31.5 33.0 34.9 30.7 29.5

Assuming that the initial standard deviation has not changed, test at the 2% level of
significance whether John’s typing speed has improved after the training course. [4]

Explain what is meant by 2% level of significance in the context of the question. [1]

4
7 An Extreme Ironing Club consists of 11 members: 6 male and 5 female. It is also known
that 4 of the male members are local while 3 of the female members are foreigners.

(a) Find the number of ways for all 11 members to stand in a row so that no two
foreigners are adjacent. [2]

(b) 4 members are to be chosen to enter a competition. In how many ways can this be
done if:

(i) there are no restrictions, [1]

(ii) there must be at least 1 male member and at least 1 female member, [2]

(iii) there must be at least 1 female member and at least 1 foreign member
(a female member who is a foreigner will satisfy both conditions). [3]

8 Temasek United Football Club (TUFC) is participating in a football competition. 3 points


are awarded for a win, 1 point is awarded for a draw and no points are awarded for a loss.
In this competition, TUFC plays three matches in the first round. Being a good team, it
may be assumed that for any match in the first round, the probability that TUFC wins is
1
and the probability that TUFC loses is 1 . All the matches are played independently.
2 5

(i) Find the probability that TUFC draws a match. [1]

(ii) Find the probability that after three games, TUFC has exactly three points. [3]

(iii) To qualify for the second round, TUFC needs to get at least five points. Given that
TUFC does not win the first game, find the probability that they will still qualify for
the second round. [4]

9 The times taken, in seconds, for two swimmers, A and B, to complete a 100-metre
freestyle race are independent and normally distributed with means 48.0 and 47.2 and
standard deviations 0.5 and 0.8 respectively. The two swimmers compete in a 100-
metre race for which the world record is 46.9 seconds.

(i) Show that the probability that at least one of the two swimmers breaks the world
record during the race is 0.363 correct to 3 significant figures. [3]

(ii) Find the probability of Swimmer B beating Swimmer A. [2]

(iii) If A and B are to meet 20 times for the 100m freestyle race, how many times do
you expect A to beat B? Give your answer correct to the nearest integer. [2]

(iv) Find the probability that the total sum of four randomly chosen timings of A is
more than 4 times a randomly chosen timing of B. [3]

5
10 At a stall in a fun-fair, games of chance are played. The probability that a participant
wins a prize in each game is 0.05.

(a) If the stall holder wants the probability of more than 5 prizes to be won on a
particular day to be less than 0.1, find the largest number of games that can be
played on that day. [3]

(b) On another day, N games are played. Using a suitable approximation, find the
probability that prizes are won in at most 5% of the games if :
(i) N = 60, [3]
(ii) N = 200. [3]

(c) The stall is open for 70 days and on each day, 60 games are played. Find the
probability that the average number of prizes won each day is between 2.5 and 3
inclusive. [4]

11 Mary recorded the length of time, y minutes, taken to travel to her office when leaving
home x minutes after 7 am on nine selected mornings. The results are as follows.

x 0 5 10 20 25 30 40 50 60
y 20 23 29 33 35 39 40 48 51

(i) State, giving a reason, which of the least squares regression lines, x on y or y on x,
should be used to express possible relation between x and y. [1]

(ii) Find the equation of the regression line chosen in part (i) and interpret the slope of
your regression line. [3]

(iii) Calculate the product moment correlation coefficient and interpret your value in
the context of this question. [2]

Mary needs to arrive at her office no later than 8.30 am. The number of minutes by
which Mary arrives at her office early, when leaving home x minutes after 7 am, is
denoted by z.

(iv) Write z in terms of x and y. [1]

(v) Estimate, to the nearest minute, the latest time that Mary can leave home without
arriving late at office. [3]
Comment on the reliability of this value. [2]

6
TEMASEK JUNIOR COLLEGE, SINGAPORE
Preliminary Examination
Higher 2

MATHEMATICS 9740/01
Paper 1 15 September 2010

Solutions
1 ⎛ 0 ⎞ ⎛ 2 ⎞ ⎛ −2 ⎞ ⎛1⎞
uuur ⎜ ⎟ ⎜ ⎟ ⎜ ⎟ ⎜ ⎟
AB = ⎜ 2 ⎟ − ⎜ 0 ⎟ = ⎜ 2 ⎟ = −2 ⎜ −1⎟
⎜ −1⎟ ⎜ 3 ⎟ ⎜ −4 ⎟ ⎜2⎟
⎝ ⎠ ⎝ ⎠ ⎝ ⎠ ⎝ ⎠
⎛ α ⎞ ⎛ 2⎞ ⎛ α − 2 ⎞ ⎛1⎞
uuur ⎜ ⎟ ⎜ ⎟ ⎜ ⎟ ⎜ ⎟
AC = ⎜ −α + 2 ⎟ − ⎜ 0 ⎟ = ⎜ −α + 2 ⎟ = (α − 2 ) ⎜ −1⎟
⎜ 2α − 1 ⎟ ⎜ 3 ⎟ ⎜ 2α − 4 ⎟ ⎜2⎟
⎝ ⎠ ⎝ ⎠ ⎝ ⎠ ⎝ ⎠
Therefore AB is parallel to AC and since A is a common point, we have A, B and C
collinear.

From above, we have


uuur 2 − α uuur
AC = AB
2
uuur 2 uuur
Since C divides the line segment AB in the ratio 2:1, AC = AB .
3
2 −α 2 2
∴ = ⇒α =
2 3 3
Alternative 1for last part:
Since C divides the line segment AB in the ratio 2:1, we have AB:BC = 2:1
uuur uuur ⎛ α ⎞ ⎡⎛ 2 ⎞ ⎛ 0 ⎞ ⎤
uuur OA + 2OB ⎜ ⎟ 1 ⎢⎜ ⎟ ⎜ ⎟ ⎥ 2
∴ OC = ⇒ ⎜ 2 − α ⎟ = ⎢⎜ 0 ⎟ + 2 ⎜ 2 ⎟ ⎥ ⇒ α =
3 ⎜ 2α − 1⎟ 3 ⎢⎜ 3 ⎟ ⎜ −1⎟ ⎥ 3
⎝ ⎠ ⎣⎝ ⎠ ⎝ ⎠ ⎦

2 dx
= k (120 − x )
2

dt

∫ (120 − x )
−2
dx = ∫ k dt

1
(120 − x )
−1
= kt + c
1
When t = 0, (120 − 0 ) = k ( 0 ) + c ⇒ c =
−1

120
1 1
When t =1, ( 20 ) = k (1) +
−1
⇒ k=
120 24

1 1 1 5t + 1
= t+ =
120 − x 24 120 120

600t
x=
5t + 1

So p = 600 .

3 (i) When z = 1,

LHS = (1) + a (1) − a (1) − 1 = 0 = RHS


3 2

Hence z = 1 is a root of the equation.

(ii) Since 2i is a root,

∴ ( 2i ) + a ( 2i ) − a ( 2i ) − 1 = 0
3 2

−8i − 4a − 2ai − 1 = 0

a ( −4 − 2i ) = 1 + 8i

1 + 8i 3
a= = −1 − i
−4 − 2i 2

(iii) Let z = b be the other root.

( z − b )( z − 1)( z − 2i ) = z 3 + az 2 − az − 1
Comparing the constant term, we have

1
−2bi = −1 ⇒ b = − i
2

4
(i) Clearly AF = AB 2 + BC 2 + CF 2

= 2 x2 + 4

2
(ii) Let V = 2 x 2

dV dx dx 26
⇒ = 4x ⇒ = - - - - - - (1)
dt dt dt x

Let y = AF .
dy 2x
Differentiate (i) wrt x : = - - - - - - (2)
dx 2 x2 + 4

When t = 3 , V = 23 + 104(3) = 320

⇒ x = 4 10 or x = −4 10 (NA since x > 0)

dy dy dx 52 26
So = ⋅ = =
dt dx dt 18 9
26
Rate of increase of AF at t = 3 is cms−1.
9

Alternative solution:
dV
= 104
dt
⇒ V = 104t + c

When t =0, V = 8. This gives us c = 8.


V = 104t + 8.

Volume of cuboid = 2x 2 .
From (i), let AF = y = 2x 2 + 4 = V + 4

( )
1
dy 1 −
⇒ = V +4 2 .
dV 2
dy ⎛ dy ⎞ ⎛ dV ⎞ 104 52
=⎜ ⎟ ⎜ ⎟ = = .
dt ⎝ dV ⎠ ⎝ dt ⎠ 2 V + 4 V +4

When t = 3, V = 104(3) + 8 = 320.


dy 52 26
Therefore, = = cm s-1.
dt 320 + 4 9

3
5 (i) fh ( x ) exist ⇒ Rh ⊆ Df ⇒ ( λ , ∞ ) ⊆ [ −2, ∞ ) ⇒ least λ = −2

(ii) For g −1 ( x ) ,
Let y = ln ( − x )
⇒ − x = e y ⇒ x = −e y
⇒ g −1 ( x ) = −e x x∈
y
(iii)
y = g ( x)

y =0
x

y = g −1 ( x )
y = g g ( x)
−1

x =0

6 f ( x) x2 + 4x + 6 ( 5x + 8) ≤ 0
(a) ≤1⇒ 2 −1 ≤ 0 ⇒
g ( x) x − x−2 ( x − 2 )( x + 1)
⇒ ( 5 x + 8 )( x − 2 )( x + 1) ≤ 0 [ x ≠ −1, 2]
8
⇒x≤− or − 1 < x < 2
5
f ( e− x ) 8
≤ 1 ⇒ e− x ≤ − or − 1 < e− x < 2
g (e −x
) 5
⇒ 0 < e− x < 2
1
⇒ x > ln
2

(b) y = h ( x) + f ( x)
⇒ y = ax3 + bx 2 + c + x 2 + 4 x + 6 = ax 3 + ( b + 1) x 2 + 4 x + ( c + 6 )

Since it passes through the points ( −1, −18 ) , (1, −14 ) and ( 3,30 )
a ( −1) + ( b + 1)( −1) + 4 ( −1) + ( c + 6 ) = −18
3 2

⇒ −a + b + c = −21 − − − (1)
a (1) + ( b + 1)(1) + 4 (1) + ( c + 6 ) = −14
3 2

⇒ a + b + c = −25 − − − (2)
a ( 3) + ( b + 1)( 3) + 4 ( 3) + ( c + 6 ) = 30
3 2

⇒ 27 a + 9b + c = 3 − − − (3)
Solving (1),(2) and (3) gives a = −2, b = 10, c = −33
h ( x ) = −2 x 3 + 10 x 2 − 33 ⇒ h (101) = −1958625

4
7
(i) S1 = 1 = 3 − 5 , S2 = 5 = 3 − 7 , S3 = 15 = 3 − 9 and S4 = 37 = 3 − 11 .
2 2 4 4 8 8 16 16
Conjecture for S n = 3 − 2n +
n
3.
2

(ii) Let Pn be the statement “ S n = 3 − 2n + 3 ” , for all n ∈ + .


2n
2(1) + 3 1
When n = 1, LHS of P1 = S1 = 1 , RHS of P1 = 3 − = . ∴ P1 is true.
2 21 2

Assume that Pk is true for some k ∈ . i.e. S k = 3 − 2k +


+
k
3.
2
k +1
2r − 1
When n = k + 1, LHS of Sk +1 = ∑ r
r =1 2
k
2r − 1 2 ( k + 1) − 1
= ∑
r =1 2r
+
2 k +1
2 k + 3 2k + 1
= 3− + k +1
2k 2
4 k + 6 − 2k − 1
= 3−
2 k +1
2 ( k + 1) + 3
= 3− = RHS of Sk +1
2 k +1
Hence, Pk is true ⇒ Pk +1 is true and since P1 is true, by mathematical induction,
+
Pn is true for all n ∈ .

N
2r + 1 N +1 2r − 1 2 ( N + 1) + 3 2N + 5
(iii) ∑ r +1 = ∑
r= 0 2 r=1 2
r
= 3−
2 N +1
= 3 − N +1 .
2
8 ⎛ 5 ⎞⎛ 7 ⎞
⎜ − ⎟⎜ − ⎟
⎛ 5⎞
(1 − 2 x ) = 1 + ⎜ − ⎟ ( −2 x ) + ⎝ 2 ⎠ ⎝ 2 ⎠ ( −2 x ) + ... ≈ 1 + 5 x + x 2
5
− 2 35
(i) 2
⎝ 2⎠ 2! 2
1 1
Expansion is valid if − < x <
2 2
3 5 5
dy 3
y = (1 − 2 x ) = − (1 − 2 x ) 2 ( −2 ) = 3 (1 − 2 x ) 2 = 3 y (1 − 2 x )
− − − −1
(ii) 2 ⇒
dx 2

dy
⇒ (1 − 2 x ) = 3 y − − − (1)
dx

Differentiate (1) wrt x,

5
dy d2y dy d2y dy
⇒ −2 + (1 − 2 x ) 2 = 3 ⇒ (1 − 2 x ) 2 = 5 − − − (2)
dx dx dx dx dx

Differentiate (2) wrt x,

d2y d3y d2y d3y d2y


⇒ −2 2 + (1 − 2 x ) 3 = 5 2 ⇒ (1 − 2 x ) 3 = 7 2 − − − (3)
dx dx dx dx dx

dy d2y d3y
When x = 0, y = 1, = 3, 2 = 15, 3 = 105
dx dx dx

Hence,

15 2 105 3 15 35
y ≈ 1 + 3x + x + x = 1 + 3x + x 2 + x3
2! 3! 2 2

15 35
p= ,q =
2 2
3
15 2 35 3
(iii) From (ii), (1 − 2 x )

2 ≈ 1 + 3x + x + x
2 2

⎛ 3⎞
Diff wrt x, ⎜ − ⎟ (1 − 2 x ) 2 ( −2 ) ≈ 3 + ( 2 x ) + ( 3 x 2 )
5
− 15 35
⎝ 2⎠ 2 2

5
35 2
⇒ (1 − 2 x )

2 ≈ 1 + 5x + x same as (i)
2
5
35 2 315 3
Alternative 1: Now (1 − 2 x )

2 = 1 + 5x + x + x + ...
2 8
3 5
⇒ (1 − 2 x ) = (1 − 2 x ) (1 − 2 x )
− −
2 2

⎛ 35 315 3 ⎞
= ⎜1 + 5 x + x 2 + x + ... ⎟ (1 − 2 x )
⎝ 2 8 ⎠
15 35
= 1 + 3x + x 2 + x3 + ... same as (ii)
2 2
5 3
Alternative 2: (1 − 2 x ) = (1 − 2 x ) (1 − 2 x )
− − −1
2 2

⎛ ⎞
= ⎜ 1 + 3x + x 2 + x3 + ... ⎟ (1 + 2 x + 4 x 2 + 8 x3 + ...)
15 35
⎝ 2 2 ⎠
35 2
= 1 + 5 x + x + ... same as (i)
2

6
9 (a)(i) x5 ( x16 ) = 81 ⇒ ar 4 ( ar 15 ) = 81 ⇒ a 2 r 19 = 81

20
∑ log3 xk = log 3 x1 + log 3 x2 + log 3 x3 + ... + log 3 x20 = log 3 ( x1 x2 x3 ... x20 )
k =1

(
= log 3 a ( ar ) ( ar 2 ) ... ( ar19 ) )
= log 3 ( a 20 r1+ 2+...+19 )

19( 20 )
⎛ ⎞
= log 3 ⎜ a 20 r 2 ⎟
⎜ ⎟
⎝ ⎠

= log 3 ( a 2 r19 )
10

= log 3 ( 81) = 10(4) = 40


10

xn
(ii) yn − yn −1 = log 3 ( xn ) − log 3 ( xn −1 ) = log 3
xn −1

ar n −1
= log 3 = log 3 r is a constant free from n.
ar n − 2

Hence, { yn } is an arithmetic sequence.

(b) Let the amount of coal mined in the first year be a.

The maximum total amount of coal mined = a + 0.9a + 0.92 a + 0.93 a + ....

= a = 10a
1 − 0.9
Let n be the number of year at which the mine will be in operation.
a(1 − 0.9n )
> 9a
1 − 0.9
⇒ 0.9n < 0.1
lg 0.1
⇒ n> ≈ 21.854
lg 0.9
The mine will be in operation for at most 22 years.

7
10 ⎛1⎞
The vector equation of p1 is r ⎜⎜ 1 ⎟⎟ = 6 ---(1)
⎜ −3 ⎟
⎝ ⎠

The vector equation of the line passing thru A and perpendicular to p1 is


⎛1⎞ ⎛1⎞
⎜ ⎟ ⎜ ⎟
r = ⎜0⎟ + λ ⎜ 1 ⎟, λ ∈ ----(2)
⎜ 2⎟ ⎜ −3 ⎟
⎝ ⎠ ⎝ ⎠

Sub (2) into (1):

⎛ 1 + λ ⎞⎛ 1 ⎞
⎜ ⎟⎜ ⎟
⎜ λ ⎟⎜ 1 ⎟ = 6
⎜ 2 − 3λ ⎟⎜ −3 ⎟
⎝ ⎠⎝ ⎠

1 + λ + λ − 6 + 9λ = 6 ⇒ λ = 1

⎛1⎞ ⎛ 1 ⎞ ⎛ 2 ⎞
uuur ⎜ ⎟ ⎜ ⎟ ⎜ ⎟
Therefore OB = ⎜ 0 ⎟ + ⎜ 1 ⎟ = ⎜ 1 ⎟
⎜ 2 ⎟ ⎜ −3 ⎟ ⎜ −1⎟
⎝ ⎠ ⎝ ⎠ ⎝ ⎠

⎛1⎞ ⎛ 0 ⎞ ⎛ 1 ⎞
⎜ ⎟ ⎜ ⎟ ⎜ ⎟
Q ⎜ 0 ⎟ × ⎜ −1⎟ = ⎜ −1⎟ and origin is in p2,
⎜ 1 ⎟ ⎜ 1 ⎟ ⎜ −1⎟
⎝ ⎠ ⎝ ⎠ ⎝ ⎠

Therefore p2 has Cartesian equation x − y − z = 0 ---(3)

And p1 has Cartesion equation x + y − 3 z = 6 ---(4)

Using GC to solve (3) and (4):

We have x = 3+2λ, y = 3+λ, z = λ.

⎛ 3⎞ ⎛ 2⎞
Therefore the equation of l is r = ⎜ 3 ⎟ + λ ⎜⎜ 1 ⎟⎟ , λ ∈
⎜ ⎟
⎜0⎟ ⎜1⎟
⎝ ⎠ ⎝ ⎠

8
⎛⎛ 3⎞ ⎛ 1⎞⎞ ⎛ 2⎞
⎜⎜ ⎟ ⎜ ⎟⎟ ⎜ ⎟
⎜⎜ 3⎟ − ⎜ 0⎟⎟×⎜ 1⎟
⎜⎜0⎟ ⎜ 2⎟⎟ ⎜1⎟
⎝⎝ ⎠ ⎝ ⎠⎠ ⎝ ⎠
Shortest distance =
22 + 12 + 12

⎛ 2 ⎞ ⎛ 2⎞ ⎛ 5⎞
⎜ ⎟ ⎜ ⎟ ⎜ ⎟
⎜ 3 ⎟×⎜1⎟ ⎜ −6 ⎟
⎜ −2 ⎟ ⎜ 1 ⎟ ⎜ −4 ⎟
⎝ ⎠ ⎝ ⎠ ⎝ ⎠
= =
6 6

5 2 + ( −6 ) + ( − 4 )
2 2
77
= =
6 6

⎛1⎞
⎜ ⎟
Since p3 passes through the points A and B, therefore p3 // ⎜ 1 ⎟ .
⎜ ⎟
⎝ −3 ⎠
⎛ 2⎞
Since p1, p2 and p3 do not have a common point, therefore p3 // ⎜⎜ 1 ⎟⎟ .
⎜1⎟
⎝ ⎠
⎛1⎞ ⎛1⎞ ⎛ 2⎞
An equation of p3 is r = ⎜ 0 ⎟ + α ⎜ 1 ⎟ + β ⎜⎜ 1 ⎟⎟ , α , β ∈
⎜ ⎟ ⎜ ⎟
.
⎜ 2⎟ ⎜ −3 ⎟ ⎜1⎟
⎝ ⎠ ⎝ ⎠ ⎝ ⎠

1
11 (a) ∫ sec x cosec x dx = 2∫ 2sin x cos x dx = 2∫ cosec 2 x dx

= − ln ( cot 2 x + cosec2 x ) + c
1 sec 2 x
Alternative 1: ∫ sec x cosec x dx = ∫ sin x cos x dx = ∫ tan x dx = ln | tan x | +c
sin 2 x + cos 2 x
Alternative 2: ∫ sec x cosec x dx = ∫ sin x cos x
dx = ∫ tan x dx + ∫ cot x dx

= ln | sec x | + ln | sin x | +c

du 1
(b) Let u = ln x ⇒ =
dx x

9
1
∫ x ln ( ln x ) dx = ∫ ln u du
1
= u ln u − ∫ u dx = u ln u − u + c
u
= ( ln x ) ( ln ( ln x ) − 1) + c

(c) (i) Solving x 2 + y 2 = 4 and y =


1
x (
⇒ A is − 2 + 3 , − 2 − 3 ) [2]

or (−1.931852,−0.5176381) from GC.


[3]
Solving x 2 + y 2 = 4 and x = −1 ⇒ B is (−1,− 3 ) or (−1,−1.73205).
[4]

− 2+ 3 2
− 2+ 3

−1

− 3

−1 −1
1
(ii) Area = ∫ 4 − x 2 dx − ∫ x
dx
− 2+ 3 − 2+ 3

= 0.546 [From GC]


− 2− 3 − 2− 3
1
(iv) Volume = π ∫ 4 − y d y − π (1) ( 3 − 1) − π ∫ = 4.74
2 2
dy
− 3 −1
y2

[From GC]

END OF PAPER

10
TEMASEK JUNIOR COLLEGE, SINGAPORE
Preliminary Examination
Higher 2

MATHEMATICS 9740/02
Paper 2 20 September 2010

Solutions
Section A: Pure Mathematics [40 marks]

dy dy dx 3t 2 − 9
1 = / =
dx dt dt 2t − 2

For tangent parallel to the y-axis, 2t − 2 = 0 ⇒ t = 1


Equation of tangent is x = −1

When y = 0, t 3 − 9t = 0 ⇒ (t + 3)t (t − 3) = 0
∴ t = −3 , 0 or 3 [NA since t ≤ 2 ]
When t = −3 ⇒ x = ( −3) − 2 ( −3) = 15,
2
y=0
∴ C intersects the x-axis at the point (15, 0).
15
Area of R = ∫ y dx
0
−3
= ∫ (t − 9t ) ( 2t − 2 ) dt
3

0
0
= − ∫ ( 2t 4 − 2t 3 − 18t 2 + 18t ) dt
−3
0
⎡ 2t 5 t 4 ⎤
= −⎢ − − 6t 3 + 9t 2 ⎥ = 105.3
⎣ 5 2 ⎦ −3

2 ex − 2x − 2 = 0 ⇒ ex = 2x + 2
From the diagram, there are two intersection points.
∴ e x − 2 x − 2 = 0 has two roots.

1
(i) Since graph of y = 2 x + 2 cuts x-axis at x = − 1,
β ∈ ( −1, 0 ) , ∴ a = −1 .
1
Given that xn + 1 = (e xn − 2) , xn < 0 , for n ∈ + .
2
(ii) If the sequence converges to a number L, then when n → ∞ , xn → L , xn+1 → L .
1
⇒ L = (e L − 2)
2
L
⇒ e − 2L − 2 = 0
∴ L = β as xn < 0 , for all n ∈ + .

(iii) If β < xn < 0 , (iv) xn > β


1
xn +1 − xn = (e xn − 2) − xn ⇒ e xn > e β as y = e x is an ↑ function.
2
⇒ e xn − 2 > e β − 2
1
(
= e xn − 2 xn − 2 )
2 ⇒
1 xn
2
e −2 > ( 1 β
2
e −2 ) ( )
1
2
(
= e xn − ( 2 xn + 2 ) ) 1
⇒ xn +1 > ( 2 β ) from (ii)
<0 2
⇒ xn +1 > β
as e xn < 2 xn + 2 from the above diagram
Hence, xn +1 < xn .

From (iii) and (iv), 0 > x1 > x2 > x3 > .... > β . It is a decreasing sequence which converges to β .

3 3
⎛ z ⎞ 3+i ⎛ z ⎞ 3+i
3 (a) Mtd1 ⎜ ⎟ + =0 ⇒ ⎜ ⎟ =−
⎝ 3+i⎠ z ⎝ 3+i⎠ z

( ) ( )
4 4

⇒ z4 = − 3+i = − 2e 6

⇒ z 4 = 24 e
(
i 23π +π ) = 24 ei 53π = 24 ei( − π3 ) = 24 ei( − π3 + 2 kπ )
⎛ − π + 2 kπ ⎞
i⎜ 3 ⎟ ( 6 k −1)π
⎜ 4 ⎟ i
⇒ z = 2e ⎝ ⎠
= 2e 12
, k = 0, ± 1, 2
Mtd2
3 4
⎛ z ⎞ 3+i ⎛ z ⎞ iπ i ( π + 2 kπ )
⎜ ⎟ + =0 ⇒ ⎜ ⎟ = −1 = e = e
⎝ 3+i⎠ z ⎝ 3+i⎠
( π + 2 kπ )
z z i
⇒ = π =e 4
k = 0, ± 1, − 2
3+i i
2e 6
( π + 2 kπ ) π
± i ±

z i i
⇒ π
=e 4
=e 4
,e 4
i
2e 6
π 5π 7π 11π
− i i − i i
⇒ z = 2e 12
, 2e 12
, 2e 12
, 2e 12

2
Mtd3
3 4
⎛ z ⎞ 3+i ⎛ z ⎞
⎜ ⎟ + =0 ⇒ ⎜ ⎟ +1 = 0
⎝ 3+i⎠ z ⎝ 3+i⎠
z
⇒ = − 2 + 2i, − 2 − 2i, 2 + 2i, 2 − 2i
3+i
⇒ z= ( )(
3 + i − 2 + 2i , )( )(
3 + i − 2 − 2i , ) ( 3 + i )( 2 + 2i ) , ( 3+i )( 2 − 2i )
( ) (
⇒ z = ⎡ − 6 − 2 + 6 − 2 i⎤ ,
⎣ ⎦ ) (
⎡ 2−

6 ) + (− 6 − 2 ) i⎤ ,

( ) (
⎡ 6 − 2 + 6 + 2 i⎤ ,
⎣ ⎦ ) (
⎡ 6+

2 ) − ( 6 − 2 ) i⎤ .

Or −1.93 + 0.518i, − 0.518 − 1.93i, 0.518 + 1.93i, 1.93 − 0.518i .
Mtd4
3
⎛ z ⎞ 3+i 3+i π
z3
( )
− i
⎜ ⎟ + =0 ⇒ =− ⇒ z 4 = −8i 3 + i = 8 − 8 3i = 16e 3

⎝ 3+i⎠ z 8i z
⎛ − π + 2 kπ ⎞
i⎜ 3 ⎟ ( 6 k −1)π
⎜ 4 ⎟ i
⇒ z = 2e ⎝ ⎠
= 2e 12
, k = 0, ± 1, 2
Mtd5
3
⎛ z ⎞ 3+i
( ) ( )
4 4
⎜ ⎟ + = 0 ⇒ z4 = − 3+i ⇒ z2 = ± − 3+i = ±4e −0.5236i
⎝ 3+i⎠ z
⇒ z = ± ±4e −0.5236i = 1.93 − 0.518i, −1.93 + 0.518i, 0.518 + 1.93i, −0.518 − 1.93i
y

(b) iz + 3 = 5 ⇒ z − 3i = 5

3• P

Q θ

Mtd1 O x
S •
R
Let Q represent the number − 3 in the Argand Diagram.
Minimum of z + 3 is equal to the length QR.
To find z, we find the complex number represented by R in the above Argand Diagram.
3 π
From diagram, tan θ = = 3 ⇒ θ= .
3 3
⎛1⎞ 5
x-coordinates of R is −5cos θ = −5 ⎜ ⎟ = −
⎝2⎠ 2
⎛ 3 ⎞ 6−5 3
y-coordinates of R is 3 − 5sin θ = 3 − 5 ⎜⎜ ⎟⎟ =
⎝ 2 ⎠ 2
5 6−5 3
Therefore, required z = − + i
2 2
3
Mtd2
By similar triangle:
5 5
PQ = 2 3 ; QR = 5 − 2 3 ; PS = 3 ; RS =
2 2
⎛ 5 ⎛5 ⎞⎞ 5 ⎛5 ⎞
So P = ⎜ − , − ⎜ 3 − 3⎟ ⎟ ⇒ z = − − ⎜ 3 − 3 ⎟ i or z = −2.5 − 1.33i
⎝ 2 ⎝2 ⎠⎠ 2 ⎝2 ⎠
Mtd3
R is the intersection of Line: y = 3x + 3 and Circle: ( y − 3) + x 2 = 52 .
2

⎛ 5 ⎛5 ⎞⎞ 5 ⎛5 ⎞
Solving gives P = ⎜ − , − ⎜ 3 − 3⎟ ⎟ ⇒ z = − − ⎜ 3 − 3 ⎟ i or z = −2.5 − 1.33i
⎝ 2 ⎝2 ⎠⎠ 2 ⎝2 ⎠

4 (a)(ii)

y2 = f ( x)

0 1 x
−2

x = −1

− x2 − 2x + 5 2
(b) Note: y = = −x − 3 +
x −1 x −1
2 A 2 2
Mtd1 y = x − 4 − ⎯⎯ → y = −x − 4 − = −x − 4 +
x −x x
2 2
⎯⎯ B
→ y = − ( x − 1) − 4 + = −x − 3 +
( x − 1) x −1
A: a reflection about y-axis
B: a translation of 1 unit in the positive direction of x-axis.

Mtd2 A: a translation of 1 unit in the negative direction of x-axis.


B: a reflection in y-axis.

Mtd3 A: a translation of 1 unit in the positive direction of x-axis.


B: a reflection in x-axis.
C: a translation of 8 units in the negative direction of y-axis.

4
Mtd4 A: a translation of 1 unit in the positive direction of x-axis.
B: a translation of 8 units in the positive direction of y-axis.
C: a reflection in x-axis. x =1

− x2 − 2x + 5 2
(ii) y= = −x − 3 + ,
x −1 x −1 y = −x − 3
Asymptotes are: x = 1
and y = −x−3

− x2 − 2 x + 5
y=
( x − 1)
2
x −1 + ( y + 4) = 1
2
2
2
( x − 1)
2

+ ( y + 4) = 1
2
(iii) Adding graph of 2
2
to part (ii), there are 4 roots for the
( x − 1)
2 2
⎛ − x2 − 2 x + 5 ⎞
equation +⎜ + 4 ⎟ = 1.
22 ⎝ x −1 ⎠

Section B: Statistics [60 marks]

5 (i) − The sample may not represent all levels of college’s population.
− Students who do not go to the college canteen during that one hour will be excluded
from the sample.
− The sample is not representative because it excludes students who are not willing to
take part in the survey.

(ii) Quota sampling.

(iii) Use Stratified sampling:

Draw random samples from each level as follows:

Level IP1 IP2 JC1 JC2

78 70 692 760
Number of × 160 × 160 × 160 × 160
1600 1600 1600 1600
students
= 7.8 ≈ 8 =7 = 69.2 ≈ 69 = 76

5
6 H o : μ = 34
H1 : μ < 34
Level of significance: 2%
X −μ
Test statistic: ~ N (0, 1)
σ/ n
32.15 − 34
Assume H o is true, zcal = = −2.4917 (5 s.f.) , p –value = 0.00636 < 0.02
2.1/ 8
Since the p –value is less than the level of significance, we reject H o at 2% level of significance.
There is sufficient evidence to conclude that John’s typing speed has improved after the training
course.

2% level of significance means there is 2% chance that we wrongly support the claim that John
has improved his typing speed after the training course.

7
Male Female
Local 4 2
Foreign 2 3

(a) Number of ways = 6! × 7P5 = 1814400


(b) (i) Number of ways = 11C4 = 330

(ii) Number of ways = No. w/o restriction − No. w/o males − No. w/o females
= 11C4 − 5C4 − 6C4 = 310

(iii) No. of ways = No. w/o restriction − No. w/o females − No. w/o foreigners
+ No. with only local males
= C4 − 6C4 − 6C4 + 4C4 = 301.
11

1 1 3
8(i) P(TUFC draws a match) = 1− − =
2 5 10

(ii) P(three points after 3 games)


= P(WLL) + P(DDD)
2 3
1⎛1⎞ ⎛ 3⎞
= ⎜ ⎟ ( 3) + ⎜ ⎟
2⎝5⎠ ⎝ 10 ⎠
= 0.087

(iii) P(qualifies for second round | TUFC did not win the first game)
P ( qualifies and did not win the first game )
=
P ( did not win the first game )

6
P ( DWW ) +P ( DWD ) +P ( DDW ) +P ( LWW )
=
1
1−
2
2 2 2
3 ⎛1⎞ ⎛ 3 ⎞ ⎛1⎞ 1⎛1⎞
⎜ ⎟ + 2⎜ ⎟ ⎜ ⎟ + ⎜ ⎟
10 ⎝ 2 ⎠ ⎝ 10 ⎠ ⎝ 2 ⎠ 5 ⎝ 2 ⎠
=
1
2
= 0.43

9 Let A be the timing of swimmer A for the 100m freestyle. A ~ N (48.0, 0.52 )
Let B be the timing of swimmer B for the 100m freestyle. B ~ N (47.2, 0.82 )

(i) Mtd1
Required Probability = 1− P(non among the two broke the world record)
= 1 − P ( A ≥ 46.9 ) P ( B ≥ 46.9 )
= 1 − ( 0.986097 )( 0.646170 ) = 0.36281 ≈ 0.363
Mtd2
Required Probability = P(A breaks record) + P(B breaks record) – P(both break record)
= P ( A < 46.9 ) + P ( B < 46.9 ) − P ( A < 46.9 ) P ( B < 46.9 )
≈ 0.363
Mtd3
Required Probability = P(A breaks record, B don’t break record)
+ P(A don’t break record, B breaks record) + P(both break record)
= P ( A < 46.9, B ≥ 46.9 ) + P ( A ≥ 46.9, B < 46.9 )
+ P ( A < 46.9 ) P ( B < 46.9 )
≈ 0.363

(ii) A − B ~ N (0.8, 0.89)


P ( A > B ) = P ( A − B > 0 ) = 0.801781 ≈ 0.802

(iii) 20 × (1 − 0.801781) ≈ 4
Expected times out of 20 is 4 times.

(iv) Let W = A1 + A2 + A3 + A4 − 4 B ~ N (3.2,11.24)


P ( A1 + A2 + A3 + A4 > 4 B ) = P (W > 0 ) = 0.83008 ≈ 0.830

10 (a) Let X be the number of prizes won out of N games. X ~ B ( N , 0.05 )


P ( X > 5 ) < 0.1 ⇒ P ( X ≤ 5 ) ≥ 0.9
From G.C., When N = 63, P ( X ≤ 5 ) = 0.90551 > 0.9
When N = 64, P ( X ≤ 5 ) = 0.89990 < 0.9
Therefore, largest number of games is 63.
7
(b)(i) Let Y be the number of prizes won out of 60 games. Y ~ B ( 60, 0.05 )
Since 60 × 0.05 = 3 < 5 ,
Y ~ Po ( 3) approximately
P (Y ≤ 5% ( 60 ) ) = P (Y ≤ 3) = 0.64723 = 0.647 (3 s.f.)

(ii) Let S be the number of prizes won out of 200 games. S ~ B ( 200, 0.05 )
Since 200 × 0.05 = 10 > 5 , 200 × 0.95 = 190 > 5
S ~ N (10, 9.5 ) approximately
P ( S ≤ 5% ( 200 ) ) = P ( S ≤ 10 ) ≈ P ( S < 10.5 )
= 0.564434 = 0.564 (3 s.f.)

(c) Mtd1:
Y ~ B ( 60, 0.05 ) , E(Y) = 60 × 0.05 = 3 , Var(Y) = 60(0.05)(0.95) = 2.85
Since sample size 70 is large, by CLT,
⎛ 2.85 ⎞
Y ~ N ⎜ 3, ⎟ , i.e. Y ~ N ( 3, 0.04071)
⎝ 70 ⎠
( )
P 2.5 ≤ Y ≤ 3 = 0.49340 = 0.493 (3 s.f.)
Mtd2:
Let T be the total number of prizes won out of 70 × 60 games.
T ~ B ( 4200, 0.05 )
⎛ T ⎞
P ⎜ 2.5 ≤ ≤ 3 ⎟ = P (175 ≤ T ≤ 210 ) = 0.51333 ≈ 0.513
⎝ 70 ⎠

11
(i) The least squares regression line of y on x should be used as x is an independent
variable and y depends on x.

(ii) Equation of the regression line of y on x is y = 21.9 + 0.504x


The slope of this regression line means Mary’s travelling time will increase by 0.504
minutes for each minute she leaves home after 7 am.

(iii) The product moment correlation coefficient, r = 0.988


which indicates the time that Mary leave home after 7 am and the time that she takes
to travel are strongly positively linearly correlated.

(iv) z = Time available – Time taken = 90 – x – y.

(v) For Mary not to be late, x + y ≤ 90


⇒ x + 21.881 + 0.504x ≤ 90
⇒ x ≤ 45.292
The latest time that Mary can leave home is 7.45am.

This value is reliable as the value of r is close to 1 and x = 45 is within the given set of
data. The estimate is an interpolation.

8
VICTORIA JUNIOR COLLEGE
Preliminary Examination
Higher 2
MATHEMATICS 9740/ 01
(Paper 1)

September 2010

3 hours
Additional materials: Answer paper
Graph Paper
List of Formulae (MF15)

READ THESE INSTRUCTIONS FIRST

Write your name and CT group on all the work you hand in.
Write in dark blue or black pen on both sides of the paper.
You may use a soft pencil for any diagrams or graphs.
Do not use staples, paper clips, highlighters, glue or correction fluid.

Answer all the questions.


Give non-exact numerical answers correct to 3 significant figures, or 1 decimal place in the case of angles in
degrees, unless a different level of accuracy is specified in the question.
You are expected to use a graphic calculator.
Unsupported answers from a graphic calculator are allowed unless a question specifically states otherwise.
Where unsupported answers from a graphic calculator are not allowed, you are required to present the
mathematical steps using mathematical notations and not calculator commands.
You are reminded of the need for clear presentation in your answers.

The number of marks is given in brackets [ ] at the end of each question or part question.
At the end of the examination, fasten all your work securely together.

This document consists of 5 printed pages

© VJC 2010 VICTORIA JUNIOR COLLEGE [Turn over


1 The equation of a curve C is x 2  3 xy  y 2  5 . Show that every line parallel to the x-axis
cuts C at two distinct points. [3]

Without differentiating, explain, giving a reason, if there is any point on the curve at which
the tangent is parallel to the x-axis. [1]

2 In the game of TapFarm, a player is given plots of land to grow tomatoes, pumpkins and
cherries. The player can only plant a type of fruits on a plot of land each time and each plot
of land produces 1 kg of fruits. The time taken from planting to harvesting the fruits, cost
price and selling price for 1kg of fruits are as follows:

Time required (Hours) Cost price ($) Selling price ($)


Tomatoes 3 15 35
Pumpkin 5 a 50
Cherries 16 45 100

Tommy has only 1 plot of land. He used $545 and spent 154 hours playing the game before
harvesting a total of 23 kg of fruits and earned a profit of $735. Showing your working
clearly, find the value of a. [4]

[You are to assume that Tommy harvests the fruits once they are ready and there is no time
lapse between harvesting and planting new fruits.]

x2

3
3 Find the exact value of dx by using the substitution x  tan  . [5]
1 ( x 2  1) 2

1
4 (i) Given that x   , using the substitution y  2 x  1, show algebraically that
2
22  8 2 x  1  2 x is always positive, [2]

(ii) Hence, solve the inequality


22 x  8 x 2 x  1  2 x 2
 0,
 x  k  x  m 
2

where k and m are real and 0 < k < m. [4]

5 (a) Solve the equation


i( z  2) 4  (1  i)  0
giving the roots in the form z  p  re i , where p is a real number, r > 0 and
     . [4]

(b) Given that 1  i 2 is a root of the equation


3z 3  az 2  bz  3  0 ,
find the values of the real numbers a and b. [3]

2
2
6 (i) Express in partial fractions. [2]
(r  1)r (r  1)
n
1
(ii) Hence, find  . (There is no need to express your answer as a single
r  2 ( r  1) r ( r  1)
algebraic fraction.) [4]
n
1
(iii) Given that   k , for all n  2 , find the least value of k, showing your
r  2 ( r  1) r ( r  1)
working clearly. [2]

7 Positive odd integers, starting at 1, are grouped into sets containing 1, 3, 9, … odd integers ,
as indicated below, so that the number of odd integers in each set after the first is thrice the
number of odd integers in the previous set.

{1}, {3, 5, 7}, {9, 11, 13, 15, 17, 19, 21, 23, 25}, …

Find, in terms of k,
(i) the number of terms in the kth set, [1]
(ii) the number of terms in the first k sets, [2]
(iii) the first integer in the kth set, [1]
(iv) the last integer in the kth set, [2]
(v) the sum of all integers in the first k sets. [3]

8 The planes 1 and  2 have equations r.  i  j  k   6 and r.  2i  4 j  k   12


respectively. The point A has position vector 9i  7 j  5k .
(i) Find the position vector of the foot of perpendicular from A to  2 . [3]
(ii) Find a vector equation of the line of intersection of 1 and  2 [2]

The plane 3 has equation


r   i  j  k     i  3 j  3k     i  9 j  bk  ,
where  ,  are real parameters and b is a constant.
Given that 1 ,  2 and 3 have no point in common, find the value of b. [3]
3 meets 1 and  2 in lines l1 and l2 respectively. Without finding the equations of l1
and l2 , describe the relationship between l1 and l2 , giving a reason. [2]

9 A sequence of real positive numbers u1 , u2 , u3 , ... satisfies the recurrence relation


un 1  un  4  4 un  1, n   and u1  15 .
(i) Prove by induction that un  4  n  1 1 for all n   .
2
[4]
(ii) Determine, giving your reasons, if this is a converging or diverging sequence. [2]

The sequence is modified to


1 1 1
vn 1   4  4  1, n   and v1  .
vn vn 15
As n  , vn   .
(iii) Find the value of  correct to 3 decimal places. [2]
(iv) Show, graphically or otherwise, that if vn   , vn  vn 1 . [3]
[Turn over
3
10 (a) Sketch the curve given by the equation
x2 y 2
  1, 0  k  3. [2]
9 k2
Given that m is a constant and the equation
x 2 m2 x 2
 2 1
9 k
has 2 real roots, use your sketch above to find, in terms of k, an inequality satisfied
by | m | . [3]

3x( x  4)
(b) A curve C has equation y  .
x2
dy
(i) By considering , show that the gradient of C is always positive. [3]
dx
(ii) Find the equations of the asymptotes of C. [2]
(iii) Sketch C. [1]

11 (a) y

1
y=1

1 e x
y
2 1  e x
R
y=0 x
O

e x
(i) The diagram shows the curve C with equation y  . The region R is
1  e x
bounded by C, the positive x-axis and the positive y-axis. Find the exact area
of R. [3]

(ii) Denoting the answer you have obtained in part (i) by q, write down, in terms
of q, the area of R if C is scaled by factor 2 parallel to the y-axis. [1]
x
e 1
(iii) Sketch the graph of y 2  x
 , indicating clearly the equations of the
1 e 2
asymptotes and the shape of the curve for points near y = 0. [3]

4
y
11 (b) y = ln x

A (a, ln a)

x
O

A girl intends to design a bowl by rotating a section AB of the curve y  ln x completely


about the y-axis. She wants the bowl to hold 300 cm3 of fluid.

Given that the diameter of the rim is twice that of the base, write down, in terms of a, the
coordinates of the point B. Hence find the exact value of a. [5]

dy
12 Verify that the general solution of the differential equation, x  ( y  1)(1  x) is
dx
y  Axe x  1 . [1]

(i) Sketch three members of the family of solution curves, one for positive A, one for
negative A and one for A = 0. [3]

(ii) It is given that A = 3 and k is a positive constant.


 
k
Find 3 xe  x dx in terms of k. Hence, state the value of 3 xe  x dx . [4]
0 0
Indicate, on a clearly labeled diagram, the region whose area is given by

 0
3 xe  x dx . [1]

(iii) A point P is conditioned to move along the curve y  3xe x  1 such that the x-
coordinate of P increases at a constant rate of 2 units per second.
dy
(a) State the range of values of x for which 0. [1]
dt
(b) Find the x-coordinate of the point on the curve at which P is moving such
dy
that  2 . [3]
dt

5
1

VICTORIA JUNIOR COLLEGE


Preliminary Examination
Higher 2

MATHEMATICS 9740/ 02
Paper 2
September 2010

3 hours
Additional materials: Answer paper
Graph paper
List of Formulae (MF15)

READ THESE INSTRUCTIONS FIRST

Write your name and CT group on all the work you hand in.
Write in dark blue or black pen on both sides of the paper.
You may use a soft pencil for any diagrams or graphs.
Do not use staples, paper clips, highlighters, glue or correction fluid.

Answer all the questions.


Give non-exact numerical answers correct to 3 significant figures, or 1 decimal place in the case of angles in
degrees, unless a different level of accuracy is specified in the question.
You are expected to use a graphic calculator.
Unsupported answers from a graphic calculator are allowed unless a question specifically states otherwise.
Where unsupported answers from a graphic calculator are not allowed, you are required to present the
mathematical steps using mathematical notations and not calculator commands.
You are reminded of the need for clear presentation in your answers.

At the end of the examination, fasten all your work securely together.
The number of marks is given in brackets [ ] at the end of each question or part question.

This document consists of 6 printed pages

© VJC 2010 VICTORIA JUNIOR COLLEGE [Turn over


2

Section A: Pure Mathematics [40 marks]

1 Sketch, on an Argand diagram, the locus representing the complex number z for which

z  4  3i  2. [2]

(i) Given that a is the least possible value of z , find a. [2]


(ii) The complex number p is such that
p  4  3i  2 and p  a.
State the exact value of arg  p  . [1]
z
(iii) Deduce the greatest value of arg   , giving your answer correct to 2 decimal
 p
places. [2]

2 (i) A curve C has equation y  f ( x ).

y 2  f ( x) yf x
y y

y=x–4
y=–x–4
–3 O 3
–1 x  x
O 3
(–1, –1) (1, –1)

x = –3

The diagram shows the curve C 1 with equation y 2  f ( x ) and the curve C 2 with
equation y  f  x  . The line x  3 is an asymptote to C 1 and C 1 passes through
the points (  1, 0) and (3, 0).
The lines y  x  4 and y   x  4 are asymptotes to C 2. C 2 passes through the
 2
points (  3, 0), (3, 0) and  0,   . The minimum points on C 2 have coordinates
 3
 1,  1 and 1,  1 .
 7 
Given that f ( x)  0 for 1  x  0 and the point   ,  9  is the only other
 2 
turning point on C, sketch C, indicating clearly the intercepts, the equations of the
asymptotes and the coordinates of the turning points. [4]

Find the x-coordinates of the stationary points on the curve y   f ( x)  .


2
(ii) [3]
3

1 1 1


     
3 The plane  and the line l have equations r.  1   4 and r   2     0  respectively,
a  2 1
     
where  is a real parameter and a is a constant.

(a) It is given that a  1 . Find


(i) the acute angle between  and the plane z  0, [2]
(ii) the exact perpendicular distance of the point (1, 3, 2) from  . [3]
(b) It is given that a  3. Find the acute angle between l and  . [3]

1
4 (i) Obtain the expansion of up to and including the term in x 4 . [2]
1 x 2

(ii) Given that x is small such that powers of x above x3 could be ignored, use your
answer in part (i) to show that sin 1 x  x  bx3 where b is a constant to be found.
[3]
1
(iii) State the equation of the tangent to the curve y  sin x at the origin. On a single
diagram, sketch this tangent and the graph of y  x  bx3 .
You should make clear the relationship between the two graphs for points near the
origin. [3]

5 The functions f, g and h are defined by

1
f :x  x , x , x  0,
x
g : x  x ,
2
x ,
2010
h:x  , x , x  0.
x

(i) Sketch the graph of y = f(x). [2]


(ii) Define f 1 in a similar form. [3]
(iii) Use a non-calculator method to solve f 1 ( x)   4. [2]
(iv) State, giving a reason, whether fg exists. [2]
(v) Find h 21 ( x). [1]

[Turn over
4

Section B: Probability and Statistics [60 marks]

6 A circular disc is divided into n equal sectors which are labeled 1, 2, 3, … n –1 and a
“skunk”. A game is played by spinning a pointer pivoted at the centre of the disc at
most three times. If the result is a “skunk”, the game ends and the player loses all his
previous winnings. If the result from a spin is k where k = 1, 2, 3, … or n – 1, then the
player wins $k. He continues spinning and adding to his winnings until a maximum of
three spins or a “skunk” is spun.

1
Show that the probability of winning $3 is . [1]
n3
Find, in terms of n, the probability that the player
(i) wins nothing when the game ends, [2]
(ii) wins nothing when the game ends, given that he wins $3 in his first spin. [2]
[There is no need for you to simplify your answer in both cases.]

7 In a statistical investigation, a researcher wants to find out how a person’s maximum


walking speed varies with age. He selects a random sample of 12 males of certain ages
and measures their individual maximum walking speeds. Their ages, t years and
maximum walking speeds, w ms–1 are shown in the table below.

t 20 25 30 35 40 45 50 55 60 65 70 75

w 2.59 2.55 2.85 2.62 2.48 2.43 2.32 2.27 2.34 2.28 2.19 2.10

(i) Draw a scatter diagram for the data. [1]


(ii) State, giving a reason, whether a regression line of w on t or t on w could be used
to estimate the age of a male who has a maximum walking speed of 2.65 ms–1.
(There is no need to do any calculations.) [1]

The researcher decides to study the maximum walking speed of males between the age
of 30 and 55 inclusive. It is given that the correlation coefficient for the six data points
is  0.965 .

(iii) State, giving a reason, whether the regression line stated in (ii) is suitable for this
study. [1]
1
(iv) For this study, the variables y is defined by y  . For the variables y and w,
t
(a) calculate the product moment correlation coefficient and comment on its
value, [2]
(b) calculate the equation of the appropriate regression line, [1]
(c) determine the best estimate that you can of the maximum walking speed
when the age is 43. [1]
5

8 The distribution of the masses of Perayaan balls has a mean of 420 g and a standard
deviation of 6 g. Find the probability that 50 randomly selected Perayaan balls have a
combined mass which is more than 20.9 kg. [2]
State, with a reason, whether it is necessary to assume that the masses of Perayaan balls
follow a normal distribution. [1]
The masses of Jubalani balls follow a normal distribution with a mean of 440 g and a
standard deviation of 1 g. Find the probability that the combined mass of 50 randomly
selected Perayaan balls differs from 50 times the mass of one Jubalani ball by less than
900 g. [3]
State the assumption that you have made in arriving at your answer. [1]

9 The number of arrivals per minute at a fast food drive-through outlet has a Poisson
distribution with mean  . On a weekend evening,   0.8 . Find the probability that in
a 10-minute interval, there will be at most 10 arrivals given that there are more than 5
arrivals. [3]
In view of space constraints, the management wants to control the number of arrivals
during the peak period which spans over a 30-minute interval. By using a normal
distribution to approximate the Poisson distribution, find, to 4 decimal places, the
largest value of  such that the probability of having more than 30 arrivals during the
peak period is less than 0.05. [5]

10 Annabel has 7 tiles each lettered A, N, N, A, B, E, L respectively. A code-word is


formed when some tiles are picked and arranged to form a “word”.
(a) Find the number of different ways in which a 4-letter code-word can be formed
(i) if the first letter is N and last letter is E, [3]
(ii) if there are no restrictions. [5]
(b) Annabel picks up 4 tiles and arranges them in a random order. Find the probability
that the tiles spell ANNA. [2]

11 The weekly earnings, in dollars, at two casinos are modeled by independent normal
distributions with means and standard deviations as shown in the table.
Mean Earnings Standard Deviation
casino 1 600 000 50 000
casino 2 700 000 75 000

(i) Find the probability that in 2 randomly chosen weeks, the total earnings at casino
2 exceed 1 500 000 dollars. [2]
(ii) Find the probability that in a 12-week period, the weekly earnings at casino 1
exceeds $650 000 in at least 3 weeks. [3]
(iii) The government imposes a weekly tax on the earnings at casino 1 and 2 at a rate
of 7% and 10% respectively. Find the probability that the tax exceeds $99 000 in
any given week. Hence, by using a suitable approximation, find the probability
that in a year consisting of 52 weeks, the weekly tax received by the government
from the two casinos exceeds $99 000 in at least 45 weeks. [6]
[Turn over
6

12 (a) Club Gunpla emailed all of its 1600 members to find out which of the four
Celestial Being mechs was the most popular. The members were asked to select
their favourite mech. 226 members responded and it was concluded that Gundam
Exia is the most popular of the four mechs among the club members.
Explain if the sampling method used is a random one. [2]

(b) A large department store wants to find out how much its customers spend on
Gundam model kits. From a random selection of 100 transactions, the results are
summarized by

 x  3500 , x 2
 220 400 ,

where $x is the amount spent on Gundam model kits in a single transaction.


The distributor claimed that the mean amount a customer spent on Gundam model
kits is $40.
Test whether the distributor has overstated his claim at the 5% significance level.
[6]
State, giving a reason, whether any assumption is needed for the test to be valid. [1]

(c) In testing the mean breaking strain of a type of fishing line, a researcher measured
the breaking strain of 80 fishing lines. He carried out a t-test at the 5%
significance level and, based on the sample results, he concluded that the
population mean breaking strain is significantly different from 0 kN.
If the researcher had carried out a z-test instead, determine which of the following
2 statements is correct, giving clear reasons to support your claim.
(I) The researcher would have concluded that there is significant evidence at the
5% significance level that the population mean breaking strain is different
from 0 kN.
(II) It is not possible for the researcher to conclude, by using only the
information given, whether there is significant evidence at the 5%
significance level that the population mean breaking strain is different from
0 kN . [3]
Yishun Junior College♦ 2010 Preliminary Exam ♦H2 Maths 9740 Paper 1

f ( x)
1 A curve has equation given by y = , where f(x) is a quadratic function. Given
x +1
that the curve passes through the points (1, 4), (−3, −12) and (−2, −14), find the
equation of the curve.
Hence sketch the curve, showing clearly the coordinates of the turning points and the
equations of the asymptotes. [5]

(2n + 1)(2n + 3) (2n − 1)(2n + 1) 2(2n + 1)


2 (i) Show that − = . [2]
(n + 1)(n + 2) n(n + 1) n(n + 1)(n + 2)
N
(2n + 1)
(ii) Find ∑ n(n + 1)(n + 2) , in terms of N.
n =1
[3]

5 7 9 11
(iii) Hence, find the value of + + + + ... [2]
2 × 3× 4 3× 4 × 5 4 × 5× 6 5× 6 × 7

a
3 The curve C has equation y = , where a and b are positive constants.
x − bx
2

(i) Find, by differentiation, the coordinates of the turning point(s) of C. [3]


(ii) Sketch the graph of C, showing clearly the coordinates of any turning point(s)
and the equations of the asymptotes. [2]
(iii) Hence, find the range of values of k, where k is a constant, for which the
( )
equation k x 2 − bx − a = 0 has no real root. [2]
(iv) On separate diagrams, draw sketches of the graphs of
a
(a) y= , [2]
(x + b ) 2
− b( x + b )
a
(b) y2 = − . [2]
x 2 − bx

4 The functions f and g are defined by

f : x  ( x − 3) + 8 , x ∈ ℜ , x ≤ 3 ,
2

g : x  e2x + 1, x∈ℜ.

(i) Show that f −1 exists and define f −1 , giving its rule and domain. [4]

(ii) Determine, with a reason, whether the composite function f −1g exists or not.
[2]
(iii) If α and β are real numbers such that α < β ≤ 3 and gf(α) > gf(β), show that
α < 6 − β. [3]

1
Yishun Junior College♦ 2010 Preliminary Exam ♦H2 Maths 9740 Paper 1

5 The nth term of a sequence is given by un = n2n for n ≥ 1 and the sum of the first n
terms is denoted by Sn.
(i) Write down the values of S1, S2, S3, S4, S5. [2]
(ii) By considering Sn – 2, find a conjecture for the general term Sn in the form
of Sn = p2(p + 2) + 2 , where p is in terms of n. [2]
(iii) Prove the conjecture by mathematical induction for all positive integers n. [4]

6 (i) By using the substitution x = a sin θ , find, in terms of a and π ,


a
⌠ 2
 a − x 2 dx . [4]
⌡0
n

(ii) Find, in terms of n and e,  ( x + n ) e nx dx , where n ≠ 0 . [4]
⌡0
(iii) Hence, find the exact area of the region R bounded by the line x = 1 and by
the curves y = 1 − x 2 and y = ( x + 1)e x . [2]

6x
7 A curve C has equation y 2 = . The region R is enclosed by C and the line
1 + 2x 2
x = 2. Another region S is enclosed by C and the lines x = 2 and x = k (k > 2). The
volume of solid formed by region R is equal to the volume of solid formed by region
S when both R and S are rotated completely about the x-axis. Find the exact value of
k. [4]

8 Adam has many marbles that he wants to put in boxes.


(i) If he puts 13 marbles in the first box and for each subsequent box, he puts
double the number of marbles he puts in the previous box, he will need
(2k + 1) boxes for all his marbles. Given that he has 104 marbles in the
kth box, find the number of marbles he has. [4]
(ii) If he puts 13 marbles in the first box and for each subsequent box, he puts 13
marbles more than what he puts in the previous box, how many boxes will he
need and what is the number of marbles in the last box? [4]

9 (a) The complex numbers z and w are such that z = 1 + i p, w = 1 + i q, where p


and q are real and p is positive. Given that zw = 3 – 4i, find the exact values
of p and q. [4]
(b) A complex number a is given by a = 1 + i 3 . By using De Moivre’s
theorem, express 1 + a + a 2 + a 3 + .... + a 9 in the form x + i y, where x and y
are exact values to be determined. [4]

2
Yishun Junior College♦ 2010 Preliminary Exam ♦H2 Maths 9740 Paper 1

10 It is given that y = ln (1 + sin x ) .


d2 y
in terms of x and show that (1 + sin x ) 2 + cos x
dy dy
(i) Find + sin x = 0 . [2]
dx dx dx
(ii) By further differentiation of the result in (i), find the Maclaurin’s series for y,
up to and including the term in x3. [3]
 e sin x 
(iii) Deduce the Maclaurin’s series for ln  , up to and including the term

 1 + sin x 
in x3. [3]

11 The rate at which a substance evaporates is proportional to the volume of the


substance which has not yet evaporated. The initial volume of the substance is A m 3
and the volume which has evaporated at time t minutes is x m 3 . Given that it takes
 − t
1
(2ln2) minutes for half of its initial volume to evaporate, show that x = A 1 − e 2  .
 
Find the additional time needed for three quarters of the substance to evaporate,
giving your answer in exact form. [6]

12 The diagram shows a prism with the horizontal rectangular base PQRS. The
triangular planes APS and BQR are vertical and AB is horizontal.
A B

S θ
R
k j θ
2
P
i 3 Q

Given that PQ = SR = AB = 3 units, PS = QR = 2 units and each of the planes ABQP


3
and ABRS is inclined at an angle θ to the horizontal, where tan θ = .
4
The point P is taken as the origin for position vectors, with unit vectors i and j
parallel to PQ and PS respectively and unit vector k perpendicular to the plane
PQRS.

(i) Find PA . [1]
(ii) Find the exact value of the cosine of the angle PAR. [3]
(iii) Find a vector equation of the line AR. [1]
(iv) Show that the foot of perpendicular from P to the line AR has coordinates
 − 21 162 132 
 , , . [3]
 169 169 169 

3
Yishun Junior College♦ 2010 Preliminary Exam ♦H2 Maths 9740 Paper 1

13

The diagram shows the cross-section of a cone of radius r and height h which is
inscribed in a sphere of fixed radius R. Show that
V = πh 2 (2 R − h ) ,
1
3
where V is the volume of the cone.
Prove that, as r and h varies, the maximum value of V is obtained when h 2 = 2r 2 . [8]

~ End of Paper ~

4
Yishun Junior College♦ 2010 Preliminary Exam ♦H2 Maths 9740 Paper 2

Section A: Pure Mathematics [40 marks]

A curve has parametric equations x = 3(1 − t ) , y =


1
1 for t ≠ 0 .
t3
dy
(i) Find in terms of t and deduce that the curve is an increasing function. [2]
dx
 1
(ii) Find the equation of L1, the tangent to the curve at the point  3 − 3t , 3  .
 t 
Hence, find the coordinates of point P on the curve at which L1 passes through
the origin O. [4]
(iii) The line L2 is another tangent to the curve which is parallel to L1. Find the
equation of L2. [3]
(iv) The line L2 cuts the y-axis at Q. Find the exact area of triangle OPQ. [2]

2 (i) The equation 2x2 – x – ln(x + 1) = 0 has 2 real roots α and β , where α <β.
Find the values of α and β, giving any non-exact answers correct to 3 decimal
places. [1]
(ii) A sequence of positive real numbers x1 , x2 , x3 ,  satisfies the recurrence
ln( x n + 1) + x n
relation xn+1 = for n ≥ 1 .
2
Prove algebraically that, if the sequence converges, then it converges to β . [3]

(iii) If x1 = 2, write down the values of x2 and x3 .


ln( x + 1) + x
Sketch the graphs of y = and y = x on the same axes.
2
Illustrate on your diagram how the sequence x1 , x2 , x3 ,  will converge to β
starting with x1 = 2. [3]

3 (a) Solve the equation z 4 = −4 , expressing each of the roots in the form a + ib ,
where a and b are real. [4]
Hence write down the solutions of the equation ( w − 2 i) 4 = −4 . [2]
(b) On a single, clearly labelled diagram, sketch the loci of w and z defined by
z − 1 = z − 5 and w + 1 − i = 2 . [2]
(i) State the minimum value of z − w . [1]
(ii) Find the exact values of the modulus and argument of w for which
π
arg(w + 1 − i) = − .
4
Hence, express w in the form x + iy , where x and y are exact real values.
[3]

1
Yishun Junior College♦ 2010 Preliminary Exam ♦H2 Maths 9740 Paper 2

4 The plane p1 has equation x + y – 2 z = 4.


(i) A plane p2 with equation 2x + ay + bz = −4 (where a and b are constants) is
parallel to p1, find the values of a and b and also the exact distance between p1
and p2 . [3]
(ii) Another plane p3 contains a point with coordinates (2, 4, 1) and a line with
1  2 
   
equation r = 1 + s 3  , where s ∈ℜ. Show that the equation of p3 can be
1  − 3 
   
expressed as 3x – y + z = 3.
Find also a vector equation of the line l where p3 meets p1. [4]
(iii) The plane p4 has equation 5x + y + λ z = µ, where λ and µ are constants.
(a) If all three planes p1 , p3 and p4 intersect in the line l , find the values of
λ and µ . [2]
(b) Deduce the geometrical relationship of the three planes p1, p3 and p4
if λ = −3 and µ ≠ 11. [1]

Section B: Statistics [60 marks]

5 A Residents’ Committee wants to propose improvements to the recreational facilities


in Punggol. In order to find out the needs of the adults and children of both genders, a
survey is to be carried out on a sample of 120 residents who are at least 5 years old.
Given that of the 6525 male residents who are at least 5 years old, there are 1450
children. On the other hand, there are 7975 females who are at least 5 years old, out of
which 2175 are children. Describe how the sample can be obtained using quota
sampling. [1]
Name and describe another method of sampling in which each group is represented
proportionately. [3]
State an advantage of quota sampling over the above method. [1]

6 A recent research study indicates that 16% of the total population are aged 60 years or
more and that 18% of the total population have a measurable hearing defect.
Furthermore, 65% of those aged 60 years or more have a measurable hearing defect.
Find the probability that a randomly chosen person from the population
(i) has a measurable hearing defect, given that he is less than 60 years old, [3]
(ii) is either aged 60 years or more, or has a measurable hearing defect, or both. [2]
State, with a reason, whether or not the events ‘a person is 60 years old or more’ and
‘a person has a measurable hearing defect’ are independent. [1]
Find, correct to 3 decimal places, the probability that, if two persons are chosen at
random from the population, at least one of them will be aged 60 years or more and at
least one of them will have a measurable hearing defect. [4]

2
Yishun Junior College♦ 2010 Preliminary Exam ♦H2 Maths 9740 Paper 2

7 A manufacturer claims that the breaking strength of a climbing rope is normally


distributed with mean 1702 N and standard deviation 105 N. A random sample of 10
climbing ropes is tested and the mean breaking strength of the sample is x N. A test is
then carried out at the 5% significance level to determine whether the manufacturer’s
claim is valid. Given that the null hypothesis is rejected in favour of the alternative
hypothesis, find the range of possible values of x . [4]
The manufacturer believes that adding a special chemical to the ropes will increase the
mean breaking strength and change the standard deviation. A random sample of 10
such ropes is found to have a mean breaking strength of 1724 N and a standard
deviation of 35 N. Carry out a significance test at the 5% level to decide whether this
result provides sufficient evidence to confirm the manufacturer’s belief that the mean
breaking strength has increased. [4]

8 An experiment carried out to see how the intensity of radiation from a particular
radioactive source, I, varies with time, t. In the following table, the values of t may be
considered to be exact, while the values of I are subject to experimental errors.
I 22.5 25.0 28.0 30.5 38.0 40.5 42.5 48.0 54.5 55.0 70.0
t 44.0 42.0 33.5 28.0 18.0 13.6 15.0 10.3 9.0 6.3 4.0
(i) Sketch the scatter diagram for the given data. [2]
(ii) State, with a reason, which of the following models is more appropriate to fit
the data points:
A: I = a +bt2, where a > 0 and b < 0;
B: I = atb, where a > 0 and b < 0. [2]
(iii) For the appropriate model, find the product-moment correlation coefficient for
the transformed data. Find an estimate for a, correct to 3 decimal places. [2]
(iv) Hence, estimate the time when the value of intensity of radiation is 35.0 and
comment on the reliability of the estimate found. [2]

9 After production, blocks of butter are wrapped by a machine. Each block is supposed
to weigh 220 g but the blocks produced have masses which are normally distributed. A
worker then packs every six dozen randomly chosen blocks of butter in a box of mass
175 g. It is known that the total mass of each box of butter follows a normal
distribution with mean 16.375 kg and standard deviation 0.149 kg.
(i) Three such boxes of butter are chosen at random. Find the probability that each
box of butter weighs more than its mean mass. [1]
(ii) Find the percentage of blocks of butter which weigh at least 220 g. [3]
(iii) To increase the consumers’ confidence, the company wishes to adjust the mean
mass of a block of butter such that more than 75 % of the blocks will have a
mass of not less than 220 g while the standard deviation remains unchanged.
Find the least value of the mean mass after the adjustment. [3]

3
Yishun Junior College♦ 2010 Preliminary Exam ♦H2 Maths 9740 Paper 2

10 (a) Find the number of 8-letter code-words that can be formed using the letters
A, B, C, D, E if
(i) there are no restrictions, [1]
(ii) each vowel (A, E) appears once and each consonant (B, C, D) appears
twice, [1]
(iii) each letter occurs at least once and the letters appear in alphabetical
order. [3]
(b) At a particular reception, 9 guests are to stand at 3 identical round cocktail
tables. How many ways can this be done if there must be at least two people at
each table? [3]

11 A multiple choice test consists of 20 questions, each with four possible answers, of
which only one is correct. If a student randomly chooses the answer to each question,
find the probability of getting at least four but less than nine correct answers. [2]
Suppose that each correct answer is awarded five marks and each incorrect answer
carries a penalty of one mark, what is the expected score obtained by a student? [2]
50 students took the test. Using a suitable approximation, find the probability that the
mean score is more than 12 marks. [3]

12 At a post office, the number of customers purchasing postal products in a half-hour


period during peak hours follows a Poisson distribution with mean 5. During the off-
peak period, the number of customers purchasing postal products in a two-hour period
is an independent Poisson distribution with mean 10. The peak period for the post
office is from 12 pm to 1.30 pm and the post office is open from 8 am to 5 pm.
(i) Find the probability that there are more than 15 customers from 12.30 pm to
1.30 pm. [2]
(ii) Using suitable approximations, find the probability that the total number of
customers during off-peak hours is less than thrice the total number of
customers during peak hours on a particular day. [5]

~ End of Paper ~

Você também pode gostar